Patho/Pharm 4 Week 1, 2, 5 Combined (Does not include week 3-Mental Health)

अब Quizwiz के साथ अपने होमवर्क और परीक्षाओं को एस करें!

A patient is about to receive pentobarbital (Nembutal) 100 mg IV as preoperative sedation. The medication is available in a concentration of 50mg/mL. How many milliliters will the patient receive for this dose?

2 mL 50mg:1mL::100mg:x mL (50 x X)=(1 x 100); 50x=100; x=2mL

A patient is to receive midazolam (Versed) 2 mg IV push over 2 minutes just before an endoscopy procedure. The medication is available in a strength of 1 mg/mL. Identify how many milliliters of medication will the nurse draw up into the syringe for this dose. _______

2 mL If there is 1 mg in 1 mL and you need 2 mg, than you need 2 mL (1mg/mL x 2 mg = 2mL)

The medication order for a 4-year-old child reads, "Give digoxin elixir, 15 mcg/kg, PO now." Convert the micrograms to milligrams. _______

0.015

An agitated patient is to receive an intravenous dose of diazepam (Valium). The order reads, "Give diazepam, 2 mg, IV push, now. Repeat in 15 minutes if needed." Identify how many milliliters will the nurse administer for this dose. The medication vial contains 5 mg/mL.

0.4 mL

A patient is to receive a daily dose of fludarabine (Fludara), 25 mg/m2 /day for 5 consecutive days. Each dose is diluted in a 125-mL bag of normal saline and is to infuse over 30 minutes. The nurse will set the infusion pump to what rate in milliliters per hour? ______

250 mL/hr

The nurse working in the clinic receives telephone calls from several patients who want appointments as soon as possible. Which patient should be seen first? a. 71-yr-old who has noticed increasing loss of peripheral vision b. 74-yr-old who has difficulty seeing well enough to drive at night c. 60-yr-old who is reporting dry eyes with decreased tear formation d. 64-yr-old who states that it is becoming difficult to read news print

ANS: A

Which instruction should the nurse include in a teaching plan for a patient with herpes simplex keratitis? a. Wash hands frequently and avoid touching the eyes. b. Apply antibiotic drops to the eye several times daily. c. Apply a new occlusive dressing to the affected eye at bedtime. d. Use corticosteroid ophthalmic ointment to decrease inflammation.

ANS: A

During interleukin drug therapy, a patient is showing signs of severe fluid retention, with increasing dyspnea and severe peripheral edema. The next dose of the interleukin is due now. Which action will the nurse take next? A. Hold the drug, and notify the prescriber. B. Give the drug, and notify the prescriber. C. Give the drug along with acetaminophen and diphenhydramine (Benadryl). D. Monitor the patient for 2 hours, and then give the drug if the patient's condition improves.

A

The nurse notes in the patient's medication history that the patient is taking cyclobenzaprine (Flexeril). Based on this finding, the nurse interprets that the patient has which disorder? a. A musculoskeletal injury b. Insomnia c. Epilepsy d. Agitation

A Cyclobenzaprine (Flexeril) is the muscle relaxant most commonly used to reduce spasms following musculoskeletal injuries. It is not appropriate for insomnia, epilepsy, or agitation.

When a male patient is receiving androgen therapy, the nurse will monitor for signs of excessive androgens such as: a. fluid retention. b. dehydration. c. restlessness. d. visual changes.

A Fluid retention is an undesirable effect of androgens. The other options are incorrect.

In assessing a patient before administration of a cardiac glycoside, the nurse knows that which lab result can increase the toxicity of the drug? a. Potassium level 2.8 mEq/L b. Potassium level 4.9 mEq/L c. Sodium level 140 mEq/L d. Calcium level 10 mg/dL

A Hypokalemia increases the chance of digitalis toxicity. The other levels listed are incorrect.

Which action should the nurse include in the plan of care for a patient admitted with acute decompensated heart failure (ADHF) who is receiving nesiritide (Natrecor)? a. Monitor blood pressure frequently. b. Encourage patient to ambulate in room. c. Teach patient to drink at least 3 liters of fluid daily. d. Titrate nesiritide dose down slowly before stopping.

A. Monitor blood pressure frequently.

A 62-year-old male has fallen while trimming tree branches sustaining tissue injury. He describes his condition as an aching, throbbing back. This is characteristic of what type of pain? A. Nociceptive pain B. Chronic pain C. Mixed pain syndrome D. Neuropathic pain

A. Nociceptive pain

Which assessment finding in a patient admitted with acute decompensated heart failure (ADHF) requires the most immediate action by the nurse? a. O2 saturation of 88% b. Weight gain of 1 kg (2.2 lb) c. Heart rate of 106 beats/min d. Urine output of 50 mL over 2 hours

A. O2 saturation of 88%

3. The nurse recognizes that which patients are at highest risk for physiologic fatigue? (Select all that apply.) A. Parents of a newborn B. Adolescent with anorexia C. 25-year-old pregnant female D. Grandmother who takes mile walks E. Businessman who consumes six cups coffee/day

A. Parents of a newborn B. Adolescent with anorexia C. 25-year-old pregnant female E. Businessman who consumes six cups coffee/day

The nurse enters a patient's room and the patient startles easily and appears to jerk his arms and legs before awakening. Which stage of non-rapid eye movement sleep did the patient most likely awaken from? A. Stage 1 B. Stage 2 C. Stage 3 D. Stage 4

A. Stage 1

The lack of weight bearing leads to what effects on the skeletal system? a. Demineralization, calcium loss b. Thickened bones c. Increased range of motion d. Increased calcium deposition in the bones

ANS: A Weight bearing helps to strengthen the bone. Lack of weight bearing means that the bone is losing minerals and calcium that strengthen it.

A nurse recalls that the most frequent location of a neuroblastoma is the: a. retroperitoneal region. b. mediastinum. c. cervical ganglion. d. lung.

ANS: A 65% of neuroblastomas occur in the retroperitoneal region. 15% of neuroblastomas occur in the mediastinum. 4% of neuroblastomas occur in the cervical ganglion. Neuroblastomas do not occur in the lung. REF: p. 435

A patient tells the nurse "My doctor thinks my problems with stress relate to the negative way I think about things, and he wants me to learn a new way of thinking." Which response would be in keeping with the doctor's recommendations? A. Teaching the patient to recognize, reconsider, and reframe irrational thoughts B. Encouraging the patient to imagine being in calming circumstances C. Teaching the patient to use instruments that give feedback about bodily functions D. Provide the patient with a blank journal and guidance about journaling

A. Teaching the patient to recognize, reconsider, and reframe irrational thoughts

The occupational health nurse is making rounds in a factory. Which employees need further education about energy conservation strategies? (Select all that apply.) A. Workers reaching up to high shelf often to obtain cleaning supplies B. Workers bending over to lift boxes onto conveyer belt C. Workers who have delegated out parts of their activities D. Workers standing for long periods E. Workers standing at desks at waist level

A. Workers reaching up to high shelf often to obtain cleaning supplies B. Workers bending over to lift boxes onto conveyer belt D. Workers standing for long periods

A patient will be receiving mannitol (Osmitrol), 1.5 g/kg IV 1 hour before ocular surgery. The patient weighs 110 pounds. The medication is available as a 25% solution. Based on this patient's dose, calculate how many milliliters of mannitol this patient will receive, using the 25% solution.

ANS: 300 mL

The percent of retinoblastomas that are inherited as an autosomal dominant trait is ____%.

ANS: 40 REF: p. 436

A 65-yr-old patient is being evaluated for glaucoma. Which information given by the patient has implications for the patient's treatment plan? a. "I take metoprolol (Lopressor) for angina." b. "I take aspirin when I have a sinus headache." c. "I have had frequent episodes of conjunctivitis." d. "I have not had an eye examination for 10 years."

ANS: A

A patient who underwent eye surgery must wear an eye patch until the scheduled postoperative clinic visit. Which patient problem will the nurse address in the plan of care? a. Risk for falls b. Difficulty coping c. Disturbed body image d. Inability to care for home

ANS: A

Assessment of a patient's visual acuity reveals that the left eye can see at 20 feet what a person with normal vision can see at 50 feet and the right eye can see at 20 feet what a person with normal vision can see at 40 feet. Which finding should the nurse record? a. OS 20/50; OD 20/40 b. OU 20/40; OS 50/20 c. OD 20/40; OS 20/50 d. OU 40/20; OD 50/20

ANS: A

The nurse is assessing a 65-yr-old patient for presbyopia. Which instruction will the nurse give the patient before the test? a. "Hold this card and read the print out loud." b. "Cover one eye while reading the wall chart." c. "You'll feel a short burst of air directed at your eyeball." d. "A light will be used to look for a change in your pupils."

ANS: A

The nurse is testing the visual acuity of a patient in the outpatient clinic. Which instructions should the nurse give for this test? a. "Stand 20 feet away from the wall chart." b. "Look at an object far away and then near to you." c. "Follow the examiner's finger with your eyes only." d. "Look straight ahead while I check your eyes with a light."

ANS: A

Which action can the nurse working in the emergency department delegate to an experienced unlicensed assistive personnel (UAP)? a. Ask a patient with decreased visual acuity about medications taken at home. b. Perform Snellen testing of visual acuity for a patient with a history of cataracts. c. Obtain information from a patient about any history of childhood ear infections. d. Inspect a patient's external ear for redness, swelling, or presence of skin lesions.

ANS: B

Which action could the registered nurse (RN) who is working in the clinic delegate to a licensed practical/vocational nurse (LPN/VN)? a. Evaluate a patient's ability to administer eyedrops. b. Check a patient's visual acuity using a Snellen chart. c. Inspect a patient's external ear for signs of irritation caused by a hearing aid. d. Teach a patient with otosclerosis about use of sodium fluoride and vitamin D.

ANS: B

Which assessment finding alerts the nurse to provide patient teaching about cataract development? a. Unequal pupil size b. Sensitivity to light c. Loss of peripheral vision d. History of hyperthyroidism

ANS: B

Which information will the nurse provide to the patient scheduled for refractometry? a. "You should not take any of your eye medicines before the examination." b. "You will need to wear sunglasses for a few hours after the examination." c. "The doctor will shine a bright light into your eye during the examination." d. "The surface of your eye will be numb while the doctor does the examination."

ANS: B

Which nursing activity is appropriate for the registered nurse (RN) working in the eye clinic to delegate to experienced unlicensed assistive personnel (UAP)? a. Instilling antiviral drops for a patient with a corneal ulcer b. Application of a warm compress to a patient's hordeolum c. Instruction about hand washing for a patient with herpes keratitis d. Checking for eye irritation in a patient with possible conjunctivitis

ANS: B

Which patient arriving at the urgent care center will the nurse assess first? a. Patient who is reporting that the left eyelid has just started to droop b. Patient with acute right eye pain that began while using power tools c. Patient with purulent left eye discharge and conjunctival inflammation d. Patient who has redness, crusting, and swelling along the lower right lid margin

ANS: B

Which prescribed medication should the nurse give first to a patient who has just been admitted to a hospital with acute angle-closure glaucoma? a. Morphine sulfate 4 mg IV b. Mannitol (Osmitrol) 100 mg IV c. Betaxolol (Betoptic) 1 drop in each eye d. Acetazolamide (Diamox) 250 mg orally

ANS: B

Which statement by a patient with bacterial conjunctivitis indicates a need for further teaching? a. "I will wash my hands often during the day." b. "I will remove my contact lenses at bedtime." c. "I will not share towels with my friends or family." d. "I will monitor my family for eye redness or drainage."

ANS: B

Which topic will the nurse teach after a patient has had outpatient cataract surgery and lens implantation? a. Use of oral opioids for pain control b. Administration of corticosteroid drops c. Need for bed rest for 1 to 2 days after the surgery d. Importance of coughing and deep breathing exercises

ANS: B

Which patient requires the most rapid assessment and care by the emergency department nurse? a. The patient with hemochromatosis who reports abdominal pain b. The patient with neutropenia who has a temperature of 101.8 F c. The patient with sickle cell anemia who has had nausea and diarrhea for 24 hours d. The patient with thrombocytopenia who has oozing after having a tooth extracted

ANS: B A neutropenic patient with a fever is assumed to have an infection and is at risk for rapidly developing sepsis. Rapid assessment, cultures, and initiation of antibiotic therapy are needed. The other patients also require rapid assessment and care but not as urgently as the neutropenic patient.

A 40-year-old female developed adenocarcinoma of the vagina. Which prenatal event is the most likely cause of her cancer? a. Rb gene mutation b. Exposure to diethylstilbestrol (DES) c. Exposure to solvents d. Exposure to radiation

ANS: B The patient with adenocarcinoma of the vagina most likely experienced prenatal exposure to DES, not an Rb gene mutation or exposure to solvents or radiation. REF: pp. 303-304

Which set of assessment data is consistent for a patient with severe infection that could lead to system failure? a. Blood pressure (BP) 92/52, pulse (P) 56 beats/min, respiratory rate (RR) 10 breaths/min, urine output 1200 mL in past 24 hours b. BP 90/48, P 112 beats/min, RR 26 breaths/min, urine output 240 mL in past 24 hours c. BP 112/64, P 98 beats/min, RR 18 breaths/min, urine output 2400 mL in past 24 hours d. BP 152/90, P 52 beats/min, RR 12 breaths/min, urine output 4800 mL in past 24 hours

ANS: B The patient with severe infection presents with low BP and compensating elevations in pulse to move lower volumes of blood more rapidly and respiration to increase access to oxygen. Urine output decreases to counteract the decreased circulating blood volume and hypotension. These vital signs are all too low: Blood pressure (BP) 92/52, pulse (P) 56 beats/min, respiratory rate (RR) 10 breaths/min, urine output 1200 mL in past 24 hours. The patient with severe infection does have a low BP, but the pulse and respiratory rate increase to compensate. This data is all within normal limits: BP 112/64, P 98 beats/min, RR 18 breaths/min, urine output 2400 mL in past 24 hours. This set of data reflects an elevated BP with a decrease in pulse and respiratory rates along with normal urine output: BP 152/90, P 52 beats/min, RR 12 breaths/min, urine output 4800 mL in past 24 hours. None of these is a typical response to severe infection.

27. After being hospitalized for 3 days with a right femur fracture, a 32-year-old patient suddenly develops shortness of breath and tachypnea. The patient tells the nurse, "I feel like I am going to die!" Which action should the nurse take first? a. Stay with the patient and offer reassurance. b. Administer the prescribed PRN oxygen at 4 L/min. c. Check the patient's legs for swelling or tenderness. d. Notify the health care provider about the symptoms.

ANS: B The patient's clinical manifestations and history are consistent with a pulmonary embolus, and the nurse's first action should be to ensure adequate oxygenation. The nurse should offer reassurance to the patient, but meeting the physiologic need for oxygen is a higher priority. The health care provider should be notified after the oxygen is started and pulse oximetry and assessment for fat embolus or venous thromboembolism (VTE) are obtained. DIF: Cognitive Level: Apply (application) REF: 1528 OBJ: Special Questions: Prioritization TOP: Nursing Process: Implementation MSC: NCLEX: Physiological Integrity

32. During routine hemodialysis, a patient complains of nausea and dizziness. Which action should the nurse take first? a. Slow down the rate of dialysis. b. Check the blood pressure (BP). c. Review the hematocrit (Hct) level. d. Give prescribed PRN antiemetic drugs.

ANS: B The patient's complaints of nausea and dizziness suggest hypotension, so the initial action should be to check the BP. The other actions may also be appropriate based on the blood pressure obtained. DIF: Cognitive Level: Analyze (analysis) REF: 1090 OBJ: Special Questions: Prioritization TOP: Nursing Process: Implementation MSC: NCLEX: Physiological Integrity

3. The nurse is planning care for a patient with severe heart failure who has developed elevated blood urea nitrogen (BUN) and creatinine levels. The primary treatment goal in the plan will be a. augmenting fluid volume. b. maintaining cardiac output. c. diluting nephrotoxic substances. d. preventing systemic hypertension.

ANS: B The primary goal of treatment for acute kidney injury (AKI) is to eliminate the cause and provide supportive care while the kidneys recover. Because this patient's heart failure is causing AKI, the care will be directed toward treatment of the heart failure. For renal failure caused by hypertension, hypovolemia, or nephrotoxins, the other responses would be correct. DIF: Cognitive Level: Apply (application) REF: 1073 TOP: Nursing Process: Planning MSC: NCLEX: Physiological Integrity

The treatment of Osgood-Schlatter disease includes: a. steroids. b. restriction from physical activity. c. intense physical therapy. d. knee replacement surgery.

ANS: B The primary treatment of Osgood-Schlatter disease includes restriction from physical activity. The primary treatment does not include steroids, intense physical therapy, or knee replacement. REF: p. 1045

A patient is receiving high doses of methotrexate and is experiencing severe bone marrow suppression. The nurse expects which intervention to be ordered with this drug to reduce this problem? a. A transfusion of whole blood b. Leucovorin rescue c. Therapy with filgrastim (Neupogen) d. Administration of allopurinol (Zyloprim)

B (High-dose methotrexate is associated with bone marrow suppression, and it is always given in conjunction with the rescue drug leucovorin, which is an antidote for folic acid antagonists. Basically, leucovorin rescues the healthy cells from methotrexate. The other options are incorrect.)

A patient has an order for the monoclonal antibody adalimumab (Humira). The nurse notes that the patient does not have a history of cancer. What is another possible reason for administering this drug? A. Severe anemia B. Rheumatoid arthritis C. Thrombocytopenia D. Osteoporosis

B

Abatacept (Orencia) is prescribed for a patient with severe rheumatoid arthritis. The nurse checks the patient's medical history, knowing that this medication would need to be used cautiously if which condition is present? A. Coronary artery disease B. Chronic obstructive pulmonary disease C. Diabetes mellitus D. Hypertension

B

When monitoring a patient's response to interferon therapy, the nurse notes that the major dose-limiting factor for interferon therapy is which condition? A. Diarrhea B. Fatigue C. Anxiety D. Nausea and vomiting

B

The nurse is administering a combination of three different antineoplastic drugs to a patient who has metastatic breast cancer. Which statement best describes the rationale for combination therapy? a. There will be less nausea and vomiting. b. Increased cancer-cell killing will occur. c. The drugs will prevent metastasis. d. Combination therapy reduces the need for radiation therapy.

B (Because drug-resistant cells commonly develop, exposure to multiple drugs with multiple mechanisms and sites of action will destroy more subpopulations of cells. The other options are incorrect.)

A patient, diagnosed with lymphoma, has an allergy to one of the proposed chemotherapy drugs. The tumor has not responded to other types of treatment. The nurse expects the oncologist to follow which course of treatment? a. The physician will choose another drug to use. b. The chemotherapy will be given along with supportive measures to treat a possible allergic reaction. c. The patient will receive reduced doses of chemotherapy for a longer period of time. d. The chemotherapy cannot be given because of the patient's allergy.

B (Even if a patient has a known allergic reaction to a given antineoplastic medication, the urgency of treating the patient's cancer may still necessitate administering the medication and then treating any allergic symptoms with supportive medications, such as antihistamines, corticosteroids, and acetaminophen.)

Which problem reported by a patient with hemophilia is most important for the nurse to communicate to the physician? a. Leg bruises b. Tarry stools c. Skin abrasions d. Bleeding gums

ANS: B Melena is a sign of gastrointestinal bleeding and requires collaborative actions such as checking hemoglobin and hematocrit and administration of coagulation factors. The other problems indicate a need for patient teaching about how to avoid injury, but are not indicators of possible serious blood loss.

A patient who is receiving methotrexate for severe rheumatoid arthritis develops a megaloblastic anemia. The nurse will anticipate teaching the patient about increasing oral intake of a. iron b. folic acid. c. cobalamin (vitamin B12). d. ascorbic acid (vitamin C).

ANS: B Methotrexate use can lead to folic acid deficiency. Supplementation with oral folic acid supplements is the usual treatment. The other nutrients would not correct folic acid deficiency, although they would be used to treat other types of anemia.

An appropriate nursing intervention for a hospitalized patient with severe hemolytic anemia is to a. provide a diet high in vitamin K. b. alternate periods of rest and activity. c. teach the patient how to avoid injury. d. place the patient on protective isolation.

ANS: B Nursing care for patients with anemia should alternate periods of rest and activity to encourage activity without causing undue fatigue. There is no indication that the patient has a bleeding disorder, so a diet high in vitamin K or teaching about how to avoid injury is not needed. Protective isolation might be used for a patient with aplastic anemia, but it is not indicated for hemolytic anemia.

An assessment finding for a 55-yr-old patient that alerts the nurse to the presence of osteoporosis is a. bowed legs. b. a loss of height. c. the report of frequent falls. d. an aversion to dairy products.

ANS: B Osteoporosis occurring in the vertebrae produces a gradual loss of height. Bowed legs are associated with osteomalacia. Low intake of dairy products is a risk factor for osteoporosis, but it does not indicate that osteoporosis is present. Frequent falls increase the risk for fractures but are not an indicator of osteoporosis.

A patient has been taking iron supplements for anemia for 2 months. During a follow-up assessment, the nurse will observe for which therapeutic response? a. Decreased weight b. Increased activity tolerance c. Decreased palpitations d. Increased appetite

ANS: B Absence of fatigue, increased activity tolerance and well-being, and improved nutrition status are therapeutic responses to iron supplementation. The other options are incorrect.

A 50-year-old man who has been taking phenobarbital for 1 week is found very lethargic and unable to walk after eating out for dinner. His wife states that he has no other prescriptions and that he did not take an overdose—the correct number of pills is in the bottle. The nurse suspects that which of these may have happened? a. He took a multivitamin. b. He drank a glass of wine. c. He took a dose of aspirin. d. He developed an allergy to the drug.

ANS: B Alcohol has an additive effect when combined with barbiturates and causes central nervous system (CNS) depression. Multivitamins and aspirin do not interact with barbiturates, and this situation does not illustrate an allergic reaction. DIF: COGNITIVE LEVEL: Applying (Application) REF: p. 195 TOP: NURSING PROCESS: Assessment MSC: NCLEX: Physiological Integrity: Pharmacological and Parenteral Therapies

The nurse notes in a patient's medication history that the patient is taking allopurinol (Zyloprim). Based on this finding, the nurse interprets that the patient has which disorder? a. Rheumatoid arthritis b. Gout c. Osteoarthritis d. Systemic lupus erythematosus

ANS: B Allopurinol is indicated for the treatment of gout but is not indicated for the other disorders listed.

The nurse is administering intravenous iron dextran for the first time to a patient with anemia. After giving a test dose, how long will the nurse wait before administering the remaining portion of the dose? a. 30 minutes b. 1 hour c. 6 hours d. 24 hours

ANS: B Although anaphylactic reactions usually occur within a few moments after the test dose, it is recommended that a period of at least 1 hour elapse before the remaining portion of the initial dose is given. The other options are incorrect.

When a 70-year-old female presents with a hip fracture she is diagnosed with osteoporosis. One factor that most likely contributed to her condition is: a. increased androgen levels. b. decreased estrogen levels. c. strenuous exercise. d. excessive dietary calcium.

ANS: B Osteoporosis can be attributed to decreased estrogen levels. Osteoporosis is not attributed to increased androgen levels, excessive exercise, or excessive dietary calcium.

Osteosarcoma is most likely present in the _____ of long bones. a. epiphyses b. metaphyses c. marrow d. osteocytes

ANS: B Osteosarcoma occurs mainly in the metaphyses of long bones near sites of active physical growth, not in the epiphyses. Osteosarcoma is not associated with the marrow or osteocytes. REF: p. 1049

17. The nurse is administering oxytocin (Pitocin). Which situation is an indication for the use of oxytocin? a. Decreased fetal heart rate and movements b. Stimulation of contractions during labor c. Cervical ripening near term in pregnant patients d. To reverse premature onset of labor

ANS: B Oxytocin is used to induce labor at or near full-term gestation and to enhance labor when uterine contractions are weak and ineffective. DIF: COGNITIVE LEVEL: Applying (Application) REF: p. 545 TOP: NURSING PROCESS: Implementation MSC: NCLEX: Physiological Integrity: Pharmacological and Parenteral Therapies

4. Which assessment finding in a 36-year-old patient is most indicative of a need for further evaluation? a. Bilateral breast nodules that are tender with palpation b. A breast nodule that is 1 cm in size, nontender, and fixed c. A breast lump that increases in size before the menstrual period d. A breast lump that is small, mobile, with a rubbery consistency

ANS: B Painless and fixed lumps suggest breast cancer. The other findings are more suggestive of benign processes such as fibrocystic breasts and fibroadenoma. DIF: Cognitive Level: Apply (application) TOP: Nursing Process: Assessment MSC: NCLEX: Physiological Integrity

Palliative care used in the management of a patient with symptomatic chronic obstructive pulmonary disease (COPD) is an example of which of the following? a. Palliative care is used when the patient is beginning to die. b. Palliative care is used to help manage the symptoms that often accompany COPD. c. Hospice nurses must be involved to provide palliative care in a cancer patient. d. Patient must be enrolled into the Medicare Hospice Benefit to receive palliative care.

ANS: B Palliation is the relief or management of symptoms without providing a cure. To palliate is to reduce the severity of an actual or potential life-threatening condition or a chronic debilitating illness. Palliation is not equivalent to cure, but it is the reduction of undesirable effects resulting from the incurable disease or condition.

11. A patient who is to have no weight bearing on the left leg is learning to walk using crutches. Which observation by the nurse indicates that the patient can safely ambulate independently? a. The patient moves the right crutch with the right leg and then the left crutch with the left leg. b. The patient advances the left leg and both crutches together and then advances the right leg. c. The patient uses the bedside chair to assist in balance as needed when ambulating in the room. d. The patient keeps the padded area of the crutch firmly in the axillary area when ambulating.

ANS: B Patients are usually taught to move the crutches and the injured leg forward at the same time and then to move the unaffected leg. Patients are discouraged from using furniture to assist with ambulation. The patient is taught to place weight on the hands, not in the axilla, to avoid nerve damage. If the 2- or 4-point gaits are to be used, the crutch and leg on opposite sides move forward, not the crutch and same-side leg. DIF: Cognitive Level: Apply (application) REF: 1521 TOP: Nursing Process: Evaluation MSC: NCLEX: Safe and Effective Care Environment

12. The nurse is providing preoperative teaching about the transverse rectus abdominis musculocutaneous (TRAM) procedure to a patient. Which information will the nurse include? a. Saline-filled implants are placed under the pectoral muscles. b. Recovery from the TRAM surgery takes at least 6 to 8 weeks. c. Muscle tissue removed from the back is used to form a breast. d. TRAM flap procedures may be done in outpatient surgery centers.

ANS: B Patients take at least 6 to 8 weeks to recover from the TRAM surgery. Tissue from the abdomen is used to reconstruct the breast. The TRAM procedure can take up to 8 hours and requires postoperative hospitalization. Saline implants are used in mammoplasty. DIF: Cognitive Level: Apply (application) TOP: Nursing Process: Implementation MSC: NCLEX: Physiological Integrity

2. When a patient with acute kidney injury (AKI) has an arterial blood pH of 7.30, the nurse will expect an assessment finding of a. persistent skin tenting b. rapid, deep respirations. c. hot, flushed face and neck. d. bounding peripheral pulses.

ANS: B Patients with metabolic acidosis caused by AKI may have Kussmaul respirations as the lungs try to regulate carbon dioxide. Bounding pulses and vasodilation are not associated with metabolic acidosis. Because the patient is likely to have fluid retention, poor skin turgor would not be a finding in AKI. DIF: Cognitive Level: Apply (application) REF: 1072 TOP: Nursing Process: Assessment MSC: NCLEX: Physiological Integrity

A patient asks about his cancer treatment with monoclonal antibodies. The nurse tells him that which is the major advantage of treating certain cancers with monoclonal antibodies? A. They will help the patient improve more quickly than will other antineoplastic drugs. B. They are more effective against metastatic tumors. C. Monoclonal antibodies target certain tumor cells and yet bypass normal cells. D. There are fewer incidences of opportunistic infections with monoclonal antibodies.

C

A patient who has received chemotherapy has a critically low platelet count. The nurse expects which drug or drug class to be used to stimulate platelet cell production? A. Filgrastim (Neupogen) B. Interferons C. Oprelvekin (Neumega) D. Epoetin alfa (Epogen)

C

A patient who has received chemotherapy has a steadily decreasing white blood cell count. The chemotherapy will end on Tuesday afternoon. The oncologist has mentioned that a colony-stimulating factor will be started soon. The nurse knows that the appropriate time to start this medication is when? A. While the patient is still receiving chemotherapy B. Two hours after the chemotherapy ends C. Wednesday afternoon, 24 hours after the chemotherapy ends D. In 2 to 4 days, after the white blood cells have reached their nadir

C

A patient who has been on methotrexate therapy is experiencing mild pain. The patient is asking for aspirin for the pain. The nurse recognizes that which of these is true in this situation? a. The aspirin will aggravate diarrhea. b. The aspirin will masks signs of infection. c. Aspirin can lead to methotrexate toxicity. d. The aspirin will cause no problems for the patient on methotrexate.

C (Methotrexate interacts with weak organic acids, such as aspirin, and can lead to toxicity by displacing the methotrexate from protein-binding sites.)

The nurse is reviewing drug therapy for hypertension. According to the JNC-8 guidelines, antihypertensive drug therapy for a newly diagnosed hypertensive African-American patient would most likely include which drug or drug classes? a. Vasodilators alone b. ACE inhibitors alone c. Calcium channel blockers with thiazide diuretics d. Beta blockers with thiazide diuretics

C According to the JNC-8 guidelines, calcium channel blockers and diuretics are recommended as first-line therapy for management of hypertension in African-American patients. The other drugs are not recommended as first-line drugs for this group.

A patient has been taking haloperidol (Haldol) for 3 months for a psychotic disorder, and the nurse is concerned about the development of extrapyramidal symptoms. The nurse will monitor the patient closely for which effects? a. Increased paranoia b. Drowsiness and dizziness c. Tremors and muscle twitching d. Dry mouth and constipation

C Extrapyramidal symptoms are manifested by tremors and muscle twitching, and the incidence of such symptoms is high during haloperidol therapy. The other options are incorrect.

A patient has been diagnosed with angina and will be given a prescription for sublingual nitroglycerin tablets. When teaching the patient how to use sublingual nitroglycerin, the nurse will include which instruction? a. Take up to 5 doses at 15-minute intervals for an angina attack. b. If the tablet does not dissolve quickly, chew the tablet for maximal effect. c. If the chest pain is not relieved after one tablet, call 911 immediately. d. Wait 1 minute between doses of sublingual tablets, up to 3 doses.

C According to current guidelines, if the chest pain or discomfort is not relieved in 5 minutes, after 1 dose, the patient (or family member) must call 911 immediately. The patient may take one more tablet while awaiting emergency care and may take a third tablet 5 minutes later, but no more than a total of three tablets. The sublingual dose is placed under the tongue, and the patient needs to avoid swallowing until the tablet has dissolved.

A patient presents with poison ivy on the extremities, face, and buttocks after an initial exposure 48 hours ago. This condition is an example of: a. anaphylaxis. b. serum sickness. c. delayed hypersensitivity. d. viral disease.

C delayed hypersensitivity.

A nurse recalls that an example of an immune-complex-mediated disease is: a. bronchial asthma. b. contact dermatitis. c. serum sickness. d. rheumatoid arthritis

C serum sickness.

A 15-year-old male suffers from severe hemorrhage following a motor vehicle accident. He is given a blood transfusion, but shortly afterward the red blood cells are destroyed by agglutination and lysis. Which of the following blood type transfusion type matches would cause this? a. A-A b. B-O c. AB-O d. A-AB

D A-AB

A 30-year-old male is having difficulty breathing and has been spitting blood. He reports that he began experiencing this reaction after cleaning his pigeons' cages. Testing reveals he is suffering from allergic alveolitis. Which of the following is he experiencing? a. Serum sickness b. Raynaud phenomenon c. Antibody-dependent cytotoxicity d. Arthus reaction

D Arthus reaction

Which information would indicate more teaching is needed regarding hypersensitivity reactions? Type _______ hypersensitivity reactions involve an antibody response. a. I b. II c. III d. IV

D IV

A 40-year-old female is diagnosed with SLE. Which of the following findings would be considered a symptom of this disease? a. Gastrointestinal ulcers b. Decreased glomerular filtration rate c. Rash on trunk and extremities d. Photosensitivity

D Photosensitivity

Which statement by a patient with newly diagnosed heart failure indicates to the nurse that teaching was effective? a. "I will take furosemide (Lasix) every day just before bedtime." b. "I will use the nitroglycerin patch whenever I have chest pain." c. "I will use an additional pillow if I am short of breath at night." d. "I will call the clinic if my weight goes up 3 pounds in a week."

D. "I will call the clinic if my weight goes up 3 pounds in a week."

The nurse has been asked to administer a coping measurement instrument to a patient. What education would the nurse present to the patient related to this tool? A. "This tool will let us compare your stress to other patients in the hospital." B. "This tool is short because it only measures the negative stressors you are experiencing." C. "You will need to ask your parents about stressors you had as a child to complete this tool." D. "This tool will help assess recent positive and negative events you are experiencing."

D. "This tool will help assess recent positive and negative events you are experiencing."

A client is experiencing severe pain in the left lower quadrant of the abdomen that is rated as a 10 on a pain scale of 0-10. The client is also experiencing nausea, vomiting, and restlessness. Based on this data, the nurse concludes that the client is experiencing which phenomenon? A. Chronic pain B. Fibromyalgia pain C. End-of-life pain D. Acute pain

D. Acute pain

Following an acute myocardial infarction, a previously healthy 63-yr-old develops heart failure. What medication topic should the nurse anticipate including in discharge teaching? a. Beta-Adrenergic blockers b. Calcium channel blockers c. Digitalis and potassium therapy regimen d. Angiotensin-converting enzyme (ACE) inhibitors

D. Angiotensin-converting enzyme (ACE) inhibitors

An outpatient who has chronic heart failure returns to the clinic after 2 weeks of therapy with metoprolol (Toprol XL). Which assessment finding is most important for the nurse to report to the health care provider? a. 2+ bilateral pedal edema b. Heart rate of 52 beats/min c. Report of increased fatigue d. Blood pressure (BP) of 88/42 mm Hg

D. Blood pressure (BP) of 88/42 mm Hg

Based on the significant effects of chronic idiopathic fatigue, what is the nurse's priority assessment? A. Cholesterol level and lipid profile B. Creatinine and BUN levels C. Memory loss testing D. Mental health evaluation

D. Mental health evaluation

A 70-year-old retired nurse is interested in nondrug, mind-body therapies, self-management, and alternative strategies to deal with joint discomfort from rheumatoid arthritis. What options should you consider in her plan of care considering her expressed wishes? A. Stationary exercise bicycle, free weights, and spinning class B. Chamomile tea and IcyHot gel C. Acupuncture and attending church services D. Mind-body therapies such as music therapy, distraction techniques, meditation, prayer, hypnosis, guided imagery, relaxation techniques, and pet therapy

D. Mind-body therapies such as music therapy, distraction techniques, meditation, prayer, hypnosis, guided imagery, relaxation techniques, and pet therapy

The nurse palpates swollen nodes in a patient's neck who presented to the clinic with complaints of fatigue lasting at least 2 weeks. What is the nurse's best action? A. Advise patient this finding is normal. B. Review patient's thyroid lab work. C. Perform deep tendon reflexes. D. Notify the healthcare provider.

D. Notify the healthcare provider.

A patient who has just been admitted with pulmonary edema is scheduled to receive the following medications. Which medication should the nurse question before giving? a. captopril (Capoten) 25 mg b. furosemide (Lasix) 60 mg c. digoxin (Lanoxin) 0.125 mg d. carvedilol (Coreg) 3.125 mg

D. carvedilol (Coreg) 3.125 mg

Aldesleukin [IL-2] (Proleukin) is prescribed for a patient. The nurse reviews the patient's medication list and would note a potential drug interaction if which drug class is also ordered? A. Anticoagulants B. Antiepileptic drugs C. Oral hypoglycemic drugs D. Antihypertensive drugs

D

During a patient's therapy with interleukins, the nurse monitors the patient for capillary leak syndrome. Which assessment finding, if present, would indicate this problem? A. Bradycardia B. A dry cough C. Bruising on the skin D. A sudden, 15-pound weight gain

D

To make a diagnosis of fibromyalgia, tenderness must be present in ____ of the 18 points necessary for diagnosis.

ANS: 11

A patient is to receive codeine, 40 mg subcutaneously, every 6 hours as needed for pain. The solution is available in a concentration of 30 mg/mL. Identify how many milliliters of codeine will be drawn up for this dose. (record answer to one decimal place) _______

1.3 mL 40 mg divided by 30 mg/mL = 1.3333333... Rounded to nearest tenth (one decimal place) = 1.3

There is a new order for Naproxen (Naprosyn) 250 mg PO every 6 hours. The drug is available as an oral suspension that contains 125 mg/5 mL. Identify how many milliliters will the nurse administer for 1 dose of this medication. _______

10 mL

A patient is to receive an infusion of 250 mL of platelets over 2 hours through tubing that is labeled 1 mL equals 10 drops. How many drops per minute will the nurse infuse?

21 drops/min

A 30-year-old female is diagnosed with systemic lupus erythematosus (SLE). Which symptoms are a result of a type II hypersensitivity? a. Anemia b. Seizures c. Lymphopenia d. Facial rash e. Photosensitivity

A, C

A patient will be receiving mitoxantrone (Novantrone), 12 mg/m2 every 3 weeks, as part of treatment for prostate cancer. Each dose is mixed into a 50-mL bag of D5W and needs to infuse over 15 minutes. The infusion pump delivers the dose at milliliters per hour. Identify the nurse will set the pump to infuse at what rate. _______

200 mL/hr

A 2-year-old child will be receiving ferrous sulfate oral drops (Fer-Iron) 5 mg/kg/day in three divided doses. The child weighs 26 pounds. Identify how many milligrams will the nurse administer per dose. (record answer using one decimal place) _______

ANS: 19.7 mg

A patient will be receiving testosterone cypionate (Depo-Testosterone) 400 mg intramuscularly every 4 weeks. The medication is available in a 200-mg/mL strength. Identify how many milliliters will the nurse draw up for each injection. ______

ANS: 2mL

A patient is to receive enalapril (Vasotec) 5 mg IV every 6 hours. Each dose is given over 5 minutes. The medication is available in an injectable form, 1.25 mg/mL. Identify how many milliliters of medication will the nurse draw up for each dose. _______

4 mL

A person is given an attenuated antigen as a vaccine. When the person asks what was given in the vaccine, how should the nurse respond? The antigen is: a. alive, but less infectious. b. mutated, but highly infectious. c. normal, but not infectious. d. inactive, but infectious.

A alive, but less infectious.

When a nurse cares for a patient with systemic lupus erythematosus (SLE), the nurse remembers this disease is an example of: a. autoimmunity. b. alloimmunity. c. homoimmunity. d. alleimmunity.

A autoimmunity

A patient complains of not being able to fall asleep at night and asks the nurse if there is a safe, non-prescription medication he can try. After consulting the healthcare provider, the nurse should recommend which naturally occurring hormone? A. Melatonin B. Cortisol C. Luteinizing hormone D. Estrogen

A. Melatonin

Polyarthritis is a type of juvenile arthritis in which more than _____ joints are affected.

ANS: 3 REF: p. 1043

A patient will be receiving aldesleukin [IL-2] (Proleukin), 600,000 IU/kg every 8 hours for 14 doses. The patient weighs 220 pounds. Identify how many IU of medication this patient will receive per dose.

60 million

A patient is to receive filgrastim (Neupogen) 5 mcg/kg/day. The patient weighs 198 pounds. Identify how many micrograms of medication this patient will receive each day.

450 mcg

A patient will be receiving metoprolol (Lopressor) 5 mg IV push for angina. The medication is available in a strength of 1 mg/mL. Identify how much medication will the nurse draw up for each dose. _______

5 mL

: Nursing Process: Implementation MSC: NCLEX: Physiological Integrity 16. A chemotherapy drug that causes alopecia is prescribed for a patient. Which action should the nurse take to support the patient's self-esteem? a. Encourage the patient to purchase a wig or hat to wear when hair loss begins. b. Suggest that the patient limit social contacts until regrowth of the hair occurs. c. Teach the patient to wash hair gently with mild shampoo to minimize hair loss. d. Inform the patient that hair usually grows back once chemotherapy is complete.

: A The patient is taught to anticipate hair loss and to be prepared with wigs, scarves, or hats. Limiting social contacts is not appropriate at a time when the patient is likely to need a good social support system. The damage occurs at the hair follicles and will occur regardless of gentle washing or use of a mild shampoo. The information that the hair will grow back is not immediately helpful in maintaining the patient's self-esteem. DIF: Cognitive Level: Apply (application) REF: 256

6. A patient who is diagnosed with cervical cancer classified as Tis, N0, M0 asks the nurse what the letters and numbers mean. Which response by the nurse is accurate? a. "The cancer involves only the cervix." b. "The cancer cells look like normal cells." c. "Further testing is needed to determine the spread of the cancer." d. "It is difficult to determine the original site of the cervical cancer."

: A Cancer in situ indicates that the cancer is localized to the cervix and is not invasive at this time. Cell differentiation is not indicated by clinical staging. Because the cancer is in situ, the origin is the cervix. Further testing is not indicated given that the cancer has not spread. DIF: Cognitive Level: Apply (application) REF: 241

: Nursing Process: Evaluation MSC: NCLEX: Physiological Integrity 22. Interleukin-2 (IL-2) is used as adjuvant therapy for a patient with metastatic renal cell carcinoma. Which information should the nurse include when explaining the purpose of this therapy to the patient? a. IL-2 enhances the body's immunologic response to tumor cells. b. IL-2 prevents bone marrow depression caused by chemotherapy. c. IL-2 protects normal cells from harmful effects of chemotherapy. d. IL-2 stimulates malignant cells in the resting phase to enter mitosis.

: A IL-2 enhances the ability of the patient's own immune response to suppress tumor cells. IL-2 does not protect normal cells from damage caused by chemotherapy, stimulate malignant cells to enter mitosis, or prevent bone marrow depression. DIF: Cognitive Level: Understand (comprehension) REF: 258

37. The nurse assesses a patient with non-Hodgkin's lymphoma who is receiving an infusion of rituximab (Rituxan). Which assessment finding would require the most rapid action by the nurse? a. Shortness of breath b. Shivering and chills c. Muscle aches and pains d. Temperature of 100.2° F (37.9° C)

: A Rituximab (Rituxan) is a monoclonal antibody. Shortness of breath should be investigated rapidly because anaphylaxis is a possible reaction to monoclonal antibody administration. The nurse will need to rapidly take actions such as stopping the infusion, assessing the patient further, and notifying the health care provider. The other findings will also require action by the nurse, but are not indicative of life-threatening complications. DIF: Cognitive Level: Analyze (analysis) REF: 258 OBJ: Special Questions: Prioritization

5. During a routine health examination, a 40-yr-old patient tells the nurse about a family history of colon cancer. Which action should the nurse take next? a. Obtain more information about the family history. b. Schedule a sigmoidoscopy to provide baseline data. c. Teach the patient about the need for a colonoscopy at age 50. d. Teach the patient how to do home testing for fecal occult blood.

: A The patient may be at increased risk for colon cancer, but the nurse's first action should be further assessment. The other actions may be appropriate, depending on the information that is obtained from the patient with further questioning. DIF: Cognitive Level: Analyze (analysis) REF: 241

33. When caring for a patient who is pancytopenic, which action by unlicensed assistive personnel (UAP) indicates a need for the nurse to intervene? a. The UAP assists the patient to use dental floss after eating. b. The UAP adds baking soda to the patient's saline oral rinses. c. The UAP puts fluoride toothpaste on the patient's toothbrush. d. The UAP has the patient rinse after meals with a saline solution.

: A Use of dental floss is avoided in patients with pancytopenia because of the risk for infection and bleeding. The other actions are appropriate for oral care of a pancytopenic patient. DIF: Cognitive Level: Apply (application) REF: 261 OBJ: Special Questions: Delegation

MULTIPLE RESPONSE 1. The nurse at the clinic is interviewing a 64-yr-old woman who is 5 feet, 3 inches tall and weighs 125 lb (57 kg). The patient has not seen a health care provider for 20 years. She walks 5 miles most days and has a glass of wine two or three times a week. Which topics will the nurse plan to include in patient teaching about cancer screening and decreasing cancer risk (select all that apply)? a. Pap testing b. Tobacco use c. Sunscreen use d. Mammography e. Colorectal screening

: A, C, D, E The patient's age, gender, and history indicate a need for screening and teaching about colorectal cancer, mammography, Pap smears, and sunscreen. The patient does not use tobacco or excessive alcohol, she is physically active, and her body weight is healthy. DIF: Cognitive Level: Analyze (analysis) REF: 235

2. A patient develops neutropenia after receiving chemotherapy. Which information about ways to prevent infection will the nurse include in the teaching plan (select all that apply)? a. Cook food thoroughly before eating. b. Choose low fiber, low residue foods. c. Avoid public transportation such as buses. d. Use rectal suppositories if needed for constipation. e. Talk to the oncologist before having any dental work.

: A, C, E Eating only cooked food and avoiding public transportation will decrease infection risk. A high-fiber diet is recommended for neutropenic patients to decrease constipation. Because bacteria may enter the circulation during dental work or oral surgery, the patient may need to postpone dental work or take antibiotics. DIF: Cognitive Level: Apply (application) REF: 253

35. The nurse receives change-of-shift report on the oncology unit. Which patient should the nurse assess first? a. A 35-yr-old patient who has wet desquamation associated with abdominal radiation b. A 42-yr-old patient who is sobbing after receiving a new diagnosis of ovarian cancer c. A 24-yr-old patient who received neck radiation and has blood oozing from the neck d. A 56-yr-old patient who developed a new pericardial friction rub after chest radiation

: C Because neck bleeding may indicate possible carotid artery rupture in a patient who is receiving radiation to the neck, this patient should be seen first. The diagnoses and clinical manifestations for the other patients are not immediately life threatening. DIF: Cognitive Level: Analyze (analysis) REF: 263 OBJ: Special Questions: Multiple Patients

34. The nurse supervises the care of a patient with a temporary radioactive cervical implant. Which action by unlicensed assistive personnel (UAP), if observed by the nurse, would require an intervention? a. The UAP flushes the toilet once after emptying the patient's bedpan. b. The UAP stands by the patient's bed for 30 minutes talking with the patient. c. The UAP places the patient's bedding in the laundry container in the hallway. d. The UAP gives the patient an alcohol-containing mouthwash to use for oral care.

: B Because patients with temporary implants emit radioactivity while the implants are in place, exposure to the patient is limited. Laundry and urine and feces do not have any radioactivity and do not require special precautions. Cervical radiation will not affect the oral mucosa, and alcohol-based mouthwash is not contraindicated. DIF: Cognitive Level: Apply (application) REF: 250 OBJ: Special Questions: Delegation

30. The nurse assesses a patient who is receiving interleukin-2. Which finding should the nurse report immediately to the health care provider? a. Generalized muscle aches b. Crackles heard at the lung bases c. Complaints of nausea and anorexia d. Oral temperature of 100.6° F (38.1° C)

: B Capillary leak syndrome and acute pulmonary edema are possible toxic effects of interleukin-2. The patient may need oxygen and the nurse should rapidly notify the health care provider. The other findings are common side effects of interleukin-2. DIF: Cognitive Level: Analyze (analysis) REF: 257

: Nursing Process: Implementation MSC: NCLEX: Physiological Integrity 23. The home health nurse is caring for a patient who has been receiving interferon therapy for treatment of cancer. Which statement by the patient indicates a need for further assessment? a. "I have frequent muscle aches and pains." b. "I rarely have the energy to get out of bed." c. "I experience chills after I inject the interferon." d. "I take acetaminophen (Tylenol) every 4 hours."

: B Fatigue can be a dose-limiting toxicity for use of immunotherapy. Flulike symptoms, such as muscle aches and chills, are common side effects with interferon use. Patients are advised to use acetaminophen every 4 hours. DIF: Cognitive Level: Apply (application) REF: 258

: Nursing Process: Diagnosis MSC: NCLEX: Psychosocial Integrity 28. A hospitalized patient who has received chemotherapy for leukemia develops neutropenia. Which observation by the nurse would indicate a need for further teaching? a. The patient ambulates around the room. b. The patient's visitors bring in fresh peaches. c. The patient cleans with a warm washcloth after having a stool. d. The patient uses soap and shampoo to shower every other day.

: B Fresh, thinned-skin fruits are not permitted in a neutropenic diet because of the risk of bacteria being present. The patient should ambulate in the room rather than the hospital hallway to avoid exposure to other patients or visitors. Because overuse of soap can dry the skin and increase infection risk, showering every other day is acceptable. Careful cleaning after having a bowel movement will help prevent skin breakdown and infection. DIF: Cognitive Level: Apply (application) REF: 253

4. The nurse should suggest which food choice when providing dietary teaching for a patient scheduled to receive external-beam radiation for abdominal cancer? a. Fruit salad c. Creamed broccoli b. Baked chicken d. Toasted wheat bread

: B Protein is needed for wound healing. To minimize the diarrhea that is commonly associated with bowel radiation, the patient should avoid foods high in roughage, such as fruits and whole grains. Lactose intolerance may develop secondary to radiation, so dairy products should also be avoided. DIF: Cognitive Level: Apply (application) REF: 254

38. A patient who is being treated for stage IV lung cancer tells the nurse about new-onset back pain. Which action should the nurse take first? a. Give the patient the prescribed PRN opioid. b. Assess for sensation and strength in the legs. c. Notify the health care provider about the symptoms. d. Teach the patient how to use relaxation to reduce pain.

: B Spinal cord compression, an oncologic emergency, can occur with invasion of tumor into the epidural space. The nurse will need to assess the patient further for symptoms such as decreased leg sensation and strength and then notify the health care provider. Administration of opioids or the use of relaxation may be appropriate but only after the nurse has assessed for possible spinal cord compression. DIF: Cognitive Level: Analyze (analysis) REF: 264 OBJ: Special Questions: Prioritization

15. The nurse administers an IV vesicant chemotherapeutic agent to a patient. Which action is most important for the nurse to take? a. Infuse the medication over a short period of time. b. Stop the infusion if swelling is observed at the site. c. Administer the chemotherapy through a small-bore catheter. d. Hold the medication unless a central venous line is available.

: B Swelling at the site may indicate extravasation, and the IV should be stopped immediately. The medication generally should be given slowly to avoid irritation of the vein. The size of the catheter is not as important as administration of vesicants into a running IV line to allow dilution of the chemotherapy drug. These medications can be given through peripheral lines, although central vascular access devices are preferred. DIF: Cognitive Level: Analyze (analysis) REF: 246

12. The nurse is caring for a patient with colon cancer who is scheduled for external radiation therapy to the abdomen. Which information obtained by the nurse would indicate a need for patient teaching? a. The patient has a history of dental caries. b. The patient swims several days each week. c. The patient snacks frequently during the day. d. The patient showers each day with mild soap.

: B The patient is instructed to avoid swimming in salt water or chlorinated pools during the treatment period. The patient does not need to change habits of eating frequently or showering with a mild soap. A history of dental caries will not impact the patient who is scheduled for abdominal radiation. DIF: Cognitive Level: Apply (application) REF: 255

: Nursing Process: Assessment MSC: NCLEX: Physiological Integrity 24. A patient with leukemia is considering whether to have hematopoietic stem cell transplantation (HSCT). The nurse will include which information in the patient's teaching plan? a. Donor bone marrow is transplanted through a sternal or hip incision. b. Hospitalization is required for several weeks after the stem cell transplant. c. The transplant procedure takes place in a sterile operating room to minimize the risk for infection. d. Transplant of the donated cells can be very painful because of the nerves in the tissue lining the bone.

: B The patient requires strict protective isolation to prevent infection for 2 to 4 weeks after HSCT while waiting for the transplanted marrow to start producing cells. The transplanted cells are infused through an IV line so the transplant is not painful, nor is an operating room or incision required. DIF: Cognitive Level: Understand (comprehension) REF: 261

: Nursing Process: Planning MSC: NCLEX: Physiological Integrity 20. A widowed mother of four school-age children is hospitalized with metastatic ovarian cancer. The patient is crying and tells the nurse that she does not know what will happen to her children when she dies. Which response by the nurse is most appropriate? a. "Don't you have any friends that will raise the children for you?" b. "Would you like to talk about options for the care of your children?" c. "For now you need to concentrate on getting well and not worrying about your children." d. "Many patients with cancer live for a long time, so there is time to plan for your children."

: B This response expresses the nurse's willingness to listen and recognizes the patient's concern. The responses beginning "Many patients with cancer live for a long time" and "For now you need to concentrate on getting well" close off discussion of the topic and indicate that the nurse is uncomfortable with the topic. In addition, the patient with metastatic ovarian cancer may not have a long time to plan. Although it is possible that the patient's friends will raise the children, more assessment information is needed before making plans. DIF: Cognitive Level: Apply (application) REF: 265

11. A patient with Hodgkin's lymphoma who is undergoing external radiation therapy tells the nurse, "I am so tired I can hardly get out of bed in the morning." Which intervention should the nurse add to the plan of care? a. Minimize activity until the treatment is completed. b. Establish time to take a short walk almost every day. c. Consult with a psychiatrist for treatment of depression. d. Arrange for delivery of a hospital bed to the patient's home.

: B Walking programs are used to keep the patient active without excessive fatigue. Having a hospital bed does not necessarily address the fatigue. The better option is to stay as active as possible while combating fatigue. Fatigue is expected during treatment and is not an indication of depression. Minimizing activity may lead to weakness and other complications of immobility. DIF: Cognitive Level: Apply (application) REF: 253

: Nursing Process: Planning MSC: NCLEX: Physiological Integrity 25. The nurse teaches a patient with cancer of the liver about high-protein, high-calorie diet choices. Which snack choice by the patient indicates that the teaching has been effective? a. Lime sherbet c. Fresh strawberries b. Blueberry yogurt d. Cream cheese bagel

: B Yogurt has high biologic value because of the protein and fat content. Fruit salad does not have high amounts of protein or fat. Lime sherbet is lower in fat and protein than yogurt. Cream cheese is low in protein. DIF: Cognitive Level: Apply (application) REF: 261

7. The nurse teaches a patient who is scheduled for a prostate needle biopsy about the procedure. Which statement, if made by the patient, indicates that teaching was effective? a. "The biopsy will remove the cancer in my prostate gland." b. "The biopsy will determine how much longer I have to live." c. "The biopsy will help decide the treatment for my enlarged prostate." d. "The biopsy will indicate whether the cancer has spread to other organs."

: C A biopsy is used to determine whether the prostate enlargement is benign or malignant and determines the type of treatment that will be needed. A biopsy does not give information about metastasis, life expectancy, or the impact of cancer on the patient's life. DIF: Cognitive Level: Apply (application) REF: 238

10. External-beam radiation is planned for a patient with cervical cancer. What instructions should the nurse give to the patient to prevent complications from the effects of the radiation? a. Test all stools for the presence of blood. b. Maintain a high-residue, high-fiber diet. c. Clean the perianal area carefully after every bowel movement. d. Inspect the mouth and throat daily for the appearance of thrush.

: C Radiation to the abdomen will affect organs in the radiation path, such as the bowel, and cause frequent diarrhea. Careful cleaning of this area will help decrease the risk for skin breakdown and infection. Stools are likely to have occult blood from the inflammation associated with radiation, so routine testing of stools for blood is not indicated. Radiation to the abdomen will not cause stomatitis. A low-residue diet is recommended to avoid irritation of the bowel when patients receive abdominal radiation. DIF: Cognitive Level: Apply (application) REF: 251

39. The nurse is caring for a patient with left-sided lung cancer. Which finding would be most important for the nurse to report to the health care provider? a. Hematocrit of 32% b. Pain with deep inspiration c. Serum sodium of 126 mEq/L d. Decreased breath sounds on left side

: C The syndrome of inappropriate antidiuretic hormone (and the resulting hyponatremia) is an oncologic metabolic emergency and requires rapid treatment to prevent complications such as seizures and coma. The other findings also require intervention but are common in patients with lung cancer and not immediately life threatening. DIF: Cognitive Level: Analyze (analysis) REF: 263 OBJ: Special Questions: Prioritization

9. A patient with a large stomach tumor attached to the liver is scheduled for a debulking procedure. Which information should the nurse teach the patient about the outcome of this procedure? a. Pain will be relieved by cutting sensory nerves in the stomach. b. Relief of pressure in the stomach will promote better nutrition. c. Decreasing the tumor size will improve the effects of other therapy. d. Tumor growth will be controlled by the removal of malignant tissue.

: C A debulking surgery reduces the size of the tumor and makes radiation and chemotherapy more effective. Debulking surgeries do not control tumor growth. The tumor is debulked because it is attached to the liver, a vital organ (not to relieve pressure on the stomach). Debulking does not sever the sensory nerves, although pain may be lessened by the reduction in pressure on the abdominal organs. DIF: Cognitive Level: Understand (comprehension) REF: 245

13. A patient undergoing external radiation has developed a dry desquamation of the skin in the treatment area. The nurse teaches the patient about the management of the skin reaction. Which statement, if made by the patient, indicates the teaching was effective? a. "I can use ice packs to relieve itching." b. "I will scrub the area with warm water." c. "I can buy aloe vera gel to use on my skin." d. "I will expose my skin to a sun lamp each day."

: C Aloe vera gel and cream may be used on the radiated skin area. Ice and sunlamps may injure the skin. Treatment areas should be cleaned gently to avoid further injury. DIF: Cognitive Level: Apply (application) REF: 255

: Nursing Process: Implementation MSC: NCLEX: Psychosocial Integrity 21. A patient who has severe pain associated with terminal pancreatic cancer is being cared for at home by family members. Which finding by the nurse indicates that teaching regarding pain management has been effective? a. The patient uses the ordered opioid pain medication whenever the pain is greater than 5 (0 to 10 scale). b. The patient agrees to take the medications by the IV route in order to improve analgesic effectiveness. c. The patient takes opioids around the clock on a regular schedule and uses additional doses when breakthrough pain occurs. d. The patient states that nonopioid analgesics may be used when the maximal dose of the opioid is reached without adequate pain relief.

: C For chronic cancer pain, analgesics should be taken on a scheduled basis, with additional doses as needed for breakthrough pain. Taking the medications only when pain reaches a certain level does not provide effective pain control. Although nonopioid analgesics may also be used, there is no maximum dose of opioid. Opioids are given until pain control is achieved. The IV route is not more effective than the oral route, and usually the oral route is preferred. DIF: Cognitive Level: Apply (application) REF: 264

: Nursing Process: Evaluation MSC: NCLEX: Physiological Integrity 29. The nurse is caring for a patient diagnosed with stage I colon cancer. When assessing the need for psychologic support, which question by the nurse will provide the most information? a. "How long ago were you diagnosed with this cancer?" b. "Do you have any concerns about body image changes?" c. "Can you tell me what has been helpful to you in the past when coping with stressful events?" d. "Are you familiar with the stages of emotional adjustment to a diagnosis like cancer of the colon?"

: C Information about how the patient has coped with past stressful situations helps the nurse determine usual coping mechanisms and their effectiveness. The length of time since the diagnosis will not provide much information about the patient's need for support. The patient's knowledge of typical stages in adjustment to a critical diagnosis does not provide insight into patient needs for assistance. Because surgical interventions for stage I cancer of the colon may not cause any body image changes, this question is not appropriate at this time. DIF: Cognitive Level: Apply (application) REF: 265

41. After change-of-shift report on the oncology unit, which patient should the nurse assess first? a. Patient who has a platelet count of 82,000/μL after chemotherapy b. Patient who has xerostomia after receiving head and neck radiation c. Patient who is neutropenic and has a temperature of 100.5° F (38.1° C) d. Patient who is worried about getting the prescribed long-acting opioid on time

: C Temperature elevation is an emergency in neutropenic patients because of the risk for rapid progression to severe infections and sepsis. The other patients also require assessments or interventions but do not need to be assessed as urgently. Patients with thrombocytopenia do not have spontaneous bleeding until the platelets are 20,000/μL. Xerostomia does not require immediate intervention. Although breakthrough pain needs to be addressed rapidly, the patient does not appear to have breakthrough pain. DIF: Cognitive Level: Analyze (analysis) REF: 253 OBJ: Special Questions: Prioritization | Special Questions: Multiple Patients

2. The nurse is caring for a patient receiving intravesical bladder chemotherapy. The nurse should monitor for which adverse effect? a. Nausea c. Hematuria b. Alopecia d. Xerostomia

: C The adverse effects of intravesical chemotherapy are confined to the bladder. The other adverse effects are associated with systemic chemotherapy. DIF: Cognitive Level: Apply (application) REF: 252

1. A patient who is scheduled for a breast biopsy asks the nurse the difference between a benign tumor and a malignant tumor. Which answer by the nurse is correct? a. "Benign tumors do not cause damage to other tissues." b. "Benign tumors are likely to recur in the same location." c. "Malignant tumors may spread to other tissues or organs." d. "Malignant cells reproduce more rapidly than normal cells."

: C The major difference between benign and malignant tumors is that malignant tumors invade adjacent tissues and spread to distant tissues and benign tumors do not metastasize. The other statements are inaccurate. Both types of tumors may cause damage to adjacent tissues. Malignant cells do not reproduce more rapidly than normal cells. Benign tumors do not usually recur. DIF: Cognitive Level: Understand (comprehension) REF: 240

: Nursing Process: Evaluation MSC: NCLEX: Physiological Integrity 26. A patient with cancer has a nursing diagnosis of imbalanced nutrition: less than body requirements related to altered taste sensation. Which nursing action would address the cause of the patient problem? a. Add protein powder to foods such as casseroles. b. Tell the patient to eat foods that are high in nutrition. c. Avoid giving the patient foods that are strongly disliked. d. Add spices to enhance the flavor of foods that are served.

: C The patient will eat more if disliked foods are avoided and foods that the patient likes are included instead. Additional spice is not usually an effective way to enhance taste. Adding protein powder does not address the issue of taste. The patient's poor intake is not caused by a lack of information about nutrition. DIF: Cognitive Level: Apply (application) REF: 262

8. The nurse teaches a postmenopausal patient with stage III breast cancer about the expected outcomes of cancer treatment. Which patient statement indicates that the teaching has been effective? a. "After cancer has not recurred for 5 years, it is considered cured." b. "The cancer will be cured if the entire tumor is surgically removed." c. "I will need follow-up examinations for many years after treatment before I can be considered cured." d. "Cancer is never cured, but the tumor can be controlled with surgery, chemotherapy, and radiation."

: C The risk of recurrence varies by the type of cancer. Some cancers are considered cured after a shorter time span or after surgery, but stage III breast cancer will require additional therapies and ongoing follow-up. DIF: Cognitive Level: Apply (application) REF: 243

14. A patient with metastatic cancer of the colon experiences severe vomiting after each administration of chemotherapy. Which action, if taken by the nurse, is appropriate? a. Have the patient eat large meals when nausea is not present. b. Offer dry crackers and carbonated fluids during chemotherapy. c. Administer prescribed antiemetics 1 hour before the treatments. d. Give the patient a glass of a citrus fruit beverage during treatments.

: C Treatment with antiemetics before chemotherapy may help prevent nausea. The patient should eat small, frequent meals. Offering food and beverages during chemotherapy is likely to cause nausea. The acidity of citrus fruits may be further irritating to the stomach. DIF: Cognitive Level: Apply (application) REF: 251

: Nursing Process: Implementation MSC: NCLEX: Physiological Integrity 19. A patient has been assigned the nursing diagnosis of imbalanced nutrition: less than body requirements related to painful oral ulcers. Which nursing action will be most effective in improving oral intake? a. Offer the patient frequent small snacks between meals. b. Assist the patient to choose favorite foods from the menu. c. Provide teaching about the importance of nutritional intake. d. Apply prescribed anesthetic gel to oral lesions before meals.

: D Because the etiology of the patient's poor nutrition is the painful oral ulcers, the best intervention is to apply anesthetic gel to the lesions before the patient eats. The other actions might be helpful for other patients with impaired nutrition but would not be as helpful for this patient. DIF: Cognitive Level: Analyze (analysis) REF: 254

31. The nurse obtains information about a hospitalized patient who is receiving chemotherapy for colorectal cancer. Which information about the patient alerts the nurse to discuss a possible change in cancer therapy with the health care provider? a. Frequent loose stools b. Nausea and vomiting c. Elevated white blood count (WBC) d. Increased carcinoembryonic antigen (CEA)

: D An increase in CEA indicates that the chemotherapy is not effective for the patient's cancer and may need to be modified. Gastrointestinal adverse effects are common with chemotherapy. The nurse may need to address these, but they would not necessarily indicate a need for a change in therapy. An elevated WBC may indicate infection but does not reflect the effectiveness of the colorectal cancer therapy. DIF: Cognitive Level: Apply (application) REF: 236

40. An older adult patient who has colorectal cancer is receiving IV fluids at 175 mL/hr in conjunction with the prescribed chemotherapy. Which finding by the nurse is most important to report to the health care provider? a. Patient complains of severe fatigue. b. Patient voids every hour during the day. c. Patient takes only 50% of meals and refuses snacks. d. Patient has crackles up to the midline posterior chest.

: D Rapid fluid infusions may cause heart failure, especially in older patients. The other findings are common in patients who have cancer or are receiving chemotherapy. DIF: Cognitive Level: Analyze (analysis) REF: 266 OBJ: Special Questions: Prioritization

3. The nurse is caring for a patient who smokes two packs/day. Which action by the nurse could help reduce the patient's risk of lung cancer? a. Teach the patient about the seven warning signs of cancer. b. Plan to monitor the patient's carcinoembryonic antigen (CEA) level. c. Teach the patient about annual chest x-rays for lung cancer screening. d. Discuss risks associated with cigarette smoking during each patient encounter.

: D Teaching about the risks associated with cigarette smoking is recommended at every patient encounter because cigarette smoking is associated with multiple health problems. The other options may detect lung cancer that is already present but do not reduce the risk. DIF: Cognitive Level: Apply (application) REF: 237

: Nursing Process: Planning MSC: NCLEX: Psychosocial Integrity 17. A patient who has ovarian cancer is crying and tells the nurse, "My husband rarely visits. He just doesn't care." The husband indicates to the nurse that he does not know what to say to his wife. Which nursing diagnosis is appropriate for the nurse to add to the plan of care? a. Compromised family coping related to disruption in lifestyle b. Impaired home maintenance related to perceived role changes c. Risk for caregiver role strain related to burdens of caregiving responsibilities d. Dysfunctional family processes related to effect of illness on family members

: D The data indicate that this diagnosis is most appropriate because poor communication among the family members is affecting family processes. No data suggest a change in lifestyle or its role as an etiology. The data do not support impairment in home maintenance or a burden caused by caregiving responsibilities. DIF: Cognitive Level: Apply (application) REF: 265

32. The nurse reviews the laboratory results of a patient who is receiving chemotherapy. Which laboratory result is most important to report to the health care provider? a. Hematocrit 30% b. Platelets 95,000/μL c. Hemoglobin 10 g/L d. White blood cells (WBC) 2700/μL

: D The low WBC count places the patient at risk for severe infection and is an indication that the chemotherapy dose may need to be lower or that WBC growth factors such as filgrastim (Neupogen) are needed. Although the other laboratory data indicate decreased levels, they do not indicate any immediate life-threatening adverse effects of the chemotherapy. DIF: Cognitive Level: Apply (application) REF: 235 OBJ: Special Questions: Prioritization

: Nursing Process: Diagnosis MSC: NCLEX: Psychosocial Integrity 18. A patient receiving head and neck radiation for larynx cancer has ulcerations over the oral mucosa and tongue and thick, ropey saliva. Which instructions should the nurse give to this patient? a. Remove food debris from the teeth and oral mucosa with a stiff toothbrush. b. Use cotton-tipped applicators dipped in hydrogen peroxide to clean the teeth. c. Gargle and rinse the mouth several times a day with an antiseptic mouthwash. d. Rinse the mouth before and after each meal and at bedtime with a saline solution.

: D The patient should rinse the mouth with a saline solution frequently. A soft toothbrush is used for oral care. Hydrogen peroxide may damage tissues. Antiseptic mouthwashes may irritate the oral mucosa and are not recommended. DIF: Cognitive Level: Apply (application) REF: 251

: Nursing Process: Implementation MSC: NCLEX: Physiological Integrity 27. During the teaching session for a patient who has a new diagnosis of acute leukemia, the patient is restless and looks away without making eye contact. The patient asks the nurse to repeat the information about the complications associated with chemotherapy. Based on this assessment, which nursing diagnosis is appropriate for the patient? a. Risk for ineffective adherence to treatment related to denial of need for chemotherapy b. Acute confusion related to infiltration of leukemia cells into the central nervous system c. Deficient knowledge: chemotherapy related to a lack of interest in learning about treatment d. Risk for ineffective health maintenance related to possible anxiety about leukemia diagnosis

: D The patient who has a new cancer diagnosis is likely to have high anxiety, which may impact learning and require that the nurse repeat and reinforce information. The patient's history of a recent diagnosis suggests that infiltration of the leukemia is not a likely cause of the confusion. The patient asks for the information to be repeated, indicating that lack of interest in learning and denial are not etiologic factors. DIF: Cognitive Level: Apply (application) REF: 265

36. Which action should the nurse take when caring for a patient who is receiving chemotherapy and complains of problems with concentration? a. Teach the patient to rest the brain by avoiding new activities. b. Teach that "chemo-brain" is a short-term effect of chemotherapy. c. Report patient symptoms immediately to the health care provider. d. Suggest use of a daily planner and encourage adequate rest and sleep.

: D Use of tools to enhance memory and concentration such as a daily planner and adequate rest are helpful for patients who develop "chemo-brain" while receiving chemotherapy. Patients should be encouraged to exercise the brain through new activities. Chemo-brain may be short or long term. There is no urgent need to report common chemotherapy side effects to the provider. DIF: Cognitive Level: Apply (application) REF: 252

One patient has cancer of the bone; another has cancer in the connective tissues of the thigh muscles; a third patient has cancer in the vascular tissues. Which of these is the correct term for these tumors? a. Sarcoma b. Leukemia c. Carcinoma d. Lymphoma

A (Sarcomas are malignant tumors that arise from connective tissues. These tissues can be found in bone, cartilage, muscle, blood, lymphatic, and vascular tissues. The other options are incorrect.)

The nurse is monitoring a patient who has severe bone marrow suppression following antineoplastic drug therapy. Which is considered a principal early sign of infection? a. Fever b. Diaphoresis c. Tachycardia d. Elevated white blood cell count

A (Fever and/or chills may be the first sign of an oncoming infection. Elevated white blood cell count will not occur because of the bone marrow suppression. The other options are incorrect.)

During treatment of a patient who has brain cancer, the nurse hears the oncologist mention that the patient has reached the "nadir." The nurse knows that this term means which of these? a. The lowest level of neutrophils reached during therapy. b. The highest level of neutrophils reached during therapy. c. The point at which the adverse effects of chemotherapy will stop. d. The point at which the cytotoxic action against cancer cells is the highest.

A (The lowest neutrophil count reached after a course of chemotherapy is known as the nadir. The other options are incorrect.)

A 22-year-old patient has been taking lithium for 1 year, and the most recent lithium level is 0.9 mEq/L. Which statement about the laboratory result is correct? a. The lithium level is therapeutic. b. The lithium level is too low. c. The lithium level is too high. d. Lithium is not usually monitored with blood levels.

A Desirable long-term maintenance lithium levels range between 0.6 and 1.2 mEq/L. The other responses are incorrect.

The nurse is reviewing the medication list of a patient who will be starting androgen therapy. Which drug classes, if taken with androgens, may have an interaction with them? a. Oral anticoagulants b. Nitrates c. Beta blockers d. Proton pump inhibitors

A Androgens, when used with oral anticoagulants, can significantly increase or decrease anticoagulant activity. The other options are incorrect.

When applying transdermal nitroglycerin patches, which instruction by the nurse is correct? a. "Rotate application sites with each dose." b. "Use only the chest area for application sites." c. "Temporarily remove the patch if you go swimming." d. "Apply the patch to the same site each time."

A Application sites for transdermal nitroglycerin patches need to be rotated. Apply the transdermal patch to any nonhairy area of the body; the old patch should first be removed. The patch may be worn while swimming, but if it does come off, it should be replaced after the old site is cleansed.

A patient who has received some traumatic news is panicking and asks for some medication to help settle down. The nurse anticipates giving which drug that is most appropriate for this situation? a. Diazepam (Valium) b. Zolpidem (Ambien) c. Phenobarbital d. Cyclobenzaprine (Flexeril)

A Benzodiazepines such as diazepam are used as anxiolytics, or sedatives. Zolpidem is used as a hypnotic for sleep. Phenobarbital is not used as an anxiolytic but is used for seizure control. Cyclobenzaprine is a muscle relaxant and is not used to reduce anxiety.

The nurse is preparing for a community education program on hypertension. Which of these parameters determine the regulation of arterial blood pressure? a. Cardiac output and systemic vascular resistance b. Heart rate and peripheral resistance c. Blood volume and renal blood flow d. Myocardial contractility and arteriolar constriction

A Blood pressure is determined by the product of cardiac output and systemic vascular resistance. The other options are incorrect.

The nurse reads in the patient's medication history that the patient is taking buspirone (BuSpar). The nurse interprets that the patient may have which disorder? a. Anxiety disorder b. Depression c. Schizophrenia d. Bipolar disorder

A Buspirone is indicated for the treatment of anxiety disorders, not depression, schizophrenia, or bipolar disorder.

A 74-year-old professional golfer has chest pain that occurs toward the end of his golfing games. He says the pain usually goes away after one or two sublingual nitroglycerin tablets and rest. What type of angina is he experiencing? a. Classic b. Variant c. Unstable d. Prinzmetal's

A Classic, or chronic stable, angina is triggered by either exertion or stress and usually subsides within 15 minutes with either rest or drug therapy.

A patient arrives in the emergency department with severe chest pain. The patient reports that the pain has been occurring off and on for a week now. Which assessment finding would indicate the need for cautious use of nitrates and nitrites? a. Blood pressure of 88/62 mm Hg b. Apical pulse rate of 110 beats/min c. History of renal disease d. History of a myocardial infarction 2 years ago

A Hypotension is a possible contraindication to the use of nitrates because the medications may cause the blood pressure to decrease. The other options are incorrect.

A patient is recovering from a minor automobile accident that occurred 1 week ago. He is taking cyclobenzaprine (Flexeril) for muscular pain and goes to physical therapy three times a week. Which nursing diagnosis would be appropriate for him? a. Risk for injury related to decreased sensorium b. Risk for addiction related to psychologic dependency c. Decreased fluid volume related to potential adverse effects d. Disturbed sleep pattern related to the drug's interference with REM sleep

A Musculoskeletal relaxants have a depressant effect on the CNS; thus, the patient needs to be taught the importance of taking measures to minimize self-injury and falls related to decreased sensorium. "Risk for addiction" is not a NANDA nursing diagnosis. The other nursing diagnoses are not appropriate for this situation.

A patient is taking flurazepam (Dalmane) three to four nights a week for sleeplessness. She is concerned that she cannot get to sleep without taking the medication. What nonpharmacologic measures should the nurse suggest to promote sleep for this patient? a. Providing a quiet environment b. Exercising before bedtime to become tired c. Consuming heavy meals in the evening to promote sleepiness d. Drinking hot tea or coffee just before bedtime

A Nonpharmacologic approaches to induce sleep include providing a quiet environment, avoiding heavy exercise before bedtime, avoiding heavy meals late in the evening, and drinking warm decaffeinated drinks, such as warm milk, before bedtime.

When administering digoxin immune Fab (Digibind) to a patient with severe digoxin toxicity, the nurse knows that each vial can bind with how much digoxin? a. 0.5 mg b. 5 mg c. 5.5 mg d. 15 mg

A One vial of digoxin immune Fab binds 0.5 mg of digoxin. The other options are incorrect.

A 21-year-old male athlete admits to using androgenic steroids. The nurse tells him that which of these is a possible adverse effect of these drugs? a. Liver damage b. Renal failure c. Heart failure d. Stevens-Johnson syndrome

A Peliosis of the liver, the formation of blood-filled cavities, is a potential effect of androgenic anabolic steroid therapy and may be life threatening. Other serious hepatic effects are hepatic neoplasms (liver cancer), cholestatic hepatitis, jaundice, and abnormal liver function. The other options are incorrect.

A patient is taking digoxin (Lanoxin) and a loop diuretic daily. When the nurse enters the room with the morning medications, the patient states, "I am seeing a funny yellow color around the lights." What is the nurse's next action? a. Assess the patient for symptoms of digoxin toxicity. b. Withhold the next dose of the diuretic. c. Administer the digoxin and diuretic together as ordered. d. Document this finding, and reassess in 1 hour.

A Seeing colors around lights is one potential indication of developing digoxin toxicity. If a patient complains of this, the nurse needs to assess for other signs and symptoms of digoxin toxicity including bradycardia, headache, dizziness, confusion, nausea, and blurred vision, and then notify the prescriber. Administering the drug or withholding the diuretic are incorrect options.

When counseling a male patient about the possible adverse effects of antihypertensive drugs, the nurse will discuss which potential problem? a. Impotence b. Bradycardia c. Increased libido d. Weight gain

A Sexual dysfunction is a common complication of antihypertensive medications and may be manifested in men as decreased libido or impotence. The other options are incorrect.

A patient's blood pressure elevates to 270/150 mm Hg, and a hypertensive emergency is obvious. He is transferred to the intensive care unit and started on a sodium nitroprusside (Nipride) drip to be titrated per his response. With this medication, the nurse knows that the maximum dose of this drug should be infused for how long? a. 10 minutes b. 30 minutes c. 1 hour d. 24 hours

A Sodium nitroprusside is a potent vasodilator and may lead to extreme decreases in the patient's blood pressure. For this reason, it is never infused at the maximum dose for more than 10 minutes. If this drug does not control a patient's blood pressure after 10 minutes, it will most likely be ordered to be discontinued. The other times listed are incorrect.

An immunologist is discussing endotoxin production. Which information should the immunologist include? Endotoxins are produced by: a. gram-negative bacteria. b. gram-positive bacteria. c. gram-negative fungi. d. gram-positive fungi.

A gram-negative bacteria.

Which drug classes are considered first-line treatment for heart failure? (Select all that apply.) a. Angiotensin-converting enzyme (ACE) inhibitors b. Angiotensin II receptor blockers (ARBs) c. Digoxin (cardiac glycoside) d. Beta blockers e. Nesiritide (Natrecor), the B-type natriuretic peptide

A, B, D ACE inhibitors, ARBs, and beta blockers are now considered the first-line treatments for heart failure. Digoxin is used when the first-line treatments are not successful; nesiritide is considered a last-resort treatment.

The barbiturate phenobarbital is prescribed for a patient with epilepsy. While assessing the patient's current medications, the nurse recognizes that interactions may occur with which drugs? (Select all that apply.) a. Antihistamines b. Opioids c. Diuretics d. Anticoagulants e. Oral contraceptives f. Insulin

A, B, D, E The co-administration of barbiturates and alcohol, antihistamines, benzodiazepines, opioids, and tranquilizers may result in additive CNS depression. Co-administration of anticoagulants and barbiturates can result in decreased anticoagulation response and possible clot formation. Coadministration of barbiturates and oral contraceptives can result in accelerated metabolism of the contraceptive drug and possible unintended pregnancy. There are no interactions with diuretics and insulin

Which nursing actions for the care of a dying patient can the nurse delegate to a licensed practical/vocational nurse (LPN/LVN) (select all that apply)? a. Provide postmortem care to the patient. b. Encourage the family members to talk with and reassure the patient. c. Determine how frequently physical assessments are needed for the patient. d. Teach family members about commonly occurring signs of approaching death. e. Administer the prescribed morphine sulfate sublingual as necessary for pain control.

A, B, E a. Provide postmortem care to the patient. b. Encourage the family members to talk with and reassure the patient. e. Administer the prescribed morphine sulfate sublingual as necessary for pain control. Medication administration, psychosocial care, and postmortem care are included in LPN/LVN education and scope of practice. Patient and family teaching and assessment and planning of frequency for assessments are skills that require registered nurse level education and scope of practice.

The nurse is preparing to administer dexmedetomidine (Precedex) to a patient. Which is an appropriate indication for dexmedetomidine? (Select all that apply.) a. Procedural sedation b. Surgeries of short duration c. Surgeries of long duration d. Postoperative anxiety e. Sedation of mechanically ventilated patients

A, B, E Dexmedetomidine (Precedex) is used for procedural sedation and for surgeries of short duration, and it is also used in the intensive care setting for sedation of mechanically ventilated patients. The other options are incorrect.

A patient who has been taking a selective serotonin reuptake inhibitor (SSRI) is complaining of "feeling so badly" when he started taking an over-the-counter St. John's wort herbal product at home. The nurse suspects that he is experiencing serotonin syndrome. Which of these are symptoms of serotonin syndrome? (Select all that apply.) a. Agitation b. Drowsiness c. Tremors d. Bradycardia e. Sweating f. Constipation

A, C, E Common symptoms of serotonin syndrome include delirium, agitation, tachycardia, sweating, hyperreflexia, shivering, coarse tremors, and others. See Box 16-1 for a full list of symptoms.

The nurse is instructing a male patient about application of transdermal testosterone gel (AndroGel). Which body location is preferred for this medication? (Select all that apply.) a. Back b. Chest c. Thigh d. Scrotum e. Abdomen f. Upper arms

A, C, E, F AndroGel is applied to the skin of the back, abdomen, upper arms, or thighs. Testoderm patches are applied to the scrotal skin.

The nurse is providing education about the use of sublingual nitroglycerin tablets. She asks the patient, "What would you do if you experienced chest pain while mowing your yard? You have your bottle of sublingual nitroglycerin with you." Which actions by the patient are appropriate in this situation? (Select all that apply.) a. Stop the activity, and lie down or sit down. b. Call 911 immediately. c. Call 911 if the pain is not relieved after taking one sublingual tablet. d. Call 911 if the pain is not relieved after taking three sublingual tablets in 15 minutes. e. Place a tablet under the tongue. f. Place a tablet in the space between the gum and cheek. g. Take another sublingual tablet if chest pain is not relieved after 5 minutes, up to three total

A, C, E, G With sublingual forms, the medication is taken at the first sign of chest pain, not delayed until the pain is severe. The patient needs to sit down or lie down and take one sublingual tablet. According to current guidelines, if the chest pain or discomfort is not relieved in 5 minutes, after 1 dose, the patient (or family member) must call 911 immediately. The patient can take one more tablet while awaiting emergency care and may take a third tablet 5 minutes later, but no more than a total of three tablets. These guidelines reflect the fact that angina pain that does not respond to nitroglycerin may indicate a myocardial infarction. The sublingual dose is placed under the tongue, and the patient needs to avoid swallowing until the tablet has dissolved. Placing a tablet between the gum and cheek is the buccal route.

When teaching a patient about antihypertensive drug therapy, which statements by the nurse are correct? (Select all that apply.) a. "You need to have your blood pressure checked once a week and keep track of the readings." b. "If you notice that the symptoms have gone away, you should be able to stop taking the drug." c. "An exercise program may be helpful in treating hypertension, but let's check with your doctor first." d. "If you experience severe side effects, stop the medicine and let us know at your next office visit." e. "Most over-the-counter decongestants are compatible with antihypertensive drugs." f. "Please continue taking the medication, even if you are feeling better."

A, C, F Keeping a record of weekly blood pressure checks helps to monitor the effectiveness of the therapy. Remind the patient not to stop taking the medication just because he or she is feeling better. Abruptly stopping the medication may lead to rebound hypertension. Therapy is often lifelong, even though symptoms may improve. Many over-the-counter drugs, especially decongestants, have serious interactions with antihypertensive drugs. The patient needs to consult his or her prescriber before taking any other medication.

The nurse notes in a patient's medication history that the patient is taking the synthetic androgen danazol (Danocrine). Indications for danazol include which conditions? (Select all that apply.) a. Endometriosis b. Decreased sexual libido c. Postpartum breast engorgement d. Fibrocystic breast disease in women e. Hereditary angioedema f. Metastatic breast cancer

A, D, E Danazol is used to treat hereditary angioedema and to treat women who have endometriosis or fibrocystic breast disease. The other options are incorrect.

A patient is the primary caregiver for a disabled family member at home, and has now been unexpectedly hospitalized for surgery. What action can the nurse take to enhance the coping ability of the patient? A. Ask if there is another family member who can help at home while the patient is in the hospital. B. Plan to transfer the patient to a rehabilitation unit after surgery to allow uninterrupted time to recover. C. Coordinate an ambulance transfer of the family member to an alternate family member's home. D. Ask social services to assess what the patient's needs will be after discharge to home.

A. Ask if there is another family member who can help at home while the patient is in the hospital.

An 80-year-old male patient is in the intensive care unit has suffered a fractured femur. You are making rounds and notice he is somnolent, with no response to verbal or physical stimulation. He has been on round the clock opioid doses q 4 hours. What is the nurse's first action? A. Call the rapid response team to care for the patient immediately. B. Start a second intravenous line with a large bore catheter. C. Discontinue the opioids on the medication administration record. D. Assess the patient's blood pressure and pain level.

A. Call the rapid response team to care for the patient immediately.

The nurse is assessing a patient for risk factors of chronic fatigue syndrome. Which factors should the nurse identify as placing the patient at risk for chronic fatigue syndrome? (Select all that apply.) A. Feeling tired upon awakening B. Chronic migraines C. Tenderness under the jaw D. 5 episodes tonsillitis/past year E. Swollen, painful knees

A. Feeling tired upon awakening C. Tenderness under the jaw D. 5 episodes tonsillitis/past year E. Swollen, painful knees

A patient in the intensive care unit who has acute decompensated heart failure (ADHF) reports severe dyspnea and is anxious, tachypneic, and tachycardic. Several drugs have been prescribed for the patient. Which action should the nurse take first? a. Give PRN IV morphine sulfate 4 mg. b. Give PRN IV diazepam (Valium) 2.5 mg. c. Increase nitroglycerin infusion by 5 mcg/min. d. Increase dopamine infusion by 2 mcg/kg/min.

A. Give PRN IV morphine sulfate 4 mg.

The nurse is reviewing the care plan for a patient experiencing difficulty coping with stress. Which action should the nurse implement to assist the patient? A. Identifying the cause of fear B. Accessing a community support group C. Identifying relaxation methods D. Reviewing an educational pamphlet

A. Identifying the cause of fear

The nurse is caring for a child with tonsillar enlargement. What is the nurse's priority concern? A. Low oxygen saturation B. Daytime fatigue C. Increased temperature D. Antibiotic administration

A. Low oxygen saturation

Methotrexate is ordered for a patient with a malignant tumor, and the nurse is providing education about self-care after the chemotherapy is given. Which statements by the nurse are appropriate for the patient receiving methotrexate? (Select all that apply.) a. Report unusual bleeding or bruising. b. Hair loss is not expected with this drug. c. Prepare for hair loss. d. Avoid areas with large crowds or gatherings. e. Avoid foods that are too hot or too cold or rough in texture. f. Restrict fluid intake to reduce nausea and vomiting.

A.C.D.E (Counsel patients who are taking methotrexate to expect hair loss and to report any unusual bleeding or bruising. Because of the possibility of infection, avoid areas with large crowds or gatherings. Foods that are too hot or too cold or rough in texture may be irritating to the oral mucosa. Fluid intake is to be encouraged to prevent dehydration.)

1. A patient is to receive medroxyprogesterone (Depo-Provera) 700 mg weekly, intramuscularly, as part of palliative therapy for endometrial cancer. The medication is available in vials of 400 mg/mL. Identify how many milliliters will the nurse draw up and administer with each injection. (record answer using one decimal place) _______

ANS: 1.8 mL DIF: COGNITIVE LEVEL: Applying (Application) REF: N/A TOP: NURSING PROCESS: Implementation MSC: NCLEX: Physiological Integrity: Pharmacological and Parenteral Therapies

When giving chemotherapy as cancer treatment, the nurse recognizes that toxicity to rapidly growing normal cells also occurs. Which rapidly growing normal cells are also harmed by chemotherapy? (Select all that apply.) a. Bone marrow cells b. Retinal cells c. Hair follicle cells d. Nerve myelin cells e. Gastrointestinal (GI) mucous membrane cells

A.C.E (Chemotherapy toxicities generally stem from the fact that chemotherapy drugs affect rapidly dividing cells—both harmful cancer cells and healthy, normal cells. Three types of rapidly dividing human cells are the cells of hair follicles, GI tract cells, and bone marrow cells. The other options are incorrect.)

1. A patient will be receiving monthly injections of cyanocobalamin (Nascobal). The dose is 100 mcg/month IM. The medication is available in a strength of 1000 mcg/mL. How many milliliters will the nurse draw up into the syringe? (Record answer using one decimal place.)

ANS: 0.1 mL 1000 mcg : 1 mL :: 100 mcg : x mL (1000 ´ x) = (1 ´ 100); 1000x = 100; x = 0.1 mL

Incidence of Legg-Calvé-Perthes (LCP) disease peaks at age______.

ANS: 6 LCP disease is a common osteochondrosis usually occurring in children between the ages of 3 and 10 years, with a peak incidence at 6 years. REF: p. 1044

1. A patient in the oliguric phase after an acute kidney injury has had a 250-mL urine output and an emesis of 100 mL in the past 24 hours. What is the patient's fluid restriction for the next 24 hours?

ANS: 950 mL The general rule for calculating fluid restrictions is to add all fluid losses for the previous 24 hours, plus 600 mL for insensible losses: (250 + 100 + 600 = 950 mL). DIF: Cognitive Level: Understand (comprehension) REF: 1073 TOP: Nursing Process: Implementation MSC: NCLEX: Physiological Integrity

In which order will the nurse take these actions when caring for a patient in the emergency department with a right leg fracture after a motor vehicle accident? (Put a comma and a space between each answer choice [A, B, C, D, E, F].) a. Obtain x-rays. b. Check pedal pulses. c. Assess lung sounds. d. Take blood pressure. e. Apply splint to the leg. f. Administer tetanus prophylaxis.

ANS: C, D, B, E, A, F The initial actions should be to ensure that airway, breathing, and circulation are intact. This should be followed by checking the neurovascular status of the leg (before and after splint application). Application of a splint to immobilize the leg should be done before sending the patient for x-rays. The tetanus prophylaxis is the least urgent of the actions. DIF: Cognitive Level: Analyze (analysis) REF: 1518 OBJ: Special Questions: Prioritization TOP: Nursing Process: Implementation MSC: NCLEX: Physiological Integrity

A patient will be receiving epoetin alfa (Epogen) 8000 units IV three times a week. The medication is available in a vial that contains 10,000 units/mL. How many milliliters will the nurse draw up for this dose? _______

ANS: 0.8 mL

Which action will the nurse include in the plan of care for a 72-year-old woman admitted with multiple myeloma? a. Monitor fluid intake and output. b. Administer calcium supplements. c. Assess lymph nodes for enlargement. d. Limit weight bearing and ambulation.

ANS: A A high fluid intake and urine output helps prevent the complications of kidney stones caused by hypercalcemia and renal failure caused by deposition of Bence-Jones protein in the renal tubules. Weight bearing and ambulation are encouraged to help bone retain calcium. Lymph nodes are not enlarged with multiple myeloma. Calcium supplements will further increase the patients calcium level and are not used.

An appropriate nursing intervention for a patient with non-Hodgkins lymphoma whose platelet count drops to 18,000/L during chemotherapy is to a. check all stools for occult blood. b. encourage fluids to 3000 mL/day. c. provide oral hygiene every 2 hours. d. check the temperature every 4 hours.

ANS: A Because the patient is at risk for spontaneous bleeding, the nurse should check stools for occult blood. A low platelet count does not require an increased fluid intake. Oral hygiene is important, but it is not necessary to provide oral care every 2 hours. The low platelet count does not increase risk for infection, so frequent temperature monitoring is not indicated.

9. A patient will be starting vitamin D supplements. The nurse reviews his medical record for contraindications, including which condition? a. Renal disease b. Cardiac disease c. Hypophosphatemia d. There are no contraindications to vitamin D supplements.

ANS: A Contraindications to vitamin D products include known allergy to the product, hypercalcemia, renal dysfunction, and hyperphosphatemia.

Which menu choice indicates that the patient understands the nurses teaching about best dietary choices for iron-deficiency anemia? a. Omelet and whole wheat toast b. Cantaloupe and cottage cheese c. Strawberry and banana fruit plate d. Cornmeal muffin and orange juice

ANS: A Eggs and whole grain breads are high in iron. The other choices are appropriate for other nutritional deficiencies but are not the best choice for a patient with iron-deficiency anemia.

Which information obtained by the nurse assessing a patient admitted with multiple myeloma is most important to report to the health care provider? a. Serum calcium level is 15 mg/dL. b. Patient reports no stool for 5 days. c. Urine sample has Bence-Jones protein. d. Patient is complaining of severe back pain.

ANS: A Hypercalcemia may lead to complications such as dysrhythmias or seizures, and should be addressed quickly. The other patient findings will also be discussed with the health care provider, but are not life threatening.

1. The nurse is reviewing conditions caused by nutrient deficiencies. Conditions such as infantile rickets, tetany, and osteomalacia are caused by a deficiency in which vitamin or mineral? a. Vitamin D b. Vitamin C c. Zinc d. Cyanocobalamin (vitamin B12)

ANS: A Infantile rickets, tetany, and osteomalacia are all a result of long-term vitamin D deficiency. The other options are incorrect.

Which patient statement to the nurse indicates a need for additional instruction about taking oral ferrous sulfate? a. I will call my health care provider if my stools turn black. b. I will take a stool softener if I feel constipated occasionally. c. I should take the iron with orange juice about an hour before eating. d. I should increase my fluid and fiber intake while I am taking iron tablets.

ANS: A It is normal for the stools to appear black when a patient is taking iron, and the patient should not call the doctor about this. The other patient statements are correct.

Which action will the nurse take when caring for a patient with osteomalacia? a. Teach about the use of vitamin D supplements. b. Educate about the need for weight-bearing exercise. c. Discuss the use of medications such as bisphosphonates. d. Emphasize the importance of sunscreen use when outside.

ANS: A Osteomalacia is caused by inadequate intake or absorption of vitamin D. Weight-bearing exercise and bisphosphonate administration may be used for osteoporosis but will not be beneficial for osteomalacia. Because ultraviolet light is needed for the body to synthesize vitamin D, the patient might be taught that 20 minutes/day of sun exposure is beneficial.

Which patient should the nurse assign as the roommate for a patient who has aplastic anemia? a. A patient with chronic heart failure b. A patient who has viral pneumonia c. A patient who has right leg cellulitis d. A patient with multiple abdominal drains

ANS: A Patients with aplastic anemia are at risk for infection because of the low white blood cell production associated with this type of anemia, so the nurse should avoid assigning a roommate with any possible infectious process.

A 19-year-old woman with immune thrombocytopenic purpura (ITP) has an order for a platelet transfusion. Which information indicates that the nurse should consult with the health care provider before obtaining and administering platelets? a. The platelet count is 42,000/mL. b. Petechiae are present on the chest. c. Blood pressure (BP) is 94/56 mm Hg. d. Blood is oozing from the venipuncture site.

ANS: A Platelet transfusions are not usually indicated until the platelet count is below 10,000 to 20,000/mL unless the patient is actively bleeding. Therefore the nurse should clarify the order with the health care provider before giving the transfusion. The other data all indicate that bleeding caused by ITP may be occurring and that the platelet transfusion is appropriate.

The nurse caring for a patient with type A hemophilia being admitted to the hospital with severe pain and swelling in the right knee will a. immobilize the joint. b. apply heat to the knee. c. assist the patient with light weight bearing. d. perform passive range of motion to the knee.

ANS: A The initial action should be total rest of the knee to minimize bleeding. Ice packs are used to decrease bleeding. Range of motion (ROM) and weight-bearing exercise are contraindicated initially, but after the bleeding stops, ROM and physical therapy are started.

The nurse has obtained the health history, physical assessment data, and laboratory results shown in the accompanying figure for a patient admitted with aplastic anemia. Which information is most important to communicate to the health care provider? a. Neutropenia b. Increasing fatigue c. Thrombocytopenia d. Frequent constipation

ANS: A The low white blood cell count indicates that the patient is at high risk for infection and needs immediate actions to diagnose and treat the cause of the leucopenia. The other information may require further assessment or treatment, but does not place the patient at immediate risk for complications.

14. Which information will the nurse include in patient teaching for a 36-year-old patient who is scheduled for stereotactic core biopsy of the breast? a. A local anesthetic will be given before the biopsy specimen is obtained. b. You will need to lie flat on your back and lie very still during the biopsy. c. A thin needle will be inserted into the lump and aspirated to remove tissue. d. You should not have anything to eat or drink for 6 hours before the procedure.

ANS: A A local anesthetic is given before stereotactic biopsy. NPO status is not needed because no sedative drugs are given. The patient is placed in the prone position. A biopsy gun is used to obtain the specimens. DIF: Cognitive Level: Apply (application) TOP: Nursing Process: Implementation MSC: NCLEX: Physiological Integrity

12. A woman who lives in Seattle is preparing to take a plane trip to London. She has been taking the SERM raloxifene (Evista) for 6 months. The nurse will provide which instructions to this patient? a. She needs to stop taking the drug at least 72 hours before the trip. b. She must remember to take this drug with a full glass of water each morning. c. She will not take the drug while traveling on the plane. d. No change in how the drug is taken will be needed.

ANS: A A patient taking a SERM must be informed to discontinue the drug 72 hours before and during prolonged immobility so as to prevent the development of a thrombosis. DIF: COGNITIVE LEVEL: Applying (Application) REF: p. 549 TOP: NURSING PROCESS: Implementation MSC: NCLEX: Physiological Integrity: Reduction of Risk Potential

What term is used to describe a torn ligament? a. Sprain b. Strain c. Disunion d. Subluxation

ANS: A A torn ligament is also called a sprain. Tearing or stretching of a muscle or tendon is commonly known as a strain. Disunion occurs when fracture ends fail to heal. Subluxation occurs when contact between the opposing joint surfaces of a fracture are partially lost.

A 21-year-old is diagnosed with ankylosing spondylitis (AS). The most likely joint to be affected would be the: a. sacroiliac. b. carpal. c. shoulder. d. knee.

ANS: A AS is a chronic, inflammatory joint disease characterized by stiffening and fusion (ankylosis) of the spine and sacroiliac joints. AS is not associated initially with the carpal joints, the shoulder, or the knees.

What term is used to document a herniation of brain and meninges through a defect in the occipital area of the skull? a. Encephalocele b. Meningocele c. Myelomeningocele d. Craniosynostosis

ANS: A An encephalocele refers to a herniation or protrusion of brain and meninges through a defect in the occipital region of the skull, resulting in a saclike structure. A meningocele is a saclike cyst of meninges filled with spinal fluid, a mild form of posterior neural tube closure defect. These can occur throughout the cervical, thoracic, and lumbar spine areas. A myelomeningocele is a hernial protrusion of a saclike cyst (containing meninges, spinal fluid, and a portion of the spinal cord with its nerves) through a defect in the posterior arch of a vertebra in the lower spine. Craniosynostosis is the premature closure of one or more of the cranial sutures (saggital, coronal, lambdoid, and metopic) during the first 18-20 months of the infant's life. REF: p. 425

A female patient is receiving palliative therapy with androgen hormones as part of treatment for inoperable breast cancer. The nurse will discuss with the patient which potential body image changes that may occur as adverse effects? a. Hirsutism and acne b. Weight gain c. Flushing and hot flashes d. Alopecia and body odor A female patient is receiving palliative therapy with androgen hormones as part of treatment for inoperable breast cancer. The nurse will discuss with the patient which potential body image changes that may occur as adverse effects? a. Hirsutism and acne b. Weight gain c. Flushing and hot flashes d. Alopecia and body odor

ANS: A Androgens used for cancer treatment, such as fluoxymesterone and testolactone, can cause menstrual irregularities, virilization of female, gynecomastia, hirsutism, acne, anxiety, headache, and nausea. The patient needs to be told of these effects before therapy begins. The other options are incorrect. DIF: COGNITIVE LEVEL: Applying (Application) REF: p. 738 TOP: NURSING PROCESS: Planning MSC: NCLEX: Physiological Integrity: Pharmacological and Parenteral Therapies

11. A patient will need vascular access for hemodialysis. Which statement by the nurse accurately describes an advantage of a fistula over a graft? a. A fistula is much less likely to clot. b. A fistula increases patient mobility. c. A fistula can accommodate larger needles. d. A fistula can be used sooner after surgery.

ANS: A Arteriovenous (AV) fistulas are much less likely to clot than grafts, although it takes longer for them to mature to the point where they can be used for dialysis. The choice of an AV fistula or a graft does not have an impact on needle size or patient mobility. DIF: Cognitive Level: Understand (comprehension) REF: 1088 TOP: Nursing Process: Implementation MSC: NCLEX: Physiological Integrity

Which patient statement indicates a need for further teaching about extended-release zolpidem (Ambien CR)? a. "I will take the medication 1-2 hours before bedtime." b. "I should take the medication on an empty stomach." c. "I should not take this medication unless I can sleep for at least 6 hours." d. "I will schedule activities that require mental alertness for later in the day."

ANS: A Benzodiazepine receptor agonists such as zolpidem work quickly and should be taken immediately before bedtime. The other patient statements are correct

A patient who has received some traumatic news is panicking and asks for some medication to help settle down. The nurse anticipates giving which drug that is most appropriate for this situation? a. Diazepam (Valium) b. Zolpidem (Ambien) c. Phenobarbital d. Cyclobenzaprine (Flexeril)

ANS: A Benzodiazepines such as diazepam are used as anxiolytics, or sedatives. Zolpidem is used as a hypnotic for sleep. Phenobarbital is not used as an anxiolytic but is used for seizure control. Cyclobenzaprine is a muscle relaxant and is not used to reduce anxiety. DIF: COGNITIVE LEVEL: Applying (Application) REF: p. 191 TOP: NURSING PROCESS: Planning MSC: NCLEX: Physiological Integrity: Pharmacological and Parenteral Therapies

Children with osteogenesis imperfecta (OI) are at high risk for frequent: a. bone fractures. b. shoulder dislocations. c. bone infections. d. joint injuries.

ANS: A Children with OI experience frequent bone fractures. They are not as susceptible for shoulder dislocations, bone infections, or other joint injuries. REF: p. 1039

What should the nurse teach the patient before fluorescein angiography? a. Hold a card and fixate on the center dot. b. Report any burning or pain at the IV site. c. Remain still while the cornea is anesthetized. d. Let the examiner know when images shown appear clear.

ANS: B

A 60-year-old male presents with swelling and pain in the knee. CT reveals a tumor has developed from a pre-existing benign bone lesion. This supports which diagnosis? a. Secondary chondrosarcoma b. Rhabdomyoma c. Rhabdomyosarcoma d. Fibrosarcoma

ANS: A Chondrosarcoma is the second most common primary malignant bone tumor and is a tumor of middle-aged and older adults. Chondrosarcomas that develop from a pre-existing benign bone lesion are known as secondary chondrosarcomas. Rhabdomyoma is an extremely rare benign tumor of muscle that generally occurs in the tongue. The malignant tumor of striated muscle is called rhabdomyosarcoma. A fibrosarcoma is a solitary tumor that most often affects the metaphyseal region of the femur or tibia.

An infant is diagnosed with congenital hydrocephalus. Which of the following characteristics would the nurse expect to find? a. Enlarged ventricles b. Decreased cerebrospinal fluid (CSF) production c. Increased resorption of CSF d. Smaller than average head circumference

ANS: A Congenital hydrocephalus is characterized by enlargement of the cerebral ventricles. Increased, not decreased, CSF production would lead to hydrocephalus. Decreased resorption of CSF would lead to hydrocephalus, not increased CSF. An infant with congenital hydrocephalus would have increased head circumference. REF: p. 428

During assessment of a patient with osteoarthritis pain, the nurse knows that which condition is a contraindication to the use of nonsteroidal anti-inflammatory drugs (NSAIDs)? a. Renal disease b. Diabetes mellitus c. Headaches d. Rheumatoid arthritis

ANS: A Contraindications to NSAIDs include known drug allergy and conditions that place a patient at risk for bleeding, such as vitamin K deficiency, and peptic ulcer disease. Patients with documented aspirin allergy must not receive NSAIDs. Other common contraindications are those that apply to most drugs, including severe renal or hepatic disease. The other options are not contraindications.

What term is used to describe a premature closure of one or more of the cranial sutures during the first 18 months of life? a. Craniosynostosis b. Congential hydrocephalus c. Microcephaly d. Acrania

ANS: A Craniosynostosis is the premature closure of one or more of the cranial sutures. Congenital hydrocephalus is characterized by enlargement of the cerebral ventricles. Microcephaly is lack of brain growth with retarded mental and motor development. In acrania, the cranial vault is almost completely absent; an extensive defect of the vertebral column is often present. REF: p. 426

The nurse notes in the patient's medication history that the patient is taking cyclobenzaprine (Flexeril). Based on this finding, the nurse interprets that the patient has which disorder? a. A musculoskeletal injury b. Insomnia c. Epilepsy d. Agitation

ANS: A Cyclobenzaprine (Flexeril) is the muscle relaxant most commonly used to reduce spasms following musculoskeletal injuries. It is not appropriate for insomnia, epilepsy, or agitation. DIF: COGNITIVE LEVEL: Understanding (Comprehension) REF: p. 197 TOP: NURSING PROCESS: Assessment MSC: NCLEX: Physiological Integrity: Pharmacological and Parenteral Therapies

A patient is receiving doxorubicin (Adriamycin) as part of treatment for ovarian cancer. Which nursing diagnosis is related to this antineoplastic drug? a. Decreased cardiac output related to the adverse effect of cardiotoxicity b. Ineffective breathing pattern related to the adverse effect of pulmonary toxicity c. Risk for injury related to the effects of neurotoxicity (ataxia, numbness of hands and feet) d. Impaired urinary elimination pattern related to hyperuricemia

ANS: A Decreased cardiac output related to the adverse effect of cardiotoxicity is a nursing diagnosis related to doxorubicin because adverse effects of doxorubicin include liver and cardiovascular toxicities. The other options are incorrect. DIF: COGNITIVE LEVEL: Applying (Application) REF: p. 742 TOP: NURSING PROCESS: Nursing Diagnosis MSC: NCLEX: Physiological Integrity: Physiological Adaptation

Which type of scoliosis accounts for the majority of the cases of scoliosis? a. Idiopathic b. Infectious c. Iatrogenic d. Secondary

ANS: A Eighty percent of all scoliosis is idiopathic. REF: p. 1046

A 30-year-old male with HIV is diagnosed with Epstein-Barr virus. After 2 months, the virus is still active. Based upon the Epstein-Barr virus, which of the following cancers is most likely to develop in this patient? a. B-cell lymphoma b. Kaposi sarcoma c. T-cell leukemia d. T-cell lipoma

ANS: A Epstein-Barr virus is associated with B-cell lymphoma. Kaposi sarcoma is associated with HIV. Retroviruses are associated with leukemia. Lipomas are not associated with HIV.

3. A 51-year-old patient with a small immobile breast lump is considering having a fine-needle aspiration (FNA) biopsy. The nurse explains that an advantage to this procedure is that a. FNA is done in the outpatient clinic and results are available in 1 to 2 days. b. only a small incision is needed, resulting in minimal breast pain and scarring. c. if the biopsy results are negative, no further diagnostic testing will be needed. d. FNA is guided by a mammogram, ensuring that cells are taken from the lesion.

ANS: A FNA is done in outpatient settings and results are available in 24 to 48 hours. No incision is needed. FNA may be guided by ultrasound, but not by mammogram. Because the immobility of the breast lump suggests cancer, further testing will be done if the FNA is negative. DIF: Cognitive Level: Apply (application) TOP: Nursing Process: Implementation MSC: NCLEX: Physiological Integrity

When should the nurse assess for the vomiting and headache that are the classic symptoms of childhood brain tumors? a. Morning b. Early afternoon c. As the sun goes down and darkness begins d. During the middle of the night

ANS: A Headache and vomiting occur more commonly in the morning. REF: p. 434

4. The nurse is teaching a patient about the adverse effects of fertility drugs such as clomiphene (Clomid). Which is a potential adverse effect of this drug? a. Headache b. Drowsiness c. Dysmenorrhea d. Hypertension

ANS: A Headache is one of the possible adverse effects of the fertility drugs. They may also cause vomiting, restlessness, and urticaria. Drowsiness, dysmenorrhea, and hypertension are not potential adverse effects. See Table 34-5 for other adverse effects. DIF: COGNITIVE LEVEL: Understanding (Comprehension) REF: p. 544 TOP: NURSING PROCESS: Assessment MSC: NCLEX: Physiological Integrity: Pharmacological and Parenteral Therapies

The most prominent goal of palliative care is to a. integrate into chronic disease management sooner rather than later. b. enroll the patient into the Medicare Hospice Benefit. c. ensure that the patient has a 6-month prognosis. d. reserve this type of care until the patient is actively dying.

ANS: A The goal of palliative care is to integrate symptom management interventions earlier into the course of chronic disease sooner rather than later. This helps to promote optimal quality of life.

7. The nurse teaches a patient who is experiencing stress at work how to use imagery as a relaxation technique. Which statement by the nurse would be appropriate? a. "Think of a place where you feel peaceful and comfortable." b. "Place the stress in your life into an image that you can destroy." c. "Repeatedly visualize yourself experiencing the distress in your workplace." d. "Bring what you hear and sense in your work environment into your image."

ANS: A Imagery is the use of one's mind to generate images that have a calming effect on the body. When using imagery for relaxation, the patient should visualize a comfortable and peaceful place. The goal is to offer a relaxing retreat from the actual work environment. Imagery that is not intended for relaxation purposes can target a disease, problem, or stressor. DIF: Cognitive Level: Apply (application) REF: 84 TOP: Nursing Process: Implementation MSC: NCLEX: Psychosocial Integrity

Which of the following indicates a nurse understands a proto-oncogene? A proto-oncogene is best defined as a(n) _____ gene. a. normal b. altered c. inactive d. tumor-suppressor

ANS: A In its normal, nonmutant state, an oncogene is referred to as a proto-oncogene. A proto-oncogene is not an altered gene, an inactive gene, or a tumor-suppressor gene.

A 34-year-old female was recently diagnosed with rheumatoid arthritis (RA). Physical examination revealed that inflammation is noted in various: a. synovial membranes. b. short bones. c. subchondral bones. d. bursae sacs.

ANS: A Inflammation of RA starts in the synovial membrane. The inflammation is not associated with short bones, subchondral bones, or bursae sacs.

A 52-year-old male with hepatitis C recently developed hepatic cancer. Which of the following markers should be increased? a. Alpha-fetoprotein (AFP) b. Catecholamines c. Prostate-specific antigen d. Homovanillic acid

ANS: A Liver and germ cell tumors secrete a protein known as AFP, not catecholamines. Prostate tumors secrete prostate-specific antigen. Homovanillic acid is a catecholamine marker.

One of the biggest challenges facing current nursing practice is a. the number of aging Americans living with chronic disease. b. the number of patients entering into hospice programs. c. the number of cancer patients receiving supportive care. d. reduced length of stay in hospice care.

ANS: A Millions of Americans are living with one or more chronic debilitating diseases, and 7 out of 10 can expect to live with their diseases several years before dying. When coupled with the advancing age of the eight million baby boomers who now qualify for Medicare, this will soon create a huge demand on health care resources and community-based services.

What event occurs in about 70% of the cases of childhood cancers? a. Cured b. Required only chemotherapy c. Participated in clinical trails d. Developed a secondary malignancy

ANS: A More than 70% of children diagnosed with cancer are cured. Some of the factors leading to improved cure rates in pediatric oncology include the use of combination chemotherapy or multimodal treatment for solid childhood tumors and improvements in nursing and supportive care. A partial explanation for the relative lack of progress in curing the adolescent population at the same rate as that realized in the younger pediatric population is the lack of participation in clinical trials. While survivors of childhood cancer are at increased risk of developing a second malignancy later in life, it is not as frequent as 70%. REF: p. 305

A patient is recovering from a minor automobile accident that occurred 1 week ago. He is taking cyclobenzaprine (Flexeril) for muscular pain and goes to physical therapy three times a week. Which nursing diagnosis would be appropriate for him? a. Risk for injury related to decreased sensorium b. Risk for addiction related to psychologic dependency c. Decreased fluid volume related to potential adverse effects d. Disturbed sleep pattern related to the drug's interference with REM sleep

ANS: A Musculoskeletal relaxants have a depressant effect on the CNS; thus, the patient needs to be taught the importance of taking measures to minimize self-injury and falls related to decreased sensorium. "Risk for addiction" is not a NANDA nursing diagnosis. The other nursing diagnoses are not appropriate for this situation. DIF: COGNITIVE LEVEL: Applying (Application) REF: p. 198 TOP: NURSING PROCESS: Nursing Diagnosis MSC: NCLEX: Physiological Integrity: Reduction of Risk Potential

A patient is taking flurazepam (Dalmane) three to four nights a week for sleeplessness. She is concerned that she cannot get to sleep without taking the medication. What nonpharmacologic measures should the nurse suggest to promote sleep for this patient? a. Providing a quiet environment b. Exercising before bedtime to become tired c. Consuming heavy meals in the evening to promote sleepiness d. Drinking hot tea or coffee just before bedtime

ANS: A Nonpharmacologic approaches to induce sleep include providing a quiet environment, avoiding heavy exercise before bedtime, avoiding heavy meals late in the evening, and drinking warm decaffeinated drinks, such as warm milk, before bedtime. DIF: COGNITIVE LEVEL: Applying (Application) REF: p. 201 TOP: NURSING PROCESS: Implementation MSC: NCLEX: Physiological Integrity: Basic Care and Comfort

16. A patient wants to try an oral soy product to relieve perimenopausal symptoms. The nurse will assess the patient's medication history for which potential drug interaction? a. Thyroid replacement therapy b. Oral anticoagulant therapy c. Nonsteroidal anti-inflammatory drugs d. Beta blockers

ANS: A Orally administered soy may interfere with thyroid hormone absorption, so concurrent use must be avoided. The other options are incorrect. DIF: COGNITIVE LEVEL: Applying (Application) REF: p. 536 TOP: NURSING PROCESS: Assessment MSC: NCLEX: Physiological Integrity: Pharmacological and Parenteral Therapies

The nurse and a student nurse are discussing the effects of bed immobility on patients. The nurse knows that the student nurse understands the concept of mobility when she states, "Patients with impaired bed mobility a. have an increased risk for pressure ulcers." b. like to have extra visitors." c. need to have a mechanical soft diet." d. are prone to constipation."

ANS: A Patients who cannot move themselves in bed are more susceptible to pressure ulcers because they cannot relieve the pressure they feel. Extra visitors or diet consistency do not have any bearing on mobility. Constipation should not be a by-product of immobility if a bowel regimen is instituted.

Which of the following people is at highest risk for the development of gout? a. Men aged 40-50 years b. Premenopausal women c. Male adolescents d. Female children

ANS: A People at highest risk for gout are men aged 40-50 years of age. Gout is rare in children, adolescent males, and premenopausal women.

15. A patient with a right lower leg fracture will be discharged home with an external fixation device in place. Which information will the nurse teach? a. "You will need to check and clean the pin insertion sites daily." b. "The external fixator can be removed for your bath or shower." c. "You will need to remain on bed rest until bone healing is complete." d. "Prophylactic antibiotics are used until the external fixator is removed."

ANS: A Pin insertion sites should be cleaned daily to decrease the risk for infection at the site. An external fixator allows the patient to be out of bed and avoid the risks of prolonged immobility. The device is surgically placed and is not removed until the bone is stable. Prophylactic antibiotics are not routinely given when an external fixator is used. DIF: Cognitive Level: Apply (application) REF: 1516 TOP: Nursing Process: Implementation MSC: NCLEX: Physiological Integrity

3. The occupational health nurse will teach the patient whose job involves many hours of typing about the need to a. obtain a keyboard pad to support the wrist. b. do stretching exercises before starting work. c. wrap the wrists with compression bandages every morning. d. avoid using nonsteroidal antiinflammatory drugs (NSAIDs) for pain.

ANS: A Repetitive strain injuries caused by prolonged times working at a keyboard can be prevented by the use of a pad that will keep the wrists in a straight position. Stretching exercises during the day may be helpful, but these would not be needed before starting. Use of a compression bandage is not needed, although a splint may be used for carpal tunnel syndrome. NSAIDs are appropriate to use to decrease swelling. DIF: Cognitive Level: Apply (application) REF: 1509 TOP: Nursing Process: Implementation MSC: NCLEX: Health Promotion and Maintenance

30. Which nursing action for a patient who has had right hip replacement surgery can the nurse delegate to experienced unlicensed assistive personnel (UAP)? a. Reposition the patient every 1 to 2 hours. b. Assess for skin irritation on the patient's back. c. Teach the patient quadriceps-setting exercises. d. Determine the patient's pain level and tolerance.

ANS: A Repositioning of orthopedic patients is within the scope of practice of UAP (after they have been trained and evaluated in this skill). The other actions should be done by licensed nursing staff members. DIF: Cognitive Level: Apply (application) REF: 1514 OBJ: Special Questions: Delegation TOP: Nursing Process: Planning MSC: NCLEX: Safe and Effective Care Environment

13. The nurse recognizes that the risk of osteoporosis is higher in an individual with which risk factor? a. White or Asian race b. African-American race c. History of participation in active sports d. Obesity

ANS: A Risk factors for postmenopausal osteoporosis include white or Asian descent, slender body build, early estrogen deficiency, smoking, alcohol consumption, low-calcium diet, sedentary lifestyle, and family history of osteoporosis. DIF: COGNITIVE LEVEL: Remembering (Knowledge) REF: p. 541 TOP: NURSING PROCESS: Assessment MSC: NCLEX: Physiological Integrity: Reduction of Risk Potential

7. Sodium polystyrene sulfonate (Kayexalate) is ordered for a patient with hyperkalemia. Before administering the medication, the nurse should assess the a. bowel sounds. b. blood glucose. c. blood urea nitrogen (BUN). d. level of consciousness (LOC).

ANS: A Sodium polystyrene sulfonate (Kayexalate) should not be given to a patient with a paralytic ileus (as indicated by absent bowel sounds) because bowel necrosis can occur. The BUN and creatinine, blood glucose, and LOC would not affect the nurse's decision to give the medication. DIF: Cognitive Level: Apply (application) REF: 1080 TOP: Nursing Process: Assessment MSC: NCLEX: Physiological Integrity

A patient has a tissue growth that was diagnosed as cancer. Which of the following terms best describes this growth? a. Malignant tumor b. Lipoma c. Meningioma d. Hypertrophy

ANS: A Some tumors initially described as benign can progress to cancer and then are referred to as malignant tumors. Lipomas are benign growths, while a meningioma is a benign tumor. Hypertrophy refers to tissue overgrowth, but not cancer. REF: p. 234

11. A patient who is taking antiretroviral medication to control human immunodeficiency virus (HIV) infection tells the nurse about feeling mildly depressed and anxious. Which additional information about the patient is most important to communicate to the health care provider? a. The patient takes vitamin supplements and St. John's wort. b. The patient recently experienced the death of a close friend. c. The patient's blood pressure has increased to 152/88 mm Hg. d. The patient expresses anxiety about whether the drugs are effective.

ANS: A St. John's wort interferes with metabolism of medications that use the cytochrome P450 enzyme system, including many HIV medications. The health care provider will need to check for toxicity caused by the drug interactions. Teaching is needed about drug interactions. The other information will also be reported but does not have immediate serious implications for the patient's health. DIF: Cognitive Level: Analyze (analysis) REF: 80 TOP: Nursing Process: Assessment MSC: NCLEX: Safe and Effective Care Environment

What is the primary organism responsible for osteomyelitis? a. Staphylococcus aureus b. Salmonella c. Mycobacterium d. Haemophilus influenza

ANS: A Staphylococcus aureus remains the primary microorganism responsible for osteomyelitis. While the other options are possible causes, they are not as predominant as is Staphylococcus aureus.

What teaching should be included in the plan of care for a patient with narcolepsy? a. Driving an automobile may be possible with appropriate treatment of narcolepsy. b. Changes in sleep hygiene are ineffective in improving sleep quality in narcolepsy. c. Antidepressant drugs are prescribed to treat the depression caused by the disorder. d. Stimulant drugs should be used for only a short time because of the risk for abuse.

ANS: A The accident rate for patients with narcolepsy who are receiving appropriate treatment is similar to the general population. Stimulant medications are used on an ongoing basis for patients with narcolepsy. The purpose of antidepressant drugs in the treatment of narcolepsy is the management of cataplexy, not to treat depression. Changes in sleep hygiene are recommended for patients with narcolepsy to improve sleep quality

A patient calls the clinic to ask about taking a glucosamine-chondroitin supplement for arthritis. The nurse reviews the medication history and notes that there will be a concern for drug interactions if the patient is also taking medications for which disorder? a. Type 2 diabetes mellitus b. Hypothyroidism c. Hypertension d. Angina

ANS: A The glucosamine in glucosamine-chondroitin supplements may cause an increase in insulin resistance, necessitating the need for higher doses of oral hypoglycemics or insulin.

3. An adult patient who is hospitalized after a motorcycle crash tells the nurse, "I didn't sleep last night because I worried about missing work at my new job and losing my insurance coverage." Which nursing diagnosis is appropriate to include in the plan of care? a. Anxiety b. Defensive coping c. Ineffective denial d. Risk prone health behavior

ANS: A The information about the patient indicates that anxiety is an appropriate nursing diagnosis. The patient data do not support defensive coping, ineffective denial, or risk prone health behavior as problems for this patient. DIF: Cognitive Level: Apply (application) REF: 78 TOP: Nursing Process: Diagnosis MSC: NCLEX: Psychosocial Integrity

A nurse is preparing to teach the most common malignancy in children. Which malignancy should the nurse discuss? a. Leukemia b. Neuroblastoma c. Wilms tumor d. Retinoblastoma

ANS: A The most common malignancy in children is leukemia. Tumors of the nervous system, Wilms tumors, and retinoblastoma occur less frequently. REF: p. 302, Table 12-1

What is the first action the nurse should take in addressing the patient's concerns about insomnia and daytime fatigue? a. Question the patient about sleep and rest patterns b. Suggest that the patient decrease intake of caffeine-containing beverages. c. Advise the patient to get out of bed if unable to fall asleep in 10 to 20 minutes. d. Recommend that the patient use any prescribed sleep aids for no more then 2 weeks

ANS: A The nurse's first action should be assessment of the patient for factors that may contribute to poor sleep quality or daytime fatigue such as the use of OTC medications. The other actions may be appropriate, but assessment is needed first to choose appropriate interventions to improve the patient's sleep.

A patient with sleep apnea who uses a continuous positive airway pressure (CPAP) device is preparing to have inpatient surgery. Which instructions should the nurse provide to the patient? a. Take your home CPAP device to the hospital. b. Plan to schedule a nighttime polysomnography (PSG) study before surgery. c. Discourage the patient from requesting pain medication while hospitalized. d. Call the hospital to ensure that mechanical ventilation will be available for the patient.

ANS: A The patient should be told to take the CPAP device to the hospital if an overnight stay is expected. Many patients will be able to use their own CPAP equipment, but hospital policy should be checked to make sure it can be used. Patients should be treated for pain and monitored for respiratory depression. Another PSG is not required before surgery. There is no need to call the hospital if the patient takes the CPAP device to the hospital

12. A 32-year-old patient who has had an open reduction and internal fixation (ORIF) of left lower leg fractures continues to complain of severe pain in the leg 15 minutes after receiving the prescribed IV morphine. Pulses are faintly palpable and the foot is cool. Which action should the nurse take next? a. Notify the health care provider. b. Assess the incision for redness. c. Reposition the left leg on pillows. d. Check the patient's blood pressure.

ANS: A The patient's clinical manifestations suggest compartment syndrome and delay in diagnosis and treatment may lead to severe functional impairment. The data do not suggest problems with blood pressure or infection. Elevation of the leg will decrease arterial flow and further reduce perfusion. DIF: Cognitive Level: Apply (application) REF: 1522 TOP: Nursing Process: Implementation MSC: NCLEX: Physiological Integrity

24. A 72-yr-old patient with a history of benign prostatic hyperplasia (BPH) is admitted with acute urinary retention and elevated blood urea nitrogen (BUN) and creatinine levels. Which prescribed therapy should the nurse implement first? a. Insert urethral catheter. b. Obtain renal ultrasound. c. Draw a complete blood count. d. Infuse normal saline at 50 mL/hour.

ANS: A The patient's elevation in BUN is most likely associated with hydronephrosis caused by the acute urinary retention, so the insertion of a retention catheter is the first action to prevent ongoing postrenal failure for this patient. The other actions also are appropriate but should be implemented after the retention catheter. DIF: Cognitive Level: Analyze (analysis) REF: 1071 OBJ: Special Questions: Prioritization TOP: Nursing Process: Implementation MSC: NCLEX: Physiological Integrity

A 45-year-old male presents with persistent, severe stomach pain. Testing reveals a peptic ulcer. Further laboratory tests reveal the presence of Helicobacter pylori. Which of the following is of concern for this patient? a. Gastric cancer b. Leukemia c. Lung cancer d. Adenocarcinoma of the colon

ANS: A The presence of Helicobacter pylori is associated with gastric cancer, not leukemia, lung cancer, or colon cancer.

12. When caring for a patient with a left arm arteriovenous fistula, which action will the nurse include in the plan of care to maintain the patency of the fistula? a. Auscultate for a bruit at the fistula site. b. Assess the quality of the left radial pulse. c. Compare blood pressures in the left and right arms. d. Irrigate the fistula site with saline every 8 to 12 hours.

ANS: A The presence of a thrill and bruit indicates adequate blood flow through the fistula. Pulse rate and quality are not good indicators of fistula patency. Blood pressures should never be obtained on the arm with a fistula. Irrigation of the fistula might damage the fistula, and typically only dialysis staff would access the fistula. DIF: Cognitive Level: Understand (comprehension) REF: 1087 TOP: Nursing Process: Planning MSC: NCLEX: Physiological Integrity

What is the effect of telomere caps on cancer cells? a. Repeated divisions b. Clonal distinction c. Limited mitosis d. Mutation abilities

ANS: A The presence of telomere caps gives cancer cells the ability to divide over and over again, thus cancer cells have unlimited mitosis. Telomere caps do not give cells clonal distinction. Mutation capability is a characteristic of cancer cells, but this property is not related to telomeres.

Which reflex of infancy will disappear first? a. Stepping b. Rooting c. Palmar grasp d. Moro reflex

ANS: A The stepping reflex should no longer be obtainable at 6 weeks. The rooting reflex should no longer be obtainable at 4 months. The palmar grasp should no longer be obtainable at 6 months. The Moro reflex should no longer be obtainable at 3 months. REF: p. 423, Table 17-1

28. When using the accompanying illustration to teach a patient about breast self-examination, the nurse will include the information that most breast cancers are located in which part of the breast? a. 1 b. 2 c. 3 d. 4 e. 5

ANS: A The upper outer quadrant is the location of most of the glandular tissue of the breast. DIF: Cognitive Level: Understand (comprehension) TOP: Nursing Process: Planning MSC: NCLEX: Physiological Integrity

26. A patient who is scheduled for a lumpectomy and axillary lymph node dissection tells the nurse, I would rather not know much about the surgery. Which response by the nurse is best? a. Tell me what you think is important to know about the surgery. b. It is essential that you know enough to provide informed consent. c. Many patients do better after surgery if they have more information. d. You can wait until after surgery for teaching about pain management.

ANS: A This response shows sensitivity to the individual patients need for information about the surgery. The other responses are also accurate, but the nurse should tailor patient teaching to individual patient preferences. DIF: Cognitive Level: Apply (application) OBJ: Special Questions: Prioritization TOP: Nursing Process: Assessment MSC: NCLEX: Psychosocial Integrity

Transchondral fractures are most prevalent in: a. adolescents. b. older adults. c. infants. d. premenopausal females.

ANS: A Transchondral fractures are most prevalent in adolescents. Such fractures are not associated with older adults, infants, or premenopausal females.

What is the most common aggravating trigger of gouty arthritis attacks? a. Trauma b. Anemia c. High-fat foods d. Lack of exercise

ANS: A Trauma is the most common aggravating factor of an acute gouty exacerbation. Gout is not triggered by anemia or lack of exercise. Gout is not caused by high-fat foods, but rather high-purine foods.

A patient with gout has been treated with allopurinol (Zyloprim) for 2 months. The nurse will monitor laboratory results for which therapeutic effect? a. Decreased uric acid levels b. Decreased prothrombin time c. Decreased white blood cell count d. Increased hemoglobin and hematocrit levels

ANS: A Treatment of gout with allopurinol should result in decreased uric acid levels. The other options are incorrect.

15. A student nurse prepares a list of teaching topics for a patient with a new diagnosis of breast cancer. Which item should the charge nurse suggest that the student nurse omit from the teaching topic list about breast cancer diagnostic testing? a. CA 15-3 level testing b. HER-2 receptor testing c. Estrogen receptor testing d. Oncotype DX assay testing

ANS: A Tumor markers such as CA 15-3 are used to monitor response to treatment for breast cancer, not to detect or diagnose breast cancer. The other tests are likely to be used for additional diagnostic testing in a patient with breast cancer. DIF: Cognitive Level: Apply (application) TOP: Nursing Process: Planning MSC: NCLEX: Safe and Effective Care Environment

A 3-year-old female was diagnosed with Wilms tumor. This disease is a tumor of the: a. kidney. b. brain. c. bone marrow. d. liver.

ANS: A Wilms tumor is a tumor found in the kidney, not the brain, the bone marrow, or the liver.

When teaching a patient about the proper application of timolol (Timoptic) eyedrops, the nurse will include which instruction? a. "Apply the drops into the conjunctival sac instead of directly onto the eye." b. "Apply the drops directly to the eyeball (cornea) for the best effect." c. "Blot your eye with a tissue immediately after applying the drops." d. "Tilt your head forward before applying the eyedrops."

ANS: A All ophthalmic drugs should be administered in the conjunctival sac. Gently use a tissue to remove excess eye medication—do not blot the eye after giving the medication. Tilt the head back before giving the eyedrops. DIF: COGNITIVE LEVEL: Applying (Application) REF: p. 918 TOP: NURSING PROCESS: Implementation MSC: NCLEX: Safe and Effective Care Environment: Management of Care

A 70-yr-old patient who has had a transurethral resection of the prostate (TURP) for benign prostatic hyperplasia (BPH) is being discharged from the hospital today. Which patient statement indicates a need for the nurse to provide additional instruction? a. "I should call the doctor if I have incontinence at home." b. "I will avoid driving until I get approval from my doctor." c. "I should schedule yearly appointments for prostate examinations." d. "I will increase fiber and fluids in my diet to prevent constipation."

ANS: A Because incontinence is common for several weeks after a TURP, the patient does not need to call the health care provider if this occurs. The other patient statements indicate that the patient has a good understanding of post-TURP instructions.

When performing discharge teaching for a patient after a vasectomy, the nurse instructs the patient that he a. should continue to use other methods of birth control for 6 weeks. b. should not have sexual intercourse until his 6-week follow-up visit. c. may have temporary erectile dysfunction (ED) because of swelling. d. will notice a decrease in the appearance and volume of his ejaculate.

ANS: A Because it takes about 6 weeks to evacuate sperm that are distal to the vasectomy site, the patient should use contraception for 6 weeks. ED that occurs after vasectomy is psychologic in origin and not related to postoperative swelling. The patient does not need to abstain from intercourse. The appearance and volume of the ejaculate are not changed because sperm are a minor component of the ejaculate.

A patient with urinary obstruction from benign prostatic hyperplasia (BPH) tells the nurse, "My symptoms are much worse this week." Which response by the nurse is appropriate? a. "Have you taken any over-the-counter (OTC) medications recently?" b. "I will talk to the doctor about a prostate specific antigen (PSA) test." c. "Have you talked to the doctor about surgery such as transurethral resection of the prostate (TURP)?" d. "The prostate gland changes in size from day to day, and this may be making your symptoms worse."

ANS: A Because the patient's increase in symptoms has occurred abruptly, the nurse should ask about OTC medications that might cause contraction of the smooth muscle in the prostate and worsen obstruction. The prostate gland does not vary in size from day to day. A TURP may be needed, but more assessment about possible reasons for the sudden symptom change is a more appropriate first response by the nurse. PSA testing is done to differentiate BPH from prostatic cancer.

The nurse will inform a patient with cancer of the prostate that side effects of leuprolide (Lupron) may include a. flushing. b. dizziness. c. infection. d. incontinence.

ANS: A Hot flashes may occur with decreased testosterone production. Dizziness may occur with the -blockers used for benign prostatic hyperplasia. Urinary incontinence may occur after prostate surgery, but it is not an expected side effect of medication. Risk for infection is increased in patients receiving chemotherapy.

The nurse is performing an eye examination on a 76-yr-old patient. Which finding indicates that the nurse should refer the patient for a more extensive assessment? a. The patient's sclerae are light yellow. b. The patient reports persistent photophobia. c. The pupil recovers slowly after responding to a bright light. d. There is a whitish gray ring encircling the periphery of the iris.

ANS: B

Several patients call the urology clinic requesting appointments with the health care provider as soon as possible. Which patient will the nurse schedule to be seen first? a. A 22-yr-old patient who has noticed a firm, nontender lump on his scrotum b. A 35-yr-old patient who is concerned that his scrotum "feels like a bag of worms" c. A 40-yr-old patient who has pelvic pain while being treated for chronic prostatitis d. A 70-yr-old patient who is reporting frequent urinary dribbling after a prostatectomy

ANS: A The patient's age and symptoms suggest possible testicular cancer. Some forms of testicular cancer can be very aggressive, so the patient should be evaluated by the health care provider as soon as possible. Varicoceles do require treatment but not emergently. Ongoing pelvic pain is common with chronic prostatitis. Urinary dribbling is a common problem after prostatectomy.

The following male patients recently arrived in the emergency department. Which one should the nurse assess first? a. A 19-yr-old patient who is complaining of severe scrotal pain b. A 60-yr-old patient with a nontender ulceration of the glans penis c. A 64-yr-old patient who has dysuria after brachytherapy for prostate cancer d. A 22-yr-old patient who has purulent urethral drainage and severe back pain

ANS: A The patient's age and symptoms suggest possible testicular torsion, which will require rapid treatment to prevent testicular necrosis. The other patients also require assessment by the nurse, but their history and symptoms indicate nonemergent problems (acute prostatitis, cancer of the penis, and radiation-associated urinary tract irritation).

A patient has a new prescription for an antiglaucoma eyedrop. The next day, she calls the clinic and states, "The package insert says this medication might make my blue eyes turn brown! Is this true?" The nurse realizes that the patient has a prescription for which eye medication? a. Latanoprost (Xalatan), a prostaglandin agonist b. Dorzolamide (Trusopt), an ocular carbonic anhydrase inhibitor c. Betaxolol (Betoptic), a direct-acting beta blocker d. Pilocarpine (Pilocar), a direct-acting cholinergic

ANS: A There is one unique adverse effect associated with all prostaglandin agonists—in some people with hazel, green, or bluish-brown eye color, eye color will turn permanently brown, even if the medication is discontinued. This adverse effect appears to be cosmetic only, with no known ill effects on the eye. The other medications do not have this effect. DIF: COGNITIVE LEVEL: Understanding (Comprehension) REF: p. 911 TOP: NURSING PROCESS: Implementation MSC: NCLEX: Physiological Integrity: Pharmacological and Parenteral Therapies

(Multiple Response) The nurse is caring for a patient infected with human immunodeficiency virus (HIV) who has just been diagnosed with asymptomatic chronic HIV infection. Which prophylactic measures will the nurse include in the plan of care (select all that apply)? a. Hepatitis B vaccine b. Pneumococcal vaccine c. Influenza virus vaccine d. Trimethoprim-sulfamethoxazole e. Varicella zoster immune globulin

ANS: A, B, C Asymptomatic chronic HIV infection is a stage between acute HIV infection and a diagnosis of symptomatic chronic HIV infection. Although called asymptomatic, symptoms (e.g., fatigue, headache, low-grade fever, night sweats) often occur. Prevention of other infections is an important intervention in patients who are HIV positive, and these vaccines are recommended as soon as the HIV infection is diagnosed. Antibiotics and immune globulin are used to prevent and treat infections that occur later in the course of the disease when the CD4+ counts have dropped or when infection has occurred.

What is the term used to identify a pattern of bone destruction indicative of an aggressive malignant tumor? (select all that apply) a. Moth-eaten b. Permeative c. Geographic d. Radial e. Selective

ANS: A, B, C Three patterns of bone destruction by bone tumors have been identified: (1) the geographic pattern, (2) the moth-eaten pattern, and (3) the permeative pattern. Radial and selective are not considered patterns of malignancy

MULTIPLE RESPONSE 1. Brachytherapy is being used to treat cancer in a patient. What types of cancers respond well to brachytherapy? (select all that apply) a. Prostate b. Cervix c. Head d. Neck e. Lung

ANS: A, B, C, D Radiation sources can be temporarily placed into body cavities through a delivery method termed brachytherapy. Brachytherapy is useful in the treatment of cervical, prostate, and head and neck cancers. It is not used in the treatment of lung cancer. REF: p. 260

Assessment signs of neuroblastoma include: (select all that apply) a. weight loss. b. irritability. c. fatigue. d. fever. e. constipation.

ANS: A, B, C, D Systemic signs of neuroblastoma include weight loss, irritability, fatigue, fever, and intractable diarrhea, not constipation. REF: p. 435

The nurse is administering ophthalmic drops. Place the following administration steps in the correct order. (Select all that apply.) a. Close the eye gently. b. Apply gentle pressure to the inner canthus/lacrimal sac for 1 minute. c. Place drops into the conjunctival sac. d. Clean debris from the eye as needed. e. Have the patient tilt the head back and look up at the ceiling. f. Remove excess medication gently from around the eyes.

ANS: A, B, C, D, E, F Before applying eye medications, clean any debris from the eye, if needed, and have the patient tilt the head back and look up at the ceiling. Drops are placed into the conjunctival sac, and then the eye is closed gently. Pressure may be applied to the inner canthus/lacrimal sac for 1 minute to reduce systemic absorption, and any excess medication can then be removed from around the eyes. DIF: COGNITIVE LEVEL: Applying (Application) REF: p. 918 TOP: NURSING PROCESS: Implementation MSC: NCLEX: Safe and Effective Care Environment: Management of Care

The nurse is reviewing the medical record of a patient and notes an order for ophthalmic dexamethasone (Decadron) solution. The nurse knows that indications for ophthalmic dexamethasone include which conditions? (Select all that apply.) a. Uveitis b. Allergic conditions c. Removal of foreign bodies d. Ocular infections e. Glaucoma f. Conjunctival inflammation

ANS: A, B, C, F Dexamethasone (Decadron) is used to treat inflammation of the eye, eyelids, conjunctiva, and cornea, and it may also be used in the treatment of uveitis, iridocyclitis, allergic conditions, and burns and in the removal of foreign bodies. It is not indicated for infections or glaucoma. DIF: COGNITIVE LEVEL: Understanding (Comprehension) REF: p. 915 TOP: NURSING PROCESS: Planning MSC: NCLEX: Physiological Integrity: Pharmacological and Parenteral Therapies

The occupational health nurse is caring for an employee who reporting bilateral eye pain after a cleaning solution splashed into the employee's eyes. Which action will the nurse take? a. Apply cool compresses. b. Flush the eyes with saline. c. Apply antiseptic ophthalmic ointment to the eyes. d. Cover the eyes with dry sterile patches and shields.

ANS: B

A 6-year-old male presents with fever, pain, swelling, and warmth and is diagnosed with osteomyelitis. In addition to the clinical symptoms, the nurse would expect elevations in which lab tests? (select all that apply) a. C-reactive protein b. White blood cell count c. Red cell count d. Erythrocyte sedimentation rate (ESR) e. Liver enzymes

ANS: A, B, D Children often will present with fever, elevated white blood cell count (50-70%), elevated C-reactive protein (98%), and elevated ESR (90%). REF: p. 1042

3. During an intravenous infusion of calcium, the nurse carefully monitors the patient for symptoms of hypercalcemia. Which are symptoms of hypercalcemia? (Select all that apply.) a. Anorexia b. Nausea and vomiting c. Diarrhea d. Constipation e. Cardiac irregularities f. Drowsiness

ANS: A, B, D, E Symptoms of hypercalcemia include anorexia, nausea, vomiting, and constipation. Long-term excessive calcium intake can result in severe hypercalcemia, which can cause cardiac irregularities, delirium, and coma. The other options are incorrect.

The barbiturate phenobarbital is prescribed for a patient with epilepsy. While assessing the patient's current medications, the nurse recognizes that interactions may occur with which drugs? (Select all that apply.) a. Antihistamines b. Opioids c. Diuretics d. Anticoagulants e. Oral contraceptives f. Insulin

ANS: A, B, D, E The co-administration of barbiturates and alcohol, antihistamines, benzodiazepines, opioids, and tranquilizers may result in additive CNS depression. Co-administration of anticoagulants and barbiturates can result in decreased anticoagulation response and possible clot formation. Co-administration of barbiturates and oral contraceptives can result in accelerated metabolism of the contraceptive drug and possible unintended pregnancy. There are no interactions with diuretics and insulin. DIF: COGNITIVE LEVEL: Understanding (Comprehension) REF: p. 195 TOP: NURSING PROCESS: Assessment MSC: NCLEX: Physiological Integrity: Pharmacological Therapies

Signs and symptoms of subluxation include: (select all that apply) a. Pain b. Swelling c. Hemorrhage d. Limitation of movement e. Joint deformity

ANS: A, B, D, E Signs and symptoms of dislocations or subluxations include pain, swelling, limitation of motion, and joint deformity.

(Multiple Response) The nurse plans a presentation for community members about how to decrease the risk for antibiotic-resistant infections. Which information will the nurse include in the teaching plan (select all that apply)? a. Antibiotics may sometimes be prescribed to prevent infection. b. Continue taking antibiotics until all of the prescription is gone. c. Unused antibiotics that are more than a year old should be discarded. d. Antibiotics are effective in treating influenza associated with high fevers. e. Hand washing is effective in preventing many viral and bacterial infections.

ANS: A, B, E All prescribed doses of antibiotics should be taken. In some situations, such as before surgery, antibiotics are prescribed to prevent infection. There should not be any leftover antibiotics because all prescribed doses should be taken. However, if there are leftover antibiotics, they should be discarded immediately because the number left will not be enough to treat a future infection. Hand washing is generally considered the single most effective action in decreasing infection transmission. Antibiotics are ineffective in treating viral infections such as influenza.

The nurse is reviewing the history of a patient who has a new order for a nonsteroidal anti-inflammatory drug (NSAID) to treat tendonitis. Which conditions are contraindications to the use of NSAIDs? (Select all that apply.) a. Vitamin K deficiency b. Arthralgia c. Peptic ulcer disease d. Neuropathy e. Pericarditis

ANS: A, C Contraindications to NSAIDs include known drug allergy as well as conditions that place the patient at risk for bleeding, such as Vitamin K deficiency and peptic ulcer disease. NSAIDs may be used to treat arthralgia and pericarditis. Neuropathy is not a contraindication.

Individuals of low socioeconomic status are at an increased risk for infection because of which of the following? (Select all that apply.) a. Uninsured or underinsured status b. Easy access to health screenings c. High cost of medications d. Inadequate nutrition e. mostly female gender

ANS: A, C, D Individuals of low socioeconomic status tend to be part of the underinsured or uninsured population. Lack of insurance decreases accessibility to health care in general and health screening services specifically. High costs of medication and nutritious food also make this population at higher risk for infection.

1. Which information will be included when the nurse is teaching self-management to a patient who is receiving peritoneal dialysis (select all that apply)? a. Avoid commercial salt substitutes. b. Restrict fluid intake to 1000 mL daily. c. Take phosphate binders with each meal. d. Choose high-protein foods for most meals. e. Have several servings of dairy products daily.

ANS: A, C, D Patients who are receiving peritoneal dialysis should have a high-protein diet. Phosphate binders are taken with meals to help control serum phosphate and calcium levels. Commercial salt substitutes are high in potassium and should be avoided. Fluid intake is not limited unless weight and blood pressure are not controlled. Dairy products are high in phosphate and usually are limited. DIF: Cognitive Level: Apply (application) REF: 1087 TOP: Nursing Process: Planning MSC: NCLEX: Physiological Integrity

When a patient is receiving cisplatin (Platinol-AQ) chemotherapy, the nurse will monitor for which adverse effects? (Select all that apply.) a. Tinnitus b. Heart failure c. Hearing loss d. Elevated blood urea nitrogen and creatinine levels e. Numbness or tingling in the extremities f. Elevated glucose and ketone levels

ANS: A, C, D, E Cisplatin can cause nephrotoxicity, ototoxicity, and peripheral neuropathy. Nephrotoxicity is manifested by rising blood urea nitrogen and creatinine levels; ototoxicity is manifested by tinnitus, hearing loss, and dizziness; peripheral neuropathy is manifested by numbness or tingling of the extremities. DIF: COGNITIVE LEVEL: Applying (Application) REF: p. 736 TOP: NURSING PROCESS: Evaluation MSC: NCLEX: Physiological Integrity: Reduction of Risk Potential

What should the nurse assess to evaluate the effectiveness of treatment for the patient's myopia and presbyopia? a. Strength of the eye muscles. b. Both near and distant vision. c. Cloudiness in the eye lenses. d. Intraocular pressure changes.

ANS: B

What should the nurse teach a patient with repeated hordeolum about how to prevent further infection? a. Apply cold compresses. b. Discard all used eye cosmetics. c. Wash the eyebrows with an antiseborrheic shampoo. d. Be examined for sexually transmitted infections (STIs).

ANS: B

1. The nurse is providing patient education for a patient taking an oral contraceptive. Which drugs may cause interactions with oral contraceptives? (Select all that apply.) a. Cephalexin (Keflex) b. Guaifenesin (Robitussin) c. Warfarin (Coumadin) d. Ibuprofen (Motrin) e. Theophylline (Uniphyl)

ANS: A, C, E Patients must be educated about the need to use alternative birth control methods for at least 1 month during and after taking any of these drugs: antibiotics (especially penicillins and cephalosporins); barbiturates; isoniazid; and rifampin. The effectiveness of other drugs, such as anticonvulsants, beta blockers, hypnotics, antidiabetic drugs, warfarin, theophylline, tricyclic antidepressants, and vitamins, may be reduced when they are taken with oral contraceptives. DIF: COGNITIVE LEVEL: Applying (Application) REF: p. 540 TOP: NURSING PROCESS: Planning MSC: NCLEX: Safe and Effective Care Environment: Management of Care

1. The nurse is reviewing vitamin therapy in preparation for a nutrition class. Which statements are accurate regarding vitamin C (ascorbic acid)? (Select all that apply.) a. Vitamin C is important in the maintenance of bone, teeth, and capillaries. b. Vitamin C is essential for night vision. c.Vitamin C is important for tissue repair. d. Vitamin C is found in animal sources such as dairy products and meat. e.Vitamin C is found in tomatoes, strawberries, and broccoli. f.Vitamin C is also known as the "sunshine vitamin." g.Vitamin C deficiency is known as scurvy.

ANS: A, C, E, G These statements are true of vitamin C. Vitamin A is essential for night vision, and vitamin D is known as the sunshine vitamin. With the exception of liver, meat and dairy products are not sources of vitamin C.

Which actions will the nurse include in the plan of care for a patient with metastatic bone cancer of the left femur (select all that apply)? a. Monitor serum calcium. b. Teach about the need for strict bed rest. c. Discontinue use of sustained-release opioids. d. Support the left leg when repositioning the patient. e. Support family and patient as they discuss the prognosis.

ANS: A, D, E The nurse will monitor for hypercalcemia caused by bone decalcification. Support of the leg helps reduce the risk for pathologic fractures. Although the patient may be reluctant to exercise, activity is important to maintain function and avoid the complications associated with immobility. Adequate pain medication, including sustained-release and rapidly acting opioids, is needed for the severe pain that is frequently associated with bone cancer. The prognosis for metastatic bone cancer is poor so the patient and family need to be supported as they deal with the reality of the situation.

The nurse is assessing a patient who is receiving chemotherapy with an alkylating drug. Which assessment findings would be considered indications of an oncologic emergency? (Select all that apply.) a. Dry, "scratchy," or "swollen" throat b. Loss of hair c. Decreased red blood cell count d. White patches in the mouth or throat e. Temperature of 100.7° F (38.2° C) f. Decreased urine output

ANS: A, D, E, F Indications of an oncologic emergency include fever and/or chills with a temperature higher than 100.5° F (38.1° C); new sores or white patches in the mouth or throat; changes in bladder function or patterns; dry, burning, "scratchy," or "swollen" throat; and other signs and symptoms (see Box 46-4). The prescriber must be contacted immediately if any of the listed signs or symptoms occur. Loss of hair and decreased red blood cell count (a result of bone marrow suppression) are expected effects of chemotherapy. DIF: COGNITIVE LEVEL: Analyzing (Analysis) REF: p. 744 TOP: NURSING PROCESS: Assessment MSC: NCLEX: Physiological Integrity: Pharmacological and Parenteral Therapies

A patient will be taking oral iron supplements at home. The nurse will include which statements in the teaching plan for this patient? (Select all that apply.) a. Take the iron tablets with meals. b. Take the iron tablets on an empty stomach 1 hour before meals. c. Take the iron tablets with an antacid to prevent heartburn. d. Drink 8 ounces of milk with each iron dose. e. Taking iron supplements with orange juice enhances iron absorption. f. Stools may become loose and light in color. g. Stools may become black and tarry. h. Tablets may be crushed to enhance iron absorption.

ANS: A, E, G Iron tablets need to be taken with meals to reduce gastrointestinal distress, but antacids and milk interfere with absorption. Orange juice enhances the absorption of iron. Stools may become black and tarry in patients who are on iron supplements. Tablets need to be taken whole, not crushed, and the patient needs to be encouraged to eat foods high in iron.

A 75-yr-old patient who lives alone at home tells the nurse, "I am afraid of losing my independence because my eyes don't work as well they used to." Which action should the nurse take first? a. Discuss the increased risk for falls that is associated with impaired vision. b. Ask the patient about what type of vision problems are being experienced. c. Explain that there are many ways to compensate for decreases in visual acuity. d. Suggest ways of improving the patient's safety, such as using brighter lighting.

ANS: B

A patient with glaucoma who has been using timolol (Timoptic) drops for several days tells the nurse that the eyedrops cause eye burning and visual blurriness after administration. The best response to the patient's statement is a. "Those symptoms may indicate a need for a change in dosage of the eyedrops." b. "The drops are uncomfortable, but it is important to use them to retain your vision." c. "These are normal side effects of the drug, which should be less noticeable with time." d. "Notify your health care provider so that different eyedrops can be prescribed for you."

ANS: B

How should the nurse evaluate a patient for improvement after treatment of primary open-angle glaucoma (POAG)? a. Question the patient about blurred vision. b. Note any changes in the patient's visual field. c. Ask the patient to rate the pain using a 0 to 10 scale. d. Assess the patient's depth perception when climbing stairs.

ANS: B

The charge nurse observes a newly hired nurse performing all the following interventions for a patient who has just undergone right cataract removal and an intraocular lens implant. Which action requires that the charge nurse intervene? a. The nurse leaves the eye shield in place. b. The nurse encourages the patient to cough. c. The nurse elevates the patient's head to 45 degrees. d. The nurse applies corticosteroid drops to the right eye.

ANS: B

The nurse is caring for a patient diagnosed with adult inclusion conjunctivitis (AIC) caused by C. trachomatis. Which action should be included in the plan of care? a. Applying topical corticosteroids to decrease inflammation b. Discussing the need for sexually transmitted infection testing c. Educating about the use of antiviral eyedrops to treat the infection d. Assisting with applying for community visual rehabilitation services

ANS: B

After receiving change-of-shift report for several patients with neutropenia, which patient should the nurse assess first? a. 56-year-old with frequent explosive diarrhea b. 33-year-old with a fever of 100.8 F (38.2 C) c. 66-year-old who has white pharyngeal lesions d. 23-year old who is complaining of severe fatigue

ANS: B Any fever in a neutropenic patient indicates infection and can quickly lead to sepsis and septic shock. Rapid assessment and (if prescribed) initiation of antibiotic therapy within 1 hour are needed. The other patients also need to be assessed but do not exhibit symptoms of potentially life-threatening problems.

A patient with muscular dystrophy is hospitalized with pneumonia. Which nursing action will be included in the plan of care? a. Logroll the patient every 2 hours. b. Assist the patient with ambulation. c. Discuss the need for genetic testing with the patient. d. Teach the patient about the muscle biopsy procedure.

ANS: B Because the goal for the patient with muscular dystrophy is to keep the patient active for as long as possible, assisting the patient to ambulate will be part of the care plan. The patient will not require logrolling. Muscle biopsies are necessary to confirm the diagnosis but are not necessary for a patient who already has a diagnosis. There is no need for genetic testing because the patient already knows the diagnosis.

Which collaborative problem will the nurse include in a care plan for a patient admitted to the hospital with idiopathic aplastic anemia? a. seizures b. infection c. neurogenic shock d.pulmonary edema

ANS: B Because the patient with aplastic anemia has pancytopenia, the patient is at risk for infection and bleeding. There is no increased risk for seizures, neurogenic shock, or pulmonary edema.

A routine complete blood count indicates that an active 80-year-old man may have myelodysplastic syndrome. The nurse will plan to teach the patient about a. blood transfusion b. bone marrow biopsy. c. filgrastim (Neupogen) administration. d. erythropoietin (Epogen) administration.

ANS: B Bone marrow biopsy is needed to make the diagnosis and determine the specific type of myelodysplastic syndrome. The other treatments may be necessary if there is progression of the myelodysplastic syndrome, but the initial action for this asymptomatic patient will be a bone marrow biopsy.

11. A newly admitted patient has orders for both vitamin C and zinc supplements. The nurse reviews the patient's medical history and concludes that these supplements are ordered for which reason? a. To treat pellagra b. To aid in wound healing c. To treat osteomalacia d. As an antidote for anticoagulant overdose

ANS: B Both zinc and vitamin C are important for normal tissue growth and repair, which makes them helpful with wound healing. Vitamin B3 (niacin) is used to treat pellagra; vitamin D is used to treat osteomalacia; and vitamin K is used as an antidote for anticoagulant overdose.

Which information obtained by the nurse caring for a patient with thrombocytopenia should be immediately communicated to the health care provider? a. The platelet count is 52,000/L. b. The patient is difficult to arouse. c. There are purpura on the oral mucosa. d. There are large bruises on the patients back.

ANS: B Difficulty in arousing the patient may indicate a cerebral hemorrhage, which is life threatening and requires immediate action. The other information should be documented and reported but would not be unusual in a patient with thrombocytopenia.

A nurse who works on the orthopedic unit has just received change-of-shift report. Which patient should the nurse assess first? a. Patient who reports foot pain after hammertoe surgery b. Patient who has not voided 10 hours after a laminectomy c. Patient with low back pain and a positive straight-leg-raise test d. Patient with osteomyelitis who has a temperature of 100.5° F (38.1° C)

ANS: B Difficulty in voiding may indicate damage to the spinal nerves and should be assessed and reported to the surgeon immediately. The information about the other patients is consistent with their diagnoses. The nurse will need to assess them as quickly as possible, but the information about them does not indicate a need for immediate intervention.

A patient whose employment requires frequent lifting has a history of chronic back pain. After the nurse has taught the patient about correct body mechanics, which patient statement indicates the teaching has been effective? a. "I will keep my back straight when I lift above than my waist." b. "I will begin doing exercises to strengthen and support my back." c. "I will tell my boss I need a job where I can stay seated at a desk." d. "I can sleep with my hips and knees extended to prevent back strain."

ANS: B Exercises can help strengthen the muscles that support the back. Flexion of the hips and knees places less strain on the back. Modifications in the way the patient lifts boxes are needed, but sitting for prolonged periods can aggravate back pain. The patient should not lift above the level of the elbows.

Which intervention will be included in the nursing care plan for a patient with immune thrombocytopenic purpura (ITP)? a. Assign the patient to a private room. b. Avoid intramuscular (IM) injections. c. Use rinses rather than a soft toothbrush for oral care. d. Restrict activity to passive and active range of motion.

ANS: B IM or subcutaneous injections should be avoided because of the risk for bleeding. A soft toothbrush can be used for oral care. There is no need to restrict activity or place the patient in a private room.

A patient with acute osteomyelitis of the left femur is hospitalized for regional antibiotic irrigation. Which intervention will the nurse include in the initial plan of care? a. Quadriceps-setting exercises b. Immobilization of the left leg c. Positioning the left leg in flexion d. Assisted weight-bearing ambulation

ANS: B Immobilization of the affected leg helps decrease pain and reduce the risk for pathologic fractures. Weight-bearing exercise increases the risk for pathologic fractures. Flexion of the affected limb is avoided to prevent contractures.

The nurse notes scleral jaundice in a patient being admitted with hemolytic anemia. The nurse will plan to check the laboratory results for the a. Schilling test. b. bilirubin level. c. stool occult blood test. d. gastric analysis testing.

ANS: B Jaundice is caused by the elevation of bilirubin level associated with red blood cell (RBC) hemolysis. The other tests would not be helpful in monitoring or treating a hemolytic anemia.

A disorder similar to osteomalacia that occurs in growing bones of children is termed: a. Paget disease. b. rickets. c. osteomyelitis. d. osteosarcoma.

ANS: B Abnormal bone growth in children is termed rickets. Paget disease is a state of increased metabolic activity in bone characterized by abnormal and excessive bone remodeling, both resorption and formation. Osteomyelitis is a bone infection while osteosarcoma is a form of bone cancer.

It is important for the nurse providing care for a patient with sickle cell crisis to a. limit the patients intake of oral and IV fluids. b. evaluate the effectiveness of opioid analgesics. c. encourage the patient to ambulate as much as tolerated. d. teach the patient about high-protein, high-calorie foods.

ANS: B Pain is the most common clinical manifestation of a crisis and usually requires large doses of continuous opioids for control. Fluid intake should be increased to reduce blood viscosity and improve perfusion. Rest is usually ordered to decrease metabolic requirements. Patients are instructed about the need for dietary folic acid, but high-protein, high-calorie diets are not emphasized.

A patient with a history of a transfusion-related acute lung injury (TRALI) is to receive a transfusion of packed red blood cells (PRBCs). Which action by the nurse will decrease the risk for TRALI for this patient? a. Infuse the PRBCs slowly over 4 hours. b. Transfuse only leukocyte-reduced PRBCs. c. Administer the scheduled diuretic before the transfusion. d. Give the PRN dose of antihistamine before the transfusion.

ANS: B TRALI is caused by a reaction between the donor and the patient leukocytes that causes pulmonary inflammation and capillary leaking. The other actions may help prevent respiratory problems caused by circulatory overload or by allergic reactions, but they will not prevent TRALI.

A 28-year-old man with von Willebrand disease is admitted to the hospital for minor knee surgery. The nurse will review the coagulation survey to check the a. platelet count. b. bleeding time. c. thrombin time. d. prothrombin time.

ANS: B The bleeding time is affected by von Willebrand disease. Platelet count, prothrombin time, and thrombin time are normal in von Willebrand disease.

Which action will the admitting nurse include in the care plan for a 30-year old woman who is neutropenic? a. Avoid any injections. b. Check temperature every 4 hours. c. Omit fruits or vegetables from the diet. d. Place a No Visitors sign on the door.

ANS: B The earliest sign of infection in a neutropenic patient is an elevation in temperature. Although unpeeled fresh fruits and vegetables should be avoided, fruits and vegetables that are peeled or cooked are acceptable. Injections may be required for administration of medications such as filgrastim (Neupogen). The number of visitors may be limited and visitors with communicable diseases should be avoided, but a no visitors policy is not needed.

Which action will the nurse take first when a patient is seen in the outpatient clinic with neck pain? a. Provide information about therapeutic neck exercises. b. Ask about numbness or tingling of the hands and arms. c. Suggest the patient alternate the use of heat and cold to the neck. d. Teach about the use of nonsteroidal antiinflammatory drugs (NSAIDs).

ANS: B The nurse's initial action should be further assessment of the pain because cervical nerve root compression will require different treatment than musculoskeletal neck pain. The other actions may also be appropriate, depending on the assessment findings.

A 30-year-old man with acute myelogenous leukemia develops an absolute neutrophil count of 850/L while receiving outpatient chemotherapy. Which action by the outpatient clinic nurse is most appropriate? a. Discuss the need for hospital admission to treat the neutropenia. b. Teach the patient to administer filgrastim (Neupogen) injections. c. Plan to discontinue the chemotherapy until the neutropenia resolves. d. Order a high-efficiency particulate air (HEPA) filter for the patients home.

ANS: B The patient may be taught to self-administer filgrastim injections. Although chemotherapy may be stopped with severe neutropenia (neutrophil count less than 500/L), administration of filgrastim usually allows the chemotherapy to continue. Patients with neutropenia are at higher risk for infection when exposed to other patients in the hospital. HEPA filters are expensive and are used in the hospital, where the number of pathogens is much higher than in the patients home environment.

A patient is being discharged after 1 week of IV antibiotic therapy for acute osteomyelitis in the right leg. Which information will be included in the discharge teaching? a. How to apply warm packs to the leg to reduce pain b. How to monitor and care for a long-term IV catheter c. The need for daily aerobic exercise to help maintain muscle strength d. The reason for taking oral antibiotics for 7 to 10 days after discharge

ANS: B The patient will be on IV antibiotics for several months, and the patient will need to recognize signs of infection at the IV site and how to care for the catheter during daily activities such as bathing. IV antibiotics rather than oral antibiotics are used for acute osteomyelitis. Patients are instructed to avoid exercise and heat application because these will increase swelling and the risk for spreading infection.

Several patients call the outpatient clinic and ask to make an appointment as soon as possible. Which patient should the nurse schedule to be seen first? a. 44-year-old with sickle cell anemia who says my eyes always look sort of yellow b. 23-year-old with no previous health problems who has a nontender lump in the axilla c. 50-year-old with early-stage chronic lymphocytic leukemia who reports chronic fatigue d. 19-year-old with hemophilia who wants to learn to self-administer factor VII replacement

ANS: B The patients age and presence of a nontender axillary lump suggest possible lymphoma, which needs rapid diagnosis and treatment. The other patients have questions about treatment or symptoms that are consistent with their diagnosis but do not need to be seen urgently.

A patient with septicemia develops prolonged bleeding from venipuncture sites and blood in the stools. Which action is most important for the nurse to take? a. Avoid venipunctures. b. Notify the patients physician. c. Apply sterile dressings to the sites. d. Give prescribed proton-pump inhibitors.

ANS: B The patients new onset of bleeding and diagnosis of sepsis suggest that disseminated intravascular coagulation (DIC) may have developed, which will require collaborative actions such as diagnostic testing, blood product administration, and heparin administration. The other actions also are appropriate, but the most important action should be to notify the physician so that DIC treatment can be initiated rapidly.

After laminectomy with a spinal fusion to treat a herniated disc, a patient reports numbness and tingling of the right lower leg. The first action the nurse should take is to a. report the patient's complaint to the surgeon. b. check the chart for preoperative assessment data. c. check the vital signs for indications of hemorrhage. d. turn the patient to the left to relieve

ANS: B The postoperative movement and sensation of the extremities should be unchanged (or improved) from the preoperative assessment. If the numbness and tingling are new, this information should be immediately reported to the surgeon. Numbness and tingling are not symptoms associated with hemorrhage at the site. Turning the patient will not relieve the numbness.

3. A patient is on vitamin D supplemental therapy. The nurse will monitor for which signs of toxicity during this therapy? a. Tinnitus b. Anorexia c. Diarrhea d. Hypotension

ANS: B The toxic effects of vitamin D are those associated with hypertension, such as weakness, fatigue, headache, anorexia, dry mouth, metallic taste, nausea, vomiting, abdominal cramps, ataxia, and bone pain. If not recognized and treated, these symptoms can progress to impairment of renal function and osteoporosis. The other options listed are not signs of vitamin D toxicity.

7. A patient with a history of alcohol abuse has been admitted for severe weakness and malnutrition. The nurse will prepare to administer which vitamin preparation to prevent Wernicke's encephalopathy? a. Vitamin B3 (niacin) b. Vitamin B1 (thiamine) c. Vitamin B6 (pyridoxine) d. Folic acid

ANS: B Thiamine is necessary for the treatment of a variety of thiamine deficiencies, including Wernicke's encephalopathy. The other options are incorrect.

A 68-year-old woman with acute myelogenous leukemia (AML) asks the nurse whether the planned chemotherapy will be worth undergoing. Which response by the nurse is appropriate? a. If you do not want to have chemotherapy, other treatment options include stem cell transplantation. b. The side effects of chemotherapy are difficult, but AML frequently goes into remission with chemotherapy. c. The decision about treatment is one that you and the doctor need to make rather than asking what I would do. d. You dont need to make a decision about treatment right now because leukemias in adults tend to progress quite slowly.

ANS: B This response uses therapeutic communication by addressing the patients question and giving accurate information. The other responses either give inaccurate information or fail to address the patients question, which will discourage the patient from asking the nurse for information.

When administering alendronate (Fosamax) to a patient with osteoporosis, the nurse will a. ask about any leg cramps or hot flashes. b. assist the patient to sit up at the bedside. c. be sure that the patient has recently eaten. d. administer the ordered calcium carbonate.

ANS: B To avoid esophageal erosions, the patient taking bisphosphonates should be upright for at least 30 minutes after taking the medication. Fosamax should be taken on an empty stomach, not after taking other medications or eating. Leg cramps and hot flashes are not side effects of bisphosphonates.

The nurse is planning to administer a transfusion of packed red blood cells (PRBCs) to a patient with blood loss from gastrointestinal hemorrhage. Which action can the nurse delegate to unlicensed assistive personnel (UAP)? a. Verify the patient identification (ID) according to hospital policy. b. Obtain the temperature, blood pressure, and pulse before the transfusion. c. Double-check the product numbers on the PRBCs with the patient ID band. d. Monitor the patient for shortness of breath or chest pain during the transfusion.

ANS: B UAP education includes measurement of vital signs. UAP would report the vital signs to the registered nurse (RN). The other actions require more education and a larger scope of practice and should be done by licensed nursing staff members.

2. The nurse is preparing a plan of care for a patient undergoing therapy with vitamin A. Which nursing diagnosis is appropriate for this patient? a. Impaired tissue integrity related to vitamin deficiency b. Risk for injury related to night blindness caused by vitamin deficiency c. Impaired physical mobility (muscle weakness) related to vitamin deficiency d. Acute confusion related to vitamin deficiency

ANS: B Vitamin A deficiency causes night blindness, so risk for injury is an appropriate nursing diagnosis. The other nursing diagnoses are not appropriate for patients receiving vitamin A.

Which clinical finding would be expected in the patient with rhabdomyolysis? a. Sweating b. Dark urine c. Yellow color to the skin d. Lower extremity swelling

ANS: B A classic triad of muscle pain, weakness, and dark urine is considered typical of rhabdomyolysis. Neither sweating, yellow skin, nor lower extremity swelling is associated with rhabdomyolysis.

A 10-year-old presents with headache and seizures. CT scan reveals a tumor that is located near the pituitary gland. Which of the following is the most likely tumor type? a. Astrocytoma b. Craniopharyngioma c. Ependymoma d. Medulloblastoma

ANS: B A craniopharyngioma originates from the pituitary gland or hypothalamus. Symptoms include headache, seizures, diabetes insipidus, early onset of puberty, and growth delay. Astrocytomas are located on the surface of the right or left cerebellar hemisphere and cause unilateral symptoms (occurring on the same side as the tumor), such as head tilt, limb ataxia, and nystagmus. Ependymoma develops in the fourth ventricle and arises from the ependymal cells that line the ventricular system. Medulloblastoma occurs as an invasive malignant tumor that develops in the vermis of the cerebellum and may extend into the fourth ventricle. Because both tumors are located in the posterior fossa region along the midline, presenting signs and symptoms are similar and are usually related to hydrocephalus and increased intracranial pressure. REF: p. 434

The health care provider prescribes finasteride (Proscar) for a patient who has benign prostatic hyperplasia (BPH). When teaching the patient about the drug, the nurse informs him that a. he should change position from lying to standing slowly to avoid dizziness. b. his interest in sexual activity may decrease while he is taking the medication. c. improvement in the obstructive symptoms should occur within about 2 weeks. d. he will need to monitor his blood pressure frequently to assess for hypertension.

ANS: B A decrease in libido is a side effect of finasteride because of the androgen suppression that occurs with the drug. Although orthostatic hypotension may occur if the patient is also taking a medication for erectile dysfunction, it should not occur with finasteride alone. Improvement in symptoms of obstruction takes about 6 months. The medication does not cause hypertension.

A client is admitted to the hospital with a transverse fracture of the femur. Which statement best describes this type of fracture? a. The fracture line is parallel to the bone. b. The fracture line is straight across the bone. c. The fracture line is perpendicular to the bone. d. The fracture line is vertical to the shaft of the bone.

ANS: B A transverse fracture occurs straight across the bone. Such a fracture would not occur either perpendicularly or vertically in relationship to the bone. A linear fracture runs parallel to the long axis of the bone.

Osteoarthritis, a degenerative joint disease, is characterized by loss of: a. the epiphyses. b. articular cartilage. c. synovial fluid. d. the joint capsule.

ANS: B Osteoarthritis is caused by loss of the articular cartilage. It is not associated with the epiphyses of bones, synovial fluid, or the joint capsule.

22. A 36-year-old who has a diagnosis of fibrocystic breast changes calls the nurse in the clinic with symptoms. Which is most important to report to the health care provider? a. There is yellow-green discharge from the patients right nipple. b. There is an area on the breast that is hot, pink, and tender to touch. c. The lumps are firm and most are in the upper outer breast quadrants. d. The lumps are larger and painful before the patients menstrual period.

ANS: B An area that is hot or pink suggests an infectious process such as mastitis, which would require further assessment and treatment. The other information also will be reported, but these findings are typical in fibrocystic breasts. DIF: Cognitive Level: Apply (application) OBJ: Special Questions: Prioritization TOP: Nursing Process: Implementation MSC: NCLEX: Physiological Integrity

The interprofessional core team includes members from a. nursing, medicine, pharmacy, and nutrition. b. medicine, nursing, social work, and clergy. c. medicine, nursing, physical therapists, and volunteers. d. nursing, home health aides, volunteers, and clergy.

ANS: B An interprofessional team approach involving health care professionals from different disciplines is central to optimal palliative care practice and quality outcomes. The interdisciplinary core team includes members from medicine, nursing, social work, and clergy. Ancillary disciplines are also included.

An oncologist is discussing when a cancer cell loses differentiation. Which of the following is the oncologist describing? a. Autonomy b. Anaplasia c. Pleomorphic d. Metastasis

ANS: B Anaplasia, not autonomy, is the loss of differentiation. The term pleomorphic refers to a marked variability of size and shape. A malignant tumor has the ability to spread far beyond the tissue of origin by the process of metastasis.

19. Before administration of captopril to a patient with stage 2 chronic kidney disease (CKD), the nurse will check the patient's a. glucose. b. potassium. c. creatinine. d. phosphate.

ANS: B Angiotensin-converting enzyme (ACE) inhibitors are frequently used in patients with CKD because they delay the progression of the CKD, but they cause potassium retention. Therefore careful monitoring of potassium levels is needed in patients who are at risk for hyperkalemia. The other laboratory values would also be monitored in patients with CKD but would not affect whether the captopril was given or not. DIF: Cognitive Level: Apply (application) REF: 1075 TOP: Nursing Process: Assessment MSC: NCLEX: Physiological Integrity

8. Which nursing intervention will be included in the plan of care after a patient with a right femur fracture has a hip spica cast applied? a. Avoid placing the patient in prone position. b. Ask the patient about abdominal discomfort. c. Discuss remaining on bed rest for several weeks. d. Use the cast support bar to reposition the patient.

ANS: B Assessment of bowel sounds, abdominal pain, and nausea and vomiting will detect the development of cast syndrome. To avoid breakage, the support bar should not be used for repositioning. After the cast dries, the patient can begin ambulating with the assistance of physical therapy personnel and may be turned to the prone position. DIF: Cognitive Level: Apply (application) REF: 1516 TOP: Nursing Process: Planning MSC: NCLEX: Physiological Integrity

A nurse is preparing to teach about brain tumors. Which information should the nurse include? The most common type of brain tumor in children is: a. Neuroblastoma b. Astrocytoma c. Meningioma d. Germ cell

ANS: B Astrocytoma is the most common type of brain tumor in children. REF: p. 432

27. A 42-yr-old patient admitted with acute kidney injury due to dehydration has oliguria, anemia, and hyperkalemia. Which prescribed action should the nurse take first? a. Insert a urinary retention catheter. b. Place the patient on a cardiac monitor. c. Administer epoetin alfa (Epogen, Procrit). d. Give sodium polystyrene sulfonate (Kayexalate).

ANS: B Because hyperkalemia can cause fatal cardiac dysrhythmias, the initial action should be to monitor the cardiac rhythm. Kayexalate and Epogen will take time to correct the hyperkalemia and anemia. The catheter allows monitoring of the urine output but does not correct the cause of the renal failure. DIF: Cognitive Level: Analyze (analysis) REF: 1073 OBJ: Special Questions: Prioritization TOP: Nursing Process: Implementation MSC: NCLEX: Physiological Integrity

11. The nurse is providing patient teaching about the oral bisphosphonate alendronate (Fosamax). Which statement by the patient indicates a good understanding of when this drug should be taken? a. "I will take it in the evening just before bedtime." b. "I will take it in the morning with an 8-ounce glass of water." c. "I will take it with the first bite of the morning meal." d. "I will take it between meals on an empty stomach."

ANS: B Bisphosphonates must be taken in the morning, with 6 to 8 ounces of plain water, to prevent esophageal erosion. In addition, the patient must sit upright for 30 minutes after taking them. DIF: COGNITIVE LEVEL: Applying (Application) REF: p. 549 TOP: NURSING PROCESS: Implementation MSC: NCLEX: Physiological Integrity: Reduction of Risk Potential

6. The nurse will instruct the patient with a fractured left radius that the cast will need to remain in place a. for several months. b. for at least 3 weeks. c. until swelling of the wrist has resolved. d. until x-rays show complete bony union.

ANS: B Bone healing starts immediately after the injury, but since ossification does not begin until 3 weeks postinjury, the cast will need to be worn for at least 3 weeks. Complete union may take up to a year. Resolution of swelling does not indicate bone healing. DIF: Cognitive Level: Apply (application) REF: 1513 TOP: Nursing Process: Implementation MSC: NCLEX: Physiological Integrity

The onset of Duchenne muscular dystrophy (DMD) most often occurs at: a. 3-6 months of age. b. preschool years. c. school age. d. the onset of puberty.

ANS: B Boys with DMD will present in the preschool years with muscle weakness, difficulty walking, and large calves. REF: p. 1046

14. Which action will the nurse take in order to evaluate the effectiveness of Buck's traction for a 62-year-old patient who has an intracapsular fracture of the right femur? a. Check peripheral pulses. b. Ask about hip pain level. c. Assess for hip contractures. d. Monitor for hip dislocation.

ANS: B Buck's traction keeps the leg immobilized and reduces painful muscle spasm. Hip contractures and dislocation are unlikely to occur in this situation. The peripheral pulses will be assessed, but this does not help in evaluating the effectiveness of Buck's traction. DIF: Cognitive Level: Apply (application) REF: 1525 TOP: Nursing Process: Evaluation MSC: NCLEX: Physiological Integrity

6. Which information will the nurse monitor in order to determine the effectiveness of prescribed calcium carbonate (Caltrate) for a patient with chronic kidney disease (CKD)? a. Blood pressure b. Phosphate level c. Neurologic status d. Creatinine clearance

ANS: B Calcium carbonate is prescribed to bind phosphorus and prevent mineral and bone disease in patients with CKD. The other data will not be helpful in evaluating the effectiveness of calcium carbonate. DIF: Cognitive Level: Apply (application) REF: 1081 TOP: Nursing Process: Evaluation MSC: NCLEX: Physiological Integrity

The nurse is teaching a class about the various chemotherapy drugs. Which of these statements explains why alkylating drugs are also called "cell cycle-nonspecific drugs"? a. They are cytotoxic during a specific cell cycle. b. They are cytotoxic in any phase of the cell cycle. c. They are effective against several types of neoplasms. d. They are more highly differentiated than cell cycle-specific drugs.

ANS: B Cell cycle-nonspecific drugs kill cancer cells during any phase of the growth cycle, whereas cell cycle-specific drugs kill cancer cells during specific phases of the cell growth cycle. The other options are incorrect. DIF: COGNITIVE LEVEL: Understanding (Comprehension) REF: p. 734 TOP: NURSING PROCESS: Evaluation MSC: NCLEX: Physiological Integrity: Reduction of Risk Potential

42. Which information obtained by the nurse about a 29-year-old patient with a lumbar vertebral compression fracture is most important to report to the health care provider? a. Patient refuses to be turned due to back pain. b. Patient has been incontinent of urine and stool. c. Patient reports lumbar area tenderness to palpation. d. Patient frequently uses oral corticosteroids to treat asthma.

ANS: B Changes in bowel or bladder function indicate possible spinal cord compression and should be reported immediately because surgical intervention may be needed. The other findings are also pertinent but are consistent with the patient's diagnosis and do not require immediate intervention. DIF: Cognitive Level: Apply (application) REF: 1528 OBJ: Special Questions: Prioritization TOP: Nursing Process: Assessment MSC: NCLEX: Physiological Integrity

When giving cisplatin (Platinol-AQ), the nurse is aware that the major dose-limiting effect of this drug is which condition? a. Alopecia b. Kidney damage c. Cardiotoxicity d. Stomatitis

ANS: B Cisplatin may cause nephrotoxicity, and the patient's renal function must be monitored closely while on this drug. Ensuring hydration will help to prevent nephrotoxicity. DIF: COGNITIVE LEVEL: Understanding (Comprehension) REF: p. 735 TOP: NURSING PROCESS: Implementation MSC: NCLEX: Physiological Integrity: Pharmacological and Parenteral Therapies

30. A female patient with chronic kidney disease (CKD) is receiving peritoneal dialysis with 2-L inflows. Which information should the nurse report promptly to the health care provider? a. The patient has an outflow volume of 1800 mL. b. The patient's peritoneal effluent appears cloudy. c. The patient's abdomen appears bloated after the inflow. d. The patient has abdominal pain during the inflow phase.

ANS: B Cloudy-appearing peritoneal effluent is a sign of peritonitis and should be reported immediately so that treatment with antibiotics can be started. The other problems can be addressed through nursing interventions such as slowing the inflow and repositioning the patient. DIF: Cognitive Level: Apply (application) REF: 1087 TOP: Nursing Process: Assessment MSC: NCLEX: Physiological Integrity

Duchenne muscular dystrophy (DMD) has a(n) _____ inheritance pattern. a. autosomal recessive b. X-linked recessive c. Y-linked dominant d. autosomal dominant

ANS: B DMD is X-linked, occurring only in boys. REF: p. 1046

15. The nurse is preparing to administer the contraceptive form of medroxyprogesterone (Depo-Provera). What route is appropriate? a. Subcutaneous b. Intramuscular c. Vaginal d. Transdermal

ANS: B Depo-Provera is a progestin-only injectable contraceptive that is given by the intramuscular route. The other options are incorrect. DIF: COGNITIVE LEVEL: Understanding (Comprehension) REF: p. 549 TOP: NURSING PROCESS: Planning MSC: NCLEX: Physiological Integrity: Pharmacological and Parenteral Therapies

28. A patient has arrived for a scheduled hemodialysis session. Which nursing action is most appropriate for the registered nurse (RN) to delegate to a dialysis technician? a. Teach the patient about fluid restrictions. b. Check blood pressure before starting dialysis. c. Assess for causes of an increase in predialysis weight. d. Determine the ultrafiltration rate for the hemodialysis.

ANS: B Dialysis technicians are educated in monitoring for blood pressure. Assessment, adjustment of the appropriate ultrafiltration rate, and patient teaching require the education and scope of practice of an RN. DIF: Cognitive Level: Apply (application) REF: 1089 OBJ: Special Questions: Delegation TOP: Nursing Process: Planning MSC: NCLEX: Safe and Effective Care Environment

15. Which information in a patient's history indicates to the nurse that the patient is not an appropriate candidate for kidney transplantation? a. The patient has type 1 diabetes. b. The patient has metastatic lung cancer. c. The patient has a history of chronic hepatitis C infection. d. The patient is infected with human immunodeficiency virus.

ANS: B Disseminated malignancies are a contraindication to transplantation. The conditions of the other patients are not contraindications for kidney transplant. DIF: Cognitive Level: Understand (comprehension) REF: 1092 TOP: Nursing Process: Assessment MSC: NCLEX: Physiological Integrity

A child is diagnosed with cerebral palsy, characterized by extreme difficulty in fine motor coordination and purposeful movement. Which of the following types of cerebral palsy is the child experiencing? a. Ataxic b. Dystonic c. Spastic d. Mixed

ANS: B Dystonic cerebral palsy is associated with extreme difficulty in fine motor coordination and purposeful movements. Ataxic cerebral palsy manifests with gait disturbances and instability. The infant with this form of cerebral palsy may have hypotonia at birth, but stiffness of the trunk muscles develops by late infancy. Spastic cerebral palsy is associated with increased muscle tone, persistent primitive reflexes, hyperactive deep tendon reflexes, clonus, rigidity of the extremities, scoliosis, and contractures. Mixed cerebral palsy may have symptoms of each of the disorders. REF: p. 429

While performing an assessment of a 2-month-old, the nurse notes a positive Ortolani click. The nurse would suspect the child has: a. a hip fracture. b. hip dysplasia. c. osteogenesis imperfecta. d. osteomyelitis.

ANS: B Ortolani click is symptomatic of developmental dysplasia of the hip. A hip fracture would be evident by shortening of the leg and external rotation. Osteogenesis imperfecta is diagnosed by fractures. Osteomyelitis is diagnosed by fever and infection. REF: p. 1039

33. The nurse is titrating the IV fluid infusion rate immediately after a patient has had kidney transplantation. Which parameter will be most important for the nurse to consider? a. Heart rate b. Urine output c. Creatinine clearance d. Blood urea nitrogen (BUN) level

ANS: B Fluid volume is replaced based on urine output after transplant because the urine output can be as high as a liter an hour. The other data will be monitored but are not the most important determinants of fluid infusion rate. DIF: Cognitive Level: Analyze (analysis) REF: 1095 OBJ: Special Questions: Prioritization TOP: Nursing Process: Implementation MSC: NCLEX: Physiological Integrity

An oral iron supplement is prescribed for a patient. The nurse would question this order if the patient's medical history includes which condition? a. Decreased hemoglobin b. Hemolytic anemia c. Weakness d. Concurrent therapy with erythropoietics

ANS: B Hemolytic anemia is a contraindication to the use of iron supplements. Decreased hemoglobin and weakness are related to iron-deficiency anemia. Iron supplements are given with erythropoietic drugs to aid in the production of red blood cells.

A cancer patient is receiving drug therapy with epoetin alfa (Epogen). The nurse knows that the medication must be stopped if which laboratory result is noted? a. White blood cell count of 550 cells/mm3 b. Hemoglobin level of 12 g/dL c. Potassium level of 4.2 mEq/L d. Glucose level of 78 mg/dL

ANS: B If epoetin is continued when hemoglobin levels are above 11 g/dL, patients may experience serious adverse events, including heart attack, stroke, and death. Guidelines now recommend that the drug be stopped when the hemoglobin level reaches 10 g/dL for cancer patients. For renal patients, the target hemoglobin level is 11 g/dL for patients on dialysis and 10 g/dL for chronic renal patients not on dialysis.

29. A young man arrives in the emergency department with ankle swelling and severe pain after twisting his ankle playing basketball. Which of these prescribed collaborative interventions will the nurse implement first? a. Take the patient to have x-rays. b. Wrap the ankle and apply an ice pack. c. Administer naproxen (Naprosyn) 500 mg PO. d. Give acetaminophen with codeine (Tylenol #3).

ANS: B Immediate care after a sprain or strain injury includes the application of cold and compression to the injury to minimize swelling. The other actions should be taken after the ankle is wrapped with a compression bandage and ice is applied. DIF: Cognitive Level: Apply (application) REF: 1507 OBJ: Special Questions: Prioritization TOP: Nursing Process: Implementation MSC: NCLEX: Physiological Integrity

A nurse is giving an example of inflammation as an etiology for cancer development. What is the best example the nurse should give? a. Pneumonia and lung cancer b. Ulcerative colitis and colon cancer c. Prostatic hypertrophy and prostate cancer d. Hypercholesterolemia and leukemia

ANS: B Individuals with a 10+ year history of ulcerative colitis have a 30-fold increase in developing colon cancer. There is no relationship between pneumonia and lung cancer; between prostatic hypertrophy and cancer of the prostate; and between hypercholesterolemia and leukemia. REF: pp. 248-249

6. A 58-year-old woman tells the nurse, I understand that I have stage II breast cancer and I need to decide on a surgery, but I feel overwhelmed. What do you think I should do? Which response by the nurse is best? a. I would have a lumpectomy, but you need to decide what is best for you. b. Tell me what you understand about the surgical options that are available. c. It would not be appropriate for me to make a decision about your health. d. There is no need to make a decision rapidly; you have time to think about this.

ANS: B Inquiring about the patients understanding shows the nurses willingness to assist the patient with the decision- making process without imposing the nurses values or opinions. Treatment decisions for breast cancer do need to be made relatively quickly. Imposing the nurses opinions or showing an unwillingness to discuss the topic could cut off communication. DIF: Cognitive Level: Apply (application) TOP: Nursing Process: Implementation MSC: NCLEX: Psychosocial Integrity

26. A patient is being discharged 4 days after hip replacement surgery using the posterior approach. Which patient action requires immediate intervention by the nurse? a. The patient uses crutches with a swing-to gait. b. The patient leans over to pull shoes and socks on. c. The patient sits straight up on the edge of the bed. d. The patient bends over the sink while brushing teeth.

ANS: B Leaning over would flex the hip at greater than 90 degrees and predispose the patient to hip dislocation. The other patient actions are appropriate and do not require any immediate action by the nurse to protect the patient. DIF: Cognitive Level: Apply (application) REF: 1526 TOP: Nursing Process: Assessment MSC: NCLEX: Physiological Integrity

The nurse is administering liquid oral iron supplements. Which intervention is appropriate when administering this medication? a. Have the patient take the liquid iron with milk. b. Instruct the patient to take the medication through a plastic straw. c. Have the patient sip the medication slowly. d. Have the patient drink the medication, undiluted, from the unit-dose cup.

ANS: B Liquid oral forms of iron need to be taken through a plastic straw to avoid discoloration of tooth enamel. Milk may decrease absorption.

The nurse cares for a critically ill patient in the intensive care unit (ICU). Which intervention should the nurse include in the plan of care to improve this patient's sleep quality? a. Ask all visitors to leave the ICU for the night. b. Lower the level of lights from 8:00 PM until 7:00 AM. c. Avoid the use of opioids for pain relief during the evening hours. d. Schedule assessments to allow at least 4 hours of uninterrupted sleep.

ANS: B Lowering the level of light will help mimic normal day/night patterns and maximize the opportunity for sleep. Although frequent assessments and opioid use can disturb sleep patterns, these actions are necessary for the care of critically ill patients. For some patients, having a family member or friend at the bedside may decrease anxiety and improve sleep

A 9-month-old was diagnosed with osteogenesis imperfecta (OI). This disease is caused by: a. uterine teratogens. b. a genetic defect. c. malnutrition. d. trauma.

ANS: B OI (brittle bone disease) is a spectrum of disease caused by genetic mutation in the gene that encodes for type I collagen, the main component of bone and blood vessels. OI (brittle bone disease) is not due to teratogens, malnutrition, or a fracture. REF: p. 1039

Osgood-Schlatter disease causes inflammation of the: a. shoulder joint. b. patellar tendon. c. elbow ligaments. d. hip cartilage.

ANS: B Osgood-Schlatter affects the patella, not the shoulder, elbow, or hip. REF: pp. 1044-1045

19. The day after a having a right below-the-knee amputation, a patient complains of pain in the right foot. Which action is best for the nurse to take? a. Explain the reasons for the phantom limb pain. b. Administer prescribed analgesics to relieve the pain. c. Loosen the compression bandage to decrease incisional pressure. d. Inform the patient that this phantom pain will diminish over time.

ANS: B Phantom limb sensation is treated like any other type of postoperative pain would be treated. Explanations of the reason for the pain may be given, but the nurse should still medicate the patient. The compression bandage is left in place except during physical therapy or bathing. Although the pain may decrease over time, it still requires treatment now. DIF: Cognitive Level: Understand (comprehension) REF: 1532 TOP: Nursing Process: Implementation MSC: NCLEX: Physiological Integrity

5. A 22-year-old tennis player has an arthroscopic repair of a rotator cuff injury performed in same-day surgery. When the nurse plans postoperative teaching for the patient, which information will be included? a. "You will not be able to serve a tennis ball again." b. "You will work with a physical therapist tomorrow." c. "The doctor will use the drop-arm test to determine the success of surgery." d. "Leave the shoulder immobilizer on for the first 4 days to minimize pain."

ANS: B Physical therapy after a rotator cuff repair begins on the first postoperative day to prevent "frozen shoulder." A shoulder immobilizer is used immediately after the surgery, but leaving the arm immobilized for several days would lead to loss of range of motion (ROM). The drop-arm test is used to test for rotator cuff injury, but not after surgery. The patient may be able to return to pitching after rehabilitation. DIF: Cognitive Level: Apply (application) REF: 1510 TOP: Nursing Process: Planning MSC: NCLEX: Physiological Integrity

A nurse is preparing to teach about the most common defects of neural tube closure. Which one should the nurse discuss? a. Anterior b. Posterior c. Lateral d. Superior

ANS: B Posterior neural tube defects are the most common. Neural tube defects associated with the other options are not as common. REF: p. 424

Researchers now believe that rheumatoid arthritis (RA) is a result of: a. an ineffective antiviral agent. b. an autoimmune disease. c. a complication of rheumatic fever. d. a superficial joint injury.

ANS: B RA is now thought to be an autoimmune disease. There is no proof that RA is associated with ineffective antiviral therapy, rheumatic fever, or joint injuries.

18. Which patient statement indicates that the nurses teaching about tamoxifen (Nolvadex) has been effective? a. I can expect to have leg cramps. b. I will call if I have any eye problems. c. I should contact you if I have hot flashes. d. I will be taking the medication for 6 to 12 months.

ANS: B Retinopathy, cataracts, and decreased visual acuity should be immediately reported because it is likely that the tamoxifen will be discontinued or decreased. Tamoxifen treatment generally lasts 5 years. Hot flashes are an expected side effect of tamoxifen. Leg cramps may be a sign of deep vein thrombosis, and the patient should immediately notify the health care provider if pain occurs. DIF: Cognitive Level: Apply (application) TOP: Nursing Process: Evaluation MSC: NCLEX: Physiological Integrity

The nurse manager of a medical/surgical unit wants to improve the alertness of nurses who work the night shift. Which action will be the most helpful? a. Arrange for older staff members to work most night shifts. b. Provide a sleeping area for staff to use for napping at night. c. Post reminders about the relationship of sleep and alertness. d. Schedule nursing staff to rotate day and night shifts monthly.

ANS: B Short on-site naps will improve alertness. Rotating shifts causes the most disruption in sleep habits. Reminding staff members about the impact of lack of sleep on alertness will not improve sleep or alertness. It is not feasible to schedule nurses based on their ages

17. Which information should the nurse include in teaching a patient who is scheduled for external beam radiation to the breast? a. The radiation therapy will take a week to complete. b. Careful skin care in the radiated area will be necessary. c. Visitors are restricted until the radiation therapy is completed. d. Wigs may be used until the hair regrows after radiation therapy.

ANS: B Skin care will be needed because of the damage caused to the skin by the radiation. External beam radiation is done over a 5- to 6-week period. Scalp hair loss does not occur with breast radiation therapy. Because the patient does not have radioactive implants, no visitor restrictions are necessary. DIF: Cognitive Level: Apply (application) TOP: Nursing Process: Implementation MSC: NCLEX: Physiological Integrity

8. During a follow-up visit, a patient who has been on estrogen therapy admits that she has continued to smoke cigarettes. The nurse will remind the patient that smoking while on estrogen may lead to increased: a. incidence of nausea. b. risk for thrombosis. c. levels of triglycerides. d. tendency to bleed during menstruation.

ANS: B Smoking should be avoided during estrogen therapy because it adds to the risk for thrombosis formation. The other options are incorrect. DIF: COGNITIVE LEVEL: Applying (Application) REF: p. 537 TOP: NURSING PROCESS: Planning MSC: NCLEX: Physiological Integrity: Reduction of Risk Potential

A 35-year-old suffers a broken clavicle following a motor vehicle accident. X-ray reveals that the bone surfaces in the joint partially lost contact with each other. This condition is called: a. dislocation. b. subluxation. c. distortion. d. nonunion.

ANS: B Subluxation occurs when contact between the opposing joint surfaces of a fracture are partially lost. Dislocation is the displacement of one or more bones in a joint in which the opposing joint surfaces lose contact entirely. Distortion is not a term applicable to fracture healing. Nonunion is failure of the bone ends to grow together.

39. When assessing for Tinel's sign in a patient with possible right-sided carpal tunnel syndrome, the nurse will ask the patient about a. weakness in the right little finger. b. tingling in the right thumb and fingers. c. burning in the right elbow and forearm. d. tremor when gripping with the right hand.

ANS: B Testing for Tinel's sign will cause tingling in the thumb and first three fingers of the affected hand in patients who have carpal tunnel syndrome. The median nerve does not innervate the right little finger or elbow and forearm. Tremor is not associated with carpal tunnel syndrome. DIF: Cognitive Level: Understand (comprehension) REF: 1509 TOP: Nursing Process: Assessment MSC: NCLEX: Physiological Integrity

13. A patient with a complex pelvic fracture from a motor vehicle crash is on bed rest. Which nursing assessment finding is important to report to the health care provider? a. The patient states that the pelvis feels unstable. b. Abdomen is distended and bowel sounds are absent. c. There are ecchymoses across the abdomen and hips. d. The patient complains of pelvic pain with palpation.

ANS: B The abdominal distention and absent bowel sounds may be due to complications of pelvic fractures such as paralytic ileus or hemorrhage or trauma to the bladder, urethra, or colon. Pelvic instability, abdominal pain with palpation, and abdominal bruising would be expected with this type of injury. DIF: Cognitive Level: Apply (application) REF: 1524 TOP: Nursing Process: Assessment MSC: NCLEX: Physiological Integrity

The cause of Paget disease is: a. fluoride deficiency. b. unknown at present. c. excess vitamin A. d. osteogenic sarcoma.

ANS: B The cause of Paget disease is unknown, but studies have implicated both genetic and environmental factors. There is no known association between Paget disease and fluoride deficiency, excess vitamin A, or osteosarcoma.

31. The nurse is assessing a patient 4 hours after a kidney transplant. Which information is most important to communicate to the health care provider? a. The urine output is 900 to 1100 mL/hr. b. The patient's central venous pressure (CVP) is decreased. c. The patient has a level 7 (0- to 10-point scale) incisional pain. d. The blood urea nitrogen (BUN) and creatinine levels are elevated.

ANS: B The decrease in CVP suggests hypovolemia, which must be rapidly corrected to prevent renal hypoperfusion and acute tubular necrosis. The other information is not unusual in a patient after a transplant. DIF: Cognitive Level: Analyze (analysis) REF: 1095 OBJ: Special Questions: Prioritization TOP: Nursing Process: Assessment MSC: NCLEX: Physiological Integrity

When considering the trajectory of a specific disease, what is the most important concept? a. Hospital admissions b. Physical functioning c. Quality of life d. Symptom management

ANS: B The disease trajectory occurs from the onset of a life-limited diagnosis until death. Physical functioning determines the decline in the patient's physical status. Decline in status is used to determine when to intervene with palliative and end-of-life care.

23. The nurse notes bilateral enlargement of the breasts during examination of a 62-year-old man. Which action should the nurse take first? a. Teach the patient how to palpate the breast tissue for lumps. b. Question the patient about medications being currently used. c. Refer the patient for mammography and biopsy of the breast tissue. d. Explain that this is a temporary condition due to hormonal changes.

ANS: B The first action should be further assessment. Because gynecomastia is a possible side effect of drug therapy, asking about the current drug regimen is appropriate. The other actions may be needed, depending on the data that are obtained with further assessment. DIF: Cognitive Level: Apply (application) OBJ: Special Questions: Prioritization TOP: Nursing Process: Implementation MSC: NCLEX: Physiological Integrity

25. A 62-yr-old female patient has been hospitalized for 4 days with acute kidney injury (AKI) caused by dehydration. Which information will be most important for the nurse to report to the health care provider? a. The creatinine level is 3.0 mg/dL. b. Urine output over an 8-hour period is 2500 mL. c. The blood urea nitrogen (BUN) level is 67 mg/dL. d. The glomerular filtration rate is less than 30 mL/min/1.73 m2.

ANS: B The high urine output indicates a need to increase fluid intake to prevent hypovolemia. The other information is typical of AKI and will not require a change in therapy. DIF: Cognitive Level: Analyze (analysis) REF: 1072 OBJ: Special Questions: Prioritization TOP: Nursing Process: Assessment MSC: NCLEX: Physiological Integrity

23. A 25-yr-old male patient has been admitted with a severe crushing injury after an industrial accident. Which laboratory result will be most important to report to the health care provider? a. Serum creatinine level of 2.1 mg/dL b. Serum potassium level of 6.5 mEq/L c. White blood cell count of 11,500/µL d. Blood urea nitrogen (BUN) of 56 mg/dL

ANS: B The hyperkalemia associated with crushing injuries may cause cardiac arrest and should be treated immediately. The nurse also will report the other laboratory values, but abnormalities in these are not immediately life threatening. DIF: Cognitive Level: Analyze (analysis) REF: 1072 OBJ: Special Questions: Prioritization TOP: Nursing Process: Assessment MSC: NCLEX: Physiological Integrity

17. When doing discharge teaching for a 19-year-old patient who has had a repair of a fractured mandible, the nurse will include information about a. administration of nasogastric tube feedings. b. how and when to cut the immobilizing wires. c. the importance of high-fiber foods in the diet. d. the use of sterile technique for dressing changes.

ANS: B The jaw will be wired for stabilization, and the patient should know what emergency situations require that the wires be cut to protect the airway. There are no dressing changes for this procedure. The diet is liquid, and patients are not able to chew high-fiber foods. Initially, the patient may receive nasogastric tube feedings, but by discharge, the patient will swallow liquid through a straw. DIF: Cognitive Level: Apply (application) REF: 1529-1530 TOP: Nursing Process: Implementation MSC: NCLEX: Physiological Integrity

A 3-year-old has been diagnosed with bacterial meningitis. What should the nurse expect to find on the lab report for the most common cause of this bacterial meningitis? a. Haemophilus influenzae b. Neisseria meningitidis c. Streptococcus pneumonia d. Escherichia coli

ANS: B The most common cause of bacterial meningitis in children under 4 is Neisseria meningitidis. Haemophilus influenzae formerly was the most common cause of bacterial meningitis, but vaccines have decreased this. Streptococcus pneumonia can cause meningitis and is the most common cause of bacterial meningitis in children under 23 months, while Escherichia coli is the most common cause of meningitis in the newborn. REF: p. 431

22. Which intervention will be included in the plan of care for a patient with acute kidney injury (AKI) who has a temporary vascular access catheter in the left femoral vein? a. Start continuous pulse oximetry. b. Restrict physical activity to bed rest. c. Restrict the patient's oral protein intake. d. Discontinue the urethral retention catheter.

ANS: B The patient with a femoral vein catheter must be on bed rest to prevent trauma to the vein. Protein intake is likely to be increased when the patient is receiving dialysis. The retention catheter is likely to remain in place because accurate measurement of output will be needed. There is no indication that the patient needs continuous pulse oximetry. DIF: Cognitive Level: Apply (application) REF: 1088 TOP: Nursing Process: Planning MSC: NCLEX: Physiological Integrity

14. A patient who is taking the bisphosphonate alendronate (Fosamax) has been instructed to lie flat in bed for 2 days after having ophthalmic surgery. Which intervention is appropriate at this time? a. She will continue to take the alendronate with water. b. She cannot take the alendronate until she can sit up for 30 minutes. c. She can take the medication with breakfast. d. She will stop taking the medication 72 hours before her surgery.

ANS: B The nurse must emphasize that the patient should remain upright in either a standing or sitting position for approximately 30 minutes after taking a bisphosphonate so as to help prevent esophageal erosion or irritation. Because this patient will be required to lie flat in bed for 2 days after the surgery, the prescriber will need to be notified that the patient cannot take the medication during this time. DIF: COGNITIVE LEVEL: Applying (Application) REF: p. 549 TOP: NURSING PROCESS: Implementation MSC: NCLEX: Physiological Integrity: Reduction of Risk Potential

16. A patient who has had an open reduction and internal fixation (ORIF) of a hip fracture tells the nurse that he is ready to get out of bed for the first time. Which action should the nurse take? a. Use a mechanical lift to transfer the patient from the bed to the chair. b. Check the postoperative orders for the patient's weight-bearing status. c. Avoid administration of pain medications before getting the patient up. d. Delegate the transfer of the patient to nursing assistive personnel (NAP).

ANS: B The nurse should be familiar with the weight-bearing orders for the patient before attempting the transfer. Mechanical lifts are not typically needed after this surgery. Pain medications should be given because the movement is likely to be painful for the patient. The registered nurse (RN) should supervise the patient during the initial transfer to evaluate how well the patient is able to accomplish this skill. DIF: Cognitive Level: Apply (application) REF: 1528 TOP: Nursing Process: Implementation MSC: NCLEX: Physiological Integrity

35. A 74-yr-old patient who is progressing to stage 5 chronic kidney disease asks the nurse, "Do you think I should go on dialysis? Which initial response by the nurse is best? a. "It depends on which type of dialysis you are considering." b. "Tell me more about what you are thinking regarding dialysis." c. "You are the only one who can make the decision about dialysis." d. "Many people your age use dialysis and have a good quality of life."

ANS: B The nurse should initially clarify the patient's concerns and questions about dialysis. The patient is the one responsible for the decision, and many people using dialysis do have good quality of life, but these responses block further assessment of the patient's concerns. Referring to which type of dialysis the patient might use only indirectly responds to the patient's question. DIF: Cognitive Level: Analyze (analysis) REF: 1091 OBJ: Special Questions: Prioritization TOP: Nursing Process: Assessment MSC: NCLEX: Psychosocial Integrity

37. A patient undergoes a left above-the-knee amputation with an immediate prosthetic fitting. When the patient arrives on the orthopedic unit after surgery, the nurse should a. place the patient in a prone position. b. check the surgical site for hemorrhage. c. remove the prosthesis and wrap the site. d. keep the residual leg elevated on a pillow.

ANS: B The nurse should monitor for hemorrhage after the surgery. The prosthesis will not be removed. To avoid flexion contracture of the hip, the leg will not be elevated on a pillow. The patient is placed in a prone position after amputation to prevent hip flexion, but this would not be done during the immediate postoperative period. DIF: Cognitive Level: Apply (application) REF: 1532 TOP: Nursing Process: Implementation MSC: NCLEX: Physiological Integrity

20. Which statement by a 62-year-old patient who has had an above-the-knee amputation indicates that the nurse's discharge teaching has been effective? a. "I should elevate my residual limb on a pillow 2 or 3 times a day." b. "I should lay flat on my abdomen for 30 minutes 3 or 4 times a day." c. "I should change the limb sock when it becomes soiled or each week." d. "I should use lotion on the stump to prevent skin drying and cracking."

ANS: B The patient lies in the prone position several times daily to prevent flexion contractures of the hip. The limb sock should be changed daily. Lotion should not be used on the stump. The residual limb should not be elevated because this would encourage flexion contracture. DIF: Cognitive Level: Apply (application) REF: 1532 TOP: Nursing Process: Evaluation MSC: NCLEX: Physiological Integrity

10. The nurse provides discharge teaching for a 61-year-old patient who has had a left modified radical mastectomy and lymph node dissection. Which statement by the patient indicates that teaching has been successful? a. I will need to use my right arm and to rest the left one. b. I will avoid reaching over the stove with my left hand. c. I will keep my left arm in a sling until the incision is healed. d. I will stop the left arm exercises if moving the arm is painful.

ANS: B The patient should avoid any activity that might injure the left arm, such as reaching over a burner. If the left arm exercises are painful, analgesics should be used and the exercises continued in order to restore strength and range of motion. The left arm should be elevated at or above heart level and should be used to improve range of motion and function. DIF: Cognitive Level: Apply (application) TOP: Nursing Process: Evaluation MSC: NCLEX: Physiological Integrity

38. Before assisting a patient with ambulation 2 days after a total hip replacement, which action is most important for the nurse to take? a. Observe the status of the incisional drain device. b. Administer the ordered oral opioid pain medication. c. Instruct the patient about the benefits of ambulation. d. Change the hip dressing and document the wound appearance.

ANS: B The patient should be adequately medicated for pain before any attempt to ambulate. Instructions about the benefits of ambulation may increase the patient's willingness to ambulate, but decreasing pain with ambulation is more important. The presence of an incisional drain or timing of dressing change will not affect ambulation. DIF: Cognitive Level: Apply (application) REF: 1526 OBJ: Special Questions: Prioritization TOP: Nursing Process: Implementation MSC: NCLEX: Physiological Integrity

11. A 33-year-old patient has a saline breast implant inserted in the outpatient surgery area. Which instruction will the nurse include in the discharge teaching? a. Take aspirin every 4 hours to reduce inflammation. b. Check wound drains for excessive blood or a foul odor. c. Wear a loose-fitting bra to decrease irritation of the sutures. d. Resume normal activities 2 to 3 days after the mammoplasty.

ANS: B The patient should be taught drain care because the drains will be in place for 2 or 3 days after surgery. Normal activities can be resumed after 2 to 3 weeks. A bra that provides good support is typically ordered. Aspirin will decrease coagulation and is typically not given after surgery. DIF: Cognitive Level: Apply (application) TOP: Nursing Process: Implementation MSC: NCLEX: Physiological Integrity

6. The nurse is reviewing the use of uterine tocolytics, such as indomethacin (Indocin). Which statement best describes the indication for these drugs? a. Prevention of preterm labor in the 15th week of pregnancy b. Prevention of preterm labor in the 22nd week of pregnancy c. Stimulation of contractions in prolonged labor d. Stimulation of ovulation as part of infertility treatments

ANS: B Tocolytics relax uterine smooth muscles and stop the uterus from contracting and are used along with nonpharmacologic measures to prevent preterm labor between 20 and 37 weeks of pregnancy. DIF: COGNITIVE LEVEL: Applying (Application) REF: p. 546 TOP: NURSING PROCESS: Planning MSC: NCLEX: Physiological Integrity: Pharmacological and Parenteral Therapies

The chronic stage of gout, characterized by crystalline deposits in cartilage, synovial membranes, and soft tissue, is called: a. monoarticular arthritis. b. tophaceous gout. c. asymptomatic hyperuricemia. d. complicated gout.

ANS: B Tophaceous gout is the third and chronic stage of the disease. It is characterized by a progressive inability to excrete uric acid, which expands the urate pool. This causes urate crystal deposits (tophi) to appear in cartilage, synovial membranes, tendons, and soft tissue. The characteristic crystalline deposits are not associated with monarticular or complicated gout. Asymptomatic hyperuricemia would not lead to crystalline deposits.

What is the cause of true (primary) microcephaly? a. Viral infection b. An autosomal recessive gene c. Fetal trauma d. Hydrocephalus

ANS: B True (primary) microcephaly is usually caused by an autosomal recessive genetic or chromosomal defect. Viral infections and fetal trauma can lead to microcephaly but are not the true cause. Hydrocephalus does not lead to microcephaly. REF: p. 427

A 25-year-old male develops a tumor of the breast glandular tissue. What type of tumor will be documented on the chart? a. Carcinoma b. Adenocarcinoma c. Sarcoma d. Lymphoma

ANS: B Tumors that arise from or form ductal or glandular structures are named adenocarcinomas. Cancers arising in epithelial tissue are called carcinomas; mesenchymal tissue (including connective tissue, muscle, and bone) usually have the suffix sarcoma; lymphatic tissue are called lymphomas.

What is the most common location of brain tumors in children? a. Above the cerebellum b. In the posterior fossa c. In the anterior cerebrum d. In the ventricular lining

ANS: B Two thirds of all pediatric brain tumors are found in the posterior fossa, not above the cerebellum, in the cerebrum, or in the ventricular lining. REF: p. 432

45. When caring for a patient who is using Buck's traction after a hip fracture, which action can the nurse delegate to unlicensed assistive personnel (UAP)? a. Monitor the skin under the traction boot for redness. b. Ensure that the weight for the traction is off the floor. c. Check for intact sensation and movement in the affected leg. d. Offer reassurance that hip and leg pain are normal after hip fracture.

ANS: B UAP can be responsible for maintaining the integrity of the traction once it has been established. Assessment of skin integrity and circulation should be done by the registered nurse (RN). UAP should notify the RN if the patient experiences hip and leg pain because pain and effectiveness of pain relief measures should be assessed by the RN. DIF: Cognitive Level: Apply (application) REF: 1514 OBJ: Special Questions: Delegation TOP: Nursing Process: Planning MSC: NCLEX: Safe and Effective Care Environment

9. A patient has a long-arm plaster cast applied for immobilization of a fractured left radius. Until the cast has completely dried, the nurse should a. keep the left arm in dependent position. b. avoid handling the cast using fingertips. c. place gauze around the cast edge to pad any roughness. d. cover the cast with a small blanket to absorb the dampness.

ANS: B Until a plaster cast has dried, using the palms rather than the fingertips to handle the cast helps prevent creating protrusions inside the cast that could place pressure on the skin. The left arm should be elevated to prevent swelling. The edges of the cast may be petaled once the cast is dry, but padding the edges before that may cause the cast to be misshapen. The cast should not be covered until it is dry because heat builds up during drying. DIF: Cognitive Level: Apply (application) REF: 1515 TOP: Nursing Process: Implementation MSC: NCLEX: Physiological Integrity

It is true that viral meningitis: a. is always sudden in onset. b. often occurs with measles, mumps, or herpes. c. is effectively treated with antibiotics. d. causes increased sugar in the cerebral spinal fluid (CSF).

ANS: B Viral meningitis may result from a direct infection of a virus, or it may be secondary to disease, such as measles, mumps, or herpes. Onset may be sudden or gradual. Treatment is symptomatic; antibiotics are not used. Glucose in the CSF is normal. REF: p. 431

1. An adult patient who arrived at the triage desk in the emergency department (ED) with minor facial lacerations after a motor vehicle accident has a blood pressure (BP) of 182/94. Which action by the nurse is appropriate? a. Start an IV line to administer antihypertensive medications. b. Recheck the blood pressure after the patient has been assessed. c. Discuss the need for hospital admission to control blood pressure. d. Teach the patient about the stroke risk associated with uncontrolled hypertension.

ANS: B When a patient experiences an acute stressor, the BP increases. The nurse should plan to recheck the BP after the patient has stabilized and received treatment. This will provide a more accurate indication of the patient's usual blood pressure. Elevated BP that occurs in response to acute stress does not increase the risk for health problems such as stroke, indicate a need for hospitalization, or indicate a need for IV antihypertensive medications. DIF: Cognitive Level: Apply (application) REF: 80 TOP: Nursing Process: Planning MSC: NCLEX: Physiological Integrity

A patient who has been in the hospital for several weeks is about to be discharged. The patient is weak from the hospitalization and asks the nurse to explain why this is happening. The nurse's best response is "You are weak because a. your iron level is low. This is known as anemia." b. of your immobility in the hospital. This is known as deconditioning." c. of your poor appetite. This is known as malnutrition." d. of your medications. This is known as drug induced weakness."

ANS: B When a person is ill and immobile the body becomes weak. This is known as deconditioning. Anemia, malnutrition, and medications may have an adverse effect on the body, but this is not known as deconditioning.

What is the most common type of cerebral palsy? a. Ataxic b. Dystonic c. Spastic d. Mixed

ANS: C Spastic cerebral palsy accounts for approximately 70-80% of cerebral palsy cases. Ataxic cerebral palsy accounts for 5-10% of cerebral palsy cases. Dystonic cerebral palsy accounts for 10-20% of cerebral palsy cases. Mixed cases account for 13% of cerebral palsy cases. REF: p. 429

13. A patient who has had progressive chronic kidney disease (CKD) for several years has just begun regular hemodialysis. Which information about diet will the nurse include in patient teaching? a. Increased calories are needed because glucose is lost during hemodialysis. b. More protein is allowed because urea and creatinine are removed by dialysis. c. Dietary potassium is not restricted because the level is normalized by dialysis. d. Unlimited fluids are allowed because retained fluid is removed during dialysis.

ANS: B When the patient is started on dialysis and nitrogenous wastes are removed, more protein in the diet is encouraged. Fluids are still restricted to avoid excessive weight gain and complications such as shortness of breath. Glucose is not lost during hemodialysis. Sodium and potassium intake continues to be restricted to avoid the complications associated with high levels of these electrolytes. DIF: Cognitive Level: Apply (application) REF: 1087 TOP: Nursing Process: Implementation MSC: NCLEX: Physiological Integrity

A patient with an eye injury requires an ocular examination to detect the presence of a foreign body. The nurse anticipates that which drug will be used for this examination? a. Phenylephrine (Neo-Synephrine) b. Fluorescein sodium (AK-Fluor) c. Atropine sulfate (Isopto Atropine) d. Olopatadine (Patanol)

ANS: B Fluorescein sodium is an ophthalmic diagnostic dye used to identify corneal defects and to locate foreign objects in the eye. Phenylephrine is an ocular decongestant; atropine sulfate has mydriatic and cycloplegic effects, which are useful for examining the inner eye structures; olopatadine is an ophthalmic antihistamine. DIF: COGNITIVE LEVEL: Understanding (Comprehension) REF: p. 916 TOP: NURSING PROCESS: Planning MSC: NCLEX: Physiological Integrity: Pharmacological and Parenteral Therapies

The nurse is administering antibiotic eyedrops to a patient for the first time. After the first drop is given, the patient states, "That eyedrop is making my eye sting! Is that normal?" Which is the best response by the nurse? a. "That's unusual. Let me rinse the medication from your eye." b. "Sometimes these eyedrops may cause burning and stinging, but it should go away soon." c. "These may be serious side effects, so I will notify your doctor before the next dose is due." d. "Let's wait and see if these effects happen the next time you receive these drops."

ANS: B Ocular antibiotics may cause local inflammation, burning, stinging, urticaria, and dermatitis. These effects are transient. The other options are incorrect. DIF: COGNITIVE LEVEL: Applying (Application) REF: p. 912 TOP: NURSING PROCESS: Implementation MSC: NCLEX: Physiological Integrity: Pharmacological and Parenteral Therapies

The nurse will plan to teach the patient who is incontinent of urine following a radical retropubic prostatectomy to a. restrict oral fluid intake. b. do pelvic muscle exercises. c. perform intermittent self-catheterization. d. use belladonna and opium suppositories.

ANS: B Pelvic floor muscle training (Kegel) exercises are recommended to strengthen the pelvic floor muscles and improve urinary control. Belladonna and opium suppositories are used to reduce bladder spasms after surgery. Intermittent self-catheterization may be taught before surgery if the patient has urinary retention, but it will not be useful in reducing incontinence after surgery. The patient should have a daily oral intake of 2 to 3 L.

The nurse performing a focused examination to determine possible causes of infertility will assess for a. hydrocele. b. varicocele. c. epididymitis. d. paraphimosis.

ANS: B Persistent varicoceles are commonly associated with infertility. Hydrocele, epididymitis, and paraphimosis are not risk factors for infertility.

A patient is about to undergo ocular surgery. The preoperative nurse anticipates that which drug will be used for local anesthesia? a. Oral glycerin b. Proparacaine (Alcaine) c. Timolol (Timoptic) d. Dipivefrin (Propine)

ANS: B Proparacaine (Alcaine) and tetracaine are used as a local anesthetic for ocular surgery or other procedures. The other drugs are used for glaucoma. DIF: COGNITIVE LEVEL: Understanding (Comprehension) REF: p. 916 TOP: NURSING PROCESS: Planning MSC: NCLEX: Physiological Integrity: Pharmacological and Parenteral Therapies

The nurse in the clinic notes elevated prostate specific antigen (PSA) levels in the laboratory results of these patients. Which patient's PSA result is not expected to be elevated? a. A 38-yr-old patient who is being treated for acute prostatitis b. A 48-yr-old patient whose father died of metastatic prostate cancer c. A 52-yr-old patient who goes on long bicycle rides every weekend d. A 75-yr-old patient who uses saw palmetto to treat benign prostatic hyperplasia (BPH)

ANS: B The family history of prostate cancer and elevation of PSA indicate that further evaluation of the patient for prostate cancer is needed. Elevations in PSA for the other patients are not unusual.

A 27-yr-old patient who has testicular cancer is being admitted for a unilateral orchiectomy. The patient does not talk to his wife and speaks to the nurse only to answer the admission questions. Which action is appropriate for the nurse to take? a. Teach the patient and the wife that impotence is unlikely after unilateral orchiectomy. b. Ask the patient if he has any questions or concerns about the diagnosis and treatment. c. Inform the patient's wife that concerns about sexual function are common with this diagnosis. d. Document the patient's lack of communication on the health record and continue preoperative care.

ANS: B The initial action by the nurse should be assessment for any anxiety or questions about the surgery or postoperative care. The nurse should address the patient, not the spouse, when discussing the diagnosis and any possible concerns. Without further assessment of patient concerns, the nurse should not offer teaching about complications after orchiectomy. Documentation of the patient's lack of interaction is not an adequate nursing action in this situation.

A patient who has benign prostatic hyperplasia (BPH) with urinary retention is admitted to the hospital with elevated blood urea nitrogen (BUN) and creatinine. Which prescribed therapy should the nurse implement first? a. Infuse normal saline at 50 mL/hr. b. Insert a urinary retention catheter. c. Draw blood for a complete blood count. d. Schedule pelvic magnetic resonance imaging

ANS: B The patient data indicate that the patient may have acute kidney injury caused by the BPH. The initial therapy will be to insert a catheter. The other actions are also appropriate, but they can be implemented after the acute urinary retention is resolved.

A primary care provider is attempting to diagnose cancer and is looking for a tumor marker. Which of the following could be a possible marker? a. Red blood cells b. Apoptotic cells c. Enzymes d. Neurotransmitters

ANS: C Tumor markers include hormones, enzymes, genes, antigens, and antibodies, but not red blood cells, apoptotic cells, or neurotransmitters.

The nurse will plan to teach the patient scheduled for photovaporization of the prostate (PVP) a. that urine will appear bloody for several days. b. how to care for an indwelling urinary catheter. c. that symptom improvement takes 2 to 3 weeks. d. about complications associated with urethral stenting.

ANS: B The patient will have an indwelling catheter for 24 to 48 hours and will need teaching about catheter care. There is minimal bleeding with this procedure. Symptom improvement is almost immediate after PVP. Stent placement is not included in the procedure.

A patient tells the nurse that he decided to seek treatment for erectile dysfunction (ED) because his wife "is losing patience with the situation." The nurse's follow-up questions should focus on the man's identified concern with a. low self-esteem. b. role performance. c. increased anxiety. d. infrequent intercourse.

ANS: B The patient's statement indicates that the relationship with his wife is his primary concern. Although anxiety, low self-esteem, and ineffective sexuality patterns may also be concerns, the patient information suggests that addressing the role performance problem will lead to the best outcome for this patient.

Which information will the nurse plan to include when teaching a young adult who has a family history of testicular cancer about testicular self-examination? a. Testicular self-examination should be done at least weekly. b. Testicular self-examination should be done in a warm room. c. The only structure normally felt in the scrotal sac is the testis. d. Call the health care provider if one testis is larger than the other.

ANS: B The testes will hang lower in the scrotum when the temperature is warm (e.g., during a shower), and it will be easier to palpate. The epididymis is also normally palpable in the scrotum. One testis is normally larger. Men at high risk should perform testicular self-examination monthly.

When caring for a patient with continuous bladder irrigation after having transurethral resection of the prostate, which action could the nurse delegate to unlicensed assistive personnel (UAP)? a. Teach the patient how to perform Kegel exercises. b. Report any complaints of pain or spasms to the nurse. c. Monitor for increases in bleeding or presence of clots. d. Increase the flow rate of the irrigation if clots are noted.

ANS: B UAP education and role includes reporting patient concerns to supervising nurses. Patient teaching, assessments for complications, and actions such as bladder irrigation require more education and should be done by licensed nursing staff.

(Multiple Response) According to the Center for Disease Control and Prevention (CDC) guidelines, which personal protective equipment will the nurse put on before assessing a patient who is on contact precautions for Clostridium difficile diarrhea (select all that apply)? a. Mask b. Gown c. Gloves d. Shoe covers e. Eye protection

ANS: B, C Because the nurse will have substantial contact with the patient and bedding when doing an assessment, gloves and gowns are needed. Eye protection and masks are needed for patients in contact precautions only when spraying or splashing is anticipated. Shoe covers are not recommended in the CDC guidelines.

When an aide asks why carcinomas rarely occur in childhood, how should the nurse respond? (select all that apply) a. Carcinomas cannot develop in an immunosuppressed environment. b. Carcinomas need a long time from exposure to occurrence. c. Carcinomas are due to environmental exposures. d. Carcinomas occur primarily in secondary sexual organs. e. Carcinomas are mesodermal.

ANS: B, C Carcinomas rarely occur in children because these cancers most commonly result from environmental carcinogens and require a long period from exposure to the appearance of the carcinoma. REF: p. 301

A newborn child is diagnosed with a vertebral arch defect, spina bifida. This condition would lead the nurse to suspect the child may have which of the following as well? (select all that apply) a. Type II Chiari malformation b. Myelomeningocele c. Meningocele d. Acrania e. Craniosynostosis

ANS: B, C When meningocele and myelomeningocele occur, they are related to an accompanying vertebral defect that allows the protrusion of the neural tube contents. Type II Chiari malformation is a complex malformation of the brainstem and cerebellum in which the cerebellar tonsils are displaced downward into the cervical spinal canal. This does not occur through a spina bifida. In acrania, the cranial vault is almost completely absent; an extensive defect of the vertebral column is often present. Craniosynostosis is the premature closure of one or more of the cranial sutures. REF: pp. 424-425

2. The patient asks the nurse about taking large doses of vitamin C to improve her immunity to colds. "It's just a vitamin, right? What can happen?" Which responses by the nurse are correct? (Select all that apply.) a."Vitamin C is harmless because it is a water-soluble vitamin." b."Large doses of vitamin C can cause nausea, vomiting, headache, and abdominal cramps." c.Keep in mind that if you suddenly stop taking these large doses, you might experience symptoms similar to scurvy." d. Studies have shown that vitamin C has little value in preventing the common cold." e.Vitamin C acidifies the urine, which can lead to the formation of renal stones." f."Large doses of vitamin C may delay wound healing."

ANS: B, C, D, E Vitamin C is usually nontoxic unless excessive dosages are consumed. Large doses (megadoses) can produce nausea, vomiting, headache, and abdominal cramps, and they acidify the urine, which can result in the formation of renal stones. Furthermore, individuals who discontinue taking excessive daily doses of ascorbic acid can experience scurvylike symptoms. Studies have shown that megadoses of vitamin C have little or no value as prophylaxis against the common cold. Vitamin C is required for several important metabolic activities, including collagen synthesis and the maintenance of connective tissue and tissue repair.

MULTIPLE RESPONSE 1. A patient who is hospitalized with a pelvic fracture after a motor vehicle accident just received news that the driver of the car died from multiple injuries. What actions should the nurse take based on knowledge of the physiologic stress reactions that may occur in this patient (select all that apply)? a. Assess for bradycardia. b. Observe for decreased appetite. c. Ask about epigastric discomfort. d. Monitor for decreased respiratory rate. e. Check for elevated blood glucose levels.

ANS: B, C, E The physiologic changes associated with the acute stress response can cause changes in appetite, increased gastric acid secretion, and increase blood glucose levels. In addition, stress causes an increase in respiratory and heart rates. DIF: Cognitive Level: Analyze (analysis) REF: 78 TOP: Nursing Process: Assessment MSC: NCLEX: Physiological Integrity

The nurse is caring for a patient with a diagnosed case of Clostridium difficile. The nurse expects to implement which of the following interventions? (Select all that apply.) a. Administration of protease inhibitors b. Use of personal protective equipment c. Patient teaching on methods to inhibit transmission d. Preventing visitors from entering the room e. Administration of intravenous fluids f. Strict monitoring of intake and output

ANS: B, C, E, F Protease inhibitors are used for treatment of viral infections, not bacterial infections. The nurse wants to protect visitors from exposure to the bacteria and protect the patient from secondary infection while immunocompromised, but the patient will need the support of family and close friends. Contact isolation precautions must be strictly followed along with the use of personal protective equipment and teaching on methods to inhibit transmission to help break the chain of infection. Intravenous fluids and strict intake and output monitoring will be important for the patient suffering the effects of Clostridium difficile, because it causes diarrhea with fluid loss.

The nurse is preparing to administer a barbiturate. Which conditions or disorders would be a contraindication to the use of these drugs? (Select all that apply.) a. Gout b. Pregnancy c. Epilepsy d. Severe chronic obstructive pulmonary disease e. Severe liver disease f. Diabetes mellitus

ANS: B, D, E Contraindications to barbiturates include pregnancy, significant respiratory difficulties, and severe liver disease. The other disorders are not contraindications. DIF: COGNITIVE LEVEL: Applying (Application) REF: p. 194 TOP: NURSING PROCESS: Assessment MSC: NCLEX: Physiological Integrity: Pharmacological and Parenteral Therapies

A patient in the emergency department reports being struck in the right eye with a fist. Which finding is a priority for the nurse to communicate to the health care provider? a. The patient reports a right-sided headache. b. The sclera on the right eye has broken blood vessels. c. The patient reports "a curtain" over part of the visual field. d. The area around the right eye is bruised and tender to the touch.

ANS: C

A patient who received a corneal transplant 2 weeks ago calls the ophthalmology clinic to report that his vision has not improved with the transplant. Which action should the nurse take? a. Suggest the patient arrange a ride to the clinic immediately. b. Ask about the presence of "floaters" in the patient's visual field. c. Remind the patient it may take months to restore vision after transplant. d. Teach the patient to continue using prescribed pupil-dilating medications.

ANS: C

A patient with a head injury after a motorcycle crash arrives in the emergency department (ED) reporting shortness of breath and severe eye pain. Which action will the nurse take first? a. Assess cranial nerve functions. b. Administer the prescribed analgesic. c. Check the patient's oxygen saturation. d. Examine the eye for evidence of trauma.

ANS: C

A patient with a right retinal detachment had a pneumatic retinopexy procedure. Which information will the nurse include in the discharge teaching plan? a. The use of eye patches to reduce movement of the operative eye b. The need to wear dark glasses to protect the eyes from bright light c. The purpose of maintaining the head resting in a prescribed position d. The procedure for dressing changes when the eye dressing is saturated

ANS: C

The nurse at the outpatient surgery unit obtains the following information about a patient who is scheduled for cataract extraction and implantation of an intraocular lens. Which information is important to report to the health care provider before the procedure? a. The patient has had blurred vision for 3 years. b. The patient has not eaten anything for 8 hours. c. The patient takes antihypertensive medications. d. The patient gets nauseated with general anesthesia.

ANS: C

The nurse learns that a newly admitted patient has functional blindness and that the spouse has cared for the patient for many years. What is the nurse's most important action during the initial assessment? a. Obtain more information about the cause of the patient's vision loss. b. Obtain information from the spouse about the patient's special needs. c. Make eye contact with the patient and ask about any need for assistance. d. Perform an evaluation of the patient's visual acuity using a Snellen chart.

ANS: C

What is the safest technique for the nurse to use when assisting a blind patient to ambulate to the bathroom? a. Lead the patient slowly to the bathroom, holding on to the patient by the arm. b. Stay beside the patient and describe any obstacles on the path to the bathroom. c. Walk slightly ahead of the patient, allowing the patient to hold the nurse's elbow. d. Have the patient place a hand on the nurse's shoulder and guide the patient forward.

ANS: C

What should the nurse teach a patient with recurrent staphylococcal and seborrheic blepharitis to do? a. Irrigate the eyes with saline solution. b. Schedule an appointment for eye surgery. c. Use a gentle baby shampoo to clean the eyelids. d. Apply cool compresses to the eyes three times daily.

ANS: C

Which action for a patient with neutropenia is appropriate for the registered nurse (RN) to delegate to a licensed practical/vocational nurse (LPN/LVN)? a. Assessing the patient for signs and symptoms of infection b. Teaching the patient the purpose of neutropenic precautions c. Administering subcutaneous filgrastim (Neupogen) injection d. Developing a discharge teaching plan for the patient and family

ANS: C Administration of subcutaneous medications is included in LPN/LVN education and scope of practice. Patient education, assessment, and developing the plan of care require RN level education and scope of practice.

When a patient with splenomegaly is scheduled for splenectomy, which action will the nurse include in the preoperative plan of care? a. Discourage deep breathing to reduce risk for splenic rupture. b. Teach the patient to use ibuprofen (Advil) for left upper quadrant pain. c. Schedule immunization with the pneumococcal vaccine (Pneumovax). d. Avoid the use of acetaminophen (Tylenol) for 2 weeks prior to surgery.

ANS: C Asplenic patients are at high risk for infection with Pneumococcus and immunization reduces this risk. There is no need to avoid acetaminophen use before surgery, but nonsteroidal antiinflammatory drugs (NSAIDs) may increase bleeding risk and should be avoided. The enlarged spleen may decrease respiratory depth and the patient should be encouraged to take deep breaths.

A 52-year-old patient has a new diagnosis of pernicious anemia. The nurse determines that the patient understands the teaching about the disorder when the patient states, I a. need to start eating more red meat and liver b. will stop having a glass of wine with dinner. c. could choose nasal spray rather than injections of vitamin B12. d. will need to take a proton pump inhibitor like omeprazole (Prilosec).

ANS: C Because pernicious anemia prevents the absorption of vitamin B12, this patient requires injections or intranasal administration of cobalamin. Alcohol use does not cause cobalamin deficiency. Proton pump inhibitors decrease the absorption of vitamin B12. Eating more foods rich in vitamin B12 is not helpful because the lack of intrinsic factor prevents absorption of the vitamin.

Which instruction will the nurse plan to include in discharge teaching for the patient admitted with a sickle cell crisis? a. Take a daily multivitamin with iron. b. Limit fluids to 2 to 3 quarts per day. c. Avoid exposure to crowds when possible. d. Drink only two caffeinated beverages daily.

ANS: C Exposure to crowds increases the patients risk for infection, the most common cause of sickle cell crisis. There is no restriction on caffeine use. Iron supplementation is generally not recommended. A high-fluid intake is recommended.

A patient is receiving IV antibiotics at home to treat chronic osteomyelitis of the left femur. The nurse identifies a need for additional teaching related to health maintenance when the nurse finds that the patient a. is frustrated with the length of treatment required. b. takes and records the oral temperature twice a day. c. is unable to plantar flex the foot on the affected side. d. uses crutches to avoid weight bearing on the affected leg.

ANS: C Foot drop is an indication that the foot is not being supported in a neutral position by a splint. Using crutches and monitoring the oral temperature are appropriate self-care activities. Frustration with the length of treatment is not an indicator of ineffective health maintenance of the osteomyelitis.

Which action should the nurse take before administering gentamicin (Garamycin) to a patient with acute osteomyelitis? a. Ask the patient about any nausea. b. Obtain the patient's oral temperature. c. Review the patient's serum creatinine. d. Change the prescribed wet-to-dry dressing.

ANS: C Gentamicin is nephrotoxic and can cause renal failure. Monitoring the patient's temperature before gentamicin administration is not necessary. Nausea is not a common side effect of IV gentamicin. There is no need to change the dressing before gentamicin administration.

8. Niacin is prescribed for a patient who has hyperlipidemia. The nurse checks the patient's medical history, knowing that this medication is contraindicated in which disorder? a. Renal disease b. Cardiac disease c. Liver disease d. Diabetes mellitus

ANS: C Niacin, unlike certain other B-complex vitamins, has additional contraindications besides drug allergy. They include liver disease, severe hypotension, arterial hemorrhage, and active peptic ulcer disease. The other options are incorrect.

A 54-year-old woman with acute myelogenous leukemia (AML) is considering treatment with a hematopoietic stem cell transplant (HSCT). The best approach for the nurse to assist the patient with a treatment decision is to a. emphasize the positive outcomes of a bone marrow transplant. b. discuss the need for adequate insurance to cover post-HSCT care. c. ask the patient whether there are any questions or concerns about HSCT. d. explain that a cure is not possible with any other treatment except HSCT.

ANS: C Offering the patient an opportunity to ask questions or discuss concerns about HSCT will encourage the patient to voice concerns about this treatment and also will allow the nurse to assess whether the patient needs more information about the procedure. Treatment of AML using chemotherapy is another option for the patient. It is not appropriate for the nurse to ask the patient to consider insurance needs in making this decision.

The nurse instructs a patient who has osteosarcoma of the tibia about a scheduled above-the-knee amputation. Which statement by a patient indicates additional patient teaching is needed? a. "I will need to participate in physical therapy after surgery." b. "I wish I did not need to have chemotherapy after this surgery." c. "I did not have this bone cancer until my leg broke a week ago." d. "I can use the patient-controlled analgesia (PCA) to manage postoperative pain."

ANS: C Osteogenic sarcoma may be diagnosed following a fracture, but it is not caused by the injury. The other patient statements indicate that patient teaching has been effective.

Which nursing action included in the care of a patient after laminectomy can the nurse delegate to experienced unlicensed assistive personnel (UAP)? a. Check ability to plantar and dorsiflex the foot. b. Determine the patient's readiness to ambulate. c. Log roll the patient from side to side every 2 hours. d. Ask about pain management with the patient-controlled analgesia (PCA).

ANS: C Repositioning a patient is included in the education and scope of practice of UAP, and experienced UAP will be familiar with how to maintain alignment in the postoperative patient. Evaluation of the effectiveness of pain medications, assessment of neurologic function, and evaluation of a patient's readiness to ambulate after surgery require higher level nursing education and scope of practice.

An appropriate nursing intervention for a patient who has acute low back pain and muscle spasms is to teach the patient to a. keep both feet flat on the floor when prolonged standing is required. b. twist gently from side to side to maintain range of motion in the spine. c. keep the head elevated slightly and flex the knees when resting in bed. d. avoid the use of cold packs because they will exacerbate the muscle spasms.

ANS: C Resting with the head elevated and knees flexed will reduce the strain on the back and decrease muscle spasms. Twisting from side to side will increase tension on the lumbar area. A pillow placed under the upper back will cause strain on the lumbar spine. Alternate application of cold and heat should be used to decrease pain.

Which finding about a patient with polycythemia vera is most important for the nurse to report to the health care provider? a. Hematocrit 55% b. Presence of plethora c. Calf swelling and pain d. Platelet count 450,000/mL

ANS: C The calf swelling and pain suggest that the patient may have developed a deep vein thrombosis, which will require diagnosis and treatment to avoid complications such as pulmonary embolus. The other findings will also be reported to the health care provider but are expected in a patient with this diagnosis.

Which action will the nurse include in the plan of care for a patient who has thalassemia major? a. Teach the patient to use iron supplements. b. Avoid the use of intramuscular injections. c. Administer iron chelation therapy as needed. d. Notify health care provider of hemoglobin 11g/dL.

ANS: C The frequent transfusions used to treat thalassemia major lead to iron toxicity in patients unless iron chelation therapy is consistently used. Iron supplementation is avoided in patients with thalassemia. There is no need to avoid intramuscular injections. The goal for patients with thalassemia major is to maintain a hemoglobin of 10 g/dL or greater.

A patient with possible disseminated intravascular coagulation arrives in the emergency department with a blood pressure of 82/40, temperature 102 F (38.9 C), and severe back pain. Which physician order will the nurse implement first? a. Administer morphine sulfate 4 mg IV. b. Give acetaminophen (Tylenol) 650 mg. c. Infuse normal saline 500 mL over 30 minutes. d. Schedule complete blood count and coagulation studies.

ANS: C The patients blood pressure indicates hypovolemia caused by blood loss and should be addressed immediately to improve perfusion to vital organs. The other actions also are appropriate and should be rapidly implemented, but improving perfusion is the priority for this patient.

A postoperative patient receiving a transfusion of packed red blood cells develops chills, fever, headache, and anxiety 35 minutes after the transfusion is started. After stopping the transfusion, what action should the nurse take? a. Draw blood for a new crossmatch. b. Send a urine specimen to the laboratory. c. Administer PRN acetaminophen (Tylenol). d. Give the PRN diphenhydramine (Benadryl).

ANS: C The patients clinical manifestations are consistent with a febrile, nonhemolytic transfusion reaction. The transfusion should be stopped and antipyretics administered for the fever as ordered. A urine specimen is needed if an acute hemolytic reaction is suspected. Diphenhydramine (Benadryl) is used for allergic reactions. This type of reaction does not indicate incorrect crossmatching.

The nurse should reposition the patient who has just had a laminectomy and diskectomy by a. instructing the patient to move the legs before turning the rest of the body. b. having the patient turn by grasping the side rails and pulling the shoulders over. c. placing a pillow between the patient's legs and turning the entire body as a unit. d. turning the patient's head and shoulders first, followed by the hips, legs, and feet.

ANS: C The spine should be kept in correct alignment after laminectomy. The other positions will create misalignment of the spine.

A 32-year-old obese male begins a jogging routine. A week after beginning, he fractures his leg. This is referred to as a what type of fracture? a. Comminuted b. Greenstick c. Fatigue d. Compound

ANS: C A fatigue fracture is caused by abnormal stress or torque applied to a bone that usually occurs in individuals who engage in a new activity that is both strenuous and repetitive. A comminuted fracture is one in which a bone breaks into two or more fragments. A greenstick fracture perforates one cortex and splinters the spongy bone. A compound fracture is a complete and incomplete fracture that breaks through the skin.

A 16-year-old with aspirations of becoming a bodybuilder spends 3 hours a day in the gym. Five years later a hepatocellular carcinoma is discovered. Which of the following is the most likely cause of the cancer? a. Immunosuppressive agents b. Cytotoxic agents c. Anabolic steroids d. A viral infection

ANS: C A history of anabolic steroids, not immunosuppressive or cytotoxic agents, places the patient at risk for hepatocellular carcinoma. A viral infection does not place the patient at risk for hepatocellular carcinoma. REF: p. 305, Table 12-5

40. Which action will the urgent care nurse take when caring for a patient who has a possible knee meniscus injury? a. Encourage bed rest for 24 to 48 hours. b. Avoid palpation or movement of the knee. c. Apply a knee immobilizer to the affected leg. d. Administer intravenous narcotics for pain relief.

ANS: C A knee immobilizer may be used for several days after a meniscus injury to stabilize the knee and minimize pain. Patients are encouraged to ambulate with crutches. The knee is assessed by flexing, internally rotating, and extending the knee (McMurray's test). The pain associated with a meniscus injury will not typically require IV opioid administration; nonsteroidal antiinflammatory drugs (NSAIDs) are usually recommended for pain relief. DIF: Cognitive Level: Apply (application) REF: 1510 TOP: Nursing Process: Assessment MSC: NCLEX: Physiological Integrity

17. A 38-yr-old patient who had a kidney transplant 8 years ago is receiving the immunosuppressants tacrolimus (Prograf), cyclosporine (Sandimmune), and prednisone . Which assessment data will be of most concern to the nurse? a. Skin is thin and fragile. b. Blood pressure is 150/92. c. A nontender axillary lump. d. Blood glucose is 144 mg/dL.

ANS: C A nontender lump suggests a malignancy such as a lymphoma, which could occur as a result of chronic immunosuppressive therapy. The elevated glucose, skin change, and hypertension are possible side effects of the prednisone and should be addressed, but they are not as great a concern as the possibility of a malignancy. DIF: Cognitive Level: Analyze (analysis) REF: 1096 OBJ: Special Questions: Prioritization TOP: Nursing Process: Assessment

A 70-year-old diagnosed with osteoporosis fell and fractures the left leg at a location of pre-existing abnormality. Which term describes the fracture? a. Fatigue b. Stress c. Pathologic d. Greenstick

ANS: C A pathologic fracture is a break at the site of a pre-existing abnormality, usually by force such as a fall that would not fracture a normal bone. A fatigue fracture is caused by abnormal stress or torque applied to a bone with normal ability to deform and recover. Stress fractures occur in normal or abnormal bone that is subjected to repeated stress, such as occurs during athletics. A greenstick fracture perforates one cortex and splinters the spongy bone.

28. A patient arrived at the emergency department after tripping over a rug and falling at home. Which finding is most important for the nurse to communicate to the health care provider? a. There is bruising at the shoulder area. b. The patient reports arm and shoulder pain. c. The right arm appears shorter than the left. d. There is decreased shoulder range of motion.

ANS: C A shorter limb after a fall indicates a possible dislocation, which is an orthopedic emergency. Bruising, pain, and decreased range of motion also should be reported, but these do not indicate that emergent treatment is needed to preserve function. DIF: Cognitive Level: Apply (application) REF: 1508 OBJ: Special Questions: Prioritization TOP: Nursing Process: Assessment MSC: NCLEX: Physiological Integrity

What percentage of hip fractures are the result of falls? a. 50% b. 80% c. 90% d. 100%

ANS: C About 90% of falls end with a hip fracture

A patient asks when adjuvant chemotherapy is used. How should the nurse respond? Adjuvant chemotherapy treatment is used: a. as the primary treatment. b. before radiation therapy. c. after surgical removal of a tumor. d. in cancer with little risk of metastasis.

ANS: C Adjuvant chemotherapy is given after surgical excision of a cancer with the goal of eliminating micrometastases. Adjuvant chemotherapy is not given as the primary treatment or before radiation therapy. Adjuvant chemotherapy is indicated in the treatment of individuals with metastasis. REF: p. 261

The most common cause of toxic myopathy is: a. infection. b. a tumor. c. alcoholism. d. osteoporosis.

ANS: C Alcohol remains the most common cause of toxic myopathy.

6. A nurse prepares an adult patient with a severe burn injury for a dressing change. The nurse knows that this is a painful procedure and wants to try providing music to help the patient relax. Which action is best for the nurse to take? a. Use music composed by Mozart. b. Play music that does not have words. c. Ask the patient about music preferences. d. Select music that has 60 to 80 beats/minute.

ANS: C Although music with 60 to 80 beats/min, music without words, and music composed by Mozart are frequently recommended to reduce stress, each patient responds individually to music and personal preferences are important. DIF: Cognitive Level: Analyze (analysis) REF: 85 TOP: Nursing Process: Implementation MSC: NCLEX: Psychosocial Integrity

A nurse is giving instructions to a patient who will be receiving oral iron supplements. Which instructions will be included in the teaching plan? a. Take the iron tablets with milk or antacids. b. Crush the pills as needed to help with swallowing. c. Take the iron tablets with meals if gastrointestinal distress occurs. d. If black tarry stools occur, report it to the doctor immediately.

ANS: C Although taking iron tablets with food may decrease absorption, doing so helps to reduce gastrointestinal distress. Antacids and milk may cause decreased iron absorption; iron tablets must be taken whole and not crushed. Black, tarry stools are expected adverse effects of oral iron supplements.

A neurologist explains that arousal is mediated by the: a. cerebral cortex. b. medulla oblongata. c. reticular activating system. d. cingulate gyrus.

ANS: C Arousal is mediated by the reticular activating system, which regulates aspects of attention and information processing and maintains consciousness. The cerebral cortex affects movement. The medulla oblongata controls things such as hiccups and vomiting. The cingulate gyrus plays other roles in response. REF: p. 359

16. Which assessment finding may indicate that a patient is experiencing adverse effects to a corticosteroid prescribed after kidney transplantation? a. Postural hypotension b. Recurrent tachycardia c. Knee and hip joint pain d. Increased serum creatinine

ANS: C Aseptic necrosis of the weight-bearing joints can occur when patients take corticosteroids over a prolonged period. Increased creatinine level, orthostatic dizziness, and tachycardia are not caused by corticosteroid use. DIF: Cognitive Level: Apply (application) REF: 1096 TOP: Nursing Process: Evaluation MSC: NCLEX: Physiological Integrity

A 6-year-old child who has chickenpox also has a fever of 102.9° F (39.4° C). The child's mother asks the nurse if she should use aspirin to reduce the fever. What is the best response by the nurse? a. "It's best to wait to see if the fever gets worse." b. "You can use the aspirin, but watch for worsening symptoms." c. "Acetaminophen (Tylenol) should be used to reduce his fever, not aspirin." d. "You can use aspirin, but be sure to follow the instructions on the bottle."

ANS: C Aspirin is contraindicated in children with flu-like symptoms because the use of this drug has been strongly associated with Reye's syndrome. This is an acute and potentially life-threatening condition involving progressive neurologic deficits that can lead to coma and may also involve liver damage. Acetaminophen is appropriate for this patient. The other responses are incorrect.

5. A 53-year-old woman at menopause is discussing the use of hormone therapy (HT) with the nurse. Which information about the risk of breast cancer will the nurse provide? a. HT is a safe therapy for menopausal symptoms if there is no family history ofBRCA genes. b. HT does not appear to increase the risk for breast cancer unless there are other risk factors. c. The patient and her health care provider must weigh the benefits of HT against the risks of breast cancer. d. Natural herbs are as effective as estrogen in relieving symptoms without increasing the risk of breast cancer.

ANS: C Because HT has been linked to increased risk for breast cancer, the patient and provider must determine whether or not to use HT. Breast cancer incidence is increased in women using HT, independent of other risk factors. HT increases the risk for both nonBRCA-associated cancer and BRCA-related cancers. Alternative therapies can be used but are not consistent in relieving menopausal symptoms. DIF: Cognitive Level: Apply (application) TOP: Nursing Process: Implementation MSC: NCLEX: Physiological Integrity

2. A female patient who initially came to the clinic with incontinence was recently diagnosed with endometrial cancer. She is usually well organized and calm, but the nurse who is giving her preoperative instructions observes that the patient is irritable, has difficulty concentrating, and yells at her husband. Which action should the nurse take? a. Ask the health care provider for a psychiatric referral. b. Focus teaching on preventing postoperative complications. c. Try to calm the patient before repeating any information about the surgery. d. Encourage the patient to combine the hysterectomy with surgery for bladder repair.

ANS: C Because behavioral responses to stress include temporary changes such as irritability, changes in memory, and poor concentration, patient teaching will need to be repeated. It is also important to try to calm the patient by listening to her concerns and fears. Psychiatric referral will not necessarily be needed for her but that can better be evaluated after surgery. Focusing on postoperative care does not address the need for preoperative instruction such as the procedure, NPO instructions before surgery, date and time of surgery, medications to be taken or discontinued before surgery, and so on. The issue of incontinence is not immediately relevant in the discussion of preoperative teaching for her hysterectomy. DIF: Cognitive Level: Apply (application) REF: 81 TOP: Nursing Process: Implementation MSC: NCLEX: Psychosocial Integrity

Which patient information is most important for the nurse to monitor when evaluating the effectiveness of deferoxamine (Desferal) for a patient with hemochromatosis? a. Skin color b. Hematocrit c. Liver function d. Serum iron level

ANS: D Because iron chelating agents are used to lower serum iron levels, the most useful information will be the patients iron level. The other parameters will also be monitored, but are not the most important to monitor when determining the effectiveness of deferoxamine.

46. Based on the information shown in the accompanying figure and obtained for a patient in the emergency room, which action will the nurse take first? a. Administer the prescribed morphine 4 mg IV. b. Contact the operating room to schedule surgery. c. Check the patient's oxygen saturation using pulse oximetry. d. Ask the patient about the date of the last tetanus immunization.

ANS: C Because fat embolism can occur with tibial fracture, the nurse's first action should be to check the patient's oxygen saturation. The other actions are also appropriate, but not as important at this time as obtaining the patient's oxygen saturation. DIF: Cognitive Level: Apply (application) REF: 1523 OBJ: Special Questions: Prioritization TOP: Nursing Process: Implementation MSC: NCLEX: Physiological Integrity

A 1-year-old female was diagnosed with osteogenesis imperfecta (OI). Which complication must be monitored for? a. Congestive heart failure b. Liver failure c. Aortic aneurysm d. Pulmonary emboli

ANS: C Because type I collagen is also the main component of blood vessels, vascular deformity, such as aortic aneurysm, can occur. Neither congestive heart failure, liver failure, nor pulmonary emboli are associated with osteogenesis imperfecta (OI). REF: p. 1039

A patient has been taking temazepam (Restoril) for intermittent insomnia. She calls the nurse to say that when she takes it, she sleeps well, but the next day she feels "so tired." Which explanation by the nurse is correct? a. "Long-term use of this drug results in a sedative effect." b. "If you take the drug every night, this hangover effect will be reduced." c. "These drugs affect the sleep cycle, resulting in daytime sleepiness." d. "These drugs increase the activity of the central nervous system, making you tired the next day."

ANS: C Benzodiazepines suppress REM sleep to a degree (although not as much as barbiturates) and, thus, result in daytime sleepiness (a hangover effect). The other statements are incorrect. DIF: COGNITIVE LEVEL: Applying (Application) REF: p. 191 TOP: NURSING PROCESS: Implementation MSC: NCLEX: Physiological Integrity: Physiological Adaptation

The nurse will teach a patient who is receiving oral iron supplements to watch for which expected adverse effects? a. Palpitations b. Drowsiness and dizziness c. Black, tarry stools d. Orange-red discoloration of the urine

ANS: C Black, tarry stools and other gastrointestinal disturbances may occur with the administration of iron preparations. The other options are incorrect.

The nurse is working on a plan of care with her patient which includes turning and positioning and adequate nutrition to help the patient maintain intact skin integrity. The nurse helps the patient to realize that this breaks the chain of infection by eliminating a a. host. b. mode of transmission. c. portal of entry. d. reservoir.

ANS: C Broken or impaired skin creates a portal of entry for pathogens. By maintaining intact tissue, the patient and the nurse have broken the chain of infection by eliminating a portal of entry. Host is incorrect because you are not eliminating the person or organism. Intact tissue does not eliminate the mode of transmission. Skin can still be used to transfer pathogens regardless of it being intact or broken. Intact skin does not eliminate the location for pathogens to live and grow.

An older patient is talking with the nurse about hip fractures. The patient would like to know the best approach to strengthen the bones. The nurse's best response is which of the following? a. "Walk at least 5 miles every day for exercise." b. "Wear proper fitting shoes to prevent tripping." c. "Talk with your physician about a calcium supplement." d. "Stand up slowly so you don't feel faint."

ANS: C Calcium strengthens the bones. A calcium supplement will help strengthen bones as they may be affected by aging, illness, or trauma. Walking several miles will help strengthen the bones but a calcium supplement is a good addition. Wearing proper shoes and standing slowly to prevent dizziness is important but they will not prevent fractures.

A 30-year-old female is diagnosed with cancer. Testing reveals that the cancer cells have spread to local lymph nodes. A nurse realizes this cancer would be documented as stage: a. 1. b. 2. c. 3. d. 4.

ANS: C Cancer that has spread to regional structures, such as lymph nodes, is stage 3. Cancer confined to the organ of origin is stage 1. Cancer that is locally invasive is stage 2. Cancer that has spread to distant sites, such as a liver cancer spreading to lung or a prostate cancer spreading to bone, is stage 4.

A patient is receiving a third session of chemotherapy with daunorubicin (Cerubidine). The nurse will assess the patient for which signs of a potential severe toxic effect of this drug? a. Tinnitus and hearing loss b. Numbness and tingling in the fingers c. A weight gain of 2 pounds or more in 24 hours d. Decreased blood urea nitrogen and creatinine levels

ANS: C Cardiac toxicity may occur, so frequent checking of heart and breath sounds is necessary and daily weights need to be recorded (with reporting of an increase of 2 pounds or more in 24 hours or 5 pounds or more in 1 week). DIF: COGNITIVE LEVEL: Applying (Application) REF: p. 743 TOP: NURSING PROCESS: Assessment MSC: NCLEX: Physiological Integrity: Pharmacological and Parenteral Therapies

A 3-year-old Black child with a history of sickle cell disease and is now diagnosed with meningitis. Which is the most likely microorganism the nurse will find on the lab report? a. Virus b. Haemophilus influenzae type B c. Streptococcus pneumonia d. Neisseria meningitidis

ANS: C Children with sickle cell disease are at risk for developing bacterial meningitis. These children do not experience an increased risk for the development of a viral infection, Haemophilus influenzae, or Neisseria meningitidis. REF: p. 431

1. When teaching seniors at a community recreation center, which information will the nurse include about ways to prevent fractures? a. Tack down scatter rugs in the home. b. Most falls happen outside the home. c. Buy shoes that provide good support and are comfortable to wear. d. Range-of-motion exercises should be taught by a physical therapist.

ANS: C Comfortable shoes with good support will help decrease the risk for falls. Scatter rugs should be eliminated, not just tacked down. Activities of daily living provide range of motion exercise; these do not need to be taught by a physical therapist. Falls inside the home are responsible for many injuries. DIF: Cognitive Level: Apply (application) REF: 1506 TOP: Nursing Process: Implementation MSC: NCLEX: Safe and Effective Care Environment

21. Which action will the nurse include in the plan of care for a patient with right arm lymphedema? a. Check blood pressure (BP) on both right and left arms. b. Avoid isometric exercise on the right arm. c. Assist with application of a compression sleeve. d. Keep the right arm at or below the level of the heart.

ANS: C Compression of the arm assists in improving lymphatic flow toward the heart. Isometric exercises may be prescribed for lymphedema. BPs should only be done on the patients right arm. The arm should not be placed in a dependent position. DIF: Cognitive Level: Apply (application) TOP: Nursing Process: Planning MSC: NCLEX: Physiological Integrity

An infant has an anterior midline defect of neural tube closure. What term will the nurse observe written on the chart? a. Anencephaly b. Myelodysplasia c. Cyclopia d. Hydrocephalus

ANS: C Cyclopia is an example of an anterior midline defect that may cause brain and face abnormalities. In anencephaly, the soft, bony component of the skull and part of the brain are missing. Myelodysplasia is a posterior neural tube disorder. Hydrocephalus is associated with accelerated head growth. REF: p. 424

A 22-year-old female whose brother is diagnosed with Duchenne muscular dystrophy (DMD) wants to know if her children will inherit it. A fairly accurate test to identify female carriers of the disease is measurement of serum levels of: a. dystrophin. b. myoglobin. c. creatine kinase (CK). d. troponin 1.

ANS: C Diagnosis is confirmed by measuring the blood CK level, which is sometimes 100 times the normal level, with confirmation by genetic testing for mutations in the dystrophin gene. None of the other options is relevant to this screening. REF: p. 1046

25. The nurse is caring for a 52-year-old patient with breast cancer who is receiving chemotherapy with doxorubicin (Adriamycin) and cyclophosphamide (Cytoxan). Which assessment finding is most important to communicate to the health care provider? a. The patient complains of fatigue. b. The patient eats only 25% of meals. c. The patients apical pulse is irregular. d. The patients white blood cell (WBC) count is 5000/L.

ANS: C Doxorubicin can cause cardiac toxicity. The dysrhythmia should be reported because it may indicate a need for a change in therapy. Anorexia, fatigue, and a low-normal WBC count are expected effects of chemotherapy. DIF: Cognitive Level: Apply (application) OBJ: Special Questions: Prioritization TOP: Nursing Process: Assessment MSC: NCLEX: Physiological Integrity

2. During a well woman physical exam, a 43-year-old patient asks about her risk for breast cancer. Which question is most pertinent for the nurse to ask? a. Do you currently smoke tobacco? b. Have you ever had a breast injury? c. At what age did you start having menstrual periods? d. Is there a family history of fibrocystic breast changes?

ANS: C Early menarche and late menopause are risk factors for breast cancer because of the prolonged exposure to estrogen that occurs. Cigarette smoking, breast trauma, and fibrocystic breast changes are not associated with increased breast cancer risk. DIF: Cognitive Level: Apply (application) TOP: Nursing Process: Assessment MSC: NCLEX: Physiological Integrity

A nurse is discussing preinvasive epithelial tumors of glandular or squamous cell origin. What is the nurse describing? a. Tumor in differentiation b. Dysplastic c. Cancer in situ d. Cancer beyond (meta) situ

ANS: C Early-stage growths that are localized to the epithelium and have not invaded are called cancer in situ. Cancer in situ is early-stage growth and not a tumor in differentiation but is more mature growth. Dysplastic cells do not define cancer in situ.

4. Which discharge instruction will the emergency department nurse include for a patient with a sprained ankle? a. Keep the ankle loosely wrapped with gauze. b. Apply a heating pad to reduce muscle spasms. c. Use pillows to elevate the ankle above the heart. d. Gently move the ankle through the range of motion.

ANS: C Elevation of the leg will reduce the amount of swelling and pain. Compression bandages are used to decrease swelling. For the first 24 to 48 hours, cold packs are used to reduce swelling. The ankle should be rested and kept immobile to prevent further swelling or injury. DIF: Cognitive Level: Apply (application) REF: 1508 TOP: Nursing Process: Implementation MSC: NCLEX: Physiological Integrity

A patient with end-stage renal failure has been admitted to the hospital for severe anemia. She is refusing blood transfusions. The nurse anticipates drug therapy with which drug to stimulate the production of red blood cells? a. Folic acid b. Cyanocobalamin (vitamin B12) c. Epoetin alfa (Epogen) d. Filgrastim (Neupogen)

ANS: C Epoetin alfa is a colony-stimulating factor that is responsible for erythropoiesis, or formation of red blood cells. The other options are incorrect.

10. A 37-yr-old female patient is hospitalized with acute kidney injury (AKI). Which information will be most useful to the nurse in evaluating improvement in kidney function? a. Urine volume b. Creatinine level c. Glomerular filtration rate (GFR) d. Blood urea nitrogen (BUN) level

ANS: C GFR is the preferred method for evaluating kidney function. BUN levels can fluctuate based on factors such as fluid volume status and protein intake. Urine output can be normal or high in patients with AKI and does not accurately reflect kidney function. Creatinine alone is not an accurate reflection of renal function. DIF: Cognitive Level: Analyze (analysis) REF: 1079 TOP: Nursing Process: Evaluation MSC: NCLEX: Physiological Integrity

A patient is taking the nonsteroidal anti-inflammatory drug indomethacin (Indocin) as treatment for pericarditis. The nurse will teach the patient to watch for which adverse effect? a. Tachycardia b. Nervousness c. Nausea and vomiting d. Dizziness

ANS: C Gastrointestinal effects include dyspepsia, heartburn, epigastric distress, nausea, vomiting, anorexia, abdominal pain, and others. See Table 44-2 for the other adverse effects of nonsteroidal anti-inflammatory drugs (NSAIDs). The other options are not adverse effects of NSAIDs.

Which genetic change causes alterations in only one or a few nucleotide bases? a. Insertions b. Deletions c. Point mutations d. Amplification mutations

ANS: C Genetic changes may occur by both mutational and epigenetic mechanisms. Mutation generally means an alteration in the DNA sequence affecting expression or function of a gene. Mutations include small-scale changes in DNA, such as point mutations, which are the alteration of one or a few nucleotide base pairs. The process involved with insertions, deletions, or amplification mutations is different.

21. A 55-yr-old patient with end-stage kidney disease (ESKD) is scheduled to receive a prescribed dose of epoetin alfa (Procrit). Which information should the nurse report to the health care provider before giving the medication? a. Creatinine 1.6 mg/dL b. Oxygen saturation 89% c. Hemoglobin level 13 g/dL d. Blood pressure 98/56 mm Hg

ANS: C High hemoglobin levels are associated with a higher rate of thromboembolic events and increased risk of death from serious cardiovascular events (heart attack, heart failure, stroke) when erythropoietin (EPO) is administered to a target hemoglobin of greater than 12 g/dL. Hemoglobin levels higher than 12 g/dL indicate a need for a decrease in epoetin alfa dose. The other information also will be reported to the health care provider but will not affect whether the medication is administered. DIF: Cognitive Level: Apply (application) REF: 1081 TOP: Nursing Process: Assessment MSC: NCLEX: Physiological Integrity

During therapy with the hematopoietic drug epoetin alfa (Epogen), the nurse instructs the patient about adverse effects that may occur, such as: a. anxiety. b. drowsiness. c. hypertension. d. constipation.

ANS: C Hypertension is an adverse effect of hematopoietic drugs, along with headache, fever, pruritus, rash, nausea, vomiting, arthralgia, cough, and injection site reaction. The other options are incorrect.

10. Which statement by the patient indicates a good understanding of the nurse's teaching about a new short-arm plaster cast? a. "I can get the cast wet as long as I dry it right away with a hair dryer." b. "I should avoid moving my fingers and elbow until the cast is removed." c. "I will apply an ice pack to the cast over the fracture site off and on for 24 hours." d. "I can use a cotton-tipped applicator to rub lotion on any dry areas under the cast.

ANS: C Ice application for the first 24 hours after a fracture will help reduce swelling and can be placed over the cast. Plaster casts should not get wet. The patient should be encouraged to move the joints above and below the cast. Patients should not insert objects inside the cast. DIF: Cognitive Level: Apply (application) REF: 1520 TOP: Nursing Process: Evaluation MSC: NCLEX: Physiological Integrity

What is the nurse's priority action if extravasation of an antineoplastic drug occurs during intravenous (IV) administration? a. Reduce the infusion rate. b. Discontinue the IV, and apply warm compresses. c. Stop the infusion immediately, but leave the IV catheter in place. d. Change the infusion to normal saline, and inject the area with hydrocortisone.

ANS: C If extravasation is suspected, administration of the drug must be stopped immediately but the IV catheter left in place and the appropriate antidote instilled through the existing IV tube, after which the needle may be removed. The other options are incorrect. DIF: COGNITIVE LEVEL: Applying (Application) REF: p. 736 TOP: NURSING PROCESS: Implementation MSC: NCLEX: Safe and Effective Care Environment: Management of Care

9. Before administration of calcium carbonate to a patient with chronic kidney disease (CKD), the nurse should check laboratory results for a. potassium level. b. total cholesterol. c. serum phosphate. d. serum creatinine.

ANS: C If serum phosphate is elevated, the calcium and phosphate can cause soft tissue calcification. Calcium carbonate should not be given until the phosphate level is lowered. Total cholesterol, creatinine, and potassium values do not affect whether calcium carbonate should be administered. DIF: Cognitive Level: Apply (application) REF: 1081 TOP: Nursing Process: Implementation MSC: NCLEX: Physiological Integrity

A 56-year-old male was admitted to the hospital with a diagnosis of osteomalacia. History reveals that he underwent bariatric surgery 3 years earlier. What is the common link between the surgery and the development of osteomalacia? a. Impaired phosphate absorption b. Increased calcium excretion c. Vitamin D deficiency d. Impaired vitamin C metabolism

ANS: C Impaired nutrient absorption from bariatric surgery can result in vitamin D deficiency. Vitamin D deficiency is the most important factor in the development of osteomalacia. Neither impaired phosphate absorption, increased calcium excretion, nor impaired vitamin C metabolism is a cause of osteomalacia.

The inflammatory symptoms of classic gouty arthritis are caused by the crystallization of _____ within the synovial fluid. a. purines b. pyrimidines c. monosodium urate d. acetic acid

ANS: C In classic gouty arthritis, monosodium urate crystals form and are deposited in joints and their surrounding tissues, initiating a powerful inflammatory response. Uric acid is a result of purine metabolism, but purine is not present in the joint. Gout is not caused by an increase in pyrimidines or acetic acid.

A 70-year-old female presents with a hip fracture secondary to osteoporosis. This condition is caused by an increase in bone: a. density. b. formation. c. resorption. d. mineralization.

ANS: C In osteoporosis, old bone is being resorbed faster than new bone is being made, causing the bones to lose density, becoming thinner, and more porous. Mineralization is not increased by osteoporosis.

24. When giving home care instructions to a patient who has comminuted forearm fractures and a long-arm cast on the left arm, which information should the nurse include? a. Keep the left shoulder elevated on a pillow or cushion. b. Keep the hand immobile to prevent soft tissue swelling. c. Call the health care provider for increased swelling or numbness of the hand. d. Avoid nonsteroidal antiinflammatory drugs (NSAIDs) for 24 hours after the injury.

ANS: C Increased swelling or numbness may indicate increased pressure at the injury, and the health care provider should be notified immediately to avoid damage to nerves and other tissues. The patient should be encouraged to move the joints above and below the cast to avoid stiffness. There is no need to elevate the shoulder, although the forearm should be elevated to reduce swelling. NSAIDs are appropriate to treat pain after a fracture. DIF: Cognitive Level: Apply (application) REF: 1520 TOP: Nursing Process: Implementation MSC: NCLEX: Physiological Integrity

A patient is to receive iron dextran injections. Which technique is appropriate when the nurse is administering this medication? a. Intravenous administration mixed with 5% dextrose b. Intramuscular injection in the upper arm c. Intramuscular injection using the Z-track method d. Subcutaneous injection into the abdomen

ANS: C Intramuscular iron is given using the Z-track method deep into a large muscle mass. If given intravenously, it is given with normal saline, not 5% dextrose.

Legg-Calvé-Perthes (LCP) disease affects which of the following joints? a. Vertebral b. Shoulder c. Hip d. Knee

ANS: C LCP affects the hip. This condition is not associated with the vertebrae, the shoulder, or the knee. REF: p. 1044

18. The nurse in the dialysis clinic is reviewing the home medications of a patient with chronic kidney disease (CKD). Which medication reported by the patient indicates that patient teaching is required? a. Acetaminophen b. Calcium phosphate c. Magnesium hydroxide d. Multivitamin with iron

ANS: C Magnesium is excreted by the kidneys, and patients with CKD should not use over-the-counter products containing magnesium. The other medications are appropriate for a patient with CKD. DIF: Cognitive Level: Apply (application) REF: 1081 TOP: Nursing Process: Assessment MSC: NCLEX: Physiological Integrity

Which information indicates a nurse understands characteristics of malignant tumors? a. Grows slowly b. Has a well-defined capsule c. Cells vary in size and shape d. Is well differentiated

ANS: C Malignant tumors have cells that vary in both size and shape, and they grow rapidly. They are poorly differentiated and not encapsulated.

18. An older adult female patient is receiving the progestin drug megestrol (Megace). Which is the most likely reason megestrol is ordered for this patient? a. Migraine headaches b. Osteoporosis c. Appetite stimulant d. Reduction of hot flashes

ANS: C Megestrol can cause appetite stimulation and weight gain, and therefore is used in the management of anorexia, cachexia, or unexplained substantial weight loss in patients with acquired immunodeficiency syndrome (AIDS) and in patients with cancer. The other options are incorrect. DIF: COGNITIVE LEVEL: Applying (Application) REF: p. 539 TOP: NURSING PROCESS: Planning MSC: NCLEX: Physiological Integrity: Pharmacological and Parenteral Therapies

A major predictor of poor prognosis for Ewing sarcoma is: a. age of onset. b. size of tumor. c. presence of metastasis. d. gender of child.

ANS: C Metastasis at diagnosis is another poor prognostic indicator, with 5-year survival rate dropping to under 40%. REF: p. 1050

A 54-year-old male was recently diagnosed with rheumatoid arthritis (RA). Which of the following is the expected treatment of choice? a. Nonsteroidal anti-inflammatory drugs (NSAIDs) b. Gold salts c. Methotrexate d. Colchicine

ANS: C Methotrexate remains the first line of treatment for RA. Neither NSAIDs, gold salts, nor colchicine is a treatment of choice for RA.

What is the origin of most childhood cancers? a. Placenta b. Environment c. Mesodermal germ layer d. Neural tube

ANS: C Most childhood cancers originate from the mesodermal germ layer that gives rise to connective tissue, bone, cartilage, muscle, blood, blood vessels, the gonads, the kidney, and the lymphatic system. Most childhood cancers do not originate from the placenta or the environment. Neural tubes are generally not associated with cancer. REF: p. 301

The nurse is reviewing the therapeutic effects of nonsteroidal anti-inflammatory drugs (NSAIDs), which include which effect? a. Anxiolytic b. Sedative c. Antipyretic d. Antimicrobial

ANS: C NSAIDs have antipyretic effects but not the other effects listed.

A 9-month-old male was diagnosed with osteogenesis imperfecta (OI). This disease is caused by errors in the synthesis of: a. elastin. b. glycoproteins. c. collagen. d. calcium salts.

ANS: C OI (brittle bone disease) is a spectrum of disease caused by genetic mutation in the gene that encodes for type I collagen. OI (brittle bone disease) is not caused by faulty synthesis of elastin, glycoproteins, or calcium salts. REF: p. 1039

During chemotherapy, a patient develops severe diarrhea caused by a vasoactive intestinal peptide-secreting tumor (VIPoma). The nurse expects to administer which drug for this problem? a. Dexrazoxane (Zinecard) b. Allopurinol (Zyloprim) c. Octreotide (Sandostatin) d. Bismuth subsalicylate (Pepto-Bismol)

ANS: C Octreotide (Sandostatin) is used for the management of a cancer-related condition called carcinoid crisis and treatment of the severe diarrhea caused by vasoactive intestinal peptide-secreting tumors (VIPomas). The other options are incorrect. DIF: COGNITIVE LEVEL: Applying (Application) REF: p. 740 TOP: NURSING PROCESS: Planning MSC: NCLEX: Physiological Integrity: Pharmacological and Parenteral Therapies

The nurse explains to a parent that young children diagnosed with Down syndrome are at higher risk for developing: a. nephroblastoma. b. rhabdomyosarcoma. c. leukemia. d. retinoblastoma.

ANS: C One of the more recognized syndromes is the association of trisomy 21 and Down syndrome, with an increased susceptibility to acute leukemia, not nephroblastoma, rhabdomyosarcoma, or retinoblastoma. REF: p. 303

29. A licensed practical/vocational nurse (LPN/LVN) is caring for a patient with stage 2 chronic kidney disease. Which observation by the RN requires an intervention? a. The LPN/LVN administers the erythropoietin subcutaneously. b. The LPN/LVN assists the patient to ambulate out in the hallway. c. The LPN/LVN administers the iron supplement and phosphate binder with lunch. d. The LPN/LVN carries a tray containing low-protein foods into the patient's room.

ANS: C Oral phosphate binders should not be given at the same time as iron because they prevent the iron from being absorbed. The phosphate binder should be given with a meal and the iron given at a different time. The other actions by the LPN/LVN are appropriate for a patient with renal insufficiency. DIF: Cognitive Level: Apply (application) REF: 1082 OBJ: Special Questions: Delegation TOP: Nursing Process: Implementation MSC: NCLEX: Safe and Effective Care Environment

5. A patient is receiving oxytocin (Pitocin) to induce labor. During administration of this medication, the nurse will also implement which action? a. Giving magnesium sulfate along with the oxytocin b. Administering the medication in an intravenous (IV) bolus c. Administering the medication with an IV infusion pump d. Monitoring fetal heart rate and maternal vital signs every 6 hours

ANS: C Oxytocin is infused via an infusion pump, not via an IV bolus. Magnesium sulfate is not administered with oxytocin. Fetal heart rate and maternal vital signs should be monitored continuously. DIF: COGNITIVE LEVEL: Applying (Application) REF: p. 549 TOP: NURSING PROCESS: Implementation MSC: NCLEX: Physiological Integrity: Pharmacological and Parenteral Therapies

While reviewing the complete blood count (CBC) of a patient on her unit, the nurse notes elevated basophil and eosinophil readings. The nurse realizes that this is most indicative of a _____ infection. a. bacterial b. fungal c. parasitic d. viral

ANS: C Parasitic infections are frequently indicated on a CBC by elevated basophil and eosinophil levels. Bacterial infections do not lead to elevated basophil and eosinophil levels but elevated B and T lymphocytes, neutrophils, and monocytes. Fungal infections do not lead to elevated basophil and eosinophil levels. Viral infections create elevations in B and T lymphocytes, neutrophils, and monocytes.

14. Which action by a patient who is using peritoneal dialysis (PD) indicates that the nurse should provide more teaching about PD? a. The patient leaves the catheter exit site without a dressing. b. The patient plans 30 to 60 minutes for a dialysate exchange. c. The patient cleans the catheter while taking a bath each day. d. The patient slows the inflow rate when experiencing abdominal pain.

ANS: C Patients are encouraged to take showers rather than baths to avoid infections at the catheter insertion side. The other patient actions indicate good understanding of peritoneal dialysis. DIF: Cognitive Level: Apply (application) REF: 1086 TOP: Nursing Process: Evaluation MSC: NCLEX: Physiological Integrity

1. The nurse is providing teaching for a patient who is to receive estrogen replacement therapy. Which statement is correct to include in the teaching session? a. "If you miss a dose, double-up on the next dose." b. "There's no need to be concerned about breast lumps or bumps that occur." c. "Be sure to report any weight gain of 5 pounds or more per week." d. "Take the medication on an empty stomach to enhance absorption."

ANS: C Patients taking oral estrogen therapy should report weight gain of 5 pounds or more per week to a physician. The other statements are not true for estrogen replacement therapy. DIF: COGNITIVE LEVEL: Applying (Application) REF: p. 550 TOP: NURSING PROCESS: Implementation MSC: NCLEX: Physiological Integrity: Reduction of Risk Potential

43. When a patient arrives in the emergency department with a facial fracture, which action will the nurse take first? a. Assess for nasal bleeding and pain. b. Apply ice to the face to reduce swelling. c. Use a cervical collar to stabilize the spine. d. Check the patient's alertness and orientation.

ANS: C Patients who have facial fractures are at risk for cervical spine injury and should be treated as if they have a cervical spine injury until this is ruled out. The other actions are also necessary, but the most important action is to prevent cervical spine injury. DIF: Cognitive Level: Apply (application) REF: 1529 OBJ: Special Questions: Prioritization TOP: Nursing Process: Planning MSC: NCLEX: Physiological Integrity

A nurse is teaching a group of businesspeople about disease transmission. He knows that he needs to reeducate when one of the participants states which of the following? a. "When traveling outside of the country, I need to be sure that I receive appropriate vaccinations." b. "Food and water supplies in foreign countries can contain microorganisms to which my body is not accustomed and has no resistance." c. "If I don't feel sick, then I don't have to worry about transmitted diseases." d. "I need to be sure to have good hygiene practices when traveling in crowded planes and trains."

ANS: C People can transmit pathogens even if they don't currently feel ill. Some carriers never experience the full symptoms of a pathogen. Travelers may need different vaccinations when traveling to countries outside their own because of variations in prevalent microorganisms. Food and water supplies in foreign countries can contain microorganisms that will affect a body unaccustomed to their presence. Adequate hygiene is essential when in crowded, public spaces like planes and other forms of public transportation.

Parents of a 3-month-old bring the infant to the emergency room (ER) after a seizure has caused muscle rigidity. Both parents are of Jewish ancestry. For what genetic disease should this infant be screened? a. Juvenile myoclonic epilepsy b. Congenital encephalopathy c. Tay-Sachs disease d. PKU

ANS: C Perhaps the best known of the lysosomal storage disorders is Tay-Sachs disease, an autosomal recessive disorder related to a deficiency of the enzyme hexosaminidase A (HEX A). Approximately 80% of individuals diagnosed are of Jewish ancestry. Jewish ancestry is not necessarily associated with epilepsy, encephalopathy, or PKU. REF: p. 430

8. Which menu choice by the patient who is receiving hemodialysis indicates that the nurse's teaching has been successful? a. Split-pea soup, English muffin, and nonfat milk b. Oatmeal with cream, half a banana, and herbal tea c. Poached eggs, whole-wheat toast, and apple juice d. Cheese sandwich, tomato soup, and cranberry juice

ANS: C Poached eggs would provide high-quality protein, and apple juice is low in potassium. Cheese is high in salt and phosphate, and tomato soup is high in potassium. Split-pea soup is high in potassium, and dairy products are high in phosphate. Bananas are high in potassium, and cream is high in phosphate. DIF: Cognitive Level: Apply (application) REF: 1087 TOP: Nursing Process: Evaluation MSC: NCLEX: Physiological Integrity

36. A 42-year-old patient is admitted to the emergency department with a left femur fracture. Which information obtained by the nurse is most important to report to the health care provider? a. Ecchymosis of the left thigh b. Complaints of severe thigh pain c. Slow capillary refill of the left foot d. Outward pointing toes on the left foot

ANS: C Prolonged capillary refill may indicate complications such as arterial damage or compartment syndrome. The other findings are typical with a left femur fracture. DIF: Cognitive Level: Apply (application) REF: 1517-1518 OBJ: Special Questions: Prioritization TOP: Nursing Process: Assessment MSC: NCLEX: Physiological Integrity

Ramelteon (Rozerem) is prescribed for a patient with insomnia. The nurse checks the patient's medical history, knowing that this medication is contraindicated in which disorder? a. Coronary artery disease b. Renal insufficiency c. Liver disease d. Anemia

ANS: C Ramelteon is contraindicated in cases of severe liver dysfunction. The other conditions are not contraindications. DIF: COGNITIVE LEVEL: Understanding (Comprehension) REF: p. 193 TOP: NURSING PROCESS: Assessment MSC: NCLEX: Physiological Integrity: Pharmacological and Parenteral Therapies

4. A patient is extremely anxious about having a biopsy on a femoral lymph node. Which relaxation technique would be the best choice for the nurse to facilitate during the procedure? a. Yoga stretching c. Relaxation breathing b. Guided imagery d. Mindfulness meditation

ANS: C Relaxation breathing is an easy relaxation technique to teach and use. The patient should remain still during the biopsy and not move or stretch any of his extremities. Meditation and guided imagery require more time to practice and learn. DIF: Cognitive Level: Apply (application) REF: 83 TOP: Nursing Process: Implementation MSC: NCLEX: Psychosocial Integrity

A 46-year-old male diagnosed with gouty arthritis is at high risk for developing: a. cholelithiasis. b. myocarditis. c. renal stones. d. liver failure.

ANS: C Renal stones are 1000 times more prevalent in individuals with primary gout than in the general population. Gouty arthritis does not appear to increase the risk for the development of cholelithiasis, myocarditis, or liver failure.

When hanging a new infusion bag of a chemotherapy drug, the nurse accidentally spills a small amount of the solution onto the floor. Which action by the nurse is appropriate? a. Let it dry, and then mop the floor. b. Wipe the area with a disposable paper towel. c. Use a spill kit to clean the area. d. Ask the housekeeping department to clean the floor.

ANS: C Special spill kits are employed to clean up even the smallest chemotherapy spills. These precautions are necessary to protect the health care provider from the cytotoxic effects of these drugs. DIF: COGNITIVE LEVEL: Applying (Application) REF: p. 741 TOP: NURSING PROCESS: Implementation MSC: NCLEX: Safe and Effective Care Environment: Safety and Infection Control

8. After a 48-year-old patient has had a modified radical mastectomy, the pathology report identifies the tumor as an estrogen-receptor positive adenocarcinoma. The nurse will plan to teach the patient about a. estradiol (Estrace). b. raloxifene (Evista). c. tamoxifen (Nolvadex). d. trastuzumab (Herceptin).

ANS: C Tamoxifen is used for estrogen-dependent breast tumors in premenopausal women. Raloxifene is used to prevent breast cancer, but it is not used postmastectomy to treat breast cancer. Estradiol will increase the growth of estrogen-dependent tumors. Trastuzumab is used to treat tumors that have the HER-2 receptor. DIF: Cognitive Level: Apply (application) TOP: Nursing Process: Planning MSC: NCLEX: Physiological Integrity

A baby is stillborn after 6 hours of labor. Autopsy reveals hydrocephalus caused by cystic dilation of the fourth ventricle and aqueductal compression. Which of the following is the most likely diagnosis? a. Congenital hydrocephalus b. Microcephaly c. Dandy-Walker deformity d. Macewen sign

ANS: C The Dandy-Walker malformation is a congenital defect of the cerebellum characterized by a large posterior fossa cyst, which communicates with the fourth ventricle, and an atrophic upwardly rotated cerebellar vermis. Congenital hydrocephalus is characterized by enlargement of the cerebral ventricles. Microcephaly is lack of brain growth with retarded mental and motor development. Macewen sign is characterized by separation of the cranial sutures with a resonant sound when the skull is tapped. REF: p. 428

A 10-month-old infant presents for a well-baby visit. Which of the following reflexes should be present at this age? a. Stepping b. Sucking c. Landau d. Palmar grasp

ANS: C The Laundau reflex would be present up until 24 months. The stepping reflex should no longer be obtainable at 6 weeks. The sucking reflex should have disappeared by 10 months. The palmar grasp should no longer be obtainable at 6 months. REF: p. 423, Table 17-1

25. A patient who slipped and fell in the shower at home has a proximal humerus fracture immobilized with a left-sided long-arm cast and a sling. Which nursing intervention will be included in the plan of care? a. Use surgical net dressing to hang the arm from an IV pole. b. Immobilize the fingers of the left hand with gauze dressings. c. Assess the left axilla and change absorbent dressings as needed. d. Assist the patient in passive range of motion (ROM) for the right arm.

ANS: C The axilla can become excoriated when a sling is used to support the arm, and the nurse should check the axilla and apply absorbent dressings to prevent this. A patient with a sling would not have traction applied by hanging. The patient will be encouraged to move the fingers on the injured arm to maintain function and to help decrease swelling. The patient will do active ROM on the uninjured side. DIF: Cognitive Level: Apply (application) REF: 1524 TOP: Nursing Process: Planning MSC: NCLEX: Physiological Integrity

4. A patient who has acute glomerulonephritis is hospitalized with hyperkalemia. Which information will the nurse monitor to evaluate the effectiveness of the prescribed calcium gluconate IV? a. Urine volume b. Calcium level c. Cardiac rhythm d. Neurologic status

ANS: C The calcium gluconate helps prevent dysrhythmias that might be caused by the hyperkalemia. The nurse will monitor the other data as well, but these will not be helpful in determining the effectiveness of the calcium gluconate. DIF: Cognitive Level: Apply (application) REF: 1073 TOP: Nursing Process: Evaluation MSC: NCLEX: Physiological Integrity

A patient complains of difficulty falling asleep and daytime fatigue for the past 6 weeks. What is the best initial action for the nurse to take in determining whether this patient has chronic insomnia? a. Schedule a polysomnography (PSG) study. b. Arrange for the patient to have a sleep study. c. Ask the patient to keep a 2-week sleep diary. d. Teach the patient about the use of an actigraph.

ANS: C The diagnosis of insomnia is made on the basis of subjective complaints and an evaluation of a 1- to 2-week sleep diary completed by the patient. Actigraphy and PSG studies/sleep studies may be used for determining specific sleep disorders but are not necessary to make an initial insomnia diagnosis

31. A patient who arrives at the emergency department experiencing severe left knee pain is diagnosed with a patellar dislocation. The initial patient teaching by the nurse will focus on the need for a. a knee immobilizer. b. gentle knee flexion. c. monitored anesthesia care. d. physical activity restrictions.

ANS: C The first goal of collaborative management is realignment of the knee to its original anatomic position, which will require anesthesia or monitored anesthesia care (MAC), formerly called conscious sedation. Immobilization, gentle range-of-motion (ROM) exercises, and discussion about activity restrictions will be implemented after the knee is realigned. DIF: Cognitive Level: Apply (application) REF: 1508 OBJ: Special Questions: Prioritization TOP: Nursing Process: Implementation MSC: NCLEX: Physiological Integrity

23. A high school teacher with ulnar drift caused by rheumatoid arthritis (RA) is scheduled for a left hand arthroplasty. Which patient statement to the nurse indicates a realistic expectation for the surgery? a. "This procedure will correct the deformities in my fingers." b. "I will not have to do as many hand exercises after the surgery." c. "I will be able to use my fingers with more flexibility to grasp things." d. "My fingers will appear more normal in size and shape after this surgery."

ANS: C The goal of hand surgery in RA is to restore function, not to correct for cosmetic deformity or treat the underlying process. Hand exercises will be prescribed after the surgery. DIF: Cognitive Level: Apply (application) REF: 1535 TOP: Nursing Process: Evaluation MSC: NCLEX: Physiological Integrity

During infancy, what is the fastest growing part of the human body? a. Spinal cord b. Limb bones c. Head d. Vertebral column

ANS: C The head, not the spinal cord, limb bones, or vertebral column, is the fastest growing part of the body during infancy. REF: p. 422

32. Following a motorcycle accident, a 58-year-old patient arrives in the emergency department with massive left lower leg swelling. Which action will the nurse take first? a. Elevate the leg on 2 pillows. b. Apply a compression bandage. c. Check leg pulses and sensation. d. Place ice packs on the lower leg.

ANS: C The initial action by the nurse will be to assess the circulation to the leg and to observe for any evidence of injury such as fractures or dislocations. After the initial assessment, the other actions may be appropriate, based on what is observed during the assessment. DIF: Cognitive Level: Apply (application) REF: 1518 OBJ: Special Questions: Prioritization TOP: Nursing Process: Implementation MSC: NCLEX: Physiological Integrity

33. A pedestrian who was hit by a car is admitted to the emergency department with possible right lower leg fractures. The initial action by the nurse should be to a. elevate the right leg. b. splint the lower leg. c. check the pedal pulses. d. verify tetanus immunizations.

ANS: C The initial nursing action should be assessment of the neurovascular status of the injured leg. After assessment, the nurse may need to splint and elevate the leg, based on the assessment data. Information about tetanus immunizations should be done if there is an open wound. DIF: Cognitive Level: Apply (application) REF: 1518 OBJ: Special Questions: Prioritization TOP: Nursing Process: Implementation MSC: NCLEX: Physiological Integrity

18. After the health care provider has recommended amputation for a patient who has nonhealing ischemic foot ulcers, the patient tells the nurse that he would rather die than have an amputation. Which response by the nurse is best? a. "You are upset, but you may lose the foot anyway." b. "Many people are able to function with a foot prosthesis." c. "Tell me what you know about your options for treatment." d. "If you do not want an amputation, you do not have to have it."

ANS: C The initial nursing action should be to assess the patient's knowledge level and feelings about the options available. Discussion about the patient's option to not have the procedure, the seriousness of the condition, or rehabilitation after the procedure may be appropriate after the nurse knows more about the patient's current level of knowledge and emotional state. DIF: Cognitive Level: Apply (application) REF: 1531 TOP: Nursing Process: Implementation MSC: NCLEX: Psychosocial Integrity

A malignant tumor of striated muscle tissue origin commonly found on the neck is called a: a. myelogenic tumor. b. giant cell tumor. c. rhabdomyosarcoma. d. rhabdomyoma.

ANS: C The malignant tumor of striated muscle is called rhabdomyosarcoma. They are located in the muscle tissue of the head, neck, and genitourinary tract in 75% of cases. Myelogenic tumors originate from various bone marrow cells. Giant cell tumor is the sixth most common of the primary bone tumors. A rhabdomyoma is an extremely rare benign tumor of muscle that generally occurs in the tongue.

Which statement indicates the patient has a correct understanding of metastasis? The most common route of metastasis is through the blood vessels and: a. lung tissue. b. body cavities. c. lymphatics. d. connective tissues.

ANS: C The most common route of metastasis is through the lymphatics, not lung tissue, body cavities, or connective tissues. REF: p. 253

The most common type of tissue cancer occurring between ages 15 and 19 is: a. sarcoma. b. squamous cell. c. carcinoma. d. neuroma.

ANS: C The most common type of tissue cancer after adolescence is carcinoma, not sarcoma, squamous cell, or neuroma. REF: p. 301

5. A patient who has frequent migraines tells the nurse, "My life feels chaotic and out of my control. I could not manage if anything else happens." Which response should the nurse make initially? a. "Regular exercise may get your mind off the pain." b. "Guided imagery can be helpful in regaining control." c. "Tell me more about how your life has been recently." d. "Your previous coping resources can be helpful to you now."

ANS: C The nurse's initial strategy should be further assessment of the stressors in the patient's life. Exercise, guided imagery, or understanding how to use coping strategies that worked in the past may be of assistance to the patient, but more assessment is needed before the nurse can determine this. DIF: Cognitive Level: Apply (application) REF: 86 OBJ: Special Questions: Prioritization TOP: Nursing Process: Implementation MSC: NCLEX: Psychosocial Integrity

9. Which nursing action should be included in the plan of care for a patient returning to the surgical unit following a left modified radical mastectomy with dissection of axillary lymph nodes? a. Obtain a permanent breast prosthesis before the patient is discharged from the hospital. b. Teach the patient to use the ordered patient-controlled analgesia (PCA) every 10 minutes. c. Post a sign at the bedside warning against venipunctures or blood pressures in the left arm. d. Insist that the patient examine the surgical incision when the initial dressings are removed.

ANS: C The patient is at risk for lymphedema and infection if blood pressures or venipuncture are done on the right arm. The patient is taught to use the PCA as needed for pain control rather than at a set time. The nurse allows the patient to examine the incision and participate in care when the patient feels ready. Permanent breast prostheses are usually obtained about 6 weeks after surgery. DIF: Cognitive Level: Apply (application) TOP: Nursing Process: Planning MSC: NCLEX: Physiological Integrity

5. Which statement by a patient with stage 5 chronic kidney disease (CKD) indicates that the nurse's teaching about management of CKD has been effective? a. "I need to get most of my protein from low-fat dairy products." b. "I will increase my intake of fruits and vegetables to 5 per day." c. "I will measure my urinary output each day to help calculate the amount I can drink." d. "I need to take erythropoietin to boost my immune system and help prevent infection."

ANS: C The patient with end-stage renal disease is taught to measure urine output as a means of determining an appropriate oral fluid intake. Erythropoietin is given to increase the red blood cell count and will not offer any benefit for immune function. Dairy products are restricted because of the high phosphate level. Many fruits and vegetables are high in potassium and should be restricted in the patient with CKD. DIF: Cognitive Level: Apply (application) REF: 1082 TOP: Nursing Process: Evaluation MSC: NCLEX: Physiological Integrity

The nurse evaluating effectiveness of prescribed calcitonin and ibandronate (Boniva) for a patient with Paget's disease will consider the patient's a. oral intake. b. daily weight. c. grip strength. d. pain intensity.

ANS: D Bone pain is one of the common early manifestations of Paget's disease, and the nurse should assess the pain level to determine whether the treatment is effective. The other information will also be collected by the nurse, but will not be used in evaluating the effectiveness of the therapy.

Which information regarding a patient's sleep is most important for the nurse to communicate to the health care provider? a. 64-year-old nurse who works the night shift reports drinking hot chocolate before going to bed in the morning b. 21-year-old student who takes melatonin to assist in sleeping when traveling from the United States to Europe c. 41-year-old librarian who has a body mass index (BMI) of 42 kg/m2 says that the spouse complains about snoring d. 32-year-old accountant who is experiencing a stressful week uses diphenhydramine (Benadryl) for several nights

ANS: C The patient's BMI and snoring suggest possible sleep apnea, which can cause complications such as cardiac dysrhythmias, hypertension, and right-sided heart failure. Melatonin is safe to use as a therapy for jet lag. Short-term use of diphenhydramine in young adults is not a concern. Hot chocolate contains only 5 mg of caffeine and is unlikely to affect this patient's sleep quality

1. After the insertion of an arteriovenous graft (AVG) in the right forearm, a patient complains of pain and coldness of the right fingers. Which action should the nurse take? a. Teach the patient about normal AVG function. b. Remind the patient to take a daily low-dose aspirin tablet. c. Report the patient's symptoms to the health care provider. d. Elevate the patient's arm on pillows to above the heart level.

ANS: C The patient's complaints suggest the development of distal ischemia (steal syndrome) and may require revision of the AVG. Elevation of the arm above the heart will further decrease perfusion. Pain and coolness are not normal after AVG insertion. Aspirin therapy is not used to maintain grafts. DIF: Cognitive Level: Apply (application) REF: 1088 TOP: Nursing Process: Implementation MSC: NCLEX: Physiological Integrity

35. The second day after admission with a fractured pelvis, a 64-year-old patient suddenly develops confusion. Which action should the nurse take first? a. Take the blood pressure. b. Assess patient orientation. c. Check the oxygen saturation. d. Observe for facial asymmetry.

ANS: C The patient's history and clinical manifestations suggest a fat embolus. The most important assessment is oxygenation. The other actions are also appropriate but will be done after the nurse assesses gas exchange. DIF: Cognitive Level: Apply (application) REF: 1523 OBJ: Special Questions: Prioritization TOP: Nursing Process: Assessment MSC: NCLEX: Physiological Integrity

A 45-year-old female was recently diagnosed with cervical cancer. Which of the following is the most likely cause of her cancer? a. Herpes virus b. Rubella virus c. Human papillomavirus (HPV) d. Hepatitis B virus

ANS: C The presence of HPV is a factor in cervical cancer. The presence of herpes virus, rubella virus, or hepatitis B virus is not a factor in cervical cance

9. A hospitalized patient with diabetes tells the nurse, "I don't understand why I can keep my blood sugar under control at home with diet alone, but when I get sick, my blood sugar goes up. This is so frustrating." Which response by the nurse is accurate? a. "The liver is not able to metabolize glucose as well during stressful times." b. "Your diet at the hospital is the most likely cause of the increased glucose." c. "The stress of illness causes release of hormones that increase blood glucose." d. "It is probably coincidental that your blood glucose is higher when you are ill."

ANS: C The release of cortisol, epinephrine, and norepinephrine increase blood glucose levels. The increase in blood glucose is not coincidental. The liver does not control blood glucose. A patient with diabetes who is hospitalized will be on an appropriate diet to help control blood glucose. DIF: Cognitive Level: Apply (application) REF: 79 TOP: Nursing Process: Implementation MSC: NCLEX: Physiological Integrity

13. A patient newly diagnosed with stage I breast cancer is discussing treatment options with the nurse. Which statement by the patient indicates that additional teaching may be needed? a. There are several options that I can consider for treating the cancer. b. I will probably need radiation to the breast after having the surgery. c. Mastectomy is the best choice to decrease the chance of cancer recurrence. d. I can probably have reconstructive surgery at the same time as a mastectomy.

ANS: C The survival rates with lumpectomy and radiation or modified radical mastectomy are comparable. The other patient statements indicate a good understanding of stage I breast cancer treatment. DIF: Cognitive Level: Apply (application) TOP: Nursing Process: Assessment MSC: NCLEX: Physiological Integrity

A 32-year-old male was injured in a motor vehicle accident and confined to bed for 3 weeks. During this time, the size and strength of muscle fibers decreased, a condition referred to as: a. myelodysplasia. b. ischemic atrophy. c. disuse atrophy. d. fibromyalgia.

ANS: C The term disuse atrophy describes the pathologic reduction in normal size of muscle fibers after prolonged inactivity from bed rest, trauma (casting), or local nerve damage as can be seen with spinal cord trauma or polio. Ischemic atrophy is due to decreased blood flow. Such an experience would not result in either myelodysplasia or fibromyalgia.

9. When considering the various types of contraceptive drugs, the nurse is aware that which type most closely duplicates the normal hormonal levels of the female menstrual cycle? a. Monophasic b. Biphasic c. Triphasic d. Short acting

ANS: C The triphasic drugs most closely duplicate the normal hormonal levels of the female menstrual cycle. The other options are incorrect. DIF: COGNITIVE LEVEL: Understanding (Comprehension) REF: p. 540 TOP: NURSING PROCESS: Assessment MSC: NCLEX: Physiological Integrity: Physiological Adaptation

A patient is brought to the emergency department for treatment of a suspected overdose. The patient was found with an empty prescription bottle of a barbiturate by his bedside. He is lethargic and barely breathing. The nurse would expect which immediate intervention? a. Starting an intravenous infusion of diluted bicarbonate solution b. Administering medications to increase blood pressure c. Implementing measures to maintain the airway and support respirations d. Administrating naloxone (Narcan) as an antagonist

ANS: C There are no antagonists/antidotes for barbiturates. Treatment supports respirations and maintains the airway. The other interventions are not appropriate. DIF: COGNITIVE LEVEL: Applying (Application) REF: p. 195 TOP: NURSING PROCESS: Implementation MSC: NCLEX: Physiological Integrity: Pharmacological and Parenteral Therapies

A 3-year-old male is diagnosed with Duchenne muscular dystrophy. Treatment will include: a. aspirin. b. antivirals. c. steroids. d. chemotherapy.

ANS: C Treatment with steroids can prolong the ability to walk by several years and improves life expectancy. Treatment does not include aspirin, antivirals, or chemotherapy. REF: p. 1046

20. A patient with diabetes who has bacterial pneumonia is being treated with IV gentamicin 60 mg IV BID. The nurse will monitor for adverse effects of the medication by evaluating the patient's a. blood glucose. b. urine osmolality. c. serum creatinine. d. serum potassium.

ANS: C When a patient at risk for chronic kidney disease (CKD) receives a potentially nephrotoxic medication, it is important to monitor renal function with BUN and creatinine levels. The other laboratory values would not be useful in assessing for the adverse effects of the gentamicin. DIF: Cognitive Level: Apply (application) REF: 1083 TOP: Nursing Process: Evaluation MSC: NCLEX: Physiological Integrity

A 53-yr-old patient is scheduled for an annual physical examination. The nurse will plan to teach the patient about the purpose of a. urinalysis collection. b. uroflowmetry studies. c. prostate specific antigen (PSA) testing. d. transrectal ultrasound scanning (TRUS).

ANS: C An annual digital rectal exam (DRE) and PSA are usually recommended starting at age 50 years for men who have an average risk for prostate cancer. Urinalysis and uroflowmetry studies are done if patients have symptoms of urinary tract infection or changes in the urinary stream. TRUS may be ordered if the DRE or PSA results are abnormal.

The nurse taking a focused health history for a patient with possible testicular cancer will ask the patient about a history of a. testicular torsion. b. testicular trauma. c. undescended testicles. d. sexually transmitted infection (STI).

ANS: C Cryptorchidism is a risk factor for testicular cancer if it is not corrected before puberty. STI, testicular torsion, and testicular trauma are risk factors for other testicular conditions but not for testicular cancer.

The nurse will be giving ophthalmic drugs to a patient with glaucoma. Which drug is given intravenously to reduce intraocular pressure when other medications are not successful? a. Tobramycin (Tobrex) b. Bacitracin (AK-Tracin) c. Mannitol (Osmitrol) d. Ketorolac (Acular)

ANS: C Drugs used to reduce intraocular pressure include osmotic diuretics such as mannitol, which is given intravenously. Tobramycin and bacitracin are antibiotics; ketorolac has anti-inflammatory actions. DIF: COGNITIVE LEVEL: Remembering (Knowledge) REF: p. 911 TOP: NURSING PROCESS: Planning MSC: NCLEX: Physiological Integrity: Pharmacological and Parenteral Therapie

A patient who has been recently diagnosed with benign prostatic hyperplasia (BPH) tells the nurse that he does not want to have a transurethral resection of the prostate (TURP) because it might affect his ability to have sexual intercourse. Which action should the nurse take? a. Discuss alternative methods of sexual expression. b. Teach about medication for erectile dysfunction (ED). c. Clarify that TURP does not commonly affect erection. d. Offer reassurance that fertility is not affected by TURP.

ANS: C ED is not a concern with TURP, although retrograde ejaculation is likely, and the nurse should discuss this with the patient. Erectile function is not usually affected by a TURP, so the patient will not need information about penile implants or reassurance that other forms of sexual expression may be used. Because the patient has not asked about fertility, reassurance about sperm production does not address his concerns.

Which information will the nurse teach a patient who has chronic prostatitis? a. Ibuprofen (Motrin) should provide good pain control. b. Prescribed antibiotics should be taken for 7 to 10 days. c. Intercourse or masturbation will help relieve symptoms. d. Cold packs used every 4 hours will decrease inflammation.

ANS: C Ejaculation helps drain the prostate and relieve pain. Warm baths are recommended to reduce pain. Nonsteroidal antiinflammatory drugs (NSAIDs) are frequently prescribed but usually do not offer adequate pain relief. Antibiotics for chronic prostatitis are taken for 4 to 12 weeks.

The nurse is reviewing the use of ophthalmic preparations. Indications for the direct- and indirect-acting miotics include which condition? a. Cataracts b. Removal of foreign bodies c. Open-angle glaucoma d. Ocular infections

ANS: C Indications for the direct- and indirect-acting miotics include open-angle glaucoma, angle-closure glaucoma, ocular surgery, and convergent strabismus. DIF: COGNITIVE LEVEL: Remembering (Knowledge) REF: p. 906 TOP: NURSING PROCESS: Planning MSC: NCLEX: Physiological Integrity: Pharmacological and Parenteral Therapies

To determine the severity of the symptoms for a patient with benign prostatic hyperplasia (BPH), the nurse will ask the patient about a. blood in the urine. b. lower back or hip pain. c. force of urinary stream. d. erectile dysfunction (ED).

ANS: C The American Urological Association Symptom Index for a patient with BPH asks questions about the force and frequency of urination, nocturia, and so on. Blood in the urine, ED, and back or hip pain are not typical symptoms of BPH.

The plan of care for a patient immediately after a perineal radical prostatectomy will include decreasing the risk for infection related to a. urinary incontinence. b. prolonged urinary stasis. c. fecal wound contamination. d. suprapubic catheter placement.

ANS: C The perineal approach increases the risk for infection because the incision is located close to the anus, and contamination with feces is possible. Urinary stasis and incontinence do not occur because the patient has a retention catheter in place for 1 to 2 weeks. A urethral catheter is used after the surgery.

Which information will the nurse include when teaching a patient with acute low back pain (select all that apply)? a. Sleep in a prone position with the legs extended. b. Keep the knees straight when leaning forward to pick something up. c. Expect symptoms of acute low back pain to improve in a few weeks. d. Avoid activities that require twisting of the back or prolonged sitting. e. Use ibuprofen (Motrin, Advil) or acetaminophen (Tylenol) to relieve pain.

ANS: C, D, E Acute back pain usually starts to improve within 2 weeks. In the meantime, the patient should use medications such as nonsteroidal antiinflammatory drugs (NSAIDs) or acetaminophen to manage pain and avoid activities that stress the back. Sleeping in a prone position and keeping the knees straight when leaning forward will place stress on the back, and should be avoided.

6. The nurse will prepare to give which preparation to a newborn upon arrival in the nursery after delivery? a. vitamin B1 (thiamine) b. vitamin D (calciferol) c. folic acid d. vitamin K (AquaMEPHYTON)

ANS: D Deficiency in vitamin K can be seen in newborns because of malabsorption attributed to inadequate amounts of bile. AquaMEPHYTON is given as a single intramuscular dose for infants upon arrival in the nursery.

Which information obtained by the nurse about an older adult who complains of occasional insomnia indicates a need for patient teaching (select all that apply)? a. Drinks a cup of coffee every morning with breakfast b. Has a snack every evening 1 hour before going to bed c. Likes to read or watch television in bed on most evenings d. Usually takes a warm bath just before bedtime every night e. Uses diphenhydramine (Benadryl) as an occasional sleep aid

ANS: C, E Reading and watching television in bed may contribute to insomnia. Older adults should avoid the use of medications that have anticholinergic effects, such as diphenhydramine. Having a snack 1 hour before bedtime and/or coffee early in the day should not affect sleep quality. Rituals such as a warm bath before bedtime can enhance sleep quality

A 75-year-old woman has been given a nonsteroidal anti-inflammatory drug (an NSAID for the treatment of rheumatoid arthritis. The nurse is reviewing the patient's medication history and notes that which types of medications could have an interaction with the NSAID? (Select all that apply.) a. Antibiotics b. Decongestants c. Anticoagulants d. Beta blockers e. Diuretics f. Corticosteroids

ANS: C, E, F Anticoagulants taken with NSAIDs may cause increased bleeding tendencies because of platelet inhibition and hypoprothrombinemia. NSAIDs taken with diuretics may cause reduced hypotensive and diuretic effects. NSAIDs taken with corticosteroids may cause increased ulcerogenic effects. See Table 44-5. The other options are incorrect.

A patient with age-related macular degeneration (AMD) has just had photodynamic therapy. Which statement by the patient indicates that the discharge teaching has been effective? a. "I will use drops to keep my pupils dilated until my appointment." b. "I will need to use brighter lights to read for at least the next week." c. "I will not use facial lotions near my eyes during the recovery period." d. "I will cover up with long-sleeved shirts and pants for the next 5 days."

ANS: D

In reviewing a patient's medical record, the nurse notes that the last eye examination revealed an intraocular pressure of 28 mm Hg. What should the nurse plan to assess?a. Visual acuity b. Pupil reaction c. Color perception d. Peripheral vision

ANS: D

The nurse at the eye clinic made a follow-up telephone call to a patient who underwent cataract extraction and intraocular lens implantation the previous day. Which information is the priority to communicate to the health care provider? a. The patient reports that the vision has not improved. b. The patient requests a prescription refill for next week. c. The patient feels uncomfortable wearing an eye patch. d. The patient reports eye pain rated 5 (on a 0 to 10 scale).

ANS: D

The nurse in the eye clinic is examining a 67-yr-old patient who says, "I see small spots that move around in front of my eyes." Which action will the nurse take first? a. Immediately have the ophthalmologist evaluate the patient. b. Explain that spots and "floaters" are a normal part of aging. c. Warn the patient that these spots may indicate retinal damage. d. Use an ophthalmoscope to examine the posterior eye chambers.

ANS: D

The nurse is working in an urgent care clinic that has standardized treatment protocols for implementation by nursing staff. After reviewing the history, physical assessment, and vital signs for a 60-yr-old patient as shown in the accompanying figure, which action should the nurse take first? a. Check the patient's blood glucose level. b. Take the blood pressure on the left arm. c. Use an irrigating syringe to clean the ear canals. d. Report a vision change to the health care provider.

ANS: D

Which finding by the nurse performing an eye examination indicates that the patient has normal accommodation? a. After covering one eye for 1 minute, the pupil constricts as the cover is removed. b. Shining a light into the patient's eye causes pupil constriction in the opposite eye. c. A blink reaction occurs after touching the patient's pupil with a piece of sterile cotton. d. The pupils constrict while fixating on an object being moved toward the patient's eyes.

ANS: D

Which teaching point should the nurse plan to include when caring for a patient whose vision is corrected to 20/200? a. How to access audio books? b. How to use a white cane safely? c. Where Braille instruction is available? d. Where to obtain hand-held magnifiers?

ANS: D

A patient who has been receiving a heparin infusion and warfarin (Coumadin) for a deep vein thrombosis (DVT) is diagnosed with heparin-induced thrombocytopenia (HIT) when her platelet level drops to 110,000/L. Which action will the nurse include in the plan of care? a. Use low-molecular-weight heparin (LMWH) only. b. Administer the warfarin (Coumadin) at the scheduled time. c. Teach the patient about the purpose of platelet transfusions. d. Discontinue heparin and flush intermittent IV lines using normal saline.

ANS: D All heparin is discontinued when the HIT is diagnosed. The patient should be instructed to never receive heparin or LMWH. Warfarin is usually not given until the platelet count has returned to 150,000/L. The platelet count does not drop low enough in HIT for a platelet transfusion, and platelet transfusions increase the risk for thrombosis.

Which assessment finding for a patient who has had surgical reduction of an open fracture of the right radius requires notification of the health care provider? a. Serous wound drainage b. Right arm muscle spasms c. Right arm pain with movement d. Temperature 101.4° F (38.6° C)

ANS: D An elevated temperature is suggestive of possible osteomyelitis. The other clinical manifestations are typical after a repair of an open fracture.

Which statement by a patient indicates good understanding of the nurses teaching about prevention of sickle cell crisis? a. Home oxygen therapy is frequently used to decrease sickling. b. There are no effective medications that can help prevent sickling. c. Routine continuous dosage narcotics are prescribed to prevent a crisis. d. Risk for a crisis is decreased by having an annual influenza vaccination.

ANS: D Because infection is the most common cause of a sickle cell crisis, influenza, Haemophilus influenzae, pneumococcal pneumonia, and hepatitis immunizations should be administered. Although continuous dose opioids and oxygen may be administered during a crisis, patients do not receive these therapies to prevent crisis. Hydroxyurea (Hydrea) is a medication used to decrease the number of sickle cell crises.

Which laboratory test will the nurse use to determine whether filgrastim (Neupogen) is effective for a patient with acute lymphocytic leukemia who is receiving chemotherapy? a. Platelet count b. Reticulocyte count c. Total lymphocyte count d. Absolute neutrophil count

ANS: D Filgrastim increases the neutrophil count and function in neutropenic patients. Although total lymphocyte, platelet, and reticulocyte counts also are important to monitor in this patient, the absolute neutrophil count is used to evaluate the effects of filgrastim.

4. The nurse is counseling a patient about calcium supplements. Which dietary information is appropriate during this teaching session? a. "Take oral calcium supplements with meals." b. "There are no drug interactions with calcium products." c. "Avoid foods that are high in calcium, such as beef, egg yolks, and liver." d. "Be sure to eat foods high in calcium, such as dairy products and salmon."

ANS: D Foods high in calcium include dairy products, fortified cereals, calcium-fortified orange juice, sardines, and salmon. Patients can be encouraged to add dietary sources of calcium to their diets. Oral-dosage forms of calcium need to be given 1 to 3 hours after meals. Calcium salts will bind with tetracycline and quinolone antibiotics and result in an insoluble complex.

A patient who has non-Hodgkins lymphoma is receiving combination treatment with rituximab (Rituxan) and chemotherapy. Which patient assessment finding requires the most rapid action by the nurse? a. Anorexia b. Vomiting c. Oral ulcers d. Lip swelling

ANS: D Lip swelling in angioedema may indicate a hypersensitivity reaction to the rituximab. The nurse should stop the infusion and further assess for anaphylaxis. The other findings may occur with chemotherapy, but are not immediately life threatening.

A critical action by the nurse caring for a patient with an acute exacerbation of polycythemia vera is to a. place the patient on bed rest. b. administer iron supplements. c. avoid use of aspirin products. d. monitor fluid intake and output.

ANS: D Monitoring hydration status is important during an acute exacerbation because the patient is at risk for fluid overload or underhydration. Aspirin therapy is used to decrease risk for thrombosis. The patient should be encouraged to ambulate to prevent deep vein thrombosis (DVT). Iron is contraindicated in patients with polycythemia vera.

Which laboratory result will the nurse expect to show a decreased value if a patient develops heparin-induced thrombocytopenia (HIT)? a. Prothrombin time b. Erythrocyte count c. Fibrinogen degradation products d. Activated partial thromboplastin time

ANS: D Platelet aggregation in HIT causes neutralization of heparin, so that the activated partial thromboplastin time will be shorter and more heparin will be needed to maintain therapeutic levels. The other data will not be affected by HIT.

A 54-yr-old woman who recently reached menopause and has a family history of osteoporosis is diagnosed with osteopenia following densitometry testing. In teaching the woman, the nurse explains that a. with a family history of osteoporosis, there is no way to prevent or slow bone resorption. b. estrogen replacement therapy must be started to prevent rapid progression to osteoporosis. c. continuous, low-dose corticosteroid treatment is effective in stopping the course of osteoporosis. d. calcium loss from bones can be slowed by increasing calcium intake and weight-bearing exercise.

ANS: D Progression of osteoporosis can be slowed by increasing calcium intake and weight-bearing exercise. Estrogen replacement therapy does help prevent osteoporosis, but it is not the only treatment and is not appropriate for some patients. Corticosteroid therapy increases the risk for osteoporosis.

Which menu choice by a patient with osteoporosis indicates the nurse's teaching about appropriate diet has been effective? a. Pancakes with syrup and bacon b. Whole wheat toast and fresh fruit c. Egg-white omelet and a half grapefruit d. Oatmeal with skim milk and fruit yogurt

ANS: D Skim milk and yogurt are high in calcium. The other choices do not contain any high-calcium foods.

Following successful treatment of Hodgkins lymphoma for a 55-year-old woman, which topic will the nurse include in patient teaching? a. Potential impact of chemotherapy treatment on fertility b. Application of soothing lotions to treat residual pruritus c. Use of maintenance chemotherapy to maintain remission d. Need for follow-up appointments to screen for malignancy

ANS: D The chemotherapy used in treating Hodgkins lymphoma results in a high incidence of secondary malignancies; follow-up screening is needed. The fertility of a 55-year-old woman will not be impacted by chemotherapy. Maintenance chemotherapy is not used for Hodgkins lymphoma. Pruritus is a clinical manifestation of lymphoma, but should not be a concern after treatment.

The nurse will determine more teaching is needed if a patient with discomfort from a bunion says, "I will a. give away my high-heeled shoes." b. take ibuprofen (Motrin) if I need it." c. use the bunion pad to cushion the area." d. only wear sandals, no closed-toe shoes."

ANS: D The patient can wear shoes that have a wide forefoot. The other patient statements indicate that the teaching has been effective.

A 62-year old man with chronic anemia is experiencing increased fatigue and occasional palpitations at rest. The nurse would expect the patients laboratory findings to include a. a hematocrit (Hct) of 38%. b. an RBC count of 4,500,000/mL. c. normal red blood cell (RBC) indices. d. a hemoglobin (Hgb) of 8.6 g/dL (86 g/L).

ANS: D The patients clinical manifestations indicate moderate anemia, which is consistent with a Hgb of 6 to 10 g/dL. The other values are all within the range of normal.

A patient in the emergency department complains of back pain and difficulty breathing 15 minutes after a transfusion of packed red blood cells is started. The nurses first action should be to a. administer oxygen therapy at a high flow rate. b. obtain a urine specimen to send to the laboratory. c. notify the health care provider about the symptoms. d. disconnect the transfusion and infuse normal saline.

ANS: D The patients symptoms indicate a possible acute hemolytic reaction caused by the transfusion. The first action should be to disconnect the transfusion and infuse normal saline. The other actions also are needed but are not the highest priority.

10. A patient accidentally took an overdose of the anticoagulant warfarin (Coumadin), and the nurse is preparing to administer vitamin K as an antidote. Which statement about vitamin K is accurate? a. The vitamin K dose will be given intramuscularly. b. The patient will take oral doses of vitamin K after the initial injection. c. The vitamin K cannot be given if the patient has renal disease. d. The patient will be unresponsive to warfarin therapy for 1 week after the vitamin K is given.

ANS: D When vitamin K is used as an antidote to warfarin therapy, the patient becomes unresponsive to warfarin for approximately 1 week after vitamin K administration. The use of vitamin K products is contraindicated in patients who are in the last few weeks of pregnancy and in patients with severe hepatic disease. Vitamin K is given subcutaneously and not intramuscularly when used to reverse warfarin effects.

Which term is used to document a hernial protrusion of a saclike cyst through a defect in the posterior arch of a vertebra? a. Craniosynostosis b. Meningocele c. Encephalocele d. Myelomeningocele

ANS: D A myelomeningocele is a hernial protrusion of a saclike cyst (containing meninges, spinal fluid, and a portion of the spinal cord with its nerves) through a defect in the posterior arch of a vertebra in the lower spine. Craniosynostosis is the premature closure of one or more of the cranial sutures (saggital, coronal, lambdoid, metopic) during the first 18-20 months of the infant's life. A meningocele, a saclike cyst of meninges filled with spinal fluid, is a mild form of posterior neural tube closure defect. These can occur throughout the cervical, thoracic, and lumbar spine areas. An encephalocele refers to a herniation or protrusion of brain and meninges through a defect in the occipital region of the skull, resulting in a saclike structure. REF: p. 425

34. The day after a 60-year-old patient has an open reduction and internal fixation (ORIF) for an open, displaced tibial fracture, the priority nursing diagnosis is a. activity intolerance related to deconditioning. b. risk for constipation related to prolonged bed rest. c. risk for impaired skin integrity related to immobility. d. risk for infection related to disruption of skin integrity.

ANS: D A patient having an ORIF is at risk for problems such as wound infection and osteomyelitis. After an ORIF, patients typically are mobilized starting the first postoperative day, so problems caused by immobility are not as likely. DIF: Cognitive Level: Apply (application) REF: 1514 OBJ: Special Questions: Prioritization TOP: Nursing Process: Diagnosis MSC: NCLEX: Physiological Integrity

A patient with chronic insomnia asks the nurse about ways to improve sleep quality. What is the nurse's best response? a. Avoid aerobic exercise during the day. b. Read in bed for a few minutes each night. c. Keep the bedroom temperature warm. d. Go to bed at the same time every evening.

ANS: D A regular evening schedule is recommended to improve sleep time and quality. Aerobic exercise may improve sleep quality but should occur at least 6 hours before bedtime. Reading in bed is discouraged for patients with insomnia. The bedroom temperature should be slightly cool.

22. Which action will the nurse include in the plan of care for a patient who has had a total right knee arthroplasty? a. Avoid extension of the right knee beyond 120 degrees. b. Use a compression bandage to keep the right knee flexed. c. Teach about the need to avoid weight bearing for 4 weeks. d. Start progressive knee exercises to obtain 90-degree flexion.

ANS: D After knee arthroplasty, active or passive flexion exercises are used to obtain a 90-degree flexion of the knee. The goal for extension of the knee will be 180 degrees. A compression bandage is used to hold the knee in an extended position after surgery. Full weight bearing is expected before discharge. DIF: Cognitive Level: Apply (application) REF: 1535 TOP: Nursing Process: Planning MSC: NCLEX: Physiological Integrity

Which of the following diseases does the nurse screen for in all newborns? a. Epilepsy b. Tay-Sachs disease c. Pica d. Phenylketonuria (PKU)

ANS: D All newborns are screened for PKU. Not all newborns are screened for epilepsy, Tay-Sachs, or pica. REF: p. 430

27. The outpatient clinic receives telephone calls from four patients. Which patient should the nurse call back first? a. 57-year-old with ductal ectasia who has sticky multicolored nipple discharge and severe nipple itching b. 21-year-old with a family history of breast cancer who wants to discuss genetic testing for the BRCA gene c. 40-year-old who still has left side chest and arm pain 2 months after a left modified radical mastectomy d. 50-year-old with stage 2 breast cancer who is receiving doxorubicin (Adriamycin) and has ankle swelling and fatigue

ANS: D Although all the patients have needs that the nurse should address, the patient who is receiving a cardiotoxic medication and has symptoms of heart failure should be assessed by the nurse first. BRCA testing may be appropriate for the 21-year-old, but it does not need to be done immediately. Chest and arm pain are normal up to 3 months after mastectomy. Nipple discharge and itching is a common finding with ductal ectasia. DIF: Cognitive Level: Analyze (analysis) OBJ: Special Questions: Prioritization; Multiple Patients TOP: Nursing Process: Planning MSC: NCLEX: Safe and Effective Care Environment

A patient who has a history of coronary artery disease has been instructed to take one 81-mg aspirin tablet a day. The patient asks about the purpose of this aspirin. Which response by the nurse is correct? a. "Aspirin is given reduce anxiety." b. "It helps to reduce inflammation." c. "Aspirin is given to relieve pain." d. "It will help to prevent clot formation."

ANS: D Aspirin can reduce platelet aggregation; low doses of aspirin (81 to 325 mg once daily) are used for thromboprevention. Higher doses are required for pain relief, reduction of inflammation, and reduction of fever. The other options are incorrect.

A patient has been taking phenobarbital for 2 weeks as part of his therapy for epilepsy. He tells the nurse that he feels tense and that "the least little thing" bothers him now. Which is the correct explanation for this problem? a. These are adverse effects that usually subside after a few weeks. b. The drug must be stopped immediately because of possible adverse effects. c. This drug causes the rapid eye movement (REM) sleep period to increase, resulting in nightmares and restlessness. d. This drug causes deprivation of REM sleep and may cause the inability to deal with normal stress.

ANS: D Barbiturates such as phenobarbital deprive people of REM sleep, which can result in agitation and the inability to deal with normal stress. A rebound phenomenon occurs when the drug is stopped (not during therapy), and the proportion of REM sleep increases, sometimes resulting in nightmares. The other options are incorrect. DIF: COGNITIVE LEVEL: Understanding (Comprehension) REF: p. 194 TOP: NURSING PROCESS: Evaluation MSC: NCLEX: Physiological Integrity: Basic Care and Comfort

When caring for patients with sleep disorders, which activity can the nurse appropriately delegate to unlicensed assistive personnel (UAP)? a. Interview a new patient about risk factors for obstructive sleep disorders. b. Discuss the benefits of oral appliances in decreasing obstructive sleep apnea. c. Help a patient choose an appropriate continuous positive airway pressure (CPAP) mask. d. Help the patient put on the CPAP device at bedtime.

ANS: D Because CPAP mask placement is consistently done in the same way, this is appropriate to delegate to UAP. The other actions require critical thinking and nursing judgment, and should be done by the RN.

19. The nurse is admitting a patient scheduled this morning for lumpectomy and axillary lymph node dissection. Which action should the nurse take first? a. Teach the patient how to deep breathe and cough. b. Discuss options for postoperative pain management. c. Explain the postdischarge care of the axillary drains. d. Ask the patient to describe what she knows about the surgery.

ANS: D Before teaching, the nurse should assess the patients current knowledge level. The other teaching also may be appropriate, depending on the assessment findings. DIF: Cognitive Level: Apply (application) OBJ: Special Questions: Prioritization TOP: Nursing Process: Implementation MSC: NCLEX: Physiological Integrity

Which information should the nurse include when teaching about angiogenic factors? In cancer, angiogenic factors stimulate: a. release of growth factors. b. tumor regression. c. apoptosis. d. new blood vessel growth.

ANS: D Cancers can secrete multiple factors that stimulate new blood vessel growth called angiogenesis, not release of growth factors or tumor regression. Apoptosis is cell death. REF: pp. 245-246

The nurse is teaching a patient who is taking colchicine for the treatment of gout. Which instruction will the nurse include during the teaching session? a. "Fluids should be restricted while on colchicine therapy." b. "Take colchicine with meals." c. "The drug will be discontinued when symptoms are reduced." d. "Call your doctor if you have increased pain or blood in the urine."

ANS: D Colchicine may cause renal effects; therefore, these symptoms must be reported immediately. The drug is taken on an empty stomach for better absorption, and fluids should be increased unless contraindicated. Successful treatment depends upon continuing the medication as ordered.

A patient has used enteric aspirin for several years as treatment for osteoarthritis. However, the symptoms are now worse and she is given a prescription for a nonsteroidal anti-inflammatory drug and misoprostol (Cytotec). The patient asks the nurse, "Why am I now taking two pills for arthritis?" What is the nurse's best response? a. "Cytotec will also reduce the symptoms of your arthritis." b. "Cytotec helps the action of the NSAID so that it will work better." c. "Cytotec reduces the mucous secretions in the stomach, which reduces gastric irritation." d. "Cytotec may help to prevent gastric ulcers that may occur in patients taking NSAIDs."

ANS: D Cytotec inhibits gastric acid secretions and stimulates mucous secretions; it has proved successful in preventing the gastric ulcers that may occur in patients taking NSAIDs.

A patient has been receiving epoetin alfa (Epogen) for severe iron-deficiency anemia. Today, the provider changed the order to darbepoetin (Aranesp). The patient questions the nurse, "What is the difference in these drugs?" Which response by the nurse is correct? a. "There is no difference in these two drugs." b. "Aranesp works faster than Epogen to raise your red blood cell count." c. "Aranesp is given by mouth, so you will not need to have injections." d. "Aranesp is a longer-acting form, so you will receive fewer injections."

ANS: D Darbepoetin (Aranesp) is longer-acting than epoetin alfa (Epogen); therefore, fewer injections are required. The other options are incorrect.

2. The nurse recognizes that use of estrogen drugs is contraindicated in which patient? a. A patient who has atrophic vaginitis b. A patient who has inoperable prostate cancer c. A woman who has just given birth and wants to prevent postpartum lactation d. A woman with a history of thrombophlebitis

ANS: D Estrogenic drugs are contraindicated in people who have active thromboembolic disorders and in those with histories of thromboembolic disease. Atrophic vaginitis and inoperable prostate cancer are potential indications for estrogen therapy. Estrogen is not used to prevent lactation. DIF: COGNITIVE LEVEL: Understanding (Comprehension) REF: p. 537 TOP: NURSING PROCESS: Assessment MSC: NCLEX: Physiological Integrity: Pharmacological and Parenteral Therapies

A patient has experienced insomnia for months, and the physician has prescribed a medication to help with this problem. The nurse expects which drug to be used for long-term treatment of insomnia? a. Secobarbital (Seconal), a barbiturate b. Diazepam (Valium), a benzodiazepine c. Midazolam (Versed), a benzodiazepine d. Eszopiclone (Lunesta), a nonbenzodiazepine sleep aid

ANS: D Eszopiclone (Lunesta) is one of the newest prescription hypnotics to be approved for long-term use in treatment of insomnia. Barbiturates and benzodiazepines are not appropriate for long-term treatment of insomnia; midazolam is used for procedural (moderate) sedation. DIF: COGNITIVE LEVEL: Understanding (Comprehension) REF: p. 193 TOP: NURSING PROCESS: Planning MSC: NCLEX: Physiological Integrity: Pharmacological and Parenteral Therapies

Ewing sarcoma originates from: a. osteoblasts. b. epithelial cells. c. the spleen. d. the bone marrow.

ANS: D Ewing sarcoma originates from the bone marrow. This tumor does not originate from osteoblasts, epithelial cells, or the spleen. REF: p. 1049

Fibromyalgia is a chronic musculoskeletal disorder characterized by: a. pain resulting from joint and muscle inflammation. b. muscle pain in the back and gastrointestinal symptoms. c. neurologic pain in the skeletal muscles. d. diffuse pain, fatigue, and tender points.

ANS: D Fibromyalgia is characterized by diffuse pain, fatigue, and point tenderness and the absence of systemic or localized inflammation. Fibromyalgia is not related to specific back pain and is not manifested by gastrointestinal upset. Nor is it related to neurologic pain in muscles.

What nutrient should the nurse encourage a woman in the early stages of pregnancy to consume to prevent neural tube defects? a. Protein b. Iron c. Vitamin D d. Folic acid

ANS: D Folic acid deficiency during early stages of pregnancy increases the risk for neural tube defects. Neural tube defects are not associated with deficiencies of protein, iron, or vitamin D. REF: pp. 424-425

The nurse is administering folic acid to a patient with a new diagnosis of anemia. Which statement about treatment with folic acid is true? a. Folic acid is used to treat any type of anemia. b. Folic acid is used to treat iron-deficiency anemia. c. Folic acid is used to treat pernicious anemia. d. The specific cause of the anemia needs to be determined before treatment.

ANS: D Folic acid should not be used to treat anemias until the underlying cause and type of anemia have been identified. Administering folic acid to a patient with pernicious anemia may correct the hematologic changes of anemia, but the symptoms of pernicious anemia (which is due to a vitamin B12 deficiency, not a folic acid deficiency) may be deceptively masked. The other options are incorrect.

8. An obese female patient who had enjoyed active outdoor activities is stressed because osteoarthritis in her hips now limits her activity. Which action by the nurse will best assist the patient to cope with this situation? a. Have the patient practice frequent relaxation breathing. b. Ask the patient what outdoor activities she misses the most. c. Teach the patient to use imagery for reducing pain and stress. d. Encourage the patient to consider weight loss to improve symptoms.

ANS: D For problems that can be changed or controlled, problem-focused coping strategies, such as encouraging the patient to lose weight, are most helpful. The other strategies also may assist the patient in coping with her problem, but they will not be as helpful as a problem-focused strategy. DIF: Cognitive Level: Analyze (analysis) REF: 86 TOP: Nursing Process: Implementation MSC: NCLEX: Psychosocial Integrity

Mobility for the patient changes throughout the life span. What is the term that best describes this process? a. aging and illness. b. illness and disease. c. health and wellness. d. growth and development.

ANS: D Growth and development happens from infancy to death. Muscular changes are always happening, and these changes affect the individual and his or her performance in life. Aging, illness, health, and wellness do have an effect on a person, but they don't always affect mobility.

The nurse is reviewing the medical record of a patient before giving a new order for iron sucrose (Venofer). Which statement regarding the administration of iron sucrose is correct? a. The medication is given with food to reduce gastric distress. b. Iron sucrose is contraindicated if the patient has renal disease. c. A test dose will be administered before the full dose is given. d. The nurse will monitor the patient for hypotension during the infusion.

ANS: D Iron sucrose (Venofer) is an injectable iron product indicated for the treatment of iron-deficiency anemia in patients with chronic renal disease. It is also used for patients without kidney disease. Its risk of precipitating anaphylaxis is much less than that of iron dextran, and a test dose is not required. Hypotension is the most common adverse effect and appears to be related to infusion rate. Low-weight elderly patients appear to be at greatest risk for hypotension.

A woman who is planning to become pregnant should ensure that she receives adequate levels of which supplement to reduce the risk for fetal neural tube defects? a. Vitamin B12 b. Vitamin D c. Iron d. Folic acid

ANS: D It is recommended that administration of folic acid be begun at least 1 month before pregnancy and continue through early pregnancy to reduce the risk for fetal neural tube defects.

24. A patient has had left-sided lumpectomy (breast-conservation surgery) and an axillary lymph node dissection. Which nursing intervention is appropriate to delegate to a licensed practical/vocational nurse (LPN/LVN)? a. Teaching the patient how to avoid injury to the left arm b. Assessing the patients range of motion for the left arm c. Evaluating the patients understanding of instructions about drain care d. Administering an analgesic 30 minutes before scheduled arm exercises

ANS: D LPN/LVN education and scope of practice include administration and evaluation of the effects of analgesics. Assessment, teaching, and evaluation of a patients understanding of instructions are more complex tasks that are more appropriate to RN level education and scope of practice. DIF: Cognitive Level: Apply (application) OBJ: Special Questions: Delegation TOP: Nursing Process: Planning MSC: NCLEX: Safe and Effective Care Environment

20. When the nurse is working in the womens health care clinic, which action is appropriate to take? a. Teach a healthy 30-year-old about the need for an annual mammogram. b. Discuss scheduling an annual clinical breast examination with a 22-year-old. c. Explain to a 60-year-old that mammography frequency can be reduced to every 3 years. d. Teach a 28-year-old with a BRCA-1 mutation about magnetic resonance imaging (MRI).

ANS: D MRI (in addition to mammography) is recommended for women who are at high risk for breast cancer. A young woman should have a clinical breast exam every 3 years. Annual mammograms are recommended for women over 50. DIF: Cognitive Level: Apply (application) TOP: Nursing Process: Planning MSC: NCLEX: Health Promotion and Maintenance

3. A patient is being treated for secondary amenorrhea. The nurse expects which drug to be used to treat this problem? a. Methylergonovine (Methergine) b. Estradiol transdermal (Estraderm) c. Raloxifene (Evista) d. Medroxyprogesterone (Provera)

ANS: D Medroxyprogesterone, a progestin, is one of the drugs most commonly used for secondary amenorrhea. Secondary amenorrhea is not an indication for the other drugs listed. DIF: COGNITIVE LEVEL: Understanding (Comprehension) REF: p. 538 TOP: NURSING PROCESS: Planning MSC: NCLEX: Physiological Integrity: Pharmacological and Parenteral Therapies

34. A patient complains of leg cramps during hemodialysis. The nurse should a. massage the patient's legs. b. reposition the patient supine. c. give acetaminophen (Tylenol). d. infuse a bolus of normal saline.

ANS: D Muscle cramps during dialysis are caused by rapid removal of sodium and water. Treatment includes infusion of normal saline. The other actions do not address the reason for the cramps. DIF: Cognitive Level: Apply (application) REF: 1091 TOP: Nursing Process: Implementation MSC: NCLEX: Physiological Integrity

The nurse is teaching a patient with iron-deficiency anemia about foods to increase iron intake. Which food may enhance the absorption of oral iron forms? a. Milk b. Yogurt c. Antacids d. Orange juice

ANS: D Orange juice contains ascorbic acid, which enhances the absorption of oral iron forms; antacids, milk, and yogurt may interfere with absorption.

Osteomalacia is a result of: a. collagen breakdown in the bone matrix. b. excessive bone resorption. c. crowding of bone marrow by excessive bone growth. d. inadequate bone mineralization.

ANS: D Osteomalacia is a metabolic disease characterized by inadequate and delayed mineralization of osteoid in mature compact and spongy bone. Idiopathic osteoarthritis leads to collagen breakdown. Giant cell tumors promote excessive bone resorption. Abnormal remodeling causes crowding of bone marrow.

Today most patients are living for several years before dying with multiple chronic conditions, such as COPD, congestive heart failure, diabetes, and obesity. These concomitant diseases contribute to multiple symptoms that interfere with the patient's quality of life. What type of care would you consider for this patient? a. End-of-life care b. Supportive care c. Comfort care d. Palliative care

ANS: D Palliative care provides optimal symptom management in the setting of multiple chronic conditions. The relief and management of these symptoms help to promote improved quality of life for the patient and help to maintain physical functioning.

The nurse is talking to the unlicensed assistive personnel about moving a patient in bed. The nurse knows the unlicensed assistive personnel understands the concept of mobility and proper moving techniques when he or she states, "Patients must a. have a trapeze over the bed to move properly." b. move themselves in bed to prevent immobility." c. always have a two-person assist to move in bed." d. be moved correctly in bed to prevent shearing."

ANS: D Patients must be moved properly in bed to prevent shearing of the skin. Having a trapeze over the bed is only functional is the patient can assist in the moving process. A two-person assist is good, but the patient still needs to be moved properly. A patient may move himself or herself if he or she is able, but shearing may still occur.

A patient tells the nurse that he likes to drink kava herbal tea to help him relax. Which statement by the patient indicates that additional teaching about this herbal product is needed? a. "I will not drink wine with the kava tea." b. "If I notice my skin turning yellow, I will stop taking the tea." c. "I will not take sleeping pills if I have this tea in the evening." d. "I will be able to drive my car after drinking this tea."

ANS: D Patients should not drive after drinking this tea because it may cause sedation. Kava tea may cause skin discoloration (with long-term use). In addition, it must not be taken with alcohol, barbiturates, and psychoactive drugs. DIF: COGNITIVE LEVEL: Applying (Application) REF: p. 192 TOP: NURSING PROCESS: Evaluation MSC: NCLEX: Physiological Integrity: Reduction of Risk Potential

In order to provide the best intervention for a patient, the nurse is often responsible for obtaining a sample of exudate for culture. This test will identify a. whether a patient has an infection. b. where an infection is located. c. what cells are being utilized by the body to attack an infection. d. what specific type of pathogen is causing an infection.

ANS: D People can transmit pathogens even if they don't currently feel ill. Some carriers never experience the full symptoms of a pathogen. A CBC will identify that the patient has an infection. Inspection and radiography will help identify where an infection is located. The CBC with differential will identify the white blood cells being used by the body to fight an infection. The culture will grow the microorganisms in the sample for identification of the specific type of pathogen.

1. The nurse teaching a young womens community service group about breast self-examination (BSE) will include that a. BSE will reduce the risk of dying from breast cancer. b. BSE should be done daily while taking a bath or shower. c. annual mammograms should be scheduled in addition to BSE. d. performing BSE after the menstrual period is more comfortable.

ANS: D Performing BSE at the end of the menstrual period will reduce the breast tenderness associated with the procedure. The evidence is not clear that BSE reduces mortality from breast cancer. BSE should be done monthly. Annual mammograms are not routinely scheduled for women under age 40, and newer guidelines suggest delaying them until age 50. DIF: Cognitive Level: Apply (application) TOP: Nursing Process: Implementation MSC: NCLEX: Health Promotion and Maintenance

10. A woman visits a health center requesting oral contraceptives. Which laboratory test is most important for the nurse to assess before the patient begins oral contraceptive therapy? a. Complete blood count b. Serum potassium level c. Vaginal cultures d. Pregnancy test

ANS: D Pregnancy should be ruled out before beginning oral contraceptive therapy because the medications can be harmful to the fetus; they are classified as pregnancy category X. DIF: COGNITIVE LEVEL: Applying (Application) REF: p. 541 TOP: NURSING PROCESS: Assessment MSC: NCLEX: Safe and Effective Care Environment: Safety and Infection Control

7. A 51-year-old woman will be taking selective estrogen receptor modulators (SERMs) as part of treatment for postmenopausal osteoporosis. The nurse reviews potential contraindications, including which condition? a. Hypocalcemia b. Breast cancer c. Stress fractures d. Venous thromboembolism

ANS: D SERMs such as raloxifene are contraindicated in women with a venous thromboembolic disorder, including deep vein thrombosis, pulmonary embolism, or a history of such disorders. The other options are incorrect. DIF: COGNITIVE LEVEL: Understanding (Comprehension) REF: p. 542 TOP: NURSING PROCESS: Implementation MSC: NCLEX: Physiological Integrity: Reduction of Risk Potential

A teenager sustains a severe closed head injury following an all-terrain vehicle (ATV) accident and is in a state of deep sleep that requires vigorous stimulation to elicit eye opening. How should the nurse document this in the chart? a. Confusion b. Coma c. Obtundation d. Stupor

ANS: D Stupor is a condition of deep sleep or unresponsiveness from which a person may be aroused or caused to open his or her eyes only by vigorous and repeated stimulation. Confusion is the loss of the ability to think rapidly and clearly and is characterized by impaired judgment and decision making. Coma is a condition in which there is no verbal response to the external environment or to any stimuli; noxious stimuli such as deep pain or suctioning do not yield motor movement. Obtundation is a mild-to-moderate reduction in arousal (awakeness) with limited response to the environment. REF: p. 361, Table 15-3

44. After change-of-shift report, which patient should the nurse assess first? a. Patient with a Colles' fracture who has right wrist swelling and deformity b. Patient with a intracapsular left hip fracture whose leg is externally rotated c. Patient with a repaired mandibular fracture who is complaining of facial pain d. Patient with right femoral shaft fracture whose thigh is swollen and ecchymotic

ANS: D Swelling and bruising after a femoral shaft fracture suggest hemorrhage and risk for compartment syndrome. The nurse should assess the patient rapidly and then notify the health care provider. The other patients have symptoms that are typical for their injuries, but do not require immediate intervention. DIF: Cognitive Level: Analyze (analysis) REF: 1512 OBJ: Special Questions: Prioritization; Multiple Patients TOP: Nursing Process: Assessment MSC: NCLEX: Safe and Effective Care Environment

41. Which finding in a patient with a Colles' fracture of the left wrist is most important to communicate to the health care provider? a. Swelling is noted around the wrist. b. The patient is reporting severe pain. c. The wrist has a deformed appearance. d. Capillary refill to the fingers is prolonged.

ANS: D Swelling, pain, and deformity are common findings with a Colles' fracture. Prolonged capillary refill indicates decreased circulation and risk for ischemia. This is not an expected finding and should be immediately reported. DIF: Cognitive Level: Apply (application) REF: 1523-1524 OBJ: Special Questions: Prioritization TOP: Nursing Process: Assessment MSC: NCLEX: Physiological Integrity

16. The nurse will anticipate teaching a 56-year-old patient who is diagnosed with lobular carcinoma in situ (LCIS) about a. lumpectomy. b. lymphatic mapping. c. MammaPrint testing. d. tamoxifen (Nolvadex).

ANS: D Tamoxifen is used as a chemopreventive therapy in some patients with LCIS. The other diagnostic tests and therapies are not needed because LCIS does not usually require treatment. DIF: Cognitive Level: Apply (application) TOP: Nursing Process: Planning MSC: NCLEX: Physiological Integrity

The incidence of fractures of the pelvis is highest in: a. preadolescent boys. b. adolescent boys. c. adolescent girls. d. older adults.

ANS: D The incidence of fractures of the upper femur, upper humerus, vertebrae, and pelvis is highest in older adults and is often associated with osteoporosis. Fractures of healthy bones, particularly the tibia, clavicle, and lower humerus, tend to occur in young persons and without gender preference.

10. A middle-aged male patient with usually well-controlled hypertension and diabetes visits the clinic. Today he has a blood pressure of 174/94 mm Hg and a blood glucose level of 190 mg/dL. What patient information may indicate that additional intervention by the nurse is needed? a. The patient states that he takes his prescribed antihypertensive medications daily. b. The patient states that both of his parents have high blood pressure and diabetes. c. The patient indicates that he does blood glucose monitoring several times each day. d. The patient reports that he and his wife are disputing custody of their 8-yr-old son.

ANS: D The increase in blood pressure and glucose levels possibly suggests that stress caused by his divorce and custody battle may be adversely affecting his health. The nurse should assess this further and develop an appropriate plan to assist the patient in decreasing his stress. Although he has been very compliant with his treatment plan in the past, the nurse should assess whether the stress in his life is interfering with his management of his health problems. The family history will not necessarily explain why he has had changes in his blood pressure and glucose levels. DIF: Cognitive Level: Apply (application) REF: 79 TOP: Nursing Process: Assessment MSC: NCLEX: Physiological Integrity

26. A patient with acute kidney injury (AKI) has longer QRS intervals on the electrocardiogram (ECG) than were noted on the previous shift. Which action should the nurse take first? a. Notify the patient's health care provider. b. Document the QRS interval measurement. c. Review the chart for the patient's current creatinine level. d. Check the medical record for the most recent potassium level.

ANS: D The increasing QRS interval is suggestive of hyperkalemia, so the nurse should check the most recent potassium and then notify the patient's health care provider. The BUN and creatinine will be elevated in a patient with AKI, but they would not directly affect the electrocardiogram (ECG). Documentation of the QRS interval is also appropriate, but interventions to decrease the potassium level are needed to prevent life-threatening dysrhythmias. DIF: Cognitive Level: Analyze (analysis) REF: 1072 OBJ: Special Questions: Prioritization TOP: Nursing Process: Implementation MSC: NCLEX: Physiological Integrity

A mother brings her toddler into the emergency department and tells the nurse that she thinks the toddler has eaten an entire bottle of chewable aspirin tablets. The nurse will assess for which most common signs of salicylate intoxication in children? a. Photosensitivity and nervousness b. Tinnitus and hearing loss c. Acute gastrointestinal bleeding d. Hyperventilation and drowsiness

ANS: D The most common manifestations of chronic salicylate intoxication in adults are tinnitus and hearing loss. Those in children are hyperventilation and CNS effects, such as dizziness, drowsiness, and behavioral changes.

The diagnosis of rhabdomyolysis is based on the measurement of which laboratory value? a. White blood cell count (WBC) b. Antinuclear antibodies c. Aspartate aminotransferase d. Creatine kinase (CK)

ANS: D The most important and clinically useful measurement in rhabdomyolysis is serum CK. A level five times the upper limit of normal (about 1000 units per liter) is used to identify rhabdomyolysis. While the other options may be measured, they are not used as diagnostic criteria.

Just before the second course of chemotherapy, the laboratory calls to report that the patient's neutrophil count is 450 cells/mm3. The nurse expects that the oncologist will follow which course of treatment? a. Chemotherapy will continue as scheduled. b. Chemotherapy will resume with a lowered dosage. c. Chemotherapy will resume after a transfusion of neutrophils. d. Chemotherapy will be withheld until the neutrophil count returns toward normal levels.

ANS: D The normal range for neutrophils is above 1500 cells/mm3. If neutrophils are decreased to levels of less than 500 cells/mm3 (neutropenia), there is risk for severe infection. Chemotherapy will be held until the count returns toward normal levels. DIF: COGNITIVE LEVEL: Analyzing (Analysis) REF: p. 744 TOP: NURSING PROCESS: Evaluation MSC: NCLEX: Physiological Integrity: Pharmacological and Parenteral Therapies

7. A 48-year-old patient with a comminuted fracture of the left femur has Buck's traction in place while waiting for surgery. To assess for pressure areas on the patient's back and sacral area and to provide skin care, the nurse should a. loosen the traction and help the patient turn onto the unaffected side. b. place a pillow between the patient's legs and turn gently to each side. c. turn the patient partially to each side with the assistance of another nurse. d. have the patient lift the buttocks by bending and pushing with the right leg.

ANS: D The patient can lift the buttocks off the bed by using the left leg without changing the right-leg alignment. Turning the patient will tend to move the leg out of alignment. Disconnecting the traction will interrupt the weight needed to immobilize and align the fracture. DIF: Cognitive Level: Apply (application) REF: 1514 | 1520 TOP: Nursing Process: Assessment MSC: NCLEX: Safe and Effective Care Environmen

21. The nurse is caring for a patient who is to be discharged from the hospital 5 days after insertion of a femoral head prosthesis using a posterior approach. Which statement by the patient indicates a need for additional instruction? a. "I should not cross my legs while sitting." b. "I will use a toilet elevator on the toilet seat." c. "I will have someone else put on my shoes and socks." d. "I can sleep in any position that is comfortable for me."

ANS: D The patient needs to sleep in a position that prevents excessive internal rotation or flexion of the hip. The other patient statements indicate that the patient has understood the teaching. DIF: Cognitive Level: Apply (application) REF: 1526 TOP: Nursing Process: Evaluation MSC: NCLEX: Physiological Integrity

36. After receiving change-of-shift report, which patient should the nurse assess first? a. Patient who is scheduled for the drain phase of a peritoneal dialysis exchange b. Patient with stage 4 chronic kidney disease who has an elevated phosphate level c. Patient with stage 5 chronic kidney disease who has a potassium level of 3.4 mEq/L d. Patient who has just returned from having hemodialysis and has a heart rate of 124/min

ANS: D The patient who has tachycardia after hemodialysis may be bleeding or excessively hypovolemic and should be assessed immediately for these complications. The other patients also need assessments or interventions but are not at risk for life-threatening complications. DIF: Cognitive Level: Analyze (analysis) REF: 1091 OBJ: Special Questions: Prioritization | Special Questions: Multiple Patients TOP: Nursing Process: Assessment MSC: NCLEX: Safe and Effective Care Environment

Which characterizes an embryonic cancer tumor? a. Commonly occurring b. Often seen in adults c. Composed of mature, differentiated cells d. Usually manifested around age 5

ANS: D These types of cancers usually manifest around age 5. These types of cancers are not commonly occurring, seen in adults, or composed of undifferentiated cells. REF: p. 301

7. The nurse will teach a patient with metastatic breast cancer who has a new prescription for trastuzumab (Herceptin) that a. hot flashes may occur with the medication. b. serum electrolyte levels will be drawn monthly. c. the patient will need frequent eye examinations. d. the patient should call if she notices ankle swelling.

ANS: D Trastuzumab can lead to ventricular dysfunction, so the patient is taught to self-monitor for symptoms of heart failure. There is no need to monitor serum electrolyte levels. Hot flashes or changes in visual acuity may occur with tamoxifen, but not with trastuzumab. DIF: Cognitive Level: Apply (application) TOP: Nursing Process: Planning MSC: NCLEX: Physiological Integrity

2. A factory line worker has repetitive strain syndrome in the left elbow. The nurse will plan to teach the patient about a. surgical options. b. elbow injections. c. wearing a left wrist splint. d. modifying arm movements.

ANS: D Treatment for repetitive strain syndrome includes changing the ergonomics of the activity. Elbow injections and surgery are not initial options for this type of injury. A wrist splint might be used for hand or wrist pain. DIF: Cognitive Level: Apply (application) REF: 1509 TOP: Nursing Process: Planning MSC: NCLEX: Physiological Integrity

A 36-year-old reports pain and weakness in the elbow. MRI reveals inflammation of the tendon and the presence of microtears where it attaches to bone. This condition is called: a. periostitis. b. muscle strain. c. bursitis. d. epicondylopathy.

ANS: D When force is sufficient to cause microscopic tears (microtears) in tissue, the result is known as epicondylopathy. Such an injury is not referred to as periostitis which involves the presence of inflammation. Muscle strain is local muscle damage that occurs when the muscle is stretched beyond capacity. Bursitis is inflammation of the bursae.

A 13-year-old presents with pain at night, cough, and dyspnea and is diagnosed with a metastasizing malignant bone tumor. The most likely type of tumor is: a. nonossifying fibroma. b. chondrosarcoma. c. Ewing sarcoma. d. osteosarcoma.

ANS: D With osteosarcoma, the most common presenting complaint is pain. Night pain, awakening a child from sleep, is a particularly foreboding sign. There may be swelling, warmth, and redness caused by the vascularity of the tumor. Symptoms may also include cough, dyspnea, and chest pain if lung metastasis is present. This symptomatology is not associated with the other options. REF: p. 1049

A patient has been taking the corticosteroid dexamethasone (Decadron) but has developed bacterial conjunctivitis and has a prescription for gentamicin (Garamycin) ointment. The nurse notes that which interaction is possible if the two drugs are used together? a. The infection may become systemic. b. The gentamicin effects may become more potent. c. The corticosteroid may cause overgrowth of nonsusceptible organisms. d. Immunosuppression may make it more difficult to eliminate the eye infection.

ANS: D Concurrent use of corticosteroids, such as dexamethasone and ophthalmic antimicrobials, may cause immunosuppression that may make it more difficult to eliminate the eye infection. DIF: COGNITIVE LEVEL: Applying (Application) REF: p. 912 TOP: NURSING PROCESS: Assessment MSC: NCLEX: Physiological Integrity: Reduction of Risk Potential

The nurse will anticipate that a 61-yr-old patient who has an enlarged prostate detected by digital rectal examination (DRE) and an elevated prostate specific antigen (PSA) level will need teaching about a. cystourethroscopy. b. uroflowmetry studies. c. magnetic resonance imaging (MRI). d. transrectal ultrasonography (TRUS).

ANS: D In a patient with an abnormal DRE and elevated PSA, transrectal ultrasound is used to visualize the prostate for biopsy. Uroflowmetry studies help determine the extent of urine blockage and treatment, but there is no indication that this is a problem for this patient. Cystoscopy may be used before prostatectomy but will not be done until after the TRUS and biopsy. MRI is used to determine whether prostatic cancer has metastasized but would not be ordered at this stage of the diagnostic process.

The student demonstrates a lack of understanding of palliative care when making which statement? a. "Palliative care is designed to promote comfort." b. "Palliative care is designed to reduce disease exacerbations." c. "Palliative care is designed to decrease acute care hospital admissions." d. "Palliative care is designed to promote a cure for chronic disease."

ANS: D Palliation is the reduction of symptoms without elimination of the cause. Palliative care refers to the provision of care for patients who are diagnosed with a disease or condition without a cure.

A patient who has been diagnosed with stage 2 prostate cancer chooses the option of active surveillance. The nurse will plan to a. vaccinate the patient with sipuleucel-T (Provenge). b. provide the patient with information about cryotherapy. c. teach the patient about placement of intraurethral stents. d. schedule the patient for annual prostate-specific antigen testing.

ANS: D Patients who opt for active surveillance need to have annual digital rectal examinations and prostate-specific antigen testing. Vaccination with sipuleucel-T, cryotherapy, and stent placement are options for patients who choose to have active treatment for prostate cancer.

A patient is receiving ocular cyclosporine (Restasis) and also has an order for an artificial tears product. The nurse includes which instructions in the teaching plan for these medications? a. "These two eye drugs cannot be given together. Let's check with your prescriber." b. "You may take these two drugs together at the same time." c. "First take the artificial tears, and then take the Restasis after 5 minutes." d. "Take the Restasis first, and then wait 15 minutes before taking the artificial tears."

ANS: D Restasis can be used together with artificial tears if the drugs are given 15 minutes apart. The other options are incorrect. DIF: COGNITIVE LEVEL: Applying (Application) REF: p. 917 TOP: NURSING PROCESS: Implementation MSC: NCLEX: Physiological Integrity: Pharmacological and Parenteral Therapies

After a transurethral resection of the prostate (TURP), a 64-yr-old patient with continuous bladder irrigation complains of painful bladder spasms. The nurse observes clots in the urine. Which action should the nurse take first? a. Increase the flow rate of the bladder irrigation. b. Administer the prescribed IV morphine sulfate. c. Give the patient the prescribed belladonna and opium suppository. d. Manually instill and then withdraw 50 mL of saline into the catheter.

ANS: D The assessment suggests that obstruction by a clot is causing the bladder spasms, and the nurse's first action should be to irrigate the catheter manually and to try to remove the clots. IV morphine will not decrease the spasm, although pain may be reduced. Increasing the flow rate of the irrigation will further distend the bladder and may increase spasms. The belladonna and opium suppository will decrease bladder spasms but will not remove the obstructing blood clot.

Which information about continuous bladder irrigation will the nurse teach to a patient who is being admitted for a transurethral resection of the prostate (TURP)? a. Bladder irrigation decreases the risk of postoperative bleeding. b. Hydration and urine output are maintained by bladder irrigation. c. Antibiotics are infused continuously through the bladder irrigation. d. Bladder irrigation prevents obstruction of the catheter after surgery.

ANS: D The purpose of bladder irrigation is to remove clots from the bladder and prevent obstruction of the catheter by clots. The irrigation does not decrease bleeding or improve hydration. Antibiotics are given by the IV route, not through the bladder irrigation.

A patient with chronic pain who has been receiving morphine sulfate 20 mg IV over 24 hours is to be discharged home on oral sustained-release morphine (MS Contin) administered twice a day. What dosage of MS Contin will be needed for each dose to obtain an equinalgesic dose for the patient? (Morphine sulfate 10 mg IV is equianalgesic to morphine sulfate 30 mg orally).

ANS: MS Contin 30 mg/dose

A patient with osteomyelitis is to receive vancomycin (Vancocin) 500 mg IV every 6 hours. The vancomycin is diluted in 100 mL of normal saline and needs to be administered over 1 hour. The nurse should set the IV pump to deliver how many milliliters per minute?

Ans: 1.67

A patient who is receiving methotrexate for severe rheumatoid arthritis develops a megaloblastic anemia. Which nutrient supplement should the nurse plan to explain to the patient? a. Iron b. Folic Acid c. Cobalamin (Vitamin B12) d. Ascorbic Acid (Vitamin C)

Ans: B

What is the priority problem for a patient experiencing an acute attack with Meniere's disease? a. Being at risk for falls. b. Imbalanced nutritional intake. c. Difficulty performing self-care. d. Impaired verbal communication.

Answer: a. Being at risk for falls. Rationale: All the problems are appropriate, but because sudden attacks of vertigo can lead to "drop attacks," the major focus of nursing care is to prevent injuries associated with dizziness.

A patient who has Ménière's disease is admitted with vertigo, nausea, and vomiting. Which nursing intervention will be included in the care plan? a. Dim the lights in the patient's room. b. Encourage increased oral fluid intake. c. Change the patient's position every 2 hours. d. Keep the head of the bed elevated 45 degrees.

Answer: a. Dim the lights in the patient's room. Rationale: A darkened, quiet room will decrease the symptoms of the acute attack of Ménière's disease. Because the patient will be nauseated during an acute attack, fluids are administered IV. Position changes will cause vertigo and nausea. The head of the bed can be positioned for patient comfort.

The nurse is assessing a patient who was recently treated with amoxicillin for acute otitis media of the right ear. Which finding is a priority to report to the health care provider? a. The patient has a temperature of 100.6° F. b. The patient report frequent "popping" in the ear. c. Clear fluid is visible through the tympanic membrane. d. The patient frequently asks the nurse to repeat information.

Answer: a. The patient has a temperature of 100.6° F. Rationale: The fever indicates that the infection may not be resolved, and the patient might need further antibiotic therapy. A feeling of fullness, "popping" of the ear, decreased hearing, and fluid in the middle ear are indications of otitis media with effusion. These symptoms are normal for weeks to months after an episode of acute otitis media and usually resolve without treatment.

The nurse is observing a student who is preparing to perform an ear examination for a 30-year-old patient. Which action by the student indicates that the nurse should intervene? a. Pulls the auricle of the ear up and posterior. b. Chooses a speculum larger than the ear canal. c. Stabilizes the hand holding the otoscope on the patient's head. d. Stops inserting the otoscope after observing impacted cerumen.

Answer: b. Chooses a speculum larger than the ear canal. Rationale: The speculum should be smaller than the ear canal, so it can be inserted without damage to the external ear canal. The other actions are appropriate when performing an ear examination.

A patient with presbycusis is fitted with binaural hearing aids. Which information will the nurse include when teaching the patient how to use the hearing aids? a. Keep the volume low on the hearing aids for the first week. b. Experiment with volume and hearing in a quiet environment. c. Add the second hearing aid after making adjustments to the first hearing aid. d. Begin wearing the hearing aids for an hour a day, gradually increasing the use.

Answer: b. Experiment with volume and hearing in a quiet environment. Rationale: Initially the patient should use the hearing aids in a quiet environment such as the home, experimenting with increasing and decreasing the volume as needed. There is no need to gradually increase the time of wear. The patient should experiment with the level of volume to find what works well in various situations. Both hearing aids should be used.

Which action will the nurse take when performing ear irrigation for a patient with cerumen impaction? a. Assist the patient to a supine position for the irrigation. b. Fill the irrigation syringe with body-temperature solution. c. Use a sterile applicator to clean the ear canal before irrigating. d. Occlude the ear canal completely with the syringe while irrigating.

Answer: b. Fill the irrigation syringe with body-temperature solution. Rationale: Solution at body temperature is used for ear irrigation. The patient should be sitting for the procedure. Use of cotton-tipped applicators to clear the ear may result in forcing the cerumen deeper into the ear canal. The ear should not be completely occluded with the syringe.

Which information about a patient who had a stapedotomy yesterday is most important for the nurse to communicate to the health care provider? a. The patient reports ear "fullness." b. Oral temperature is 100.8° F (38.1° C). c. Small amount of dried drainage on dressing. d. The patient reports that hearing has gotten worse.

Answer: b. Oral temperature is 100.8° F (38.1° C). Rationale: An elevated temperature may indicate a postoperative infection. Although the nurse would report all the data, a temporary decrease in hearing, bloody drainage on the dressing, and a feeling of congestion (because of the accumulation of blood and drainage in the ear) are common after this surgery.

When the patient turns his head quickly during the admission assessment, the nurse observes nystagmus. What is the indicated nursing action? a. Assess the patient with a Rinne test. b. Place a fall-risk bracelet on the patient. c. Ask the patient to watch the mouths of staff when they are speaking. d. Remind unlicensed assistive personnel to speak loudly to the patient.

Answer: b. Place a fall-risk bracelet on the patient. Rationale: Problems with balance related to vestibular function may present as nystagmus or vertigo and indicate an increased risk for falls. The Rinne test is used to check hearing. Reading lips and louder speech are compensatory behaviors for decreased hearing.

A patient reports dizziness when bending over and of nausea and dizziness associated with physical activities. What exam should the nurse expect to prepare the patient to undergo? a. Tympanometry b. Rotary chair testing c. Pure-tone audiometry d. Bone-conduction testing

Answer: b. Rotary chair testing. Rationale: The patient's clinical manifestations of dizziness and nausea suggest a disorder of the labyrinth, which controls balance and contains three semicircular canals and the vestibule. Rotary chair testing is used to test vestibular function. The other tests are used to test for problems with hearing.

Which equipment does the nurse need to perform a Rinne test? a. Otoscope b. Tuning fork c. Audiometer d. Ticking watch

Answer: b. Tuning fork. Rationale: Rinne testing is done using a tuning fork. The other equipment is used for other types of ear examinations.

Unlicensed assistive personnel (UAP) perform the following actions when caring for a patient with Ménière's disease who is experiencing an acute attack. Which action by UAP indicates that the nurse should intervene? a. UAP raises the side rails on the bed. b. UAP turns on the patient's television. c. UAP places an emesis basin at the bedside. d. UAP helps the patient turn to the right side.

Answer: b. UAP turns on the patient's television. Rationale: Watching television may exacerbate the symptoms of an acute attack of Ménière's disease. The other actions are appropriate because the patient will be at high fall risk and may suffer from nausea during the acute attack.

What should the nurse include when teaching a patient who has undergone a left tympanoplasty? a. "Remain on bed rest." b. "Keep your head elevated." c. "Avoid blowing your nose." d. "Irrigate your left ear canal."

Answer: c. "Avoid blowing your nose." Rationale: Coughing or blowing the nose increases pressure in the eustachian tube and middle ear cavity and disrupts postoperative healing. There is no postoperative need for prolonged bed rest, elevation of the head, or continuous antibiotic irrigation.

When the nurse is taking a health history of a new patient at the ear clinic, the patient states, "I have to sleep with the television on." Which follow-up question is appropriate to obtain more information about possible hearing problems? a. "Do you grind your teeth at night?" b. "What time do you usually fall asleep?" c. "Have you noticed ringing in your ears?" d. "Are you ever dizzy when you are lying down?"

Answer: c. "Have you noticed ringing in your ears?" Rationale: Patients with tinnitus may use masking techniques, such as playing a radio, to block out the ringing in the ears. The responses "Do you grind your teeth at night?" and "Are you ever dizzy when you are lying down?" would be used to obtain information about other ear problems, such as vestibular disorders and referred temporomandibular joint pain. The response "What time do you usually fall asleep?" would not be helpful in assessing problems with the patient's ears.

A patient diagnosed with external otitis is being discharged from the emergency department with an ear wick in place. Which statement by the patient indicates a need for further teaching? a. "I will apply the eardrops to the cotton wick in the ear canal." b. "I can use aspirin or acetaminophen (Tylenol) for pain relief." c. "I will clean the ear canal daily with a cotton-tipped applicator." d. "I can use warm compresses to the outside of the ear for comfort."

Answer: c. "I will clean the ear canal daily with a cotton-tipped applicator." Rationale: Insertion of instruments such as cotton-tipped applicators into the ear should be avoided. The other patient statements indicate that the teaching has been successful.

Which statement by the patient to the home health nurse indicates a need for further teaching about self-administering eardrops? a. "I will leave the ear wick in place while administering the drops." b. "I will hold the tip of the dropper above the ear to administer the drops." c. "I will refrigerate the medication until I am ready to administer the drops." d. "I should lie down before and for 5 minutes after administering the drops."

Answer: c. "I will refrigerate the medication until I am ready to administer the drops." Rationale: Administration of cold eardrops can cause dizziness because of stimulation of the semicircular canals. The other patient actions are appropriate.

Which assessment finding should the nurse report to the health care provider? a. Visible cone of light. b. Dry skin in the ear canal. c. A blue-tinged tympanum. d. Cerumen in the auditory canal.

Answer: c. A blue-tinged tympanum. Rationale: A bluish-tinged tympanum can occur with acute otitis media, which requires immediate care to prevent perforation of the tympanum. Cerumen in the ear canal may need to be removed before proceeding with the examination but is not unusual or pathologic. The presence of a cone of light on the eardrum is normal. Dry and scaly skin in the ear canal may need further assessment but does not require urgent care.

Which action should the nurse take when teaching a patient with mild presbycusis? a. Use patient education handouts rather than discussion. b. Use a high-pitched tone of voice to provide instructions. c. Ask for permission to turn off the television before teaching. d. Wait until family members have left before initiating teaching.

Answer: c. Ask for permission to turn off the television before teaching. Rationale: Normal changes with aging make it more difficult for older patients to filter out unwanted sounds, so a quiet environment should be used for teaching. Loss of sensitivity for high-pitched tones is lost with presbycusis. Because the patient has mild presbycusis, the nurse should use both discussion and handouts. There is no need to wait until family members have left to provide patient teaching.

The nurse recording health histories in the outpatient clinic would plan a focused hearing assessment for adult patients taking which medication? a. Atenolol b. Albuterol c. Ibuprofen d. Acetaminophen

Answer: c. Ibuprofen Rationale: Nonsteroidal antiinflammatory drugs are potentially ototoxic. Acetaminophen, atenolol, and albuterol are not associated with hearing loss.

An older patient who is being admitted to the hospital repeatedly asks the nurse to "speak up so that I can hear you." Which action should the nurse take? a. Increase the speaking volume. b. Overenunciate while speaking. c. Speak normally but more slowly. d. Use more facial expressions when talking.

Answer: c. Speak normally but more slowly. Rationale: Patient understanding of the nurse's speech will be enhanced by speaking at a normal tone, but more slowly. Increasing the volume, overenunciating, and exaggerating facial expressions will not improve the patient's ability to comprehend.

To decrease the risk for future hearing loss, which action should the nurse implement with college students at the on-campus health clinic? a. Perform tympanometry. b. Schedule otoscopic examinations. c. Administer influenza immunizations. d. Discuss exposure to amplified music.

Answer: d. Discuss exposure to amplified music. Rationale: The nurse should discuss the impact of amplified music on hearing with young adults and discourage listening to highly amplified music, especially for prolonged periods. Tympanometry measures the ability of the eardrum to vibrate and would not help prevent future hearing loss. Although students are at risk for the influenza virus, being vaccinated does not help prevent future hearing loss. Otoscopic examinations are not necessary for all patients.

The charge nurse is observing a new nurse who is caring for a patient with vestibular disease. For what action by the nurse should the charge nurse intervene immediately? a. Facing the patient directly when speaking. b. Speaking slowly and distinctly to the patient. c. Administering both the Rinne and Weber tests. d. Encouraging the patient to ambulate independently.

Answer: d. Encouraging the patient to ambulate independently. Rationale: Vestibular disease affects balance, so the nurse should monitor the patient during activities that require balance. The other actions might be used for patients with hearing disorders.

Which information will the nurse include for a patient considering a cochlear implant? Cochlear implants: a. are not useful for patients with congenital deafness. b. are most helpful as an early intervention for presbycusis. c. improve hearing in patients with conductive hearing loss. d. require extensive training in order to reach the full benefit.

Answer: d. require extensive training in order to reach the full benefit. Rationale: Extensive rehabilitation is required after cochlear implants for patients to receive the maximum benefit. Hearing aids, rather than cochlear implants, are used initially for presbycusis. Cochlear implants are used for sensorineural hearing loss and would not be helpful for conductive loss. They are appropriate for some patients with congenital deafness

The nurse is reviewing infection-prevention measures with a patient who is receiving antineoplastic drug therapy. Which statement by the patient indicates the need for further teaching? a. "I will avoid those who have recently had a vaccination." b. "I will eat only fresh fruits and vegetables." c. "I will report a sore throat, cough, or low-grade temperature." d. "It is important for both my family and me to practice good hand washing."

B (Patients who are neutropenic and susceptible to infections need to adhere to a low-microbe diet by washing fresh fruits and vegetables and making sure foods are well cooked. The other options are correct.)

A patient with type 2 diabetes mellitus has been found to have trace proteinuria. The prescriber writes an order for an angiotensin-converting enzyme (ACE) inhibitor. What is the main reason for prescribing this class of drug for this patient? a. Cardioprotective effects b. Renal protective effects c. Reduces blood pressure d. Promotes fluid output

B ACE inhibitors have been shown to have a protective effect on the kidneys because they reduce glomerular filtration pressure. This is one reason that they are among the cardiovascular drugs of choice for diabetic patients. The other drugs do not have this effect.

A patient was diagnosed with pancreatic cancer last month, and has complained of a dull ache in the abdomen for the past 4 months. This pain has been gradually increasing, and the pain relievers taken at home are no longer effective. What type of pain is the patient experiencing? a. Acute pain b. Chronic pain c. Somatic pain d. Neuropathic pain

B Chronic pain is associated with cancer and is characterized by slow onset, long duration, and dull, persistent aching. The patient's symptoms are not characteristics of acute pain, somatic pain, or neuropathic pain.

A patient is receiving finasteride (Proscar) for treatment of benign prostatic hyperplasia. The nurse will tell him that a possible effect of this medication is: a. alopecia. b. increased hair growth. c. urinary retention. d. increased prostate size.

B Finasteride is given to reduce prostate size in men with benign prostatic hyperplasia. It has been noted that men taking this medication experience increased hair growth. The other options are incorrect.

While monitoring a depressed patient who has just started SSRI antidepressant therapy, the nurse will observe for which problem during the early time frame of this therapy? a. Hypertensive crisis b. Self-injury or suicidal tendencies c. Extrapyramidal symptoms d. Loss of appetite

B In 2005, the U.S. Food and Drug Administration (FDA) issued special black-box warnings regarding the use of all classes of antidepressants in both adult and pediatric patient populations. Data from the FDA indicated a higher risk for suicide in patients receiving these medications. As a result, current recommendations for all patients receiving antidepressants include regular monitoring for signs of worsening depressive symptoms, especially when the medication is started or the dosage is changed. The other options are incorrect.

A patient is recovering from abdominal surgery, which he had this morning. He is groggy but complaining of severe pain around his incision. What is the most important assessment data to consider before the nurse administers a dose of morphine sulfate to the patient? a. His pulse rate b. His respiratory rate c. The appearance of the incision d. The date of his last bowel movement

B One of the most serious adverse effects of opioids is respiratory depression. The nurse must assess the patient's respiratory rate before administering an opioid. The other options are incorrect.

While assessing a patient who is receiving intravenous digitalis, the nurse recognizes that the drug has a negative chronotropic effect. How would this drug effect be evident in the patient? a. Decreased blood pressure b. Decreased heart rate c. Decreased conduction d. Decreased ectopic beats

B A negative chronotropic effect results in a decreased heart rate; this is one effect of cardiac glycosides. The other options are incorrect.

A patient who has been taking antihypertensive drugs for a few months complains of having a persistent dry cough. The nurse knows that this cough is an adverse effect of which class of antihypertensive drugs? a. Beta blockers b. Angiotensin-converting enzyme (ACE) inhibitors c. Angiotensin II receptor blockers (ARBs) d. Calcium channel blockers

B ACE inhibitors cause a characteristic dry, nonproductive cough that reverses when therapy is stopped. The other drug classes do not cause this cough.

A 50-year-old man who has been taking phenobarbital for 1 week is found very lethargic and unable to walk after eating out for dinner. His wife states that he has no other prescriptions and that he did not take an overdose—the correct number of pills is in the bottle. The nurse suspects that which of these may have happened? a. He took a multivitamin. b. He drank a glass of wine. c. He took a dose of aspirin. d. He developed an allergy to the drug.

B Alcohol has an additive effect when combined with barbiturates and causes central nervous system (CNS) depression. Multivitamins and aspirin do not interact with barbiturates, and this situation does not illustrate an allergic reaction.

A patient is receiving gabapentin (Neurontin), an anticonvulsant, but has no history of seizures. The nurse expects that the patient is receiving this drug for which condition? a. Inflammation pain b. Pain associated with peripheral neuropathy c. Depression associated with chronic pain d. Prevention of seizures

B Anticonvulsants are often used as adjuvants for treatment of neuropathic pain to enhance analgesic efficacy. The other indications listed are not correct.

A patient wants to take a ginseng dietary supplement. The nurse instructs the patient to look for which potential adverse effect? a. Drowsiness b. Palpitations and anxiety c. Dry mouth d. Constipation

B Elevated blood pressure, chest pain or palpitations, anxiety, insomnia, headache, nausea, vomiting, and diarrhea are potential adverse effects of ginseng. Drowsiness, difficulty with urination, and constipation are not potential adverse effects of ginseng.

The nurse is reviewing herbal therapies. Which is a common use of the herb feverfew? a. Muscle aches b. Migraine headaches c. Leg cramps d. Incision pain after surgery

B Feverfew is commonly used for migraine headaches, menstrual problems, arthritis, and fever. Possible adverse effects include muscle stiffness and muscle and joint pain.

During the administration of finasteride (Proscar), the nurse must remember which important precaution? a. It must be taken on an empty stomach. b. It must not be handled by pregnant women. c. It is given by deep intramuscular injection to avoid tissue irritation. d. The patient needs to be warned that alopecia is a common adverse effect.

B Finasteride must not be handled by pregnant women because of its teratogenic effects. It is taken orally and without regard to meals. The other options are incorrect.

A 57-year-old woman being treated for end-stage breast cancer has been using a transdermal opioid analgesic as part of the management of pain. Lately, she has been experiencing breakthrough pain. The nurse expects this type of pain to be managed by which of these interventions? a. Administering NSAIDs b. Administering an immediate-release opioid c. Changing the opioid route to the rectal route d. Making no changes to the current therapy

B If a patient is taking long-acting opioid analgesics, breakthrough pain must be treated with an immediate-release dosage form that is given between scheduled doses of the long-acting opioid. The other options are not appropriate actions.

A 73-year-old male patient is in the clinic for a yearly physical and is asking for a prescription for sildenafil (Viagra). He has listed on his health history that he is taking a nitrate for angina. The nurse is aware that which problem may occur if sildenafil is taken with a nitrate? a. Significant increase in pulse rate b. Significant decrease in blood pressure c. Increased risk of bleeding d. Reduced effectiveness of the sildenafil

B In patients with pre-existing cardiovascular disease, especially those on nitrates, erectile dysfunction drugs such as sildenafil lower blood pressure substantially, potentially leading to more serious adverse events. The other options are incorrect.

A patient is being prepared for an oral endoscopy, and the nurse anesthetist reminds him that he will be awake during the procedure but probably will not remember it. What type of anesthetic technique is used in this situation? a. Local anesthesia b. Moderate sedation c. Topical anesthesia d. Spinal anesthesia

B Moderate sedation effectively reduces patient anxiety, sensitivity to pain, and recall of the medical procedure, yet it preserves a patient's ability to maintain his or her own airway and respond to verbal commands. The other options are incorrect.

The nurse notes in a patient's medical record that nesiritide (Natrecor) has been ordered. Based on this order, the nurse interprets that the patient has which disorder? a. Atrial fibrillation b. Acutely decompensated heart failure with dyspnea at rest c. Systolic heart failure d. Chronic, stable heart failure

B Nesiritide is indicated for the treatment of acutely decompensated heart failure with dyspnea at rest. Digoxin is used for the treatment of atrial fibrillation and systolic heart failure. Nesiritide is not indicated for chronic, stable heart failure.

A patient has been treated for lung cancer for 3 years. Over the past few months, the patient has noticed that the opioid analgesic is not helping as much as it had previously and more medication is needed for the same pain relief. The nurse is aware that this patient is experiencing which of these? a. Opioid addiction b. Opioid tolerance c. Opioid toxicity d. Opioid abstinence syndrome

B Opioid tolerance is a common physiologic result of long-term opioid use. Patients with opioid tolerance require larger doses of the opioid agent to maintain the same level of analgesia. This situation does not describe toxicity (overdose), addiction, or abstinence syndrome (withdrawal).

The wife of a patient who has been diagnosed with depression calls the office and says, "It's been an entire week since he started that new medicine for his depression, and there's no change! What's wrong with him?" What is the nurse's best response? a. "The medication may not be effective for him. He may need to try another type." b. "It may take up to 6 weeks to notice any therapeutic effects. Let's wait a little longer to see how he does." c. "It sounds like the dose is not high enough. I'll check about increasing the dosage." d. "Some patients never recover from depression. He may not respond to this therapy."

B Patients and family members need to be told that antidepressant drugs commonly require several weeks before full therapeutic effects are noted. The other answers are incorrect.

The nurse is creating a plan of care for a patient with a new diagnosis of hypertension. Which is a potential nursing diagnosis for the patient taking antihypertensive medications? a. Diarrhea b. Sexual dysfunction c. Urge urinary incontinence d. Impaired memory

B Sexual dysfunction is a potential nursing diagnosis related to possible adverse effects of antihypertensive drug therapy. The other nursing diagnoses are not appropriate.

Chlorpromazine (Thorazine) is prescribed for a patient, and the nurse provides instructions to the patient about the medication. The nurse includes which information? a. The patient needs to avoid caffeine while on this drug. b. The patient needs to wear sunscreen while outside because of photosensitivity. c. Long-term therapy may result in nervousness and excitability. d. The medication may be taken with an antacid to reduce gastrointestinal upset.

B Sun exposure and tanning booths need to be avoided with conventional antipsychotics because of the adverse effect of photosensitivity. Instruct the patient to apply sunscreen liberally and to wear sun-protective clothing and hats.

During the immediate postoperative period, the Post Anesthesia Care Unit nurse is assessing a patient who had hip surgery. The patient is experiencing tachycardia, tachypnea, and muscle rigidity, and his temperature is 103° F (39.4° C). The nurse will prepare for what immediate treatment? a. Naltrexone hydrochloride (Narcan) injection, an opioid reversal drug b. Dantrolene (Dantrium) injection, a skeletal muscle relaxant c. An anticholinesterase drug, such as neostigmine d. Cardiopulmonary resuscitation (CPR) and intubation

B Tachycardia, tachypnea, muscle rigidity, and raised temperature are symptoms of malignant hyperthermia, which is treated with cardiorespiratory supportive care as needed to stabilize heart and lung function as well as with immediate treatment with the skeletal muscle relaxant dantrolene. CPR is not immediately needed because the patient still has a pulse and respirations. Naltrexone and anticholinesterase drugs are not appropriate in this situation.

A patient has a digoxin level of 1.1 ng/mL. Which interpretation by the nurse is correct? a. It is below the therapeutic level. b. It is within the therapeutic range. c. It is above the therapeutic level. d. It is at a toxic level.

B The normal therapeutic drug level of digoxin is between 0.5 and 2 ng/mL. The other options are incorrect.

When a patient is receiving a second-generation antipsychotic drug, such as risperidone (Risperdal), the nurse will monitor for which therapeutic effect? a. Fewer panic attacks b. Decreased paranoia and delusions c. Decreased feeling of hopelessness d. Improved tardive dyskinesia

B The therapeutic effects of the antipsychotic drugs include improvement in mood and affect, and alleviation or decrease in psychotic symptoms (decrease in hallucinations, paranoia, delusions, garbled speech). Tardive dyskinesia is a potential adverse effect of these drugs. The other options are incorrect.

Before beginning a patient's therapy with selective serotonin reuptake inhibitor (SSRI) antidepressants, the nurse will assess for concurrent use of which medications or medication class? a. Aspirin b. Anticoagulants c. Diuretics d. Nonsteroidal anti-inflammatory drugs

B Use of selective serotonin reuptake inhibitor (SSRI) antidepressants with warfarin results in an increased anticoagulant effect. SSRI antidepressants do not interact with the other drugs or drug classes listed. See Table 16-6 for important drug interactions with SSRIs.

A 30-year-old female complains of fatigue, arthritis, rash, and changes in urine color. Laboratory testing reveals anemia, lymphopenia, and kidney inflammation. Assuming a diagnosis of SLE, which of the following is also likely to be present? a. Anti-LE antibodies b. Autoantibodies c. Antiherpes antibodies d. Anti-CMV antibodies

B Autoantibodies

In addition to matching ABO antigens, a blood transfusion must also be matched for: a. HLA type. b. Rh antigen. c. immunoglobulins. d. platelet compatibility.

B Rh antigen.

During therapy with hematopoietic drugs, the nurse will monitor the patient for which adverse effects? (Select all that apply.) A. Hypotension B. Edema C. Diarrhea D. Black, tarry stools E. Nausea and vomiting F. Headache

B, C, E, F

Vicodin (acetaminophen/hydrocodone) is prescribed for a patient who has had surgery. The nurse informs the patient that which common adverse effects can occur with this medication? (Select all that apply.) a. Diarrhea b. Constipation c. Lightheadedness d. Nervousness e. Urinary retention f. Itching

B, C, E, F Constipation (not diarrhea), lightheadedness (not nervousness), urinary retention, and itching are some of the common adverse effects that the patient may experience while taking Vicodin.

The nurse is preparing to administer a barbiturate. Which conditions or disorders would be a contraindication to the use of these drugs? (Select all that apply.) a. Gout b. Pregnancy c. Epilepsy d. Severe chronic obstructive pulmonary disease e. Severe liver disease f. Diabetes mellitus

B, D, E Contraindications to barbiturates include pregnancy, significant respiratory difficulties, and severe liver disease. The other disorders are not contraindications.

When a patient is experiencing digoxin toxicity, which clinical situation would necessitate the use of digoxin immune Fab (Digifab)? (Select all that apply.) a. The patient reports seeing colorful halos around lights. b. The patient's serum potassium level is above 5 mEq/L. c. The patient is experiencing nausea and anorexia. d. The patient is experiencing severe sinus bradycardia that does not respond to cardiac pacing. e. The patient has received an overdose of greater than 10 mg of digoxin. f. The patient reports fatigue and headaches.

B, D, E Clinical situations that would require the use of digoxin immune Fab in a patient with digoxin toxicity include serum potassium level above 5 mEq/L, severe sinus bradycardia that does not respond to cardiac pacing, or an overdose of more than 10 mg of digoxin. Seeing colorful halos around lights and experiencing nausea, anorexia, fatigue, and headaches are potential adverse effects of digoxin therapy but are not necessarily reasons for digoxin immune Fab treatment.

A patient with a history of angina will be started on ranolazine (Ranexa). The nurse is reviewing the patient's history and will note potential contraindications to this drug therapy if which condition is present? (Select all that apply.) a. Type 2 diabetes mellitus b. Prolonged QT interval on the electrocardiogram c. Heart failure d. Closed-angle glaucoma e. Decreased liver function

B, E Ranolazine is contraindicated in patients with pre-existing QT prolongation or hepatic impairment. The other options are not contraindications.

A patient has been recently diagnosed with chronic fatigue syndrome and asks the nurse about the cause of this illness. What is the nurse's best response? A. "The provider will be able to tell you when the lab results are back." B. "An exact cause may not be determined, but a treatment plan will be discussed." C. "Stress is the main cause; a referral to a counselor may be helpful." D. "This is considered a psychiatric illness requiring behavioral medicine."

B. "An exact cause may not be determined, but a treatment plan will be discussed."

After receiving change-of-shift report on four patients admitted to a heart failure unit, which patient should the nurse assess first? a. A patient who reported dizziness after receiving the first dose of captopril. b. A patient who has new-onset confusion and restlessness and cool, clammy skin. c. A patient who is receiving oxygen and has crackles bilaterally in the lung bases. d. A patient who is receiving IV nesiritide (Natrecor), with a blood pressure of 100/62.

B. A patient who has new-onset confusion and restlessness and cool, clammy skin.

Which action should the nurse take to monitor the effects of an acute stressor on a hospitalized patient? (Select all that apply.) A. Assess for bradycardia. B. Ask about epigastric pain. C. Observe for increased appetite. D. Check for elevated blood glucose levels. E. Monitor for a decrease in respiratory rate.

B. Ask about epigastric pain. C. Observe for increased appetite. D. Check for elevated blood glucose levels.

A patient with chronic heart failure who is taking a diuretic and an angiotensin-converting enzyme (ACE) inhibitor and who is on a low-sodium diet tells the home health nurse about a 5-lb weight gain in the past 3 days. What is the nurse's priority action? a. Teach the patient about restricting dietary sodium. b. Assess the patient for manifestations of acute heart failure. c. Ask the patient about the use of the prescribed medications. d. Have the patient recall the dietary intake for the past 3 days.

B. Assess the patient for manifestations of acute heart failure.

Which topic will the nurse plan to include in discharge teaching for a patient who has heart failure with reduced ejection fraction (HFrEF)? a. Need to begin an aerobic exercise program several times weekly b. Benefits and effects of angiotensin-converting enzyme (ACE) inhibitors c. Use of salt substitutes to replace table salt when cooking and at the table d. Importance of making an annual appointment with the health care provider

B. Benefits and effects of angiotensin-converting enzyme (ACE) inhibitors

The nurse is caring for a patient diagnosed with amyotrophic lateral sclerosis. The nurse should assess for which priority problem? A. Fatigue B. Bradypnea C. Hypertension D. Fall risk

B. Bradypnea

4. Which are treatment related causes of fatigue? (Select all that apply.) A. Blood transfusion B. Chemotherapy C. Radiation therapy D. Surgery E. Side effects of medications

B. Chemotherapy C. Radiation therapy D. Surgery E. Side effects of medications

The nurse is assessing for fatigue in a patient diagnosed with multiple sclerosis. Which self -reporting tool is best for the nurse to utilize? A. Multidimensional Assessment of Fatigue (MAF) B. Fatigue Severity Scale (FSS) C. Brief Fatigue Inventory (BFI) D. Multidimensional Fatigue Inventory (MFI)

B. Fatigue Severity Scale (FSS)

An elderly Chinese woman is interested in biologically based therapies to relieve osteoarthritis (OA) pain. You are preparing a plan of care for her OA. Options most conducive to her expressed wishes may include which actions or activities? A. Pilates, breathing exercises, and aloe vera B. Herbs, vitamins, and tai chi C. Alternating ice and heat to relieve pain and inflammation D. Guided imagery, relaxation breathing, and meditation

B. Herbs, vitamins, and tai chi

A 19-year-old male has sustained a transection of C-7 in a motor vehicle crash rendering him a quadriplegic. He describes his pain as burning, sharp, and shooting. What type of pain is this patient describing? A. Nociceptive pain B. Neuropathic pain C. Mixed pain syndrome D. Ghost pain

B. Neuropathic pain

After receiving change-of-shift report on a heart failure unit, which patient should the nurse assess first? a. Patient who is taking carvedilol (Coreg) and has a heart rate of 58. b. Patient who is taking digoxin and has a potassium level of 3.1 mEq/L. c. Patient who is taking captopril and has a frequent nonproductive cough. d. Patient who is taking isosorbide dinitrate/hydralazine (BiDil) and has a headache.

B. Patient who is taking digoxin and has a potassium level of 3.1 mEq/L.

The nurse must awaken a patient from Stage 4 non-rapid eye movement sleep in order to prepare the patient for a procedure. The patient is disoriented. What is the nurse's best action? A. Notify the healthcare provider. B. Re-assess the patient's orientation. C. Administer an anti-anxiety medication. D. Cancel the patient's procedure.

B. Re-assess the patient's orientation.

The nurse is making rounds on the hospital unit and observes a patient sleeping. The patient's pulse and respiratory rates are slower than baseline. The nurse realizes the patient has most likely just entered which stage of non-rapid eye movement sleep? A. Stage 1 B. Stage 2 C. Stage 3 D. Stage 4

B. Stage 2

The nurse is working with a patient who recently lost her spouse after a lengthy illness. The patient shares that she would like to sell her home and move to another state now that her spouse has passed away. Which of the following interventions would be considered a priority for this patient? (Select all that apply.) A. Notify the provider to evaluate for antidepressant therapy. B. Suggest that the patient consider a support group for widows. C. Suggest that the patient learn stress reduction breathing exercises. D. Suggest that the patient take prescribed antianxiety medications. E. Assist the patient in identifying support systems. F. Notify the provider to evaluate the need for antianxiety medications.

B. Suggest that the patient consider a support group for widows. C. Suggest that the patient learn stress reduction breathing exercises. E. Assist the patient in identifying support systems.

A patient is to receive acetylcysteine (Mucomyst) as part of the treatment for an acetaminophen (Tylenol) overdose. Which action by the nurse is appropriate when giving this medication? a. Giving the medication undiluted for full effect b. Avoiding the use of a straw when giving this medication c. Disguising the flavor with soda or flavored water d. Preparing to give this medication via a nebulizer

C Acetylcysteine has the flavor of rotten eggs and so is better tolerated if it is diluted and disguised by mixing with a drink such as cola or flavored water to help increase its palatability. The use of a straw helps to minimize contact with the mucous membranes of the mouth and is recommended. The nebulizer form of this medication is used for certain types of pneumonia, not for acetaminophen overdose.

While assessing a patient who is taking a beta blocker for angina, the nurse knows to monitor for which adverse effect? a. Nervousness b. Hypertension c. Bradycardia d. Dry cough

C Adverse effects of beta blockers include bradycardia, hypotension, dizziness, lethargy, impotence, and several other effects, but not dry cough or nervousness.

The nurse is reviewing the food choices of a patient who is taking a monoamine oxidase inhibitor ( MAOI). Which food choice would indicate the need for additional teaching? a. Orange juice b. Fried eggs over-easy c. Salami and Swiss cheese sandwich d. Biscuits and honey

C Aged cheeses, such a Swiss or cheddar cheese, and Salami contain tyramine. Patients who are taking MAOIs need to avoid tyramine-containing foods because of a severe hypertensive reaction that may occur. Orange juice, eggs, biscuits, and honey do not contain tyramine.

A patient has been taking temazepam (Restoril) for intermittent insomnia. She calls the nurse to say that when she takes it, she sleeps well, but the next day she feels "so tired." Which explanation by the nurse is correct? a. "Long-term use of this drug results in a sedative effect." b. "If you take the drug every night, this hangover effect will be reduced." c. "These drugs affect the sleep cycle, resulting in daytime sleepiness." d. "These drugs increase the activity of the central nervous system, making you tired the next day."

C Benzodiazepines suppress REM sleep to a degree (although not as much as barbiturates) and, thus, result in daytime sleepiness (a hangover effect). The other statements are incorrect.

A patient is suffering from tendonitis of the knee. The nurse is reviewing the patient's medication administration record and recognizes that which adjuvant medication is most appropriate for this type of pain? a. Antidepressant b. Anticonvulsant c. Corticosteroid d. Local anesthesia

C Corticosteroids have an anti-inflammatory effect, which may help to reduce pain. The other medications do not have anti-inflammatory properties.

The nurse is reviewing medications used for depression. Which of these statements is a reason that selective serotonin reuptake inhibitors (SSRIs) are more widely prescribed today than tricyclic antidepressants? a. SSRIs have fewer sexual side effects. b. Unlike tricyclic antidepressants, SSRIs do not have drug-food interactions. c. Tricyclic antidepressants cause serious cardiac dysrhythmias if an overdose occurs. d. SSRIs cause a therapeutic response faster than tricyclic antidepressants.

C Death from overdose of tricyclic antidepressants usually results from either seizures or dysrhythmias. SSRIs are associated with significantly fewer and less severe systemic adverse effects, especially anticholinergic and cardiovascular adverse effects. The other options are incorrect.

A patient is in the intensive care unit and receiving an infusion of milrinone (Primacor) for severe heart failure. The prescriber has written an order for an intravenous dose of furosemide (Lasix). How will the nurse give this drug? a. Infuse the drug into the same intravenous line as the milrinone. b. Stop the milrinone, flush the line, and then administer the furosemide. c. Administer the furosemide in a separate intravenous line. d. Notify the prescriber that the furosemide cannot be given at this time.

C Furosemide must not be injected into an intravenous line with milrinone because it will precipitate immediately. The infusion must not be stopped because of the patient's condition. A separate line will be needed. The other options are incorrect.

A patient will be discharged with a 1-week supply of an opioid analgesic for pain management after abdominal surgery. The nurse will include which information in the teaching plan? a. How to prevent dehydration due to diarrhea b. The importance of taking the drug only when the pain becomes severe c. How to prevent constipation d. The importance of taking the drug on an empty stomach

C Gastrointestinal (GI) adverse effects, such as nausea, vomiting, and constipation, are the most common adverse effects associated with opioid analgesics. Physical dependence usually occurs in patients undergoing long-term treatment. Diarrhea is not an effect of opioid analgesics. Taking the dose with food may help minimize GI upset.

A 38-year-old man has come into the urgent care center with severe hip pain after falling from a ladder at work. He says he has taken several pain pills over the past few hours but cannot remember how many he has taken. He hands the nurse an empty bottle of acetaminophen (Tylenol). The nurse is aware that the most serious toxic effect of acute acetaminophen overdose is which condition? a. Tachycardia b. Central nervous system depression c. Hepatic necrosis d. Nephropathy

C Hepatic necrosis is the most serious acute toxic effect of an acute overdose of acetaminophen. The other options are incorrect.

The nurse is assessing a patient for contraindications to drug therapy with acetaminophen (Tylenol). Which patient should not receive acetaminophen? a. A patient with a fever of 101° F (38.3° C) b. A patient who is complaining of a mild headache c. A patient with a history of liver disease d. A patient with a history of peptic ulcer disease

C Liver disease is a contraindication to the use of acetaminophen. Fever and mild headache are both possible indications for the medication. Having a history of peptic ulcer disease is not a contraindication.

When a patient asks the nurse what hypersensitivity is, how should the nurse respond? Hypersensitivity is best defined as: a. a reduced immune response found in most pathologic states. b. a normal immune response to an infectious agent. c. an excessive or inappropriate response of the immune system to a sensitizing antigen. d. antigenic desensitization

C an excessive or inappropriate response of the immune system to a sensitizing antigen.

A 10-year-old male is stung by a bee while playing in the yard. He begins itching and develops pain, swelling, redness, and respiratory difficulties. He is suffering from: a. immunodeficiency. b. autoimmunity. c. anaphylaxis. d. tissue-specific hypersensitivity.

C anaphylaxis.

A 78-year-old patient is in the recovery room after having a lengthy surgery on his hip. As he is gradually awakening, he requests pain medication. Within 10 minutes after receiving a dose of morphine sulfate, he is very lethargic and his respirations are shallow, with a rate of 7 per minute. The nurse prepares for which priority action at this time? a. Assessment of the patient's pain level b. Immediate intubation and artificial ventilation c. Administration of naloxone (Narcan) d. Close observation of signs of opioid tolerance

C Naloxone, an opioid-reversal agent, is used to reverse the effects of acute opioid overdose and is the drug of choice for reversal of opioid-induced respiratory depression. This situation is describing an opioid overdose, not opioid tolerance. Intubation and artificial ventilation are not appropriate because the patient is still breathing at 7 breaths/min. It would be inappropriate to assess the patient's level of pain.

The nurse is reviewing the orders for a patient and notes a new order for an angiotensinconverting enzyme (ACE) inhibitor. The nurse checks the current medication orders, knowing that this drug class may have a serious interaction with what other drug class? a. Calcium channel blockers b. Diuretics c. Nonsteroidal anti-inflammatory drugs d. Nitrates

C Nonsteroidal anti-inflammatory drugs (NSAIDs) such as ibuprofen can reduce the antihypertensive effect of ACE inhibitors. In addition, the use of NSAIDs and ACE inhibitors may also predispose patients to the development of acute renal failure.

The nurse administering the phosphodiesterase inhibitor milrinone (Primacor) recognizes that this drug will have a positive inotropic effect. Which result reflects this effect? a. Increased heart rate b. Increased blood vessel dilation c. Increased force of cardiac contractions d. Increased conduction of electrical impulses across the heart

C Positive inotropic drugs increase myocardial contractility, thus increasing the force of cardiac conduction. Positive chronotropic drugs increase the heart rate. Positive dromotropic drugs increase the conduction of electrical impulses across the heart. Blood vessel dilation is not affected.

Ramelteon (Rozerem) is prescribed for a patient with insomnia. The nurse checks the patient's medical history, knowing that this medication is contraindicated in which disorder? a. Coronary artery disease b. Renal insufficiency c. Liver disease d. Anemia

C Ramelteon is contraindicated in cases of severe liver dysfunction. The other conditions are not contraindications.

The nurse is assessing a patient who has been admitted to the emergency department for a possible opioid overdose. Which assessment finding is characteristic of an opioid drug overdose? a. Dilated pupils b. Restlessness c. Respiration rate of 6 breaths/min d. Heart rate of 55 beats/min

C The most serious adverse effect of opioid use is CNS depression, which may lead to respiratory depression. Pinpoint pupils, not dilated pupils, are seen. Restlessness and a heart rate of 55 beats/min are not indications of an opioid overdose.

When the nurse is administering topical nitroglycerin ointment, which technique is correct? a. Apply the ointment on the skin on the forearm. b. Apply the ointment only in the case of a mild angina episode. c. Remove the old ointment before new ointment is applied. d. Massage the ointment gently into the skin, and then cover the area with plastic wrap.

C The old ointment should be removed before a new dose is applied. The ointment should be applied to clean, dry, hairless skin of the upper arms or body, not below the elbows or below the knees. The ointment is not massaged or spread on the skin, and it is not indicated for the treatment of acute angina.

A patient is to receive testosterone therapy via a transdermal patch. He asks the nurse, "Why am I getting a patch? Can't I just take a pill?" Which response by the nurse is correct? a. "The patch reduces the incidence of side effects." b. "If you don't take the patch, you will have to have injections instead." c. "The patch allows for better absorption of the medication." d. "You will only have to change the patch weekly."

C The transdermal form allows for better absorption of testosterone because of its high first-pass effect. Oral forms are poorly absorbed, and the transdermal form is preferable to an injection and is preferred for hormonal replacement therapy. The patch is changed daily.

A patient is brought to the emergency department for treatment of a suspected overdose. The patient was found with an empty prescription bottle of a barbiturate by his bedside. He is lethargic and barely breathing. The nurse would expect which immediate intervention? a. Starting an intravenous infusion of diluted bicarbonate solution b. Administering medications to increase blood pressure c. Implementing measures to maintain the airway and support respirations d. Administrating naloxone (Narcan) as an antagonist

C There are no antagonists/antidotes for barbiturates. Treatment supports respirations and maintains the airway. The other interventions are not appropriate.

When teaching a patient who has a new prescription for transdermal nitroglycerin patches, the nurse tells the patient that these patches are most appropriately used for which situation? a. To prevent low blood pressure b. To relieve shortness of breath c. To prevent the occurrence of angina d. To keep the heart rate from rising too high during exercise

C Transdermal dosage formulations of nitroglycerin are used for the long-term prophylactic management (prevention) of angina pectoris. Transdermal nitroglycerin patches are not appropriate for the relief of shortness of breath, to prevent palpitations, or to control the heart rate during exercise.

A patient has been admitted to the emergency department with a suspected overdose of a tricyclic antidepressant. The nurse will prepare for what immediate concern? a. Hypertension b. Renal failure c. Cardiac dysrhythmias d. Gastrointestinal bleeding

C Tricyclic antidepressant overdoses are notoriously lethal. The primary organ systems affected are the central nervous system and the cardiovascular system, and death usually results from either seizures or dysrhythmias.

When a nurse notices that a patient has type O blood, the nurse realizes that anti-_____ antibodies are present in the patient's body. a. A only b. B only c. A and B d. O

C A and B

When the maternal immune system becomes sensitized against antigens expressed by the fetus, what type of immune reaction occurs? a. Autoimmune b. Anaphylaxis c. Alloimmune d. Allergic

C Alloimmune

When a patient presents at the emergency department for an allergic reaction, the nurse recognizes the most severe consequence of a type I hypersensitivity reaction is: a. urticaria. b. hives. c. anaphylaxis. d. antibody-dependent cell-mediated cytotoxicity (ADCC).

C anaphylaxis.

Which statements are true regarding the selective serotonin reuptake inhibitors (SSRIs)? (Select all that apply.) a. Avoid foods and beverages that contain tyramine. b. Monitor the patient for extrapyramidal symptoms. c. Therapeutic effects may not be seen for about 4 to 6 weeks after the medication is started. d. If the patient has been on an MAOI, a 2- to 5-week or longer time span is required before beginning an SSRI medication. e. These drugs have anticholinergic effects, including constipation, urinary retention, dry mouth, and blurred vision. f. Cogentin is often also prescribed to reduce the adverse effects that may occur.

C, D During SSRI medication, therapeutic effects may not be seen for 4 to 6 weeks. To prevent the potentially fatal pharmacodynamic interactions that can occur between the SSRIs and the MAOIs, a 2- to 5-week washout period is recommended between uses of these two classes of medications. The other options apply to other classes of psychotherapeutic drugs, not SSRIs.

A mother tells the nurse she is concerned because her 8-month-old infant sleeps all day and night and is only awake about 2-3 hours per day. What is the nurse's best response? A. "This sleep pattern is very normal for an infant at this age." B. "Adding an additional feeding will keep the child awake more." C. "I recommend that you notify the child's pediatrician." D. "Be sure you are laying the child on his back to sleep at night."

C. "I recommend that you notify the child's pediatrician."

A patient with chronic heart failure has a new order for captopril 12.5 mg PO. After giving the first dose and teaching the patient about the drug, which statement by the patient indicates that teaching has been effective? a. "I plan to take the medication with food." b. "I should eat more potassium-rich foods." c. "I will call for help when I need to get up to use the bathroom." d. "I can expect to feel more short of breath for the next few days."

C. "I will call for help when I need to get up to use the bathroom."

A 53-yr-old patient with stage D heart failure and type 2 diabetes asks the nurse whether heart transplant is an option. Which response is accurate? a. "Your heart failure has not reached the end stage yet." b. "You could not manage the multiple complications of that surgery." c. "The suitability of a heart transplant for you depends on many factors." d. "Because you have diabetes, you would not be a heart transplant candidate."

C. "The suitability of a heart transplant for you depends on many factors."

A patient presents to the clinic complaining of nausea, vomiting, and fatigue. Lab results reveal elevated BUN and creatinine levels. Which acute condition is this patient at most risk for developing? A. Influenza B. Mononucleosis C. Acute renal failure D. Pneumonia

C. Acute renal failure

A patient who has chronic heart failure is admitted to the emergency department with severe dyspnea and a dry, hacking cough. Which action should the nurse take first? a. Auscultate the abdomen. b. Check the capillary refill. c. Auscultate the breath sounds. d. Ask about the patient's allergies.

C. Auscultate the breath sounds.

Which diagnostic test will be most useful to the nurse in determining whether a patient admitted with acute shortness of breath has heart failure? a. Serum troponin b. Arterial blood gases c. B-type natriuretic peptide d. 12-lead electrocardiogram

C. B-type natriuretic peptide

A patient tells the nurse that he experiences daytime fatigue even after 7-8 hours of sleep each night. What is the best assessment question for the nurse to ask? A. Have you tried getting 10 hours of sleep instead of 8 hours? B. How long are you in the rapid eye movement (REM) stage? C. Do you also have any recent lifestyle or behavior changes? D. Do any of your close relatives have any sleep disorders?

C. Do you also have any recent lifestyle or behavior changes?

After a management decision to admit terminal care patients to a medical unit, the nursing manager notes that nursing staff on the unit appear tired and anxious. Staff absences from work are increasing. The nurse manager is concerned that staff may be experiencing stress and burnout at work. What action would be best for the manager to take that will help the staff? A. Ask administration to require staff to meditate daily for at least 30 minutes. B. Have a staff psychologist available on the unit once a week for required counseling. C. Have training sessions to help the staff understand their new responsibilities. D. Ask support staff from other disciplines to complete some nursing tasks to provide help.

C. Have training sessions to help the staff understand their new responsibilities.

The nurse is developing a care plan for a patient with ineffective coping skills. Which intervention would be an example of a problem-focused coping strategy? A. Scheduling a regular exercise program B. Attending a seminar on treatment options C. Identifying a confidant to share feelings D. Attending a support group for families

C. Identifying a confidant to share feelings

While assessing an older adult patient, the nurse notes jugular venous distention (JVD) with the head of the patient's bed elevated 45 degrees. What does this finding indicate? a. Decreased fluid volume b. Jugular vein atherosclerosis c. Increased right atrial pressure d. Incompetent jugular vein valves

C. Increased right atrial pressure

The school nurse is assessing coping skills of high school students who attend an alternative school for students at high risk to not graduate. What is the priority concern that the nurse has for this student population? A. Altered vital sign readings B. Inaccurate perceptions of stressors C. Increased risk for suicide D. Decreased access to alcoholic beverages

C. Increased risk for suicide

A patient who is receiving dobutamine for the treatment of acute decompensated heart failure (ADHF) has the following nursing interventions included in the plan of care. Which action will be most appropriate for the registered nurse (RN) to delegate to an experienced licensed practical/vocational nurse (LPN/VN)? a. Teach the patient the reasons for remaining on bed rest. b. Change the peripheral IV site according to agency policy. c. Monitor the patient's blood pressure and heart rate every hour. d. Titrate the dobutamine to keep the systolic blood pressure >90 mm Hg.

C. Monitor the patient's blood pressure and heart rate every hour.

The nurse plans discharge teaching for a patient with chronic heart failure who has prescriptions for digoxin (Lanoxin) and hydrochlorothiazide. Which instruction should the nurse include? a. Limit dietary sources of potassium. b. Take the hydrochlorothiazide at bedtime. c. Notify the health care provider if nausea develops. d. Take the digoxin if the pulse is below 60 beats/min.

C. Notify the health care provider if nausea develops.

A patient who has chronic heart failure tells the nurse, "I was fine when I went to bed, but I woke up in the middle of the night feeling like I was suffocating!" How should the nurse document this finding? a. Orthopnea b. Pulsus alternans c. Paroxysmal nocturnal dyspnea d. Acute bilateral pleural effusion

C. Paroxysmal nocturnal dyspnea

Controlling pain is important to promoting wellness. Unrelieved pain has been associated with which complication? A. Large tidal volumes and decreased lung capacity B. Decreased tumor growth and longevity C. Prolonged stress response and a cascade of harmful effects system-wide D. Decreased carbohydrate, protein, and fat destruction

C. Prolonged stress response and a cascade of harmful effects system-wide

The nurse is caring for a patient who is receiving IV furosemide (Lasix) and morphine for the treatment of acute decompensated heart failure (ADHF) with severe orthopnea. Which clinical finding is the best indicator that the treatment has been effective? a. Weight loss of 2 lb in 24 hours b. Hourly urine output greater than 60 mL c. Reduced dyspnea with the head of bed at 30 degrees d. Patient denies experiencing chest pain or chest pressure

C. Reduced dyspnea with the head of bed at 30 degrees

While admitting an 82-yr-old patient with acute decompensated heart failure to the hospital, the nurse learns that the patient lives alone and sometimes confuses the "water pill" with the "heart pill." What should the nurse include in the discharge plan? a. Consult with a psychologist. b. Transfer to a long-term care facility. c. Referral to a home health care agency. d. Arrangements for around-the-clock care.

C. Referral to a home health care agency.

A patient who has heart failure has recently started taking digoxin (Lanoxin) in addition to furosemide (Lasix) and captopril. Which finding by the home health nurse is a priority to communicate to the health care provider? a. Presence of 1+ to 2+ edema in the feet and ankles b. Palpable liver edge 2 cm below the ribs on the right side c. Serum potassium level 3.0 mEq/L after 1 week of therapy d. Weight increase from 120 pounds to 122 pounds over 3 days

C. Serum potassium level 3.0 mEq/L after 1 week of therapy

IV sodium nitroprusside is ordered for a patient with acute pulmonary edema. Which reassessment finding during the first hours of administration indicates that the nurse should decrease the rate of nitroprusside infusion? a. Ventricular ectopy b. Dry, hacking cough c. Systolic BP below 90 mm Hg d. Heart rate below 50 beats/min

C. Systolic BP below 90 mm Hg

When teaching a patient with heart failure on a 2000-mg sodium diet, which foods should the nurse recommend limiting? a. Chicken and eggs b. Canned and frozen fruits c. Yogurt and milk products d. Fresh or frozen vegetables

C. Yogurt and milk products

The nurse is developing a plan of care for a patient who is experiencing gastrointestinal adverse effects, including anorexia and nausea, after the first course of antineoplastic therapy. What is an appropriate outcome for this patient when dealing with this problem? a. The patient will eat three balanced meals a day within 2 days. b. The patient will return to normal eating pattern within 4 weeks. c. The patient will maintain normal weight by consuming healthy snacks as tolerated. d. The patient will maintain a diet of small, frequent feedings with nutrition supplements within 2 weeks.

D (Consuming small, frequent meals with nutritional supplements, and maintaining a bland diet help to improve nutrition during antineoplastic therapy.)

A patient is receiving her third course of 5-fluorouracil therapy and knows that stomatitis is a potential adverse effect of antineoplastic therapy. What will the nurse teach her about managing this problem? a. "You can take aspirin to prevent stomatitis." b. "Be sure to watch for and report black, tarry stools immediately." c. "You need to increase your intake of foods containing fiber and citric acid." d. "Be sure to examine your mouth daily for bleeding, painful areas, and ulcerations."

D (The symptoms of stomatitis consist of pain or burning in the mouth, difficulty swallowing, taste changes, viscous saliva, dryness, cracking, and fissures, with or without bleeding mucosa. Teach patients to avoid consuming foods containing citric acid and foods that are hot or spicy or high in fiber. Assessing stools is important but is not related to stomatitis, and aspirin must not be used during this therapy.)

When assessing patients in the preoperative area, the nurse knows that which patient is at a higher risk for an altered response to anesthesia? a. The 21-year-old patient who has never had surgery before b. The 35-year-old patient who stopped smoking 8 years ago c. The 40-year-old patient who is to have a kidney stone removed d. The 82-year-old patient who is to have gallbladder removal

D The elderly patient is more affected by anesthesia than the young or middle-aged adult patient because of the effects of aging on the hepatic, cardiac, respiratory, and renal systems.

During a follow-up visit, the health care provider examines the fundus of the patient's eye. Afterward, the patient asks the nurse, "Why is he looking at my eyes when I have high blood pressure? It does not make sense to me!" What is the best response by the nurse? a. "We need to monitor for drug toxicity." b. "We must watch for increased intraocular pressure." c. "The provider is assessing for visual changes that may occur with drug therapy." d. "The provider is making sure the treatment is effective over the long term."

D The physician would examine the fundus of a patient's eyes during antihypertensive therapy because it is a more reliable indicator than blood pressure readings of the long-term effectiveness of treatment.

A patient has a new order for the adrenergic drug doxazosin (Cardura). When providing education about this drug, the nurse will include which instructions? a. "Weigh yourself daily, and report any weight loss to your prescriber." b. "Increase your potassium intake by eating more bananas and apricots." c. "The impaired taste associated with this medication usually goes away in 2 to 3 weeks." d. "Be sure to lie down after taking the first dose, because first-dose hypotension may make you dizzy."

D A patient who is starting doxazosin should take the first dose while lying down because there is a first-dose hypotensive effect with this medication. The other options are incorrect.

A patient with severe liver disease is receiving the angiotensin-converting enzyme (ACE) inhibitor, captopril (Capoten). The nurse is aware that the advantage of this drug for this patient is which characteristic? a. Captopril rarely causes first-dose hypotensive effects. b. Captopril has little effect on electrolyte levels. c. Captopril is a prodrug and is metabolized by the liver before becoming active. d. Captopril is not a prodrug and does not need to be metabolized by the liver before becoming active.

D A prodrug relies on a functioning liver to be converted to its active form. Captopril is not a prodrug, and therefore it would be safer for the patient with liver dysfunction.

A 63-year-old male patient is scheduled for a physical examination, and he tells the nurse that he wants to start taking a vitamin formula that includes saw palmetto for prostate health. Which is the nurse's best response? a. "I've heard many good things about saw palmetto." b. "It's not a good idea to start herbal therapy at your age." c. "There are very few adverse effects with saw palmetto therapy." d. "The doctor will need to draw some blood and do a digital rectal exam first."

D A prostatic-specific antigen test and digital rectal examination needs to be performed before initiation of treatment with saw palmetto for benign prostatic hyperplasia. Adverse effects may include gastrointestinal upset, headache, back pain, and dysuria.

While monitoring a patient who had surgery under general anesthesia 2 hours ago, the nurse notes a sudden elevation in body temperature. This finding may be an indication of which problem? a. Tachyphylaxis b. Postoperative infection c. Malignant hypertension d. Malignant hyperthermia

D A sudden elevation in body temperature during the postoperative period may indicate the occurrence of malignant hyperthermia, a life-threatening emergency. The elevated temperature does not reflect the other problems listed.

A patient arrives at the urgent care center complaining of leg pain after a fall when rock climbing. The x-rays show no broken bones, but he has a large bruise on his thigh. The patient says he drives a truck and does not want to take anything strong because he needs to stay awake. Which statement by the nurse is most appropriate? a. "It would be best for you not to take anything if you are planning to drive your truck." b. "We will discuss with your doctor about taking an opioid because that would work best for your pain." c. "You can take acetaminophen, also known as Tylenol, for pain, but no more than 1000 mg per day." d. "You can take acetaminophen, also known as Tylenol, for pain, but no more than 3000 mg per day."

D Acetaminophen is indicated for mild-to-moderate pain and does not cause drowsiness, as an opioid would. Currently, the maximum daily amount of acetaminophen is 3000 mg/day. The 1000-mg amount per day is too low. Telling the patient not to take any pain medications is incorrect.

What action is often recommended to help reduce tolerance to transdermal nitroglycerin therapy? a. Omit a dose once a week. b. Leave the patch on for 2 days at a time. c. Cut the patch in half for 1 week until the tolerance subsides. d. Remove the patch at bedtime, and then apply a new one in the morning.

D To prevent tolerance, remove the transdermal patch at night for 8 hours, and apply a new patch in the morning. Transdermal patches must never be cut or left on for 2 days, and doses must not be omitted.

A patient has been taking phenobarbital for 2 weeks as part of his therapy for epilepsy. He tells the nurse that he feels tense and that "the least little thing" bothers him now. Which is the correct explanation for this problem? a. These are adverse effects that usually subside after a few weeks. b. The drug must be stopped immediately because of possible adverse effects. c. This drug causes the rapid eye movement (REM) sleep period to increase, resulting in nightmares and restlessness. d. This drug causes deprivation of REM sleep and may cause the inability to deal with normal stress.

D Barbiturates such as phenobarbital deprive people of REM sleep, which can result in agitation and the inability to deal with normal stress. A rebound phenomenon occurs when the drug is stopped (not during therapy), and the proportion of REM sleep increases, sometimes resulting in nightmares. The other options are incorrect.

A patient has been taking a beta blocker for 4 weeks as part of his antianginal therapy. He also has type II diabetes and hyperthyroidism. When discussing possible adverse effects, the nurse will include which information? a. "Watch for unusual weight loss." b. "Monitor your pulse for increased heart rate." c. "Use the hot tub and sauna at the gym as long as time is limited to 15 minutes." d. "Monitor your blood glucose levels for possible hypoglycemia or hyperglycemia."

D Beta blockers can cause both hypoglycemia and hyperglycemia. They may also cause weight gain if heart failure is developing, and decreased pulse rate. The use of hot tubs and saunas is not recommended because of the possibility of hypotensive episodes.

The nurse is reviewing discharge teaching for a patient who will be taking digoxin (Lanoxin) therapy. The nurse will teach the patient to avoid which foods when taking the digoxin? a. Leafy green vegetables b. Dairy products c. Grapefruit juice d. Bran muffins

D Bran, in large amounts, may decrease the absorption of oral digitalis drugs. The other foods do not affect digoxin levels.

A calcium channel blocker (CCB) is prescribed for a patient, and the nurse provides instructions to the patient about the medication. Which instruction is correct? a. Chew the tablet for faster release of the medication. b. To increase the effect of the drug, take it with grapefruit juice. c. If the adverse effects of chest pain, fainting, or dyspnea occur, discontinue the medication immediately. d. A high-fiber diet with plenty of fluids will help prevent the constipation that may occur.

D Constipation is a common effect of CCBs, and a high-fiber diet and plenty of fluids will help to prevent it. Grapefruit juice decreases the metabolism of CCBs. Extended-release tablets must never be chewed or crushed. These medications should never be discontinued abruptly because of the risk for rebound hypertension.

A patient about to receive a morning dose of digoxin has an apical pulse of 53 beats/min. What will the nurse do next? a. Administer the dose. b. Administer the dose, and notify the prescriber. c. Check the radial pulse for 1 full minute. d. Withhold the dose, and notify the prescriber.

D Digoxin doses are held and the prescriber notified if the apical pulse is 60 beats/min or lower or is higher than 100 beats/min. The other options are incorrect.

A patient has been taking digoxin at home but took an accidental overdose and has developed toxicity. The patient has been admitted to the telemetry unit, where the physician has ordered digoxin immune Fab (Digifab). The patient asks the nurse why the medication is ordered. What is the nurse's best response? a. "It will increase your heart rate." b. "This drug helps to lower your potassium levels." c. "It helps to convert the irregular heart rhythm to a more normal rhythm." d. "This drug is an antidote to digoxin and will help to lower the blood levels."

D Digoxin immune Fab (Digifab) is the antidote for a severe digoxin overdose. It is given intravenously. The other options are incorrect.

A patient has experienced insomnia for months, and the physician has prescribed a medication to help with this problem. The nurse expects which drug to be used for long-term treatment of insomnia? a. Secobarbital (Seconal), a barbiturate b. Diazepam (Valium), a benzodiazepine c. Midazolam (Versed), a benzodiazepine d. Eszopiclone (Lunesta), a nonbenzodiazepine sleep aid

D Eszopiclone (Lunesta) is one of the newest prescription hypnotics to be approved for long-term use in treatment of insomnia. Barbiturates and benzodiazepines are not appropriate for long-term treatment of insomnia; midazolam is used for procedural (moderate) sedation.

A patient has been taking the monoamine oxidase inhibitor (MAOI) phenelzine (Nardil) for 6 months. The patient wants to go to a party and asks the nurse, "Will just one beer be a problem?" Which advice from the nurse is correct? a. "You can drink beer as long as you have a designated driver." b. "Now that you've had the last dose of that medication, there will be no further dietary restrictions." c. "If you begin to experience a throbbing headache, rapid pulse, or nausea, you'll need to stop drinking." d. "You need to avoid all foods that contain tyramine, including beer, while taking this medication."

D Foods containing tyramine, such as beer and aged cheeses, should be avoided while a patient is taking an MAOI. Drinking beer while taking an MAOI may precipitate a dangerous hypertensive crisis. The other options are incorrect.

During a fishing trip, a patient pierced his finger with a large fishhook. He is now in the emergency department to have it removed. The nurse anticipates that which type of anesthesia will be used for this procedure? a. No anesthesia b. Topical benzocaine spray on the area c. Topical prilocaine (EMLA) cream around the site d. Infiltration of the puncture wound with lidocaine

D Infiltration anesthesia is commonly used for minor surgical procedures. It involves injecting the local anesthetic solution intradermally, subcutaneously, or submucosally across the path of nerves supplying the area to be anesthetized. The local anesthetic may be administered in a circular pattern around the operative field. The other types are not appropriate for this injury. This is a painful procedure; therefore, the option of "no anesthesia" is incorrect.

A pregnant woman is experiencing hypertension. The nurse knows that which drug is commonly used for a pregnant patient who is experiencing hypertension? a. Mannitol (Osmitrol) b. Enalapril (Vasotec) c. Hydrochlorothiazide (HydroDIURIL) d. Methyldopa (Aldomet)

D Methyldopa is used in the treatment of hypertension during pregnancy. The other options are incorrect.f

The nurse notes in a female patient's history that she has an order for the androgen methyltestosterone (Android). Based on this finding, the nurse interprets that the patient has which disorder? a. Fibrocystic breast disease b. Hereditary angioedema c. Hypertension d. Inoperable breast cancer

D Methyltestosterone can be used in cases of inoperable breast cancer in women. The other options are incorrect.

The drug nalbuphine (Nubain) is an agonist-antagonist (partial agonist). The nurse understands that which is a characteristic of partial agonists? a. They have anti-inflammatory effects. b. They are given to reverse the effects of opiates. c. They have a higher potency than agonists. d. They have a lower dependency potential than agonists.

D Partial agonists such as nalbuphine are similar to the opioid agonists in terms of their therapeutic indications; however, they have a lower risk of misuse and addiction. They do not have antiinflammatory effects, nor are they given to reverse the effects of opiates. They do not have a higher potency than agonists.

When administering a neuromuscular blocking drug, the nurse needs to remember which principle? a. It is used instead of general anesthesia during surgery. b. Only skeletal muscles are paralyzed; respiratory muscles remain functional. c. It causes sedation and pain relief while allowing for lower doses of anesthetics. d. Artificial mechanical ventilation is required because of paralyzed respiratory muscles.

D Patients receiving neuromuscular blocking drugs require artificial mechanical ventilation because of the resultant paralysis of the respiratory muscles. In addition, they do not cause sedation or pain relief. They are used along with, not instead of, general anesthesia during surgery.

A patient tells the nurse that he likes to drink kava herbal tea to help him relax. Which statement by the patient indicates that additional teaching about this herbal product is needed? a. "I will not drink wine with the kava tea." b. "If I notice my skin turning yellow, I will stop taking the tea." c. "I will not take sleeping pills if I have this tea in the evening." d. "I will be able to drive my car after drinking this tea."

D Patients should not drive after drinking this tea because it may cause sedation. Kava tea may cause skin discoloration (with long-term use). In addition, it must not be taken with alcohol, barbiturates, and psychoactive drugs.

A patient with the diagnosis of schizophrenia is hospitalized and is taking a phenothiazine drug. Which statement by this patient indicates that he is experiencing a common adverse effect of phenothiazines? a. "I can't sleep at night." b. "I feel hungry all the time." c. "Look at how red my hands are." d. "My mouth has been so dry lately."

D Phenothiazines produce anticholinergic-like adverse effects of dry mouth, urinary hesitancy, and constipation.

An 18-year-old basketball player fell and twisted his ankle during a game. The nurse will expect to administer which type of analgesic? a. Synthetic opioid, such as meperidine (Demerol) b. Opium alkaloid, such as morphine sulfate c. Opioid antagonist, such as naloxone HCL (Narcan) d. Nonopioid analgesic, such as indomethacin (Indocin)

D Somatic pain, which originates from skeletal muscles, ligaments, and joints, usually responds to nonopioid analgesics such as nonsteroidal anti-inflammatory drugs (NSAIDs). The other options are not the best choices for somatic pain.

A patient has been taking the selective serotonin reuptake inhibitor (SSRI) sertraline (Zoloft) for about 6 months. At a recent visit, she tells the nurse that she has been interested in herbal therapies and wants to start taking St. John's wort. Which response by the nurse is appropriate? a. "That should be no problem." b. "Good idea! Hopefully you'll be able to stop taking the Zoloft." c. "Be sure to stop taking the herb if you notice a change in side effects." d. "Taking St. John's wort with Zoloft may cause severe interactions and is not recommended."

D The herbal product St. John's wort must not be used with SSRIs. Potential interactions include confusion, agitation, muscle spasms, twitching, and tremors. The other responses by the nurse are inappropriate.

The nurse is giving intravenous nitroglycerin to a patient who has just been admitted because of an acute myocardial infarction. Which statement is true regarding the administration of the intravenous form of this medication? a. The solution will be slightly colored green or blue. b. The intravenous form is given by bolus injection. c. It can be given in infusions with other medications. d. Non-polyvinylchloride (non-PVC) plastic intravenous bags and tubing must be used.

D The non-PVC infusion kits are used to avoid absorption and/or uptake of the nitrate by the intravenous tubing and bag and/or decomposition of the nitrate. The medication is given by infusion via an infusion pump and not with other medications. It is not given by bolus injection. If the parenteral solution is discolored blue or green, it should be discarded.

A patient is undergoing abdominal surgery and has been anesthetized for 3 hours. Which nursing diagnosis would be appropriate for this patient? a. Anxiety related to the use of an anesthetic b. Risk for injury related to increased sensorium from general anesthesia c. Decreased cardiac output related to systemic effects of local anesthesia d. Impaired gas exchange related to central nervous system depression produced by general anesthesia

D The nursing diagnosis of impaired gas exchange is appropriately worded for this patient. Anxiety would not be appropriate while the patient is in surgery. Sensorium would be decreased during surgery, not increased. Cardiac output is affected by general anesthesia, not local anesthesia.

An older adult patient will be taking a vasodilator for hypertension. Which adverse effect is of most concern for the older adult patient taking this class of drug? a. Dry mouth b. Restlessness c. Constipation d. Hypotension

D The older adult patient is more sensitive to the blood pressure-lowering effects of vasodilators, and consequently experience more problems with hypotension, dizziness, and syncope. The other options are incorrect.

A patient has been placed on a milrinone (Primacor) infusion as part of the therapy for end-stage heart failure. What adverse effect of this drug will the nurse watch for when assessing this patient during the infusion? a. Hypertension b. Hyperkalemia c. Nausea and vomiting d. Cardiac dysrhythmias

D The primary adverse effects seen with milrinone are cardiac dysrhythmias, mainly ventricular. It may also cause hypotension, hypokalemia, and other effects, but not nausea and vomiting.

A patient with primary hypertension is prescribed drug therapy for the first time. The patient asks how long drug therapy will be needed. Which answer by the nurse is the correct response? a. "This therapy will take about 3 months." b. "This therapy will take about a year." c. "This therapy will go on until your symptoms disappear." d. "Therapy for high blood pressure is usually lifelong."

D There is no cure for the disease, and treatment will be lifelong. The other answers are not appropriate.

What is the chance that two siblings share both HLA haplotypes, making them a good match for an organ transplant from one to the other? a. 100% b. 75% c. 50% d. 25%

D 25%

A recent immigrant from Honduras comes to the clinic with a family member who has been a U.S. resident for 10 years. The family member says, "The immigration to America has been very difficult." Considering cultural background, which expression of stress by this patient would the nurse expect? a. Motor restlessness b. Somatic complaints c. Memory deficiencies d. Sensory perceptual alterations

Somatic complaints

Which comments by a nurse are likely to help a patient cope by addressing the mediators of the stress response? (Select all that apply.) a. "A divorce, while stressful, can be the beginning of a new, better phase of life." b. "You mentioned your spirituality. Are there aspects of your faith that could be helpful to you at this time?" c. "Journaling often promotes awareness of how experiences have affected people." d. "It seems to me you are overreacting to this change in your life." e. "There is a support group for newly divorced persons in your neighborhood."

a. "A divorce, while stressful, can be the beginning of a new, better phase of life." b. "You mentioned your spirituality. Are there aspects of your faith that could be helpful to you at this time?" c. "Journaling often promotes awareness of how experiences have affected people." e. "There is a support group for newly divorced persons in your neighborhood."

While collecting a health history on a patient admitted for colon cancer, which of the following questions would be a priority to ask this patient? a. "Have you noticed any blood in your stool?" b. "Have you been experiencing nausea?" c. "Do you have back pain?" d. "Have you noticed any swelling in your abdomen?"

a. "Have you noticed any blood in your stool?"

After the nurse has taught a 28-yr-old with fibromyalgia, which statement by the patient indicates a good understanding of effective self-management? a. "I will need to stop drinking so much coffee and soda." b. "I am going to join a soccer team to get more exercise." c. "I will call the doctor every time my symptoms get worse." d. "I should avoid using over-the-counter medications for pain."

a. "I will need to stop drinking so much coffee and soda." Dietitians frequently suggest patients with fibromyalgia limit their intake of caffeine and sugar because these substances are muscle irritants. Mild exercise such as walking is recommended for patients with fibromyalgia, but vigorous exercise is likely to make symptoms worse. Because symptoms may fluctuate from day to day, the patient should be able to adapt the regimen independently rather than calling the provider whenever symptoms get worse. Over-the-counter medications such as ibuprofen and acetaminophen are frequently used for symptom management.

A patient is not certain whether she and her family should participate in a genetic screening plan. She asks the nurse why the X-linked recessive disorder that has been noted in some of her family members is expressed in males more frequently than in females. What is the nurse's best response? a. "The disease tends to show up in males because they do not have a second X chromosome to balance the expression of the gene." b. "One X chromosome of a pair is always inactive in females. This inactivity effectively negates the effects of the gene." c. "Females are known to have more effective DNA repair mechanisms than males, thus negating the damage caused by the recessive gene." d. "Expression of genes from the male's Y chromosome does not occur in females, so they are essentially immune to the effects of the gene."

a. "The disease tends to show up in males because they do not have a second X chromosome to balance the expression of the gene."

A patient with chronic back pain has learned to control the pain with the use of imagery and hypnosis. The patient's spouse asks the nurse how these techniques work. Which response by the nurse is accurate? a. "The strategies work by affecting the perception of pain." b. "These techniques block the pain pathways of the nerves." c. "These strategies prevent transmission of stimuli from the back to the brain." d. "The therapies slow the release of chemicals in the spinal cord that cause pain."

a. "The strategies work by affecting the perception of pain." Cognitive therapies affect the perception of pain by the brain rather than affecting efferent or afferent pathways or influencing the release of chemical transmitters in the dorsal horn. p. 121

1. The nurse is advising a clinic patient who was exposed a week ago to human immunodeficiency virus (HIV) through unprotected sexual intercourse. The patient's antigen and antibody test has just been reported as negative for HIV. What instructions should the nurse give to this patient? a. "You will need to be retested in 2 weeks." b. "You do not need to fear infecting others." c. "Since you don't have symptoms and you have had a negative test, you do not have HIV)." d. "We won't know for years if you will develop acquired immunodeficiency syndrome (AIDS)."

a. "You will need to be retested in 2 weeks." HIV screening tests detect HIV-specific antibodies or antigens. However, there may be a delay between infection and the time a screening test is able to detect HIV. The typical "window period" for antigen-antibody combination assays is approximately 2 weeks. It is not known based on this information whether the patient is infected with HIV or can infect others. It would be best practice to have him return for repeat testing in approximately 2 weeks.

A patient who has just started taking sustained-release morphine sulfate (MS Contin) for chronic arthritic joint pain after a traumatic injury complains of nausea and abdominal fullness. Which action should the nurse take initially? a. Administer the ordered antiemetic medication. b. Order the patient a clear liquid diet until the nausea decreases. c. Tell the patient that the nausea should subside in about a week. d. Consult with the health care provider about using a different opioid.

a. Administer the ordered antiemetic medication. Nausea is frequently experienced with the initiation of opioid therapy, and antiemetics usually are prescribed to treat this expected side effect. The best choice would be to administer the antiemetic medication so the patient can eat. There is no indication that a different opioid is needed, although if the nausea persists, the health care provider may order a change of opioid. Although tolerance develops and the nausea will subside in about a week, it is not appropriate to allow the patient to continue to be nauseated. A clear liquid diet may decrease the nausea but may not provide needed nutrients for injury healing. p. 114

Which action will the nurse include in the plan of care for a patient with newly diagnosed ankylosing spondylitis? a. Advise the patient to sleep on the back with a flat pillow. b. Emphasize that application of heat may worsen symptoms. c. Schedule annual laboratory assessment for the HLA-B27 antigen. d. Assist patient to choose physical activities that involve spinal flexion.

a. Advise the patient to sleep on the back with a flat pillow. Because ankylosing spondylitis results in flexion deformity of the spine, postures that extend the spine (e.g., sleeping on the back and with a flat pillow) are recommended. HLA-B27 antigen is assessed for initial diagnosis but is not needed annually. To counteract the development of flexion deformities, the patient should choose activities that extend the spine, such as swimming. Heat application is used to decrease localized pain.

10. Which information about a patient population would be most useful to help the nurse plan for human immunodeficiency virus (HIV) testing needs? a. Age b. Lifestyle c. Symptoms d. Sexual orientation

a. Age The current Centers for Disease Control and Prevention policy is to offer routine testing for HIV to all individuals age 13 to 64 years. Although lifestyle, symptoms, and sexual orientation may suggest increased risk for HIV infection, the goal is to test all individuals in this age range.

Which finding for a patient who is taking hydroxychloroquine (Plaquenil) to treat rheumatoid arthritis is likely to be an adverse effect of the medication? a. Blurred vision b. Joint tenderness c. Abdominal cramping d. Elevated blood pressure

a. Blurred vision Plaquenil can cause retinopathy. The medication should be stopped. Other findings are not related to the medication although they will also be reported.

A 22-year-old was recently diagnosed with acquired immunodeficiency syndrome (AIDS). Which decreased lab finding would be expected to accompany this virus? a. CD4+ T-helper b. CD8 T-helper c. CDC cells d. CDC10 cells

a. CD4+ T-helper

A nurse assesses a postoperative patient 2 days after chest surgery. What findings indicate that the patient requires better pain management (select all that apply)? a. Confusion b. Hypoglycemia c. Poor cough effort d. Shallow breathing e. Elevated temperature

a. Confusion c. Poor cough effort d. Shallow breathing e. Elevated temperature Inadequate pain control can decrease tidal volume and cough effort, leading to complications such as pneumonia with increases in temperature. Poor pain control may lead to confusion through a variety of mechanism, including hypoventilation and poor sleep quality. Stressors such as pain cause increased release of corticosteroids that can result in hyperglycemia. p. 103

A female patient complains of a "scab that just won't heal" under her left breast. During your conversation, she also mentions chronic fatigue, loss of appetite, and slight cough, attributed to allergies. What is the nurse's best action? a. Continue to conduct a symptom analysis to better understand the patient's symptoms and concerns. b. End the appointment and tell the patient to use skin protection during sun exposure. c. Suggest further testing with a cancer specialist and provide the appropriate literature. d. Tell her to put a bandage on the scab and set a follow-up appointment in one week.

a. Continue to conduct a symptom analysis to better understand the patient's symptoms and concerns.

The nurse admits a terminally ill patient to the hospital. What is the first action that the nurse should complete when planning this patient's care? a. Determine the patient's wishes regarding end-of-life care. b. Emphasize the importance of addressing any family issues. c. Discuss the normal grief process with the patient and family. d. Encourage the patient to talk about any fears or unresolved issues.

a. Determine the patient's wishes regarding end-of-life care. The nurse's initial action should be to assess the patient's wishes at this time. The other actions may be implemented if the patient or the family express a desire to discuss fears, understand the grief process, or address family issues, but they should not be implemented until the assessment indicates that they are appropriate.

A patient who is using both a fentanyl (Duragesic) patch and immediate-release morphine for chronic cancer pain develops new-onset confusion, dizziness, and a decrease in respiratory rate. Which action should the nurse take first? a. Remove the fentanyl patch. b. Obtain complete vital signs. c. Notify the health care provider. d. Administer prescribed PRN naloxone

a. Remove the fentanyl patch. The assessment data indicate a possible overdose of opioid. The first action should be to remove the patch. Naloxone administration in a patient who has been chronically using opioids can precipitate withdrawal and would not be the first action. Notification of the health care provider and continued monitoring are also needed, but the patient's data indicate that more rapid action is needed. The respiratory rate alone is an indicator for immediate action before obtaining blood pressure, pulse, and temperature. p. 118

18. Eight years after seroconversion, a human immunodeficiency virus (HIV)-infected patient has a CD4+ cell count of 800/μL and an undetectable viral load. What should be included in the plan of care at this time? a. Encourage adequate nutrition, exercise, and sleep. b. Teach about the side effects of antiretroviral agents. c. Explain opportunistic infections and antibiotic prophylaxis. d. Monitor symptoms of acquired immunodeficiency syndrome (AIDS).

a. Encourage adequate nutrition, exercise, and sleep. The CD4+ level for this patient is in the normal range, indicating that the patient is the stage of asymptomatic chronic infection when the body is able to produce enough CD4+ cells to maintain a normal CD4+ count. Maintaining healthy lifestyle behaviors is an important goal in this stage. AIDS and increased incidence of opportunistic infections typically develop when the CD4+ count is much lower than normal. Although the initiation of ART is highly individual, it would not be likely that a patient with a normal CD4+ level would receive ART.

A patient with chronic kidney disease is most likely to complain of which symptom? a. Fatigue b. Thirst c. Constipation d. Excess bleeding

a. Fatigue

An individual says to the nurse, "I feel so stressed out lately. I think the stress is affecting my body also." Which somatic complaints are most likely to accompany this feeling? (Select all that apply.) a. Headache b. Neck pain c. Insomnia d. Anorexia e. Myopia

a. Headache b. Neck pain c. Insomnia d. Anorexia

A patient with terminal cancer is being admitted to a family-centered inpatient hospice. The patient's spouse visits daily and cheerfully talks with the patient about wedding anniversary plans for the next year. When the nurse asks about any concerns, the spouse says, "I'm busy at work, but otherwise things are fine." Which provisional nursing diagnosis is appropriate for the patient's spouse? a. Ineffective coping related to lack of grieving b. Anxiety related to complicated grieving process c. Hopelessness related to knowledge deficit about cancer d. Caregiver role strain related to spouse's complex care needs

a. Ineffective coping related to lack of grieving The spouse's behavior and statements indicate the absence of anticipatory grieving, which may lead to impaired adjustment as the patient progresses toward death. The spouse does not appear to feel overwhelmed, hopeless, or anxious.

20. An older adult who takes medications for coronary artery disease and hypertension is newly diagnosed with HIV infection and is starting antiretroviral therapy. Which information will the nurse include in patient teaching? a. Many drugs interact with antiretroviral medications. b. HIV infections progress more rapidly in older adults. c. Less frequent CD4+ level monitoring is needed in older adults. d. Hospice care is available for patients with terminal HIV infection.

a. Many drugs interact with antiretroviral medications. The nurse will teach the patient about potential interactions between antiretrovirals and the medications that the patient is using for chronic health problems. Treatment and monitoring of HIV infection is not affected by age. A patient beginning early ART is not a candidate for hospice. Progression of HIV is not affected by age although it may be affected by chronic disease.

Which changes reflect short-term physiological responses to stress? (Select all that apply.) a. Muscular tension, blood pressure, and triglycerides increase. b. Epinephrine is released, increasing heart and respiratory rates. c. Corticosteroid release increases stamina and impedes digestion. d. Cortisol is released, increasing glucogenesis and reducing fluid loss. e. Immune system functioning decreases, and risk of cancer increases. f. Risk of depression, autoimmune disorders, and heart disease increases.

a. Muscular tension, blood pressure, and triglycerides increase. b. Epinephrine is released, increasing heart and respiratory rates. c. Corticosteroid release increases stamina and impedes digestion. d. Cortisol is released, increasing glucogenesis and reducing fluid loss.

The nurse is examining the eyes of a newborn infant. If the nurse notes the absence of the red reflex, what is the next best action? a. Notify the physician. b. Document the finding in the records. c. Recheck the reflex after several hours. d. Monitor the eye movements and pupil reactions closely.

a. Notify the physician.

The nurse is caring for a terminally ill patient who is experiencing continuous and severe pain. How should the nurse schedule the administration of opioid pain medications? a. Plan around-the-clock routine administration of analgesics. b. Provide PRN doses of medication whenever the patient requests them. c. Suggest small analgesic doses to avoid decreasing the respiratory rate. d. Offer enough pain medication to keep the patient sedated and unaware of stimuli.

a. Plan around-the-clock routine administration of analgesics. The principles of beneficence and nonmaleficence indicate that the goal of pain management in a terminally ill patient is adequate pain relief even if the effect of pain medications could hasten death. Administration of analgesics on a PRN basis will not provide the consistent level of analgesia the patient needs. Patients usually do not require so much pain medication that they are oversedated and unaware of stimuli. Adequate pain relief may require a dosage that will result in a decrease in respiratory rate.

While planning care for a patient experiencing fatigue due to chemotherapy, which of the following is the most appropriate nursing intervention? a. Prioritization and administration of nursing care throughout the day b. Completing all nursing care in the morning so the patient can rest the remainder of the day c. Completing all nursing care in the evening when the patient is more rested d. Limiting visitors, thus promoting the maximal amount of hours for sleep

a. Prioritization and administration of nursing care throughout the day

A patient has increased intraocular pressure. Which diagnosis will the nurse observe on the chart? a. Glaucoma b. Ocular degeneration c. Diplopia d. Nystagmus

a. glaucoma

During assessment of the patient with fibromyalgia, the nurse would expect the patient to report which of the following (select all that apply)? a. Sleep disturbances b. Multiple tender points c. Cardiac palpitations and dizziness d. Multijoint inflammation and swelling e. Widespread bilateral, burning musculoskeletal pain

a. Sleep disturbances b. Multiple tender points e. Widespread bilateral, burning musculoskeletal pain These symptoms are commonly described by patients with fibromyalgia. Cardiac involvement and joint inflammation are not typical of fibromyalgia.

A patient who had arthroscopic surgery of the right knee 7 days ago is admitted with a red, swollen, hot knee. Which assessment finding by the nurse should be reported to the health care provider immediately? a. The blood pressure is 86/50 mm Hg. b. The patient says the knee pain is severe. c. The white blood cell count is 11,500/µL. d. The patient is taking ibuprofen (Motrin).

a. The blood pressure is 86/50 mm Hg. The low blood pressure suggests the patient may be developing septicemia as a complication of septic arthritis. Immediate blood cultures and initiation of antibiotic therapy are indicated. The other information is typical of septic arthritis and should also be reported to the health care provider, but it does not indicate any immediately life-threatening problems.

A patient with dermatomyositis is receiving long-term prednisone therapy. Which assessment finding by the nurse is important to report to the health care provider? a. The patient has painful hematuria. b. Acne is noted on the patient's face. c. Fasting blood glucose is 112 mg/dL. d. The patient has an increased appetite.

a. The patient has painful hematuria. Corticosteroid use is associated with an increased risk for infection, so the nurse should report the urinary tract symptoms immediately to the health care provider. The increase in blood glucose, increased appetite, and acne are also adverse effects of corticosteroid use but do not need diagnosis and treatment as rapidly as the probable urinary tract infection.

A patient who had been experiencing significant stress learned to use progressive muscle re-laxation and deep breathing exercises. When the patient returns to the clinic 2 weeks later, which finding most clearly shows the patient is coping more effectively with stress? a. The patient's systolic blood pressure has changed from the 140s to the 120s mm Hg. b. The patient reports, "I feel better, and that things are not bothering me as much." c. The patient reports, "I spend more time napping or sitting quietly at home." d. The patient's weight decreased by 3 pounds.

a. The patient's systolic blood pressure has changed from the 140s to the 120s mm Hg.

16. To evaluate the effectiveness of antiretroviral therapy (ART), which laboratory test result will the nurse review? a. Viral load testing b. Enzyme immunoassay c. Rapid HIV antibody testing d. Immunofluorescence assay

a. Viral load testing The effectiveness of ART is measured by the decrease in the amount of virus detectable in the blood. The other tests are used to detect HIV antibodies, which remain positive even with effective ART.

A patient who uses a fentanyl (Duragesic) patch for chronic abdominal pain caused by ovarian cancer asks the nurse to administer the prescribed hydrocodone tablets, but the patient is asleep when the nurse returns with the medication. Which action is best for the nurse to take? a. Wake the patient and administer the hydrocodone. b. Wait until the patient wakes up and reassess the pain. c. Suggest the use of nondrug therapies for pain relief instead of additional opioids. d. Consult with the health care provider about changing the fentanyl (Duragesic) dose.

a. Wake the patient and administer the hydrocodone. Because patients with chronic pain frequently use withdrawal and decreased activity as coping mechanisms for pain, sleep is not an indicator that the patient is pain free. The nurse should wake the patient and administer the hydrocodone. p. 107

The nurse teaching a support group of women with rheumatoid arthritis (RA) about how to manage activities of daily living suggests they should a. avoid activities requiring repetitive use of the same muscles and joints. b. protect the knee joints by sleeping with a small pillow under the knees. c. stand rather than sit when performing daily household and yard chores. d. strengthen small hand muscles by wringing out sponges or washcloths.

a. avoid activities requiring repetitive use of the same muscles and joints. Patients are advised to avoid repetitious movements. Sitting during household chores is recommended to decrease stress on joints. Wringing water out of sponges would increase joint stress. Patients are encouraged to position joints in the extended (neutral) position. Sleeping with a pillow behind the knees would decrease the ability of the knee to extend and also decrease knee range of motion.

A nurse leads a psychoeducational group for patients experiencing depression. The nurse plans to implement an exercise regime for each patient. The rationale to use when presenting this plan to the treatment team is that exercise. a. has an antidepressant effect comparable to selective serotonin reuptake inhibitors. b. prevents damage from overstimulation of the sympathetic nervous system. c. detoxifies the body by removing metabolic wastes and other toxins. d. improves mood stability for patients with bipolar disorders.

a. has an antidepressant effect comparable to selective serotonin reuptake inhibitors.

A 76-year-old with benign prostatic hyperplasia (BPH) is agitated and confused, with a markedly distended bladder. Which intervention prescribed by the health care provider should the nurse implement first? a. insert the indwelling urinary catheter b. draw blood for a serum creatinine level c. schedule an intravenous pyelogram d. administer lorazepam 0.5 mg PO

a. insert the indwelling urinary catheter

A 72-year-old patient with a history of benign prostatic hyperplasia (BPH) is admitted with acute urinary retention and elevated blood urea nitrogen (BUN) and creatinine levels. Which prescribed therapy should the nurse implement first. a. insert urethral catheter b. obtain renal ultrasound c. draw a CBC d. infuse normal saline at 50mL/hr

a. insert urethral catheter

Which group of people is most prone to red-green color blindness? a. Males b. Females c. Elderly persons d. Children

a. males

The nurse determines that colchicine has been effective for a patient with an acute attack of gout upon finding a. reduced joint pain. b. increased urine output. c. elevated serum uric acid. d. increased white blood cells (WBC).

a. reduced joint pain. Colchicine reduces joint pain in 24 to 48 hours by decreasing inflammation. The recommended increase in fluid intake of 2 to 3 L/day during acute gout would increase urine output but would not indicate the effectiveness of colchicine. Elevated serum uric acid would result in increased symptoms. The WBC count might decrease with decreased inflammation but would not increase.

A 25-yr-old female patient with systemic lupus erythematosus (SLE) who has a facial rash and alopecia tells the nurse, "I never leave my house because I hate the way I look." The nurse will plan interventions with the patient to address the nursing diagnosis of a. social isolation. b. activity intolerance. c. impaired skin integrity. d. impaired social interaction.

a. social isolation. The patient's statement about not going anywhere because of hating the way he or she looks expresses social isolation because of embarrassment about the effects of the SLE. Activity intolerance is a possible problem for patients with SLE, but the information about this patient does not support this. The rash with SLE is nonpruritic. There is no evidence of lack of social skills for this patient.

The nurse is planning to teach a patient how to use relaxation techniques to prevent elevation of blood pressure and heart rate. The nurse is teaching the patient to a. switch from the sympathetic mode of the autonomic nervous system to the parasympathetic mode. b. alter the internal state by modifying electronic signals related to physiologic processes. c. replace stress-producing thoughts and activities with daily stress-reducing thoughts and activities. d. reduce catecholamine production and promote the production of additional beta-endorphins.

a. switch from the sympathetic mode of the autonomic nervous system to the parasympathetic mode.

A person with a fear of heights drives across a high bridge. Which structure will stimulate a response from the autonomic nervous system? a. Thalamus b. Parietal lobe c. Hypothalamus d. Pituitary gland

c. Hypothalamus

The nurse is caring for an adolescent patient who is dying. The patient's parents are interested in organ donation and ask the nurse how the health care providers determine brain death. Which response by the nurse accurately describes brain death determination? a. "If CPR does not restore a heartbeat, the brain cannot function." b. "Brain death has occurred if there is not any breathing or brainstem reflexes." c. "Brain death has occurred if a person has flaccid muscles and does not awaken." d. "If respiratory efforts cease and no apical pulse is audible, brain death is present."

b. "Brain death has occurred if there is not any breathing or brainstem reflexes." The diagnosis of brain death is based on irreversible loss of all brain functions, including brainstem functions that control respirations and brainstem reflexes. The other descriptions describe other clinical manifestations associated with death but are insufficient to declare a patient brain dead.

The nurse is providing health teaching to a group of mothers of school-aged children. Which statement by a mother indicates the need for additional instruction? a. "I will take my child to the audiologist because he doesn't seem to hear me except when I look directly at him." b. "Both of my children have the same eye medication, which is a real bonus, because I only need to buy one bottle." c. "Making my child wear ear plugs when she goes to a rock concert may save her hearing!" d. "I see now why when my child has a cold, he complains about everything tasting blah!"

b. "Both of my children have the same eye medication, which is a real bonus, because I only need to buy one bottle."

When the immunologist says that pathogens possess infectivity, what is the immunologist explaining? Infectivity allows pathogens to: a. spread from one individual to others and cause disease. b. induce an immune response. c. invade and multiply in the host. d. damage tissue.

c. invade and multiply in the host.

Which question asked by the nurse will give the most information about the patient's metastatic bone cancer pain? a. "How long have you had this pain?" b. "How would you describe your pain?" c. "How often do you take pain medication?" d. "How much medication do you take for the pain?"

b. "How would you describe your pain?" Because pain is a multidimensional experience, asking a question that addresses the patient's experience with the pain will elicit more information than the more specific information asked in the other three responses. All of these questions are appropriate, but the response beginning "How would you describe your pain?" is the best initial question. p. 102

A patient tells the nurse, "I will never be happy until I'm as successful as my older sister." The nurse asks the patient to reassess this statement and reframe it. Which reframed state-ment by the patient is most likely to promote coping? a. "People should treat me as well as they treat my sister." b. "I can find contentment in succeeding at my own job level." c. "I won't be happy until I make as much money as my sister." d. "Being as smart or clever as my sister isn't really important."

b. "I can find contentment in succeeding at my own job level."

The nurse determines additional instruction is needed when a patient diagnosed with scleroderma makes which statement? a. "Paraffin baths can be used to help my hands." b. "I should lie down for an hour after each meal." c. "Lotions will help if I rub them in for a long time." d. "I should perform range-of-motion exercises daily."

b. "I should lie down for an hour after each meal." Because of the esophageal scarring, patients should sit up for 2 hours after eating. The other patient statements are correct and indicate teaching has been effective.

After the nurse has finished teaching a patient with osteoarthritis (OA) of the right hip about how to manage the OA, which patient statement indicates a need for more teaching? a. "I can exercise every day to help maintain joint motion." b. "I will take 1 g of acetaminophen (Tylenol) every 4 hours." c. "I will take a shower in the morning to help relieve stiffness." d. "I can use a cane to decrease the pressure and pain in my hip."

b. "I will take 1 g of acetaminophen (Tylenol) every 4 hours." No more than 4 g of acetaminophen (1 g every 6 hours) should be taken daily to decrease the risk for liver damage. Regular exercise, moist heat, and supportive equipment are recommended for OA management.

Which statement by a patient with systemic lupus erythematosus (SLE) indicates the patient has understood the nurse's teaching about the condition? a. "I will exercise even if I am tired." b. "I will use sunscreen when I am outside." c. "I should avoid nonsteroidal antiinflammatory drugs." d. "I should take birth control pills to avoid getting pregnant."

b. "I will use sunscreen when I am outside." Severe skin reactions can occur in patients with SLE who are exposed to the sun. Patients should avoid fatigue by balancing exercise with rest periods as needed. Oral contraceptives can exacerbate lupus. Aspirin and nonsteroidal antiinflammatory drugs are used to treat the musculoskeletal manifestations of SLE.

A patient says, "One result of my chronic stress is that I feel so tired. I usually sleep from 11:00 PM to 6:30 AM. I started setting my alarm to give me an extra 30 minutes of sleep each morning, but I don't feel any better and I'm rushed for work." Which nursing response would best address the patient's concerns? a. "You may need to speak to your doctor about taking a sedative to help you sleep." b. "Perhaps going to bed a half-hour earlier would work better than sleeping later." c. "A glass of wine in the evening might take the edge off and help you to rest." d. "Exercising just before retiring for the night may help you to sleep better."

b. "Perhaps going to bed a half-hour earlier would work better than sleeping later."

A diabetic patient who is hospitalized tells the nurse, "I don't understand why I can keep my blood sugar under control at home with diet alone, but when I get sick, my blood sugar goes up." Which response by the nurse is appropriate? a. "It is probably just coincidental that your blood sugar is high when you are ill." b. "Stressors such as illness cause the release of hormones that increase blood sugar." c. "Increased blood sugar occurs because the kidneys are not able to metabolize glucose as well during stressful times." d. "Your diet is different here in the hospital than at home, and that is the most likely cause of the increased glucose level."

b. "Stressors such as illness cause the release of hormones that increase blood sugar.

A patient with two school-age children has recently been diagnosed with rheumatoid arthritis (RA) and tells the nurse that home life is very stressful. Which initial response by the nurse is most appropriate? a. "You need to see a family therapist for some help with stress." b. "Tell me more about the situations that are causing you stress." c. "Your family should understand the impact of your rheumatoid arthritis." d. "Perhaps it would be helpful for your family to be involved in a support group."

b. "Tell me more about the situations that are causing you stress." The initial action by the nurse should be further assessment. The other three responses might be appropriate based on the information the nurse obtains with further assessment.

A patient in hospice is manifesting a decrease in all body system functions except for a heart rate of 124 beats/min and a respiratory rate of 28 breaths/min. Which statement, if made by the nurse to the patient's family member, is most appropriate? a. "These vital signs will continue to increase until death finally occurs." b. "These vital signs are an expected response now but will slow down later." c. "These vital signs may indicate an improvement in the patient's condition." d. "These vital signs are a helpful response to the slowing of other body systems."

b. "These vital signs are an expected response now but will slow down later." An increase in heart and respiratory rate may occur before the slowing of these functions in a dying patient. Heart and respiratory rate typically slow as the patient progresses further toward death. In a dying patient, high respiratory and pulse rates do not indicate improvement or compensation, and it would be inappropriate for the nurse to indicate this to the family.

A patient who has been diagnosed with inoperable lung cancer and has a poor prognosis plans a trip across the country "to settle some issues with family members." The nurse recognizes that the patient is manifesting which psychosocial response to death? a. Protesting the unfairness of death b. Anxiety about unfinished business c. Fear of having lived a meaningless life d. Restlessness about the uncertainty of prognosis

b. Anxiety about unfinished business The patient's statement indicates that there is some unfinished family business that the patient would like to address before dying. There is no indication that the patient is protesting the prognosis, feels uncertain about the prognosis, or fears that life has been meaningless.

The nurse wishes to use guided imagery to help a patient relax. Which comments would be appropriate to include in the guided imagery script? (Select all that apply.) a. "Imagine others treating you the way they should, the way you want to be treated ..." b. "With each breath, you feel calmer, more relaxed, almost as if you are float-ing ..." c. "You are alone on a beach, the sun is warm, and you hear only the sound of the surf ..." d. "You have taken control, nothing can hurt you now. Everything is going your way ..." e. "You have grown calm, your mind is still, there is nothing to disturb your well-being ..." f. "You will feel better as work calms down, as your boss becomes more under-standing ..."

b. "With each breath, you feel calmer, more relaxed, almost as if you are float-ing ..." c. "You are alone on a beach, the sun is warm, and you hear only the sound of the surf ..." e. "You have grown calm, your mind is still, there is nothing to disturb your well-being ..."

The son of a dying patient tells the nurse, "Mother doesn't really respond any more when I visit. I don't think she knows that I am here." Which response by the nurse is appropriate? a. "Cut back your visits for now to avoid overtiring your mother." b. "Withdrawal can be a normal response in the process of dying." c. "Most dying patients don't know what is going on around them." d. "It is important to stimulate your mother so she can't retreat from you."

b. "Withdrawal can be a normal response in the process of dying." Withdrawal is a normal psychosocial response to approaching death. Dying patients may maintain the ability to hear while not being able to respond. Stimulation will tire the patient and is not an appropriate response to withdrawal in this circumstance. Visitors are encouraged to be "present" with the patient, talking softly and making physical contact in a way that does not demand a response from the patient.

A patient experiencing significant stress associated with a disturbing new medical diagnosis asks the nurse, "Do you think saying a prayer would help?" Select the nurse's best answer. a. "It could be that prayer is your only hope." b. "You may find prayer gives comfort and lowers your stress." c. "I can help you feel calmer by teaching you meditation exercises." d. "We do not have evidence that prayer helps, but it wouldn't hurt."

b. "You may find prayer gives comfort and lowers your stress."

8. Which patient would benefit from education about HIV preexposure prophylaxis (PrEP)? a. A 23-yr-old woman living with HIV infection. b. A 52-yr-old recently single woman just diagnosed with chlamydia. c. A 33-yr-old hospice worker who received a needle stick injury 3 hours ago. d. A 60-yr-old male in a monogamous relationship with an HIV-uninfected partner.

b. A 52-yr-old recently single woman just diagnosed with chlamydia. Preexposure prophylaxis (PrEP) is used to prevent HIV infection. Persons who would be good candidates for PrEP include individuals with a recent diagnosis of an STI and those with more than one partner. Individuals who are not on PrEP but who have a recent high-risk exposure (such as a needle stick) would be better candidates for postexposure prophylaxis (PEP). A person in a monogamous relationship with an HIV-uninfected partner is considered low-risk for HIV infection.

Which information will the nurse include when teaching a patient with newly diagnosed systemic exertion intolerance disease (SEID) about self-management? a. Symptoms usually progress as patients become older. b. A gradual increase in daily exercise may help decrease fatigue. c. Avoid use of over-the-counter antihistamines or decongestants. d. A low-residue, low-fiber diet will reduce any abdominal distention.

b. A gradual increase in daily exercise may help decrease fatigue. A graduated exercise program is recommended to avoid fatigue while encouraging ongoing activity. Because many patients with SEID syndrome have allergies, antihistamines and decongestants are used to treat allergy symptoms. A high-fiber diet is recommended. SEID usually does not progress.

The health care provider has prescribed the following interventions for a patient who is taking azathioprine (Imuran) for systemic lupus erythematosus. Which order will the nurse question? a. Draw anti-DNA blood titer. b. Administer varicella vaccine. c. Naproxen (Aleve) 200 mg BID. d. Famotidine (Pepcid) 20 mg daily.

b. Administer varicella vaccine. Live virus vaccines, such as varicella, are contraindicated in a patient taking immunosuppressive drugs. The other orders are appropriate for the patient.

The nurse is caring for a patient who has diabetes and complains of chronic, burning leg pain even when taking oxycodone (OxyContin) twice daily. Which prescribed medication is the best choice for the nurse to administer as an adjuvant to decrease the patient's pain? a. Aspirin b. Amitriptyline c. Celecoxib (Celebrex) d. Acetaminophen (Tylenol)

b. Amitriptyline The patient's pain symptoms are consistent with neuropathic pain and the tricyclic antidepressants are effective for treating this type of pain. The other medications are more effective for nociceptive pain. p. 106

During the examination of the ear, a dark yellow substance is noted in the ear canal. The tympanic membrane is not visible. The patient's wife complains that he never hears what she says lately. These findings would suggest that the nurse prepare the patient for which procedure? a. Tympanoplasty b. Irrigation of the ear c. Pure tone test d. Otoscopic exam by a specialist

b. Irrigation of the ear

Which information will the nurse include when preparing teaching materials for a patient with an exacerbation of rheumatoid arthritis? a. Affected joints should not be exercised when pain is present. b. Applying cold packs before exercise may decrease joint pain. c. Exercises should be performed passively by someone other than the patient. d. Walking may substitute for range-of-motion (ROM) exercises on some days.

b. Applying cold packs before exercise may decrease joint pain. Cold application is helpful in reducing pain during periods of exacerbation of RA. Because the joint pain is chronic, patients are instructed to exercise even when joints are painful. ROM exercises are intended to strengthen joints and improve flexibility, so passive ROM alone is not sufficient. Recreational exercise is encouraged but is not a replacement for ROM exercises.

Which action is most important for the nurse to take to ensure culturally competent care for an alert, terminally ill Filipino patient? a. Let the family decide how to tell the patient about the terminal diagnosis. b. Ask the patient and family about their preferences for care during this time. c. Obtain information from Filipino staff members about possible cultural needs. d. Remind family members that dying patients prefer to have someone at the bedside.

b. Ask the patient and family about their preferences for care during this time. Because cultural beliefs may vary among people of the same ethnicity, the nurse's best action is to assess the expectations of both the patient and family. The other actions may be appropriate, but the nurse can only plan for individualized culturally competent care after assessment of this patient and family.

Which laboratory result will the nurse monitor to determine if prednisone has been effective for a patient with an acute exacerbation of rheumatoid arthritis? a. Blood glucose b. C-reactive protein c. Serum electrolytes d. Liver function tests

b. C-reactive protein C-reactive protein is a serum marker for inflammation, and a decrease would indicate the corticosteroid therapy was effective. Blood glucose and serum electrolytes will also be monitored to assess for side effects of prednisone. Liver function is not routinely monitored in patients receiving corticosteroids.

A patient with a deep partial thickness burn has been receiving hydromorphone through patient-controlled analgesia (PCA) for 1 week. The nurse caring for the patient during the previous shift reports that the patient wakes up frequently during the night complaining of pain. What action by the nurse is appropriate? a. Administer a dose of morphine every 1 to 2 hours from the PCA machine while the patient is sleeping. b. Consult with the health care provider about using a different treatment protocol to control the patient's pain. c. Request that the health care provider order a bolus dose of morphine to be given when the patient awakens with pain. d. Teach the patient to push the button every 10 minutes for an hour before going to sleep, even if the pain is minimal.

b. Consult with the health care provider about using a different treatment protocol to control the patient's pain. PCAs are best for controlling acute pain. This patient's history indicates a need for a pain management plan that will provide adequate analgesia while the patient is sleeping. Administering a dose of morphine when the patient already has severe pain will not address the problem. Teaching the patient to administer unneeded medication before going to sleep can result in oversedation and respiratory depression. It is illegal for the nurse to administer the morphine for a patient through PCA. p. 107

Which nursing action could the nurse delegate to unlicensed assistive personnel (UAP) when caring for a patient who is using a fentanyl (Duragesic) patch and a heating pad for treatment of chronic back pain? a. Check the skin under the heating pad. b. Count the respiratory rate every 2 hours. c. Ask the patient whether pain control is effective. d. Monitor sedation using the sedation assessment scale.

b. Count the respiratory rate every 2 hours. Obtaining the respiratory rate is included in UAP education and scope of practice. Assessment for sedation, pain control, and skin integrity requires more education and scope of practice. p. 123

A patient with psoriatic arthritis and back pain is receiving etanercept (Enbrel). Which finding is most important for the nurse to report to the health care provider? a. Red, scaly patches are noted on the arms. b. Crackles are auscultated in the lung bases. c. Hemoglobin is 11.1g/dL, and hematocrit is 35%. d. Patient has continued pain after first week of etanercept therapy.

b. Crackles are auscultated in the lung bases. Because heart failure is a possible adverse effect of etanercept, the medication may need to be discontinued. The other information will also be reported to the health care provider but does not indicate a need for a change in treatment. Red, scaly patches of skin and mild anemia are commonly seen with psoriatic arthritis. Treatment with biologic therapies requires time to improve symptoms.

A hospice nurse who has become close to a terminally ill patient is present in the home when the patient dies and feels saddened and tearful as the family members begin to cry. Which action should the nurse take at this time? a. Contact a grief counselor as soon as possible. b. Cry along with the patient's family members. c. Leave the home quickly to allow the family to grieve privately. d. Consider leaving hospice work because patient losses are common.

b. Cry along with the patient's family members. It is appropriate for the nurse to cry and express sadness in other ways when a patient dies, and the family is likely to feel that this is supportive. Contacting a grief counselor, leaving the family to grieve privately, and considering whether hospice continues to be a satisfying place to work are all appropriate actions as well, but the nurse's initial action at this time should be to share the grieving process with the family.

A patient nervously says, "Financial problems are stressing my marriage. I've heard rumors about cutbacks at work; I am afraid I might get laid off." The patient's pulse is 112/minute; respirations are 26/minute; and blood pressure is 166/88. Which nursing inter-vention will the nurse implement? a. Advise the patient, "Go to sleep 30 to 60 minutes earlier each night to increase rest." b. Direct the patient in slow and deep breathing using abdominal muscles. c. Suggest the patient consider that a new job might be better than the present one. d. Tell the patient, "Relax by spending more time playing with your pet."

b. Direct the patient in slow and deep breathing using abdominal muscles.

Which finding will the nurse expect when assessing a patient who has osteoarthritis (OA) of the knee? a. Presence of Heberden's nodules b. Discomfort with joint movement c. Redness and swelling of the knee joint d. Stiffness that increases with movement

b. Discomfort with joint movement Initial symptoms of OA include pain with joint movement. Heberden's nodules occur on the fingers. Redness of the joint is associated with inflammatory arthritis such as rheumatoid arthritis. Stiffness in OA is worse right after the patient rests and decreases with joint movement.

Which information will the nurse include when teaching a patient with newly diagnosed ankylosing spondylitis (AS) about management of the condition? a. Exercise by taking long walks. b. Do daily deep-breathing exercises. c. Sleep on the side with hips flexed. d. Take frequent naps during the day.

b. Do daily deep-breathing exercises. Deep-breathing exercises are used to decrease the risk for pulmonary complications that may result from reduced chest expansion that can occur with AS. Patients should sleep on the back and avoid flexed positions. Prolonged standing and walking should be avoided. There is no need for frequent naps.

Which nursing action can the registered nurse (RN) delegate to unlicensed assistive personnel (UAP) who are assisting with the care of a patient with scleroderma? a.Monitor for difficulty in breathing. b. Document the patient's oral intake. c. Check finger strength and movement. d. Apply capsaicin (Zostrix) cream to hands.

b. Document the patient's oral intake. Monitoring and documenting patient's oral intake is included in UAP education and scope of practice. Assessments for changes in physical status and administration of medications require more education and scope of practice, and should be done by RNs.

Which result for a patient with systemic lupus erythematosus (SLE) is most important for the nurse to communicate to the health care provider? a. Decreased C-reactive protein (CRP) b. Elevated blood urea nitrogen (BUN) c. Positive antinuclear antibodies (ANA) d. Positive lupus erythematosus cell prep

b. Elevated blood urea nitrogen (BUN) Elevated BUN and serum creatinine indicate possible lupus nephritis and a need for a change in therapy to avoid further renal damage. The positive lupus erythematosus cell prep and ANA would be expected in a patient with SLE. A drop in CRP shows decreased inflammation.

Which action will the nurse include in the plan of care for a patient with a new diagnosis of rheumatoid arthritis (RA)? a. Instruct the patient to purchase a soft mattress. b. Encourage the patient to take a nap in the afternoon. c. Teach the patient to use lukewarm water when bathing. d. Suggest exercise with light weights several times daily.

b. Encourage the patient to take a nap in the afternoon. Adequate rest helps decrease the fatigue and pain associated with RA. Patients are taught to avoid stressing joints, use warm baths to relieve stiffness, and use a firm mattress. When the disease is stabilized, a therapeutic exercise program is usually developed by a physical therapist to include exercises that improve flexibility and strength of affected joints, as well as the patient's general endurance.

A 5-year-old male becomes ill with a severe cough. Histologic examination reveals a bacterial infection, and further laboratory testing reveals cell membrane damage and decreased protein synthesis. Which of the following is the most likely cause of this illness? a. Endotoxin b. Exotoxin c. Hemolysis d. Septicemia

b. Exotoxin

What is the priority nursing diagnosis for a patient experiencing chemotherapy-induced anemia? a. Risk for injury related to poor blood clotting b. Fatigue related to decreased cellular oxygenation c. Disturbed body image related to skin color changes d. Imbalanced nutrition, less than body requirements related to anorexia

b. Fatigue related to decreased cellular oxygenation

Which of the following is a characteristic of the human immunodeficiency virus (HIV), which causes AIDS? a. HIV only infects B cells. b. HIV is a retrovirus. c. Infection does not require a host cell receptor. d. After infection, cell death is immediate.

b. HIV is a retrovirus.

A patient has scleroderma manifested by CREST (calcinosis, Raynaud's phenomenon, esophageal dysfunction, sclerodactyly, and telangiectasia) syndrome. Which action will the nurse include in the plan of care? a. Avoid use of capsaicin cream on hands. b. Keep the environment warm and draft free. c. Obtain capillary blood glucose before meals. d. Assist to bathroom every 2 hours while awake.

b. Keep the environment warm and draft free. Keeping the room warm will decrease the incidence of Raynaud's phenomenon, one aspect of the CREST syndrome. Capsaicin cream may be used to improve circulation and decrease pain. There is no need to obtain blood glucose or to assist the patient to the bathroom every 2 hours.

A patient with prostate cancer is taking estrogen daily to control tumor growth. He reports that his left calf is swollen and painful. Which of the following would be the nurse's best action? a. Instruct the client to keep the leg elevated. b. Measure the calf circumference and compare the measurement with the right calf circumference measurement. c. Apply ice to the calf after a 10-minute massage of the area. d. Document assessment findings as an expected response with estrogen therapy.

b. Measure the calf circumference and compare the measurement with the right calf circumference measurement.

The health care provider orders a patient-controlled analgesia (PCA) machine to provide pain relief for a patient with acute surgical pain who has never received opioids before. Which nursing actions regarding opioid administration are appropriate at this time (select all that apply)? a. Assess for signs that the patient is becoming addicted to the opioid. b. Monitor for therapeutic and adverse effects of opioid administration. c. Emphasize that the risk of some opioid side effects increases over time. d. Teach the patient about how analgesics improve postoperative activity levels. e. Provide instructions on decreasing opioid doses by the second postoperative day.

b. Monitor for therapeutic and adverse effects of opioid administration. d. Teach the patient about how analgesics improve postoperative activity levels. Monitoring for pain relief and teaching the patient about how opioid use will improve postoperative outcomes are appropriate actions when administering opioids for acute pain. Although postoperative patients usually need a decreasing amount of opioids by the second postoperative day, each patient's response is individual. Tolerance may occur, but addiction to opioids will not develop in the acute postoperative period. The patient should use the opioids to achieve adequate pain control, so the nurse should not emphasize the adverse effects. p. 115

5. A pregnant woman with asymptomatic chronic human immunodeficiency virus (HIV) infection is seen at the clinic. The patient states, "I am very nervous about making my baby sick." Which information will the nurse include when teaching the patient? a. The antiretroviral medications used to treat HIV infection are teratogenic. b. Most infants born to HIV-positive mothers are not infected with the virus. c. Because it is an early stage of HIV infection, the infant will not contract HIV. d. Her newborn will be born with HIV unless she uses antiretroviral therapy (ART).

b. Most infants born to HIV-positive mothers are not infected with the virus. Only 25% of infants born to HIV-positive mothers develop HIV infection, even when the mother does not use ART during pregnancy. The percentage drops to 2% when ART is used. Perinatal transmission can occur at any stage of HIV infection (although it is less likely to occur when the viral load is lower). ART can safely be used in pregnancy, although some ART drugs should be avoided.

Which task can the nurse assign to unlicensed assistive personnel (UAP) who are working in the orthopedic clinic? a. Grade leg muscle strength for a patient with back pain b. Obtain blood sample for uric acid from a patient with gout c. Perform straight-leg-raise testing for a patient with sciatica d. Check for the knee joint crepitation before arthroscopic surgery

b. Obtain blood sample for uric acid from a patient with gout

What common symptom should be assessed in individuals with immunodeficiency? a. Anemia b. Recurrent infections c. Hypersensitivity d. Autoantibody production

b. Recurrent infections

A cancer patient's susceptibility to the syndrome of inappropriate antidiuretic hormone (SIADH) can be suspected with which of the following laboratory results? a. Serum potassium of 5.2 mmol/L b. Serum sodium of 120 mmol/L c. Hematocrit of 40% d. Blood urea nitrogen (BUN) of 10 mg/dL

b. Serum sodium of 120 mmol/L

A young adult patient with metastatic cancer, who is very close to death, appears restless. The patient keeps repeating, "I am not ready to die." Which action is best for the nurse to take? a. Remind the patient that no one feels ready for death. b. Sit at the bedside and ask if there is anything the patient needs. c. Insist that family members remain at the bedside with the patient. d. Tell the patient that everything possible is being done to delay death.

b. Sit at the bedside and ask if there is anything the patient needs. Staying at the bedside and listening allows the patient to discuss any unresolved issues or physical discomforts that should be addressed. Stating that no one feels ready for death fails to address the individual patient's concerns. Telling the patient that everything is being done does not address the patient's fears about dying, especially because the patient is likely to die soon. Family members may not feel comfortable staying at the bedside of a dying patient, and the nurse should not insist that they remain there.

21. The registered nurse (RN) is caring for a patient who is living with HIV and admitted with tuberculosis. Which task can the RN delegate to unlicensed assistive personnel (UAP)? a. Teach the patient how to dispose of tissues with respiratory secretions. b. Stock the patient's room with the necessary personal protective equipment. c. Interview the patient to obtain the names of family members and close contacts. d. Tell the patient's family members the reason for the use of airborne precautions.

b. Stock the patient's room with the necessary personal protective equipment. A patient diagnosed with tuberculosis would be placed on airborne precautions. Because all health care workers are taught about the various types of infection precautions used in the hospital, the UAP can safely stock the room with personal protective equipment. Obtaining contact information and patient teaching are higher-level skills that require RN education and scope of practice.

A 5-year-old male presents with low-set ears, a fish-shaped mouth, and involuntary rapid muscular contraction. Laboratory testing reveals decreased calcium levels. Which of the following diagnosis is most likely? a. B-lymphocyte deficiency b. T-lymphocyte deficiency c. Combined immunologic deficiency d. Complement deficiency

b. T-lymphocyte deficiency

Which assessment finding for a patient using naproxen (Naprosyn) to treat osteoarthritis is likely to require a change in medication? a. The patient has gained 3 lb. b. The patient has dark-colored stools. c. The patient's pain affects multiple joints. d. The patient uses capsaicin cream (Zostrix).

b. The patient has dark-colored stools. Dark-colored stools may indicate the patient is experiencing gastrointestinal bleeding caused by the naproxen. The patient's ongoing pain and weight gain will also be reported and may indicate a need for a different treatment and/or counseling about avoiding weight gain, but these are not as large a concern as the possibility of gastrointestinal bleeding. Use of capsaicin cream with oral medications is appropriate.

A 29-yr-old woman is taking methotrexate to treat rheumatoid arthritis. Which information from the patient's health history is important for the nurse to report to the health care provider related to the methotrexate? a. The patient had a history of infectious mononucleosis as a teenager. b. The patient is trying to get pregnant before her disease becomes more severe. c. The patient has a family history of age-related macular degeneration of the retina. d. The patient has been using large doses of vitamins and health foods to treat the RA.

b. The patient is trying to get pregnant before her disease becomes more severe. Methotrexate is teratogenic, and the patient should be taking contraceptives during methotrexate therapy. The other information will not impact the choice of methotrexate as therapy.

A patient who has fibromyalgia reports pain at level 7 (0 to 10 scale). The patient tells the nurse, "I feel depressed because I ache too much to play golf." Which patient goal has the highest priority when the nurse is developing the treatment plan? a. The patient will report pain at a level 2 of 10. b. The patient will be able to play a round of golf. c. The patient will exhibit fewer signs of depression. d. The patient will say that the aching has decreased.

b. The patient will be able to play a round of golf. For chronic pain, patients are encouraged to set functional goals such as being able to perform daily activities and hobbies. The patient has identified playing golf as the desired activity, so a pain level of 2 of 10 or a decrease in aching would be less useful in evaluating successful treatment. The nurse should also assess for depression, but the patient has identified the depression as being due to the inability to play golf, so the goal of being able to play golf is the most appropriate. p. 107

Which assessment information obtained by the nurse indicates a patient with an exacerbation of rheumatoid arthritis (RA) is experiencing a side effect of prednisone? a. The patient has joint pain and stiffness. b. The patient's blood glucose is 165 mg/dL. c. The patient has experienced a recent 5-pound weight loss. d. The patient's erythrocyte sedimentation rate (ESR) has increased.

b. The patient's blood glucose is 165 mg/dL. Corticosteroids have the potential to cause diabetes mellitus. The finding of elevated blood glucose reflects this side effect of prednisone. Corticosteroids increase appetite and lead to weight gain. An elevated ESR with no improvement in symptoms would indicate the prednisone was not effective but would not be side effects of the medication.

The nurse would incorporate which of the following into the plan of care as a primary prevention strategy for reduction of the risk for cancer? a. Yearly mammography for women aged 40 years and older b. Using skin protection during sun exposure while at the beach c. Colonoscopy at age 50 and every 10 years as follow-up d. Yearly prostate specific antigen (PSA) and digital rectal exam for men aged 50 and over

b. Using skin protection during sun exposure while at the beach

The nurse suggests that a patient recently diagnosed with rheumatoid arthritis (RA) plan to start each day with a. a brief routine of isometric exercises. b. a warm bath followed by a short rest. c. active range-of-motion (ROM) exercises. d. stretching exercises to relieve joint stiffness.

b. a warm bath followed by a short rest. Taking a warm shower or bath is recommended to relieve joint stiffness, which is worse in the morning. Isometric exercises would place stress on joints and would not be recommended. Stretching and ROM should be done later in the day, when joint stiffness is decreased.

Which information indicates a correct understanding of viral vaccines? Most viral vaccines contain: a. active viruses. b. attenuated viruses. c. killed viruses. d. viral toxins.

b. attenuated viruses.

A patient with gout has a new prescription for losartan (Cozaar) to control the condition. The nurse will plan to monitor a. blood glucose. b. blood pressure. c. erythrocyte count. d. lymphocyte count.

b. blood pressure. Losartan, an angiotensin II receptor antagonist, will lower blood pressure. It does not affect blood glucose, red blood cells, or lymphocytes.

A nurse is teaching about the structure that connects the middle ear with the pharynx. Which structure is the nurse describing? a. Organ of Corti b. Eustachian tube c. Semicircular canal d. Auditory canal

b. eustachian tube

A young child presents to the ophthalmologist for visual difficulties secondary to eye deviation. One of the child's eyes deviates outward, thereby decreasing the visual field. Which of the following diagnoses is most likely? a. Entropia b. Extropia c. Diplopia d. Nystagmus

b. extropia

A 78-year-old patient has been admitted with benign prostatic hyperplasia. What is most appropriate to include in the nursing plan of care? a. limit fluid intake to no more than 1000 mL/day b. leave a light on in the bathroom during the night c. ask the patient to use a urinal so that urine can be measured d. pad the patient's bed to accommodate overflow incontinence

b. leave a light on in the bathroom during the night

A 70-year-old male presents to his primary care provider reporting loss of vision. He reports that he has hypertension and smokes cigarettes. Which of the following disorders is most likely causing his visual loss? a. Presbyopia b. Macular degeneration c. Strabismus d. Amblyopia

b. macular degeneration

Which scenario best demonstrates an example of eustress? An individual a. loses a beloved family pet. b. prepares to take a vacation to a tropical island with a group of close friends. c. receives a bank notice that there were insufficient funds in his/her account for a recent rent payment. d. receives notification that his/her current employer is experiencing financial prob-lems and some workers will be terminated.

b. prepares to take a vacation to a tropical island with a group of close friends.

A patient hospitalized with polymyositis has joint pain; erythematous facial rash; eyelid edema; and a weak, hoarse voice. The safety priority for the patient is addressing the a. acute pain. b. risk for aspiration. c. disturbed visual perception. d. risk for impaired skin integrity.

b. risk for aspiration. The patient's vocal weakness and hoarseness indicate weakness of the pharyngeal muscles and a high risk for aspiration. The other concerns are also appropriate but are not as high a priority as the maintenance of the patient's airway.

4. A patient who is diagnosed with acquired immunodeficiency syndrome (AIDS) tells the nurse, "I feel obsessed with morbid thoughts about dying." Which response by the nurse is appropriate? a. "Thinking about dying will not improve the course of AIDS." b. "Do you think that taking an antidepressant might be helpful?" c. "Can you tell me more about the thoughts that you are having?" d. "It is important to focus on the good things about your life now."

c. "Can you tell me more about the thoughts that you are having?" More assessment of the patient's psychosocial status is needed before taking any other action. The statements, "Thinking about dying will not improve the course of AIDS" and "It is important to focus on the good things in life" or suggesting an antidepressant discourage the patient from sharing any further information with the nurse and decrease the nurse's ability to develop a trusting relationship with the patient.

11. A patient who uses injectable illegal drugs asks the nurse about preventing acquired immunodeficiency syndrome (AIDS). Which response by the nurse is best? a. "Clean drug injection equipment before each use." b. "Ask those who share equipment to be tested for HIV." c. "Consider participating in a needle-exchange program." d. "Avoid sexual intercourse when using injectable drugs."

c. "Consider participating in a needle-exchange program." Participation in needle-exchange programs has been shown to decrease and control the rate of HIV infection. Cleaning drug equipment before use also reduces risk, but it might not be consistently practiced. HIV antibodies do not appear for several weeks to months after exposure, so testing drug users would not be very effective in reducing risk for HIV exposure. It is difficult to make appropriate decisions about sexual activity when under the influence of drugs.

A nurse assesses a patient with chronic cancer pain who is receiving imipramine (Tofranil) in addition to long-acting morphine (MS Contin). Which statement, if made by the patient, indicates to the nurse that the patient is receiving adequate pain control? a. "I'm not anxious during the day." b. "Every night I get 8 hours of sleep." c. "I can accomplish activities without much discomfort." d. "I feel less depressed since I've been taking the Tofranil."

c. "I can accomplish activities without much discomfort." Imipramine is being used in this patient to manage chronic pain and improve functional ability. Although the medication is also prescribed for patients with depression, insomnia, and anxiety, the evaluation for this patient is based on improved pain control and activity level. p. 116

The nurse instructs a patient who has osteosarcoma of the tibia about a scheduled above-the-knee amputation. Which patient statement indicates to the nurse the additional teaching is needed? a. "I will need to participate in physical therapy after surgery" b. "I wish I did not need to have chemotherapy after this surgery" c. "I did not have this bone cancer until my leg broke a week ago." d. "I can use the patient-controlled analgesia to manage postoperative pain."

c. "I did not have this bone cancer until my leg broke a week ago."

An 80-year-old patient is being discharged after he was diagnosed with diabetes mellitus and retinopathy. His daughter has been part of the discharge instruction process. Understanding of the instructions is evident when the daughter says which of the following? a. "I will make sure that Dad always wears warm socks." b. "Dad needs to wear his glasses so he can delay the onset of macular degeneration." c. "I will ask the home health aide to carefully inspect Dad's feet every day when she helps him bathe." d. "We will give him only warm foods, so that he doesn't burn his mouth."

c. "I will ask the home health aide to carefully inspect Dad's feet every day when she helps him bathe."

When the nurse brings medications to a patient with rheumatoid arthritis, the patient refuses the prescribed methotrexate. The patient tells the nurse, "My arthritis isn't that bad yet. The side effects of methotrexate are worse than the arthritis." The most appropriate response by the nurse is a. "You have the right to refuse to take the methotrexate." b. "Methotrexate is less expensive than some of the newer drugs." c. "It is important to start methotrexate early to decrease the extent of joint damage." d. "Methotrexate is effective and has fewer side effects than some of the other drugs."

c. "It is important to start methotrexate early to decrease the extent of joint damage." Disease-modifying antirheumatic drugs (DMARDs) are prescribed early to prevent the joint degeneration that occurs as soon as the first year with RA. The other statements are accurate, but the most important point for the patient to understand is that it is important to start DMARDs as quickly as possible.

A patient newly diagnosed as HIV-positive seeks the nurse's advice on how to reduce the risk of infections. The patient says, "I went to church years ago and I was in my best health then. Maybe I should start going to church again." Which response will the nurse offer? a. "Religion does not usually affect health, but you were younger and stronger then." b. "Contact with supportive people at a church might help, but religion itself is not especially helpful." c. "Studies show that spiritual practices can enhance immune system function and coping abilities." d. "Going to church would expose you to many potential infections. Let's think about some other options."

c. "Studies show that spiritual practices can enhance immune system function and coping abilities."

A patient reports, "I am overwhelmed by stress." Which question by the nurse would be most important to use in the initial assessment of this patient? a. "Tell me about your family history. Do you have any relatives who have problems with stress?" b. "Tell me about your exercise. How much activity do you typically get in a day?" c. "Tell me about the kinds of things you do to reduce or cope with your stress." d. "Stress can interfere with sleep. How much did you sleep last night?"

c. "Tell me about the kinds of things you do to reduce or cope with your stress."

The nurse teaches a student nurse about the action of ibuprofen. Which statement, if made by the student, indicates that teaching was effective? a. "The drug decreases pain impulses in the spinal cord." b. "The drug decreases sensitivity of the brain to painful stimuli." c. "The drug decreases production of pain-sensitizing chemicals." d. "The drug decreases the modulating effect of descending nerves."

c. "The drug decreases production of pain-sensitizing chemicals." Nonsteroidal antiinflammatory drugs (NSAIDs) provide analgesic effects by decreasing the production of pain-sensitizing chemicals such as prostaglandins at the site of injury. Transmission of impulses through the spinal cord, brain sensitivity to pain, and the descending nerve pathways are not affected by NSAIDs. p. 104

Which patient should the nurse refer for hospice care? a. A 70-yr-old patient with lymphoma whose children are unable to discuss issues related to dying b. A 60-yr-old patient with chronic severe pain as a result of spinal arthritis and vertebral collapse c. A 40-yr-old patient with AIDS-related dementia who needs palliative care and pain management d. A 50-yr-old patient with advanced liver failure whose family members can no longer provide care in the home

c. A 40-yr-old patient with AIDS-related dementia who needs palliative care and pain management Hospice is designed to provide palliative care such as symptom management and pain control for patients at the end of life. Patients who require more care than the family can provide, whose families are unable to discuss important issues related to dying, or who have severe pain are candidates for other nursing services but are not appropriate hospice patients.

Which patient seen by the nurse in the outpatient clinic is most likely to require teaching about ways to reduce the risk for osteoarthritis (OA)? a. A 56-yr-old man who has a sedentary office job b. A 38-yr-old man who plays on a summer softball team c. A 56-yr-old woman who works on an automotive assembly line d. A 38-yr-old woman who is newly diagnosed with diabetes mellitus

c. A 56-yr-old woman who works on an automotive assembly line OA is more likely to occur in women as a result of estrogen reduction at menopause and in individuals whose work involves repetitive movements and lifting. Moderate exercise, such as softball, reduces the risk for OA. Diabetes is not a risk factor for OA. Sedentary work is not a risk factor for OA.

According to the Recent Life Changes Questionnaire, which situation would most necessi-tate a complete assessment of a person's stress status and coping abilities? a. A person who has been assigned more responsibility at work b. A parent whose job required relocation to a different city c. A person returning to college after an employer ceased operations d. A man who recently separated from his wife because of marital problems

c. A person returning to college after an employer ceased operations

The nurse reviews the medication orders for an older patient with arthritis in both hips who reports level 3 (0 to 10 scale) hip pain while ambulating. Which medication should the nurse offer as initial therapy? a. Naproxen 200 mg orally b. Oxycodone 5 mg orally c. Acetaminophen 650 mg orally d. Aspirin (acetylsalicylic acid) 650 mg orally

c. Acetaminophen 650 mg orally Acetaminophen is the best first-choice medication. The principle of "start low, go slow" is used to guide therapy when treating older adults because the ability to metabolize medications is decreased and the likelihood of medication interactions is increased. Nonopioid analgesics are used first for mild to moderate pain, although opioids may be used later. Aspirin and nonsteroidal antiinflammatory drugs are associated with a high incidence of gastrointestinal bleeding in older patients. p. 112

A patient with terminal cancer-related pain and a history of opioid abuse complains of breakthrough pain 2 hours before the next dose of sustained-release morphine sulfate (MS Contin) is due. Which action should the nurse take first? a. Use distraction by talking about things the patient enjoys. b. Suggest the use of alternative therapies such as heat or cold. c. Administer the prescribed PRN immediate-acting morphine. d. Consult with the doctor about increasing the MS Contin dose.

c. Administer the prescribed PRN immediate-acting morphine. The patient's pain requires rapid treatment, and the nurse should administer the immediate-acting morphine. Increasing the MS Contin dose and use of alternative therapies and distraction may also be needed, but the initial action should be to use the prescribed analgesic medications. p. 126

A new clinic patient with joint swelling and pain is being tested for systemic lupus erythematosus. Which test will provide the most specific findings for the nurse to review? a. Rheumatoid factor (RF) b. Antinuclear antibody (ANA) c. Anti-Smith antibody (Anti-Sm) d. Lupus erythematosus (LE) cell prep

c. Anti-Smith antibody (Anti-Sm) The anti-Sm is antibody found almost exclusively in SLE. The other blood tests are also used in screening but are not as specific to SLE.

A middle-aged patient tells the nurse, "My mother died 4 months ago, and I just can't get over it. I'm not sure it is normal to still think about her every day." Which nursing diagnosis is most appropriate? a. Hopelessness related to inability to resolve grief b. Complicated grieving related to unresolved issues c. Anxiety related to lack of knowledge about normal grieving d. Chronic sorrow related to ongoing distress about loss of mother

c. Anxiety related to lack of knowledge about normal grieving The patient should be reassured that grieving activities such as frequent thoughts about the deceased are considered normal for months or years after a death. The other nursing diagnoses imply that the patient's grief is unusual or pathologic, which is not the case.

The nurse assesses that a patient receiving epidural morphine has not voided for more than 10 hours. What action should the nurse take initially? a. Place an indwelling urinary catheter. b. Monitor for signs of narcotic overdose. c. Ask if the patient feels the need to void. d. Encourage the patient to drink more fluids.

c. Ask if the patient feels the need to void. Urinary retention is a common side effect of epidural opioids. Assess whether the patient feels the need to void. Because urinary retention is a possible side effect, there is no reason for concern of overdose symptoms. Placing an indwelling catheter requires an order from the health care provider. Usually an in-and-out catheter is performed to empty the bladder if the patient is unable to void because of the risk of infection with an indwelling catheter. Encouraging oral fluids may lead to bladder distention if the patient is unable to void, but might be useful if a patient who is able to void has a fluid deficit. p. 114

The nurse notices a circular lesion with a red border and clear center on the arm of a summer camp counselor who is in the clinic complaining of chills and muscle aches. Which action should the nurse take to follow up on that finding? a. Palpate the abdomen. b. Auscultate the heart sounds. c. Ask the patient about recent outdoor activities. d. Question the patient about immunization history.

c. Ask the patient about recent outdoor activities. The patient's clinical manifestations suggest possible Lyme disease. A history of recent outdoor activities such as hikes will help confirm the diagnosis. The patient's symptoms do not suggest cardiac or abdominal problems or lack of immunization.

A patient being seen in the clinic has rheumatoid nodules on the elbows. Which action will the nurse take? a. Draw blood for rheumatoid factor analysis. b. Teach the patient about injections for the nodules. c. Assess the nodules for skin breakdown or infection. d. Discuss the need for surgical removal of the nodules.

c. Assess the nodules for skin breakdown or infection. Rheumatoid nodules can break down or become infected. They are not associated with changes in rheumatoid factor, and injection is not needed. Rheumatoid nodules are usually not removed surgically because of a high probability of recurrence.

The nurse will anticipate the need to teach a patient who has osteoarthritis (OA) about which medication? a. Prednisone b. Adalimumab (Humira) c. Capsaicin cream (Zostrix) d. Sulfasalazine (Azulfidine)

c. Capsaicin cream (Zostrix) Capsaicin cream blocks the transmission of pain impulses and is helpful for some patients in treating OA. The other medications would be used for patients with rheumatoid arthritis

The nurse is caring for a patient who had abdominal surgery yesterday and is receiving morphine through patient-controlled analgesia (PCA). What action by the nurse is a priority? a. Assessing for nausea b. Auscultating bowel sounds c. Checking the respiratory rate d. Evaluating for sacral redness

c. Checking the respiratory rate The patient's respiratory rate is the highest priority of care while using PCA medication because of the possible respiratory depression. The other areas also require assessment but do not reflect immediately life-threatening complications. p. 115

14. A patient treated for human immunodeficiency virus (HIV) infection for 6 years has developed fat redistribution to the trunk with wasting of the arms, legs, and face. What recommendation will the nurse give to the patient? a. Review foods that are higher in protein. b. Teach about the benefits of daily exercise. c. Discuss a change in antiretroviral therapy. d. Talk about treatment with antifungal agents.

c. Discuss a change in antiretroviral therapy. A frequent first intervention for metabolic disorders is a change in antiretroviral therapy (ART). Treatment with antifungal agents would not be appropriate because there is no indication of fungal infection. Changes in diet or exercise have not proven helpful for this problem.

The nurse is caring for an unresponsive terminally ill patient who has 20-second periods of apnea followed by periods of deep and rapid breathing. Which action by the nurse would be appropriate? a. Suction the patient's mouth. b. Administer oxygen via face mask. c. Document Cheyne-Stokes respirations. d. Place the patient in high Fowler's position.

c. Document Cheyne-Stokes respirations. Cheyne-Stokes respirations are characterized by periods of apnea alternating with deep and rapid breaths. Cheyne-Stokes respirations are expected in the last days of life and are not position dependent. There is also no need for supplemental oxygen by face mask or suctioning the patient.

The nurse is caring for a patient with lung cancer in a home hospice program. Which action by the nurse is appropriate? a. Discuss cancer risk factors and appropriate lifestyle modifications. b. Teach the patient about the purpose of chemotherapy and radiation. c. Encourage the patient to discuss past life events and their meanings. d. Accomplish a thorough head-to-toe assessment several times a week.

c. Encourage the patient to discuss past life events and their meanings. The role of the hospice nurse includes assisting the patient with the important end-of-life task of finding meaning in the patient's life. Frequent head-to-toe assessments are not needed for hospice patients and may tire the patient unnecessarily. Patients admitted to hospice forego curative treatments such as chemotherapy and radiation for lung cancer. Discussion of cancer risk factors and therapies is not appropriate.

9. The nurse palpates enlarged cervical lymph nodes on a patient diagnosed with acute human immunodeficiency virus (HIV) infection. Which action would be appropriate for the nurse to take? a. Instruct the patient to apply ice to the neck. b. Tell the patient a secondary infection is present. c. Explain to the patient that this is an expected finding. d. Request that an antibiotic be prescribed for the patient.

c. Explain to the patient that this is an expected finding. Persistent generalized lymphadenopathy is common in the early stages of HIV infection. No antibiotic is needed because the enlarged nodes are probably not caused by bacteria. Lymphadenopathy is common with acute HIV infection and is therefore not likely the flu. Ice will not decrease the swelling in persistent generalized lymphadenopathy

A patient diagnosed with emphysema has severe shortness of breath and needs portable ox-ygen when leaving home. Recently the patient has reduced activity because of fear that breathing difficulty will occur. A nurse suggests using guided imagery. Which image should the patient be encouraged to visualize? a. Engaging in activity without using any supplemental oxygen b. Sleeping comfortably and soundly, without respiratory distress c. Feeling relaxed and taking regular deep breaths when leaving home d. Having a younger, healthier body that knows no exercise limitations

c. Feeling relaxed and taking regular deep breaths when leaving home

A 50-year-old female experiences decreased blood pressure, decreased oxygen delivery, cardiovascular shock, and subsequent death. A complication of endotoxic shock is suspected. Which of the following is the most likely cause? a. Gram-positive bacteria b. Fungi c. Gram-negative bacteria d. Virus

c. Gram-negative bacteria

12. Which nursing action will be most useful in assisting a young adult to adhere to a newly prescribed antiretroviral therapy (ART) regimen? a. Give the patient detailed information about possible medication side effects. b. Remind the patient of the importance of taking the medications as scheduled. c. Help the patient develop a schedule to decide when the drugs should be taken. d. Encourage the patient to join a support group for adults who are HIV positive.

c. Help the patient develop a schedule to decide when the drugs should be taken. The best approach to improve adherence is to learn about important activities in the patient's life and adjust the ART around those activities. The other actions are also useful, but they will not improve adherence as much as individualizing the ART to the patient's schedule.

A patient who had been complaining of intolerable stress at work has demonstrated the ability to use progressive muscle relaxation and deep breathing techniques. He will return to the clinic for follow-up evaluation in 2 weeks. Which data will best suggest that the patient is successfully using these techniques to cope more effectively with stress? a. The patient's wife reports that he spends more time sitting quietly at home. b. He reports that his appetite, mood, and energy levels are all good. c. His systolic blood pressure has gone from the 140s to the 120s (mm Hg). d. He reports that he feels better and that things are not bothering him as much.

c. His systolic blood pressure has gone from the 140s to the 120s (mm Hg).

Which medication should the nurse administer for a patient with cancer who describes the pain as "deep, aching and at a level 8 on a 0 to 10 scale"? a. Ketorolac tablets b. Fentanyl (Duragesic) patch c. Hydromorphone (Dilaudid) IV d. Acetaminophen (Tylenol) suppository

c. Hydromorphone (Dilaudid) IV The patient's pain level indicates that a rapidly acting medication such as an IV opioid is needed. The other medications may also be appropriate to use but will not work as rapidly or as effectively as the IV hydromorphone. p. 112

6. Which patient exposure by the nurse is most likely to require postexposure prophylaxis when the patient's human immunodeficiency virus (HIV) status is unknown? a. Needle stick injury with a suture needle during a surgery b. Splash into the eyes while emptying a bedpan containing stool c. Needle stick with a needle and syringe used for a venipuncture d. Contamination of open skin lesions with patient vaginal secretions

c. Needle stick with a needle and syringe used for a venipuncture Puncture wounds are the most common means for workplace transmission of blood-borne diseases, and a needle with a hollow bore that had been contaminated with the patient's blood would be a high-risk situation. The other situations described would be much less likely to result in transmission of the virus.

19. Which of these patients who have arrived at the human immunodeficiency virus (HIV) clinic should the nurse assess first? a. Patient whose rapid HIV-antibody test is positive b. Patient whose latest CD4+ count has dropped to 250/μL c. Patient who has had 10 liquid stools in the last 24 hours d. Patient who has nausea from prescribed antiretroviral drugs

c. Patient who has had 10 liquid stools in the last 24 hours The nurse should assess the patient for dehydration and hypovolemia. The other patients also will require assessment and possible interventions, but do not require immediate action to prevent complications such as hypovolemia and shock.

The nurse on a surgical inpatient unit is caring for several patients. Which patient should the nurse assess first? a. Patient with postoperative pain who received morphine sulfate IV 15 minutes ago b. Patient who received hydromorphone (Dilaudid) 1 hour ago and is currently asleep c. Patient who was treated for pain just prior to return from the postanesthesia care unit d. Patient with neuropathic pain who is scheduled to receive a dose of hydrocodone (Lortab) now

c. Patient who was treated for pain just prior to return from the postanesthesia care unit The risk for oversedation is greatest in the first 4 hours after transfer from the postanesthesia care unit. Patients should be reassessed 30 minutes after receiving IV opioids for pain. A scheduled oral medication does not need to be administered exactly at the scheduled time. A patient who falls asleep after pain medication can be allowed to rest. p. 115

The following medications are prescribed by the health care provider for a middle-aged patient who uses long-acting morphine (MS Contin) for chronic back pain but still has ongoing pain. Which medication should the nurse question? a. Morphine b. Dexamethasone c. Pentazocine (Talwin) d. Celecoxib (Celebrex)

c. Pentazocine (Talwin) Opioid agonist-antagonists can precipitate withdrawal if used in a patient who is physically dependent on mu agonist drugs such as morphine. The other medications are appropriate for the patient. p. 114

22. The nurse designs a program to decrease the incidence of human immunodeficiency virus (HIV) infection in the adolescent and young adult populations. Which information should the nurse assign as the highest priority? a. Methods to prevent perinatal HIV transmission b. Ways to sterilize needles used by injectable drug users c. Prevention of HIV transmission between sexual partners d. Means to prevent transmission through blood transfusions

c. Prevention of HIV transmission between sexual partners Sexual transmission is the most common way that HIV is transmitted. The nurse should also provide teaching about perinatal transmission, needle sterilization, and blood transfusion, but the rate of HIV infection associated with these situations is lower.

3. A patient informed of a positive rapid antibody test result for human immunodeficiency virus (HIV) is anxious and does not appear to hear what the nurse is saying. What action by the nurse is most important at this time? a. Teach the patient how to reduce risky behaviors. b. Inform the patient about the available treatments. c. Remind the patient about the need to return for retesting to verify the results. d. Ask the patient to identify individuals who had intimate contact with the patient.

c. Remind the patient about the need to return for retesting to verify the results. After an initial positive antibody test result, the next step is retesting to confirm the results. A patient who is anxious is not likely to be able to take in new information or be willing to disclose information about the HIV status of other individuals.

A patient has not been sleeping well because he is worried about losing his job and not being able to support his family. The nurse takes the patient's vital signs and notes a pulse rate of 112 beats/min, respirations are 26 breaths/min, and his blood pressure is 166/88 instead his usual 110-120/76-84 range. Which nursing intervention or recommendation should be used first? a. Go to sleep 30 to 60 minutes earlier each night to increase rest. b. Relax by spending more time playing with his pet dog. c. Slow and deepen breathing via use of a positive, repeated word. d. Consider that a new job might be better than his present one.

c. Slow and deepen breathing via use of a positive, repeated word.

A patient with rheumatoid arthritis (RA) complains to the clinic nurse about having chronically dry eyes. Which action by the nurse is appropriate? a. Ask the HCP about discontinuing methotrexate b. Remind the patient that RA is a chronic health condition. c. Suggest the patient use over-the-counter (OTC) artificial tears. d. Teach the patient about adverse effects of the RA medications.

c. Suggest the patient use over-the-counter (OTC) artificial tears. The patient's dry eyes are consistent with Sjögren's syndrome, a common extraarticular manifestation of RA. Symptomatic therapy such as OTC eye drops is recommended. Dry eyes are not a side effect of methotrexate. A focus on the prognosis for RA is not helpful. The dry eyes are not caused by RA treatment but by the disease itself.

The patient who had a hip replacement yesterday has a visual acuity of 20/200 after correction. What is the nurse's best action to provide recreational activities during the rehabilitation phase? a. Place the television to the left or right of patient's visual field. b. Encourage the patient to learn braille. c. Suggest use of talking books. d. Provide headphones for listening to music.

c. Suggest use of talking books.

A person with a fear of heights drives across a high bridge. Which division of the autonomic nervous system will be stimulated in response to this experience? a. Limbic system b. Peripheral nervous system c. Sympathetic nervous system d. Parasympathetic nervous system

c. Sympathetic nervous system

A patient tells the nurse, "My doctor thinks my problems with stress relate to the negative way I think about things and suggested I learn new ways of thinking." Which response by the nurse would support the recommendation? a. Encourage the patient to imagine being in calm circumstances. b. Provide the patient with a blank journal and guidance about journaling. c. Teach the patient to recognize, reconsider, and reframe irrational thoughts. d. Teach the patient to use instruments that give feedback about bodily functions.

c. Teach the patient to recognize, reconsider, and reframe irrational thoughts.

An infant is experiencing hemolytic disease of the newborn. Which of the following would be found in the history and physical? a. The mother was exposed to measles. b. The father was exposed to Agent Orange. c. The baby is Rh positive. d. The baby was born 6 weeks prematurely.

c. The baby is Rh positive.

A patient with an acute attack of gout in the right great toe has a new prescription for probenecid. Which information about the patient's home routine indicates a need for teaching regarding gout management? a. The patient sleeps 8-10 hours each night. b. The patient usually eats beef once a week. c. The patient takes one aspirin a day to prevent angina. d. The patient usually drinks about 3 quarts water each day.

c. The patient takes one aspirin a day to prevent angina. Aspirin interferes with the effectiveness of probenecid and should not be taken when the patient is taking probenecid. The patient's sleep pattern will not affect gout management. Drinking 3 quarts of water and eating beef only once or twice a week are appropriate for the patient with gout.

Which laboratory result is important to communicate to the health care provider for a patient who is taking methotrexate to treat rheumatoid arthritis (RA)? a. Rheumatoid factor is positive. b. Fasting blood glucose is 90 mg/dL. c. The white blood cell (WBC) count is 1500/µL. d. The erythrocyte sedimentation rate is elevated.

c. The white blood cell (WBC) count is 1500/µL. Bone marrow suppression is a possible side effect of methotrexate, and the patient's low WBC count places the patient at high risk for infection. The elevated erythrocyte sedimentation rate and positive rheumatoid factor are expected in RA. The blood glucose is normal.

The nurse assesses that a home hospice patient with terminal cancer who complains of severe pain has a respiratory rate of 11 breaths/min. Which action should the nurse take? a. Inform the patient that increasing the morphine will cause the respiratory drive to fail. b. Tell the patient that additional morphine can be administered when the respirations are 12. c. Titrate the prescribed morphine dose up until the patient indicates adequate pain relief. d. Administer a nonsteroidal antiinflammatory drug (NSAID) to improve patient pain control.

c. Titrate the prescribed morphine dose up until the patient indicates adequate pain relief. The goal of opioid use in terminally ill patients is effective pain relief regardless of adverse effects such as respiratory depression. A nonopioid analgesic such as ibuprofen would not provide adequate analgesia or be absorbed quickly. The rule of double effect provides ethical justification for administering an increased morphine dose to provide effective pain control even though the morphine may further decrease the patient's respiratory rate. p. 125

The nurse is planning care for a patient with hypertension and gout who has a red, painful right great toe. Which nursing action will be included in the plan of care? a. Gently palpate the toe to assess swelling. b. Use pillows to keep the right foot elevated. c. Use a footboard to hold bedding away from the toe. d. Teach the patient to avoid use of acetaminophen (Tylenol).

c. Use a footboard to hold bedding away from the toe. Because any touch on the area of inflammation may increase pain, bedding should be held away from the toe, and touching the toe should be avoided. Elevation of the foot will not reduce the pain, which is caused by urate crystals. Acetaminophen can be used for pain management.

After change of shift report, which patient should the nurse assess first? a. a 40 year old with a pleural effusion who reports severe stabbing chest pain b. a 72 year old with cor pulmonale who has 4+ bilateral edema in his legs and feet c. a 64 year old with lung cancer and tracheal deviation after subclavian catheter insertion d. a 28 year old with a history of a lung transplant 1 month ago and a fever of 101F

c. a 64 year old with lung cancer and tracheal deviation after subclavian catheter insertion

After the nurse has received change-of-shift report, which patient should the nurse assess first? a. a patient with pneumonia who has crackles in the right lung base b. a patient with chronic bronchitis who has a low forced vital capacity c. a patient with possible lung cancer who has just returned after bronchoscopy d. a patient with hemoptysis and a 16-mm induration after tuberculin skin testing.

c. a patient with possible lung cancer who has just returned after bronchoscopy

The nurse would expect the patient with an alteration in proprioception to experience vertigo which is manifested by: a. Headache b. Light sensitivity c. A sensation that the room is spinning d. Loss of feeling in the lips

c. a sensation that the room is spinning

A patient had an abdominal-perineal resection for colon cancer. Which nursing action is most important to include in the plan of care of the day after surgery? a. teach about a low-residue diet b. monitor output from the stoma c. assess the perineal drainage and incision d. encourage acceptance of the colostomy stoma

c. assess the perineal drainage and incision

When the immunologist says that pathogens possess virulence, what does virulence mean? a. spreads from one individual to others and causes disease b. induces an immune response c. causes disease d. damages tissue

c. causes disease

The ophthalmologist is teaching about the structure of the eye that prevents light from scattering in the eye. What structure is the ophthalmologist describing? a. Iris b. Pupil c. Choroid d. Retina

c. choroid

Which information indicates a good understanding of bacterial vaccines? Most bacterial vaccines contain: a. fully active bacteria. b. synthetic bacteria. c. dead bacteria. d. bacterial toxins.

c. dead bacteria.

Which should the nurse ask when assessing a 60-year-old patient who has a history of benign prostatic hyperplasia? a. "Have you noticed any unusual discharge from your penis?" b. "has there been any change in your sex life in the past year?" c. has there been a decrease in the force of your urinary system?" d. "have you been experiencing difficulty in achieving an erection?"

c. has there been a decrease in the force of your urinary system?"

A patient who takes multiple medications develops acute gouty arthritis. The nurse will consult with the health care provider before giving the prescribed dose of a. sertraline (Zoloft). b. famotidine (Pepcid). c. hydrochlorothiazide. d. oxycodone (Roxicodone).

c. hydrochlorothiazide. Diuretic use increases uric acid levels and can precipitate gout attacks. The other medications are safe to administer.

A patient hospitalized with a fever and red, hot, painful knees is suspected of having septic arthritis. Information obtained during the nursing history that indicates a risk factor for septic arthritis is that the patient a. had several knee injuries as a teenager. b. recently returned from South America. c. is sexually active with multiple partners. d. has a parent who has rheumatoid arthritis.

c. is sexually active with multiple partners. Neisseria gonorrhoeae is the most common cause for septic arthritis in sexually active young adults. The other information does not point to any risk for septic arthritis.

The most common form of sensorineural hearing loss in the elderly is: a. Conductive hearing loss b. Acute otitis media c. Presbycusis d. Ménière disease

c. presbycusis

After studying about viruses, which information indicates the student has a good understanding of viruses? Viruses: a. contain no DNA or RNA. b. are capable of independent reproduction. c. replicate their genetic material inside host cells. d. are easily killed by antimicrobials.

c. replicate their genetic material inside host cells.

Anakinra (Kineret) is prescribed for a patient with rheumatoid arthritis (RA). When teaching the patient about this drug, the nurse will include information about a. avoiding concurrent aspirin use. b. symptoms of gastrointestinal (GI) bleeding. c. self-administration of subcutaneous injections. d. taking the medication with at least 8 oz of fluid.

c. self-administration of subcutaneous injections. Anakinra is administered by subcutaneous injection. GI bleeding is not a side effect of this medication. Because the medication is injected, instructions to take it with 8 oz of fluid would not be appropriate. The patient is likely to be concurrently taking aspirin or nonsteroidal antiinflammatory drugs (NSAIDs), and these should not be discontinued.

The nurse is administering an interferon and will implement which intervention? A. Giving the medication with meals B. Monitoring daily weights C. Limiting fluids while the patient is taking this medication D. Rotating injection sites

d

d

d

A patient with chronic neck pain is seen in the pain clinic for follow-up. To evaluate whether the pain management is effective, which question is best for the nurse to ask? a. "Has there been a change in pain location?" b. "Can you describe the quality of your pain?" c. "How would you rate your pain on a 0 to 10 scale?" d. "Does the pain keep you from activities that you enjoy?"

d. "Does the pain keep you from activities that you enjoy?" The goal for the treatment of chronic pain usually is to enhance function and quality of life. The other questions are also appropriate to ask, but information about patient function is more useful in evaluating effectiveness. p. 107

The nurse requests that a mother give permission for a hearing test in a newborn infant. The mother questions the importance of such a test. The nurse correctly responds with which of the following statements? a. "This will help us to identify your baby's risk for ear infections the first year of life." b. "Hearing is important so your baby hears and responds to your voice, which makes you feel like a mother." c. "Socialization skills include the need to hear in order to interpret the emotional aspect of the words that are spoken to your child." d. "Imitation of sounds is the first step in language development, and it is important to identify alterations early."

d. "Imitation of sounds is the first step in language development, and it is important to identify alterations early."

A patient tells the nurse, "I know that I should reduce the stress in my life, but I have no idea where to start." What would be the best initial nursing response? a. "Physical exercise works to elevate mood and reduce anxiety." b. "Reading about stress and how to manage it might be a good place to start." c. "Why not start by learning to meditate? That technique will cover everything." d. "Let's talk about what is going on in your life and then look at possible options."

d. "Let's talk about what is going on in your life and then look at possible options."

A patient tells the nurse, "I'm told that I should reduce the stress in my life, but I have no idea where to start." Which would be the best initial nursing response? a. "Why not start by learning to meditate? That technique will cover everything." b. "In cases like yours, physical exercise works to elevate mood and reduce anxiety." c. "Reading about stress and how to manage it might be a good place to start." d. "Let's talk about what is going on in your life and then look at possible options."

d. "Let's talk about what is going on in your life and then look at possible options."

The adult child of a patient diagnosed with major depressive disorder asks, "Do you think depression and physical illness are connected? Since my father's death, my mother has had shingles and the flu, but she's usually not one who gets sick." Which answer by the nurse best reflects current knowledge? a. "It is probably a coincidence. Emotions and physical responses travel on different tracts of the nervous system." b. "You may be paying more attention to your mother since your father died and noticing more things such as minor illnesses." c. "So far, research on emotions or stress and becoming ill more easily is unclear. We do not know for sure if there is a link." d. "Negative emotions and prolonged stress interfere with the body's ability to protect itself and can increase the likelihood of illness."

d. "Negative emotions and prolonged stress interfere with the body's ability to protect itself and can increase the likelihood of illness."

2. A patient who has a positive test for human immunodeficiency virus (HIV) antibodies is admitted to the hospital with Pneumocystis jiroveci pneumonia (PCP) and a CD4+ T-cell count of less than 200 cells/L. Based on diagnostic criteria established by the Centers for Disease Control and Prevention (CDC), which statement by the nurse is correct? a. "The patient will develop symptomatic HIV infection within 1 year." b. "The patient meets the criteria for a diagnosis of acute HIV infection." c. "The patient will be diagnosed with asymptomatic chronic HIV infection." d. "The patient has developed acquired immunodeficiency syndrome (AIDS)."

d. "The patient has developed acquired immunodeficiency syndrome (AIDS)." Development of PCP meets the diagnostic criteria for AIDS. The other responses indicate earlier stages of HIV infection than is indicated by the PCP infection.

An adult male patient is complaining of decreased appetite. He states he just finished taking his antibiotics for an episode of pneumonia. What is the nurse's best response? a. "Your wife should increase the spices in your food, as the pneumonia changes your sense of smell." b. "Notify your doctor immediately, because this is a concerning reaction to the medication." c. "You need to take an appetite stimulant, as your body will need good nutrition to recover from the infection." d. "You should see an improvement in the next week or so. Call if this continues."

d. "You should see an improvement in the next week or so. Call if this continues."

While the nurse is obtaining the health history of a 75-year-old female patient, which of the following has the greatest implication for the development of cancer? a. Being a woman b. Family history of hypertension c. Cigarette smoking as a teenager d. Advancing age

d. Advancing age

A 30-year-old male was diagnosed with HIV. Which of the following treatments would be most effective? a.Reverse transcriptase inhibitors b. Protease inhibitors c. Entrance inhibitors d. Antiretroviral therapy (ART)

d. Antiretroviral therapy (ART)

As the nurse admits a patient in end-stage renal disease to the hospital, the patient tells the nurse, "If my heart or breathing stop, I do not want to be resuscitated." Which action should the nurse take first? a. Place a "Do Not Resuscitate" (DNR) notation in the patient's care plan. b. Invite the patient to add a notarized advance directive in the health record. c. Advise the patient to designate a person to make future health care decisions. d. Ask if the decision has been discussed with the patient's health care provider.

d. Ask if the decision has been discussed with the patient's health care provider. A health care provider's order should be written describing the actions that the nurses should take if the patient requires CPR, but the primary right to decide belongs to the patient or family. The nurse should document the patient's request but does not have the authority to place the DNR order in the care plan. A notarized advance directive is not needed to establish the patient's wishes. The patient may need a durable power of attorney for health care (or the equivalent), but this does not address the patient's current concern with possible resuscitation.

A patient who has had good control for chronic pain using a fentanyl (Duragesic) patch reports rapid onset pain at a level 9 (0 to 10 scale) and requests "something for pain that will work quickly." How will the nurse document the type of pain reported by this patient? a. Somatic pain b. Referred pain c. Neuropathic pain d. Breakthrough pain

d. Breakthrough pain Pain that occurs beyond the chronic pain already being treated by appropriate analgesics is termed breakthrough pain. Neuropathic pain is caused by damage to peripheral nerves or the central nervous system. Somatic pain is localized and arises from bone, joint, muscle, skin, or connective tissue. Referred pain is pain that is localized in uninjured tissue. p. 108

hen reviewing the health record for a new patient with rheumatoid arthritis, the nurse reads that the patient has swan neck deformities. Which deformity will the nurse expect to observe when assessing the patient? a. A b. B c. C d. D

d. D Swan neck deformity involves distal interphalangeal joint hyperflexion and proximal interphalangeal joint hyperextension of the hands. The other deformities are also associated with rheumatoid arthritis: ulnar drift, boutonniere deformity, and hallux vagus.

When a nurse asks a newly admitted patient to describe social supports, the patient says, "My parents died last year and I have no family. I am newly divorced, and my former in-laws blame me. I don't have many friends because most people my age just want to go out drinking." Which action will the nurse apply? a. Advise the patient that being so particular about potential friends reduces social contact. b. Suggest using the Internet as a way to find supportive others with similar values. c. Encourage the patient to begin dating again, perhaps with members of the church. When a nurse asks a newly admitted patient to describe social supports, the patient says, d. Discuss how divorce support groups could increase coping and social support.

d. Discuss how divorce support groups could increase coping and social support.

A 40-yr-old patient is diagnosed with early Huntington's disease (HD). What information should the nurse provide when teaching the patient, spouse, and adult children about this disorder? a. Improved nutrition and exercise can delay disease progression b. Levodopa-carbidopa (Sinemet) will help reduce HD symptoms c. Prophylactic antibiotics decrease the risk for aspiration pneumonia d. Genetic testing is an option for the children to determine their HD risk

d. Genetic testing is an option for the children to determine their HD risk

In caring for a patient following lobectomy for lung cancer, which of the following should the nurse include in the plan of care? a. Position the patient on the operative side only. b. Avoid administering narcotic pain medications. c. Keep the patient on strict bed rest. d. Instruct the patient to cough and deep breathe.

d. Instruct the patient to cough and deep breathe.

The nurse assesses a 78-yr-old who uses naproxen (Aleve) daily for hand and knee osteoarthritis management. Which information requires a discussion with the health care provider about an urgent change in the treatment plan? a. Knee crepitation is noted with normal knee range of motion. b. Patient reports embarrassment about having Heberden's nodes. c. Patient's knee pain while golfing has increased over the last year. d. Laboratory results indicate blood urea nitrogen (BUN) is elevated.

d. Laboratory results indicate blood urea nitrogen (BUN) is elevated. Older patients are at increased risk for renal toxicity caused by nonsteroidal antiinflammatory drugs (NSAIDs) such as naproxen. The other information will also be reported to the health care provider but is consistent with the patient's diagnosis of osteoarthritis and will not require an immediate change in the patient's treatment plan.

The nurse is completing the medication reconciliation form for a patient admitted with chronic cancer pain. Which medication is of most concern to the nurse? a. Amitriptyline 50 mg at bedtime b. Ibuprofen 800 mg 3 times daily c. Oxycodone (OxyContin) 80 mg twice daily d. Meperidine (Demerol) 25 mg every 4 hours

d. Meperidine (Demerol) 25 mg every 4 hours Meperidine is contraindicated for chronic pain because it forms a metabolite that is neurotoxic and can cause seizures when used for prolonged periods. The ibuprofen, amitriptyline, and oxycodone are appropriate medications for long-term pain management. p. 114

The nurse is caring for a patient who received a bone marrow transplant 10 days ago. The nurse would monitor for which of the following clinical manifestations that could indicate a potentially life-threatening situation? a. Mucositis b. Confusion c. Depression d. Mild temperature elevation

d. Mild temperature elevation

A patient who is receiving sustained-release morphine sulfate (MS Contin) every 12 hours for chronic pain experiences level 9 (0 to 10 scale) breakthrough pain and anxiety. Which action by the nurse is appropriate for treating this change in assessment? a. Suggest amitriptyline 10 mg orally. b. Administer lorazepam (Ativan) 1 mg orally. c. Give ibuprofen (Motrin) 400 to 800 mg orally. d. Offer immediate-release morphine 30 mg orally.

d. Offer immediate-release morphine 30 mg orally. The severe breakthrough pain indicates that the initial therapy should be a rapidly acting opioid, such as the immediate-release morphine. Lorazepam and amitriptyline may be appropriate to use as adjuvant therapy, but they are not likely to block severe breakthrough pain. Use of antianxiety agents for pain control is inappropriate because this patient's anxiety is caused by the pain. p. 108

16. The nurse prepares to administer the following medications to a hospitalized patient with human immunodeficiency (HIV). Which medication is most important to administer at the scheduled time? a. Nystatin tablet b. Oral acyclovir (Zovirax) c. Aerosolized pentamidine (NebuPent) d. Oral tenofovir AF/emtricitabine/bictegravir (Biktarvy)

d. Oral tenofovir AF/emtricitabine/bictegravir (Biktarvy) It is important that antiretrovirals be taken at the prescribed time every day to avoid developing drug-resistant HIV. The other medications should also be given as close as possible to the correct time, but they are not as essential to receive at the same time every day.

A patient is brought to the Emergency Department after a motorcycle accident. The patient is alert, responsive, and diagnosed with a broken leg. The patient's vital signs are pulse (P) 72 and respiration (R) 16. After being informed surgery is required for the broken leg, which vital sign readings would be expected? a. P 64, R 14 b. P 68, R 12 c. P 72, R 16 d. P 80, R 20

d. P 80, R 20

A 75-year-old woman walks into the emergency department with complaints of "not feeling well." Her blood pressure is 145/95, pulse 85 beats/min, respirations 24 breaths/min, and blood sugar 300. Upon inspection, the nurse notices that the woman has an open wound on the bottom of her foot, but the patient states she is not aware of this. How should the nurse interpret these findings? a. Normal in the older adult b. A need for the patient to be evaluated for cognitive impairment c. A side effect of anti-hypertensive medication d. Pathologic impairment of sensory responses

d. Pathologic impairment of sensory responses

A patient who is human immunodeficiency virus (HIV)-infected has a CD4+ cell count of 400/μL. Which factor is most important for the nurse to determine before the initiation of antiretroviral therapy (ART) for this patient? a. CD4+ cell count b. How the patient obtained HIV c. Patient's tolerance for potential medication side effects d. Patient's ability to follow a complex medication regimen

d. Patient's ability to follow a complex medication regimen Drug resistance develops quickly unless the patient takes ART medications on a strict, regular schedule. In addition, drug resistance endangers both the patient and community. The other information is also important to consider, but patients who are unable to manage and follow a complex drug treatment regimen should not be considered for ART.

A patient who was recently diagnosed with diabetes is having trouble concentrating. This patient is usually very organized and laid back. Which action should the nurse take? a. Ask the health care provider for a psychiatric referral. b. Administer the PRN sedative medication every 4 hours. c. Suggest the use of a home caregiver to the patient's family. d. Plan to reinforce and repeat teaching about diabetes management.

d. Plan to reinforce and repeat teaching about diabetes management.

An older patient presents to the outpatient clinic with a chief complaint of headache and insomnia. In gathering the history, the nurse notes which factors as contributing to this patient's chief complaint? a. The patient is responsible for caring for two school-age grandchildren. b. The patient's daughter works to support the family. c. The patient is being treated for hypertension and is overweight. d. The patient has recently lost her spouse and needed to move in with her daughter.

d. The patient has recently lost her spouse and needed to move in with her daughter.

17. The nurse is caring for a patient who is human immunodeficiency virus (HIV) positive and taking antiretroviral therapy (ART). Which information is most important for the nurse to address when planning care? a. The patient reports feeling "constantly tired." b. The patient reports having no side effects from the medications. c. The patient is unable to explain the effects of atorvastatin (Lipitor). d. The patient reports missing doses of tenofovir AF/emtricitabine (Descovy).

d. The patient reports missing doses of tenofovir AF/emtricitabine (Descovy). Because missing doses of ART can lead to drug resistance, this patient statement indicates the need for interventions such as teaching or changes in the drug scheduling. Fatigue is a common side effect of ART. The nurse should discuss medication actions and side effects with the patient, but this is not as important as addressing the skipped doses of Descovy.

The home health nurse is making a follow-up visit to a patient with recently diagnosed rheumatoid arthritis (RA). Which assessment made by the nurse indicates more patient teaching is needed? a. The patient takes a 2-hour nap each day. b. The patient has been taking 16 aspirins each day. c. The patient sits on a stool while preparing meals. d. The patient sleeps with two pillows under the head.

d. The patient sleeps with two pillows under the head. The joints should be maintained in an extended position to avoid contractures, so patients should use a small, flat pillow for sleeping. Rest, aspirin, and energy management are appropriate for a patient with RA and indicate teaching has been effective.

13. A patient with human immunodeficiency virus (HIV) infection has developed Cryptosporidium parvum infection. Which outcome would be appropriate for the nurse to include in the plan of care? a. The patient will be free from injury. b. The patient will receive immunizations. c. The patient will have adequate oxygenation. d. The patient will maintain intact perineal skin.

d. The patient will maintain intact perineal skin. The major manifestation of C. pravum infection is loose, watery stools, which would increase the risk for perineal skin breakdown. The other outcomes would be appropriate for other complications (e.g., pneumonia, dementia, influenza) associated with HIV infection.

A 35-year-old female presents with watering and severely reddened eyes. She reports being very sensitive to light. Her primary care provider determined it was viral conjunctivitis caused by: a. Secondary bacterial infections b. Cytomegalovirus c. Herpes virus d. Adenovirus

d. adenovirus

After studying about fungi, which information indicates a correct understanding of fungi? Fungi causing deep or systemic infections: a. are easily treated with penicillin. b. are extremely rare. c. never occur with other infections. d. are commonly opportunistic.

d. are commonly opportunistic.

Which finding by the nurse will be most helpful in determining whether a 67-year-old patient with benign prostatic hyperplasia has an upper urinary tract infection? a. bladder distention b. foul-smelling urine c. suprapubic discomfort d. costovertebral tenderness

d. costovertebral tenderness

A patient reporting painful urination and knee pain is diagnosed with reactive arthritis. The nurse will plan to teach the patient about the need for several months of therapy with a. methotrexate b. anakinra (Kineret). c. etanercept (Enbrel). d. doxycycline (Vibramycin).

d. doxycycline (Vibramycin). Reactive arthritis associated with urethritis is usually caused by infection with Chlamydia trachomatis and requires 3 months of treatment with doxycycline. The other medications are used for chronic inflammatory problems such as rheumatoid arthritis.

A 15-year-old female is diagnosed with an outer ear infection. Which of the following is most likely to cause this infection? a. Haemophilus b. Streptococcus pneumonia c. Moraxella catarrhalis d. Escherichia coli

d. escherichia coli

The nurse would correctly respond that the etiology of a congenital immune deficiency is due to a(n): a. negative response to an immunization. b. adverse response to a medication. c. renal failure. d. genetic defect.

d. genetic defect.

A nurse recalls bacteria become resistant to antimicrobials by: a. proliferation. b. attenuation. c. specialization. d. mutation.

d. mutation.

A 50-year-old diabetic patient experiences visual disturbances and decides to visit his primary care provider. After examination, the primary care provider tells the patient that the cells that allow him to see are degenerated. Which of the following structures is most likely damaged? a. Lens b. Pupil c. Cornea d. Retina

d. retina


संबंधित स्टडी सेट्स

Seizure Pediatric Practice Questions (Test #2, Fall 2020)

View Set

Science - Lunar Cycle from crossword

View Set

real estate principles chapter 2

View Set

L1.8: Andropov, Chernenko, Gorbatchev and Yeltsin Era (1982-91)

View Set